201
Física BACHILLERATO Materia de Proyecto y edición: grupo edebé Dirección general: Antonio Garrido González Dirección editorial: José Luis Gómez Cutillas Dirección de edición de texto: María Banal Martínez Dirección del área de Ciencias y Tecnología: Rosa Comabella Bernat Dirección pedagógica: Santiago Centelles Cervera Dirección de producción: Juan López Navarro Equipo de edición de edebé: Redacción: Mario Suárez García, Núria Lorente Pla, Manuel Martín Doménech, Fernando Monsó Ferré, Carlos Prósper Gisbert y M. Roser Sánchez Gimeno Pedagogía: Elsa Escolano Lumbreras Corrección: Marcos Fco. Poquet Martínez Ilustración: Eva Elías Fortuny Cubierta: Luis Vilardell Panicot Colaboradores: Texto: M. Ángeles Jurado Cardelús, M. Jesús Martínez de Murguía Larrechi, Julio Domingo Pérez Tudela, Gloria Sala Cladellas y Lourdes Sindreu Galobardes Edición: 3.14. Servicios editoriales, S.L. (Artur Klein) Dibujos: Luis Bogajo Peñarroya Preimpresión: Foinsa-Edifilm, S.L. Cualquier forma de reproducción, distribución, comunicación pública o transformación de esta obra sólo puede ser realizada con la autoriza- ción de sus titulares, salvo excepción prevista por la Ley. Diríjase a CEDRO (Centro Español de Derechos Reprográficos – www.cedro.org) si necesita fotocopiar o escanear algún fragmento de esta obra. Es propiedad de grupo edebé © grupo edebé, 2009 Paseo San Juan Bosco, 62 08017 Barcelona www.edebe.com ISBN 978-84-236-9523-2 Depósito Legal. B. ????-2009 Impreso en España Printed in Spain EGS-Rosario, 2 - Barcelona Este libro forma parte del proyecto editorial Edebé y ha sido elaborado según las disposiciones y normas curriculares que desarrollan la Ley Orgánica de Educación (LOE) de 3 de mayo de 2006. Modalidades de Ciencias y de Tecnología Segundo curso de Bachillerato Orientaciones y solucionario

9523 gl fis_tx2_cas

Embed Size (px)

Citation preview

Page 1: 9523 gl fis_tx2_cas

FísicaBACHILLERATO

Materia de

Proyecto y edición: grupo edebé

Dirección general: Antonio Garrido GonzálezDirección editorial: José Luis Gómez CutillasDirección de edición de texto: María Banal MartínezDirección del área de Ciencias y Tecnología: Rosa Comabella BernatDirección pedagógica: Santiago Centelles CerveraDirección de producción: Juan López Navarro

Equipo de edición de edebé:Redacción: Mario Suárez García, Núria Lorente Pla, Manuel Martín Doménech, Fernando Monsó

Ferré, Carlos Prósper Gisbert y M. Roser Sánchez GimenoPedagogía: Elsa Escolano LumbrerasCorrección: Marcos Fco. Poquet MartínezIlustración: Eva Elías FortunyCubierta: Luis Vilardell Panicot

Colaboradores:Texto: M. Ángeles Jurado Cardelús, M. Jesús Martínez de Murguía Larrechi,

Julio Domingo Pérez Tudela, Gloria Sala Cladellas y Lourdes Sindreu GalobardesEdición: 3.14. Servicios editoriales, S.L. (Artur Klein)Dibujos: Luis Bogajo PeñarroyaPreimpresión: Foinsa-Edifilm, S.L.

Cualquier forma de reproducción, distribución, comunicación pública otransformación de esta obra sólo puede ser realizada con la autoriza-ción de sus titulares, salvo excepción prevista por la Ley. Diríjase aCEDRO (Centro Español de Derechos Reprográficos – www.cedro.org)si necesita fotocopiar o escanear algún fragmento de esta obra.

Es propiedad de grupo edebé© grupo edebé, 2009 Paseo San Juan Bosco, 6208017 Barcelonawww.edebe.com

ISBN 978-84-236-9523-2Depósito Legal. B. ????-2009Impreso en EspañaPrinted in SpainEGS-Rosario, 2 - Barcelona

Este libro forma parte del proyecto editorial Edebé y ha sido elaborado según las disposicionesy normas curriculares que desarrollan la Ley Orgánica de Educación (LOE) de 3 de mayo de 2006.

Modalidades de Ciencias y de TecnologíaSegundo curso de Bachillerato

Orientaciones y solucionario

Page 2: 9523 gl fis_tx2_cas

FísicaBACHILLERATO

Materia de

Proyecto y edición: guadiel-grupo edebé

Dirección general: Antonio Garrido GonzálezDirección editorial: José Luis Gómez CutillasDirección de edición de guadiel: Alicia Muñoz MarotoDirección de edición de texto: María Banal MartínezDirección del área de Ciencias y Tecnología: Rosa Comabella BernatDirección pedagógica: Santiago Centelles CerveraDirección de producción: Juan López Navarro

Equipo de edición de guadiel:Redacción: Mario Suárez García, Núria Lorente Pla, Manuel Martín Doménech, Fernando Monsó

Ferré, Carlos Prósper Gisbert y M. Roser Sánchez GimenoPedagogía: Elsa Escolano LumbrerasCorrección: Marcos Fco. Poquet MartínezIlustración: Eva Elías FortunyCubierta: Luis Vilardell Panicot

Colaboradores:Texto: M. Ángeles Jurado Cardelús, M. Jesús Martínez de Murguía Larrechi,

Julio Domingo Pérez Tudela, Gloria Sala Cladellas y Lourdes Sindreu GalobardesEdición: 3.14. Servicios editoriales, S.L. (Artur Klein)Dibujos: Luis Bogajo PeñarroyaPreimpresión: Foinsa-Edifilm, S.L.

Cualquier forma de reproducción, distribución, comunicación pública otransformación de esta obra sólo puede ser realizada con la autoriza-ción de sus titulares, salvo excepción prevista por la Ley. Diríjase aCEDRO (Centro Español de Derechos Reprográficos – www.cedro.org)si necesita fotocopiar o escanear algún fragmento de esta obra.

Es propiedad de guadiel-grupo edebé© guadiel-grupo edebé, 2009 Parque Industrial y de Servicios del Aljarafe (P.I.S.A.)Artesanía, 3-541927 Mairena del Aljarafe (Sevilla)www.edebe.com

ISBN 978-84-8379-197-4Depósito Legal. B. ??????-2009Impreso en EspañaPrinted in SpainEGS-Rosario, 2 - Barcelona

Este libro forma parte del proyecto editorial Edebé y ha sido elaborado según las disposicionesy normas curriculares que desarrollan la Ley Orgánica de Educación (LOE) de 3 de mayo de 2006.

Modalidades de Ciencias y de TecnologíaSegundo curso de Bachillerato

Orientaciones y solucionario

Page 3: 9523 gl fis_tx2_cas

BACHILLERATO

FísicaOrientaciones y solucionario

Page 4: 9523 gl fis_tx2_cas

ÍNDICE

Orientaciones didácticas . . . . . . . . . . . . . . . . . . . . . . . . . . . . . . . 5

Solucionario . . . . . . . . . . . . . . . . . . . . . . . . . . . . . . . . . . . . . . . . . . . . 11

Unidad 1. Dinámica de traslación y de rotación . . . . . . . . . . . . . . . . . . 13

Unidad 2. Campo gravitatorio . . . . . . . . . . . . . . . . . . . . . . . . . . . . . . . . 31

Unidad 3. Gravitación en el universo . . . . . . . . . . . . . . . . . . . . . . . . . . . 45

Unidad 4. Movimientos vibratorios . . . . . . . . . . . . . . . . . . . . . . . . . . . . . 57

Unidad 5. Movimiento ondulatorio . . . . . . . . . . . . . . . . . . . . . . . . . . . . . 69

Unidad 6. Fenómenos ondulatorios . . . . . . . . . . . . . . . . . . . . . . . . . . . . 83

Unidad 7. Campo eléctrico . . . . . . . . . . . . . . . . . . . . . . . . . . . . . . . . . . . 95

Unidad 8. Campo magnético . . . . . . . . . . . . . . . . . . . . . . . . . . . . . . . . . 109

Unidad 9. Inducción electromagnética . . . . . . . . . . . . . . . . . . . . . . . . . . 123

Unidad 10. La luz . . . . . . . . . . . . . . . . . . . . . . . . . . . . . . . . . . . . . . . . . . . 135

Unidad 11. Física relativista . . . . . . . . . . . . . . . . . . . . . . . . . . . . . . . . . . . 147

Unidad 12. Física cuántica . . . . . . . . . . . . . . . . . . . . . . . . . . . . . . . . . . . . 161

Unidad 13. Física nuclear . . . . . . . . . . . . . . . . . . . . . . . . . . . . . . . . . . . . 175

Prácticas . . . . . . . . . . . . . . . . . . . . . . . . . . . . . . . . . . . . . . . . . . . . . . . . . . 187

Propuestas de evaluación . . . . . . . . . . . . . . . . . . . . . . . . . . . . . . . . . . . . . 191

3

Page 5: 9523 gl fis_tx2_cas

Orientaciones didácticas

5

En las páginas siguientes se muestran unas orientaciones didácti-cas referidas tanto a la estructura del libro como a la de las unida-des didácticas.

En la Estructura del libro del alumno se presenta y justifica la utili-dad didáctica de:

• La organización en bloques de los contenidos.

• Las páginas de presentación de cada bloque.

• Las páginas finales: Prácticas, Propuestas de evaluación, Índicealfabético y Tablas.

En la Estructura de las unidades didácticas se presentan y justifi-can los diferentes elementos que las componen, mostrando su in-tencionalidad educativa:

• Páginas iniciales.

• Desarrollo de los contenidos.

• Física y sociedad.

• Resumen.

• Resolución de ejercicios y problemas.

• Ejercicios y problemas.

• Evaluación.

Page 6: 9523 gl fis_tx2_cas

Cada bloque se abre con una doblepágina de presentación, que recogela evolución histórica de los conteni-dos, algunas reseñas sobre hechos einvestigaciones que han contribuidoal desarrollo de la Física, y los cientí-ficos que han hecho posible dichaevolución.

El libro se iniciacon una página depresentación, titula-da Cómo es este libro.En ella se describen suestructura y la metodo-logía específica de lasunidades didácticas.

Física (2.o Bachillerato)se estructura en 13 uni-

dades agrupadas en tresgrandes bloques temáticos

que siguen el plan generalde la materia establecido.

A continuación se desarrollan las unida-des didácticas y, por último, se presentauna serie de prácticas, unas propuestas deevaluación, un índice alfabético de los tér-minos físicos empleados en el desarrollo de lasunidades y unas tablas de unidades y constan-tes físicas.

Se han creado unas páginas denominadas Herramientas matemáticasque permiten al alumno/a repasar una serie de operaciones matemáticas ne-

cesarias para este curso de Física.

Estructura del libro del alumno

7

Page 7: 9523 gl fis_tx2_cas

8

Páginas iniciales:

Las unidades que componen cada bloque se inician siempre con una doble página que incluye los si-guientes elementos:

Contenidos. Presenta los distintosapartados y subapartados de

la unidad, es decir, elrecorrido de las dis-

tintas secuencias deaprendizaje.

Preparación de la uni-dad. Definiciones,

ejemplos y/o actividadesque tienen por finalidad

hacer que el alumno/a re-cuerde, repase, consulte,

investigue... contenidos quenecesita para abordar la uni-

dad.

Imagen acompañada de un texto.Pretende ilustrar cómo laFísica está presenteen distintos ámbi-tos de la vida hu-mana, de la indus-tria y de la sociedad.

Objetivos. Formulaciónde las capacidades queel alumno/a podrá alcan-zar al finalizar la unidad.

Desarrollo de la unidad:

Exposición de los contenidos. Los contenidos están estructurados en apartados y subapartados. Los apar-tados y subapartados guardan relación entre sí y reproducen la secuencia lógica del aprendizaje.

Los contenidos se plantean a partir de hechos, experiencias o conocimientos que los alumnos ya han ad-quirido, o bien, de aquello que les es más cercano y próximo a su entorno, para asentar los conocimien-tos que adquirirán.

Mediante textos expositivos, la descripción de situaciones concretas, demostraciones… y las definiciones,se presentan de forma clara, concisa y estructurada los contenidos que el alumno/a deberá interiorizar pa-ra desarrollar las capacidades previstas.

Márgenes. Se han re-servado para incluirexplicaciones comple-mentarias que el alum-no/a necesita para se-guir correctamente elproceso de aprendizaje(fíjate) o para recordarcontenidos o procedi-mientos estudiados ante-riormente (recuerda), am-pliaciones de especial inte-rés, enlaces a Internet, bio-grafías de científicos, notashistóricas, etc.

Ejemplos. En muchos casos, el desa-rrollo de un contenido culminaen uno o varios ejem-plos. Éstos son mode-los que muestran alalumno/a una aplicaciónpráctica y directa de loscontenidos estudiados.

Actividades. Se proponen,primero, cuestiones y, lue-go, ejercicios de aplicacióny síntesis. Se encuentran alfinal de cada apartado o su-bapartado.

Estructura de las unidades didácticas

Page 8: 9523 gl fis_tx2_cas

9

Páginas finales:

Al finalizar la unidad se presentan cinco apartados: Física y sociedad, Resumen, Resolución de ejerci-cios y problemas, Ejercicios y problemas, y Evaluación.

Física y sociedad. Permite alalumno/a conocer lasrelaciones de laciencia con la tecno-logía, y las implicacio-nes de ambas en lasociedad.

También contribuye a queel alumno/a trabaje loscontenidos referentes a lasenseñanzas transversales.

Resumen. Esteapartado facilita alalumno/a el estu-dio organizado, se-gún la estructura dela unidad, de las ex-presiones matemáti-cas y las ideas claveque han aparecido alo largo de la unidad.

Evaluación.Cuestiones

y problemasque permiten

al alumno/acomprobar si

ha aprendidolos contenidos

esenciales de launidad.

Resolución de ejercicios y problemas. Incluye unaserie de ejercicios y problemas modelo que res-ponden a los contenidos de la unidad y estánresueltos de forma dirigida.

Después de cada ejercicio o problema mode-lo, se proponen ejercicios y problemas quese resuelven según el esquema dado y seindica su solución si ésta es numérica.

Ejercicios y problemas. Una pro-puesta de actividades agrupadasen dos apartados:

• Piensa y resuelve. Cuestiones yejercicios básicos. Están ordena-dos según la secuencia de conteni-dos de la unidad.

• Practica lo aprendido. Listado de pro-blemas que sigue la secuencia de loscontenidos de la unidad, cuya finalidades poner en práctica los conocimientosadquiridos, así como consolidarlos.

Las últimas actividades de esta sección,identificadas por un icono distintivo, deben resolver-se aplicando las TIC.

Page 9: 9523 gl fis_tx2_cas

11

Solucionario

Este solucionario permite al profesor/a la corrección de los ejer-cicios y problemas propuestos en el libro del alumno.

Es recomendable que los alumnos lo utilicen como método deautoevaluación de las actividades que se plantean a lo largo dellibro.

Con esta finalidad, el profesor/a puede fotocopiar las páginascorrespondientes del solucionario y proporcionarlas a los alum-nos y alumnas.

Page 10: 9523 gl fis_tx2_cas

13

La aceleración angular corresponde a la derivada dela velocidad angular respecto al tiempo:

b) La velocidad lineal se obtiene al multiplicar la veloci-dad angular por el radio:

De la misma manera, obtenemos la aceleración tan-gencial como el producto del radio por la aceleraciónangular:

c) La aceleración normal será:

Para t = 5 s, cada una de las componentes de la acele-ración será:

La aceleración total será la suma vectorial:

Su módulo es:

1. DESCRIPCIÓN DEL MOVIMIENTO

1.

� ��a a a

m s m s

t n= + =

= + =

2 2

2 2 20 4 19 6 19 6( , ) ( , ) ,/ / mm s/ 2

, , ( )

� � �

� � �a a a

a u u SI

t n

t n

= +

= +0 4 19 6

a m s

a m st

n

=

= ⋅ + ⋅ + =

0 4

0 4 5 1 6 5 1 6 19 6

2

2 2

,

, , , ,

/

/

a R t t t SIn = = + ⋅ = + +ω2 ( ) , , , , ( )2 0 4 0 4 1 6 1 62 2

a Rrad

sm

m

st = ⋅ = ⋅ =α 1 0 4 0 42 2, ,

v R t t SI= ⋅ = + ⋅ = +ω ( ) , , , ( )2 0 4 0 8 0 4

αω

= =d rad

sdt1 2

1. Dinámica de traslación y de rotación

Dinámica de traslacióny de rotación1

PREPARACIÓN DE LA UNIDAD

• Las magnitudes vectoriales se diferencian de las escalarespor tener una dirección y un sentido. Una magnitud es-calar queda totalmente determinada si sabemos su valornumérico y las unidades, mientras que una magnitud vec-torial, además de valor numérico y unidades, tiene unadirección y un sentido característicos, que la diferenciande otra magnitud vectorial con las mismas unidades y va-lor numérico pero con distinta dirección y/o sentido.

Magnitudes escalares: temperatura, energía, potencia,masa, volumen.

Magnitudes vectoriales: fuerza, campo eléctrico, campomagnético, peso, velocidad.

• Los sólidos rígidos pueden tener dos tipos de movimien-tos: de traslación y de rotación alrededor de un eje.

• En un movimiento de traslación todas las partículas delsólido efectúan el mismo desplazamiento.

En un movimiento de rotación todas las partículas delsólido describen trayectorias circulares alrededor de uneje, excepto las situadas sobre el propio eje, que perma-necen inmóviles.

• Datos: R = 40 cm = 0,4 m; φ = 2 t + 0,5 t2 (SI)

a) La velocidad angular corresponde a la derivada delángulo respecto al tiempo:

ωφ

= = +d

t SIdt

2 ( )

� � � � � �F i j N F i j N1 27 4 6 2= + = − +( ) ( )

a) b) c)d(5t)

dtd(3t )

dtd(2t2

= =5 6t33

2 3

)dt

d(3t )dt

d(t

=

− += −

+ −

6

4 16 4

8 3

2

2

t

tt

td) e)

))dt

= +3 162t t

�i

�F1

� � �R i j N= +( )6

�F2

�j

Sistema de referencia en una dimensión

−8 −6 −4 −2 0 2 4 6 8

r = 6 X

Page 11: 9523 gl fis_tx2_cas

14

3. Datos: en unidades SI

a) Obtenemos la ecuación de la trayectoria a partirde las ecuaciones paramétricas del movimiento:

b) Determinamos los vectores de posición en los ins-tantes t = 2 s y t = 5 s sustituyendo estos valores deltiempo en la expresión de

c) Calculamos el vector desplazamiento entre los dosinstantes restando los vectores de posición corres-pondientes:

d) La distancia recorrida por el móvil coincidirá conel módulo del vector desplazamiento porque setrata de una trayectoria rectilínea.

4. La celeridad es el módulo del vector velocidad. A dife-rencia de la velocidad, que es un vector, la celeridad esun escalar. Por lo tanto, la celeridad carece de direc-ción y sentido.

5. El vector velocidad no se puede descomponer enuna componente tangencial y otra componente nor-mal como la aceleración. El eje tangencial está sobrela recta tangente a la trayectoria, mientras que el ejenormal se define como el eje perpendicular a la tra-yectoria en cada punto. La velocidad es siempre tan-gente a la trayectoria, de forma que su componentenormal será siempre nula. En cambio, la componen-te tangencial coincide con el módulo del vector ve-locidad.

Δ Δs r m m m= = + =( ) ( ) ,� ��3 24 24 22 2

Δ

Δ

� � � � �

� � �r r r r s r s

r i j m

= − = −

= + −

0 5 2

2 40

( ) ( )

( ) (( ) ( )− + = +� � � �i j m i j m16 3 24

� � � � �

�r s i j i j m

r s

( ) ( ) ( )

( )

2 2 3 8 2 16

5

= − + ⋅ = − +

= (( ) ( )5 3 8 5 2 40− + ⋅ = +� � � �i j i j m

�r t( ):

x t t x

y t y x x

= − = +

= = + = +

⎧⎨⎩

3 3

8 8 3 8 24( )

� � �r t t i t j( ) ( ) ,= − +3 8

2. Datos:

a) Obtendremos los vectores de posición sustituyen-do el valor correspondiente del tiempo t en la ex-presión de

b) Para encontrar el vector desplazamiento entre losdos instantes restamos los vectores de posición co-rrespondientes:

El módulo del vector desplazamiento será:

c) Para encontrar la ecuación de la trayectoria, escri-bimos primero las ecuaciones paramétricas del mo-vimiento:

Despejando t en la primera ecuación e introducien-do su expresión en la segunda ecuación paramétrica,obtendremos la ecuación de la trayectoria:

La trayectoria es una parábola.

tx

yx x

yx x

=−

=−⎛

⎝⎜⎞⎠⎟

−−⎛

⎝⎜⎞⎠⎟

=− +

24

24

22

4

12 20

2

2( ))16

x t

y t t

= +

= −

⎧⎨⎪

⎩⎪

4 2

22

� �Δ�r m m m( ) ( ) ,= + =8 4 8 92 2

Δ

Δ

� � � � �

� � �r r r r s r s

r i j m

= − = −

= + −

0 3 1

14 3

( ) ( )

( ) (( ) ( )6 8 4� � � �i j m i j m− = +

� � � � �

�r s i j i j m

r

( ) ( ) ( ) ( )1 4 1 2 1 2 1 62= ⋅ + + − ⋅ = −

(( ) ( ) ( )

( ) (

3 4 3 2 3 2 3

3 14

2s i j

r s i

= ⋅ + + − ⋅

= +

� �

� �33�j m)

�r t( ):

� � �r t t i t t j SI( ) ( ) ( ) ( )= + + −4 2 22

Sistema de referenciaen dos dimensiones

Sistema de referenciaen tres dimensiones

2

–2

–2

2X

X

Y

Y

Z

1

1

4

2

2 4

�j

� � �r i j= − +2 2

�i

�i

� � � �r i j k= + +4 4

�j

�k

La trayectoria esuna recta

Y (m)

X (m)

70

50

30

–40 –20 20 40

10

Page 12: 9523 gl fis_tx2_cas

15

e) Hallamos la velocidad y la aceleración en el instan-te t = 1 s sustituyendo este valor del tiempo en lasexpresiones de la velocidad y la aceleración instan-táneas:

7. Datos: en unidades SI

a) La aceleración instantánea se obtiene derivandoel vector velocidad instantánea:

El vector aceleración instantánea no depende deltiempo, es constante. Por tanto, en el instantet = 2 s la aceleración será la misma que en cual-quier otro instante:

Su módulo también será constante:

b) La componente tangencial de la aceleración es laderivada del módulo de la velocidad. El módulode la velocidad en un instante t será:

y su derivada:

La aceleración tangencial no depende del tiempoen este caso. Por tanto, en t = 2 s su valor será

Además, coincide con el módulo de la aceleracióntotal, de donde se deduce que la componente nor-mal es nula.

Otra forma de ver que la componente normal escero consiste en obtener la ecuación de la posi-ción integrando la ecuación de la velocidad:

Entonces se puede obtener la ecuación de la tra-yectoria:

La trayectoria es una recta. Por lo tanto, la acele-ración normal será cero.

x x t

y y tt x x y y

x− =

− =

⎨⎪⎪

⎩⎪⎪

= − − =0

2

02

20 0

3212

23

( );−− x0

3

� � �

� � �

r t r v t dt

r r t i

t

t

t

( ) ( )

(

= +

− =

⌠⌡⎮

⌠⌡⎮

0

00

0

3 ++ = +t j dt t i t j)� � �3

212

2 2

10 2.m s/

��

a td v t

dtm

s( )

( )= =

� �10 2

� ��v t t t t( ) ( )= + =3 102 2

� � � �� �a s a m s m s m( ) ( ) ( )2 3 1 102 2 2 2= = + =/ / /ss2

� � �a s i j m s( ) ( )2 3 2= + /

�� � �

a tdv t

dti j

m

s( )

( )( )= = +3 2

� � �v t t i t j( ) ,= +3

� � � � �

�v s i j i j m s

a s

( ) ( )

( )

1 6 1 2 1 6 2

1

2= ⋅ + ⋅ = +

=

/

112 1 2 12 2 2⋅ + = +� � � �i j i j m s( ) /

6. Datos: en unidades SI

a) Obtendremos la velocidad media calculando el co-ciente entre el vector desplazamiento y el intervalode tiempo. Encontraremos el vector desplazamien-to entre los dos instantes restando los vectores deposición correspondientes:

Aplicando la definición de velocidad media:

b) Obtenemos la velocidad instantánea derivando elvector de posición:

c) Hallamos la aceleración media calculando el co-ciente entre la diferencia de los vectores velocidad,en los dos instantes, y el intervalo de tiempo. Los vec-tores velocidad en los instantes t = 3 s y t = 0 s se ob-tienen sustituyendo el tiempo t correspondiente en laexpresión de la velocidad instantánea obtenida en elapartado anterior:

Aplicando la definición de aceleración media:

d) La aceleración instantánea se obtiene derivando elvector velocidad instantánea:

�� � �

a tdv t

dtt i j SI( )

( )( ) ( )= = +12 2

���

� � � �a

vt

v s v ss s

i jm = =−

−= +

Δ

Δ

( ) ( )(

3 03 0

18 2 ))m

s2

� � �

� �v s i j m s

v s i

( )

( )

0 6 0 2 0 0

3 6 3 2

2

2

= ⋅ + ⋅ =

= ⋅ + ⋅

/

33 54 6� � �j i j m s= +( ) /

�� � �

v td r t

dtt i t j SI( )

( )( ) ( )= = +6 22

�� � � � �

vrt

r s r ss s

i jm = =−

−= +

Δ

Δ

( ) ( )( )

3 03 0

18 3 (( )SI

� � �

� � �r s i j m

r s i j

( )

( )

0 2 0 0 0

3 2 3 3

3 2

3 2

= ⋅ + =

= ⋅ + = (( )

( ) ( )

54 9

3 00

� �

� � � � �

i j m

r r r r s r s

r

+

= − = −Δ

Δ == + − = +( ) ( )54 9 0 54 9� � � �i j m m i j m

� � �r t t i t j( ) ,= +2 3 2

1. Dinámica de traslación y de rotación

Eje normal

an

at

Eje tangencial

�v

�a

Page 13: 9523 gl fis_tx2_cas

16

Las ecuaciones del movimiento de la bola son:

a) La bola llegará al suelo cuando la altura y sea ce-ro. Encontraremos el tiempo de vuelo de la bolaimponiendo esta condición en su ecuación de laposición:

La solución positiva de esta ecuación da un tiem-po de t = 6,4 s.

b) La velocidad con la que llega la bola al suelo se ob-tiene sustituyendo el tiempo de vuelo que acaba-mos de encontrar en la ecuación de la velocidad:

La bola llega al suelo con una velocidad de62,7 m/s. El signo negativo indica que la bola semueve hacia abajo.

c) A los dos segundos de dejar caer la bola, su veloci-dad viene dada por la misma ecuación con t = 2 s:

La bola se mueve con una velocidad de 19,6 m/sdirigida hacia abajo.

11. Datos:

Tomamos x = 0 e y = 0 en el punto de partida de la bar-ca. Teniendo en cuenta que:

las ecuaciones de movimiento de la barca serán:

x x v t t v tms

tx x= + − = =0 0 2( )

36 361

3 6001 000

110

kmh

kmh

hs

mkm

ms

= ⋅ ⋅ =

v s m s s m s( ) , ,2 9 8 2 19 62= − ⋅ ⋅ = −− /

v s m s s m s( , ) , , ,6 4 9 8 6 4 62 72= − ⋅ ⋅ = −− /

0 20012

9 8 22= − ,m

m

st

y y v t t g t t

y mm

= + − − −

= −

0 0 0 021

2

20012

9 8

( ) ( )

,ss

t

v v g t t vm

st

22

0 0 29 8( ); ,= − − = −

8. Datos:

La ecuación de la velocidad se obtiene integrando laecuación de la aceleración. En este caso, sólo hay unacomponente:

La ecuación de la posición se obtiene integrando laecuación de la velocidad anterior:

Las ecuaciones de la velocidad y de la posición en fun-ción del tiempo son:

9. Datos: a = 3 m/s2; t1 = 25 s; t2 − t1 = 1 min = 60 s;

x0 = 0 m; v0 = 0 m/s; t0 = 0 s

Primera etapa: MRUA. Calculamos la posición y la ve-locidad al final de esta etapa:

Segunda etapa: MRU. Calculamos la posición final dela moto, que coincide con la distancia total recorrida,ya que la posición inicial era x0 = 0:

La distancia total recorrida es de 5 437,5 m.

10. Datos:

X

x x v t t m m s s

x

2 1 1 2 11

2

937 5 75 60= + − = + ⋅ ⋅

=

−( ) , ( )

55 437 5, m

x x v t t a t t

xm

s

1 0 0 1 0 1 02

1 2

12

12

3

= + − + −

=

( ) ( )

(225 937 5

3 25

2

1 0 1 02

) ,

( )

s m

v v a t t m s s

=

= + − = ⋅ ⋅ = 775 m s/

� �

v t t i SI

r t t t

( ) ( )

( )

= +⎛⎝⎜

⎞⎠⎟

= + +⎛

32

12

12

12

4

2

3

⎝⎝⎜⎞⎠⎟

( )�i SI

� � �r t r v t dt i t

t t

( ) ( )= + = + +⎛⎝⎜

⎞⌠⌡⎮ ⌠

⌡⎮0

0 0

2412

32 ⎠⎠⎟

= + +⎛⎝⎜

⎞⎠⎟

( ) ( )

� �

i dt

r t t t i SI412

12

3

� � �

� �

v t v a t dt

v t i t

t

t

t

( ) ( )

( ) ,

= +

= +

⌠⌡⎮

⌠⌡⎮

0

0

0

0 5 3 ( )� �i dt t i SI= +

⎛⎝⎜

⎞⎠⎟

12

32

2

� � � � � �a t i SI v i m s r i m= = =3 0 5 40 0( ); , ;/

Y (m)

200

g

0

y tot

al=

100

m

O

vy

vx X

Y� � �r x i y j= +

� � �v v i v jx y= +

vkmh

ms

vms

y

x

= =

=

36 10

2

�i

�j

Page 14: 9523 gl fis_tx2_cas

17

b) La velocidad en este punto sólo tiene componen-te horizontal, vx, porque vy = 0. Entonces:

v = vx = 35,4 m/s

c) Para hallar el alcance necesitamos determinar elinstante en que el proyectil llega al suelo. Lo obte-nemos imponiendo y = 0:

La solución positiva de esta ecuación de segundogrado es t = 10,9 s.

Sustituyendo este tiempo en la ecuación de la co-ordenada x, hallamos el alcance:

x = v0x t = 35,4 m ⋅ s−1 ⋅ 10,9 s = 387,2 m

13. Datos: R = 30 cm = 0,3 m; ω = 10 rpm ;

t = 2 min = 120 s

a) Expresamos la velocidad angular de 10 rpm enrad/s:

b) Los puntos de la periferia se encuentran a una dis-tancia del centro igual al radio de la rueda. Su ve-locidad lineal será:

Los puntos situados a 10 cm del eje giran con unradio R = 10 cm = 0,1 m. Por tanto:

c) Calculamos el ángulo descrito en 2 min:

Pasamos este ángulo de radianes a revoluciones (ovueltas):

d) La componente tangencial de la aceleración esnula, ya que se trata de un MCU.

La aceleración normal de los puntos de la perife-ria es:

a Rrad

sm

m

sn = =⎛⎝⎜

⎞⎠⎟

⋅ =ωπ2 , ,3

0 3 0 332

2

4012

20ππ

radvuelta

vueltasrad

=

ϕ ω π π= = ⋅ =t13

120 40rad

ss rad

vrad

sm

ms

ms

= = ⋅ = =ω π πR13

0 1 0 03 0 10, , ,

vrad

sm

ms

ms

= = ⋅ = =ω π πR13

0 3 0 1 0 31, , ,

10 1021

160 3minmin

rpmrev

rev sra

= ⋅ ⋅ =π πrad dd

s

012

0 200 35 412

9

0 02= + −

= + −

y v t g t

mms

t

y

, ,,

, ,

8

4 9 35 4 200 0

22

2

m

st

t t− − =

a) La barca habrá cruzado el río cuando llegue a laotra orilla. En esa posición, y = ytotal = 100 m. Halla-mos el tiempo empleado en cruzar el río impo-niendo esta condición en la ecuación de y:

b) La componente y del desplazamiento es la anchu-ra del río, y = 100 m. Calculamos la componente x:

x = vx t = 2 m ⋅ s−1 ⋅ 10 s = 20 m

Por tanto, la distancia recorrida será:

c) Para determinar la ecuación de la trayectoria, des-pejamos el tiempo de la coordenada x y lo sustitui-mos en la ecuación de la coordenada y:

12. Datos: y0 = 200 m; v0 = 50 m/s; α = 45o

Calculamos las componentes de la velocidad ini-cial:

Las ecuaciones del movimiento del proyectil, escri-tas por componentes, serán:

a) El proyectil alcanza la altura máxima en el puntodonde vy = 0. Buscamos el instante en que esto seproduce:

La altura en este instante es:

y y v t g t

y mms

s

y= + −

= + ⋅ −

0 021

2

200 35 4 3 61

, ,22

9 8 3 6

263 8

22, ( , )

,

m

ss

y m

=

v v g t

tv v

gm s m s

m s

y y

y y

= −

=−

=−

0

0 35 4 0

9 8

,

,

/ /

/ 223 6= , s

x x v tms

t y y v t g t

x

x y= + = = + −0 0 0 0235 4

12

( ) , ; ( ) ( )

== + −

= =

200 35 412

9 8

35 4

22

0

, ,

,

mms

tm

st

v vx x

( ); , ,

m s

v v g t vms

ms

ty y y

= − = −

−1

0 35 4 9 8

v vms

ms

v v se

ox

oy

= = ° =

=

0

0

50 45 35 4cos cos ,α

nnms

senms

,α = ° =50 45 35 4

x t

y tt

xy

xx

=

=

⎧⎨⎩

= = =2

10 210

25;

r x y m m m= + = + =2 2 2 220 100 102 0( ) ( ) ,

y v t ty

vm

m ssy

y

= = = =;10010

10/

y y v t t v tms

ty y= + − = =0 0 10( )

1. Dinámica de traslación y de rotación

Page 15: 9523 gl fis_tx2_cas

18

Para que se mueva con velocidad constante, es necesa-rio que la fuerza resultante sea cero:

R = F − Fr = 0 ⇒ F = Fr

La fuerza que debemos aplicar será igual a la fuerza derozamiento:

F = Fr = μc N = μc p = μc m g

F = 0,1 ⋅ 20 kg ⋅ 9,8 m/s2 = 19,6 N

17. Datos:

Calculamos la fuerza resultante:

Hallamos la aceleración que adquiere el cuerpo conesta fuerza resultante:

18. Datos:

La fuerza resultante sobre el sistema es nula. Por tan-to, se conservará la cantidad de movimiento. Calcu-lamos primero la cantidad de movimiento inicial delsistema:

Si cuando las dos bolas chocan quedan unidas, su ma-sa final será:

m = m1 + m2 = 2 kg + 5 kg = 7 kg

Por tanto, la velocidad del sistema después del choqueserá:

�� �

�v

pm

kg m s ikg

ms

i= =⋅

=14

72

/

� � �

� � �p p p

p m v kg m s i kg

= +

= = ⋅ − = − ⋅1 2

1 1 1 2 3 6( ) / mm s i

p m v kg m s i kg m s i

/

/ /

� � � �2 2 2 5 4 20= = ⋅ = ⋅

�� � � � �p p p kg m s i kg m s i kg m= + = − ⋅ + ⋅ = ⋅1 2 6 20 14/ / /ss i

F m a aFm

Nkg

m

s= ⇒ = = = ,

1810

1 8 2

� � � � �F F F N i N i

F N N N

= + = − +

= − =

1 2 34 52

52 34 18

14. Datos: R = 25 cm = 0,25 m; ωo = 0,5 rev/s; t = 40 s

a) Expresamos la velocidad angular inicial en rad/s:

b) Calculamos la aceleración angular a partir de laecuación de la velocidad angular y sabiendo queserá cero en t = 40 s:

Utilizamos la ecuación del movimiento para deter-minar el ángulo girado en 40 s:

Pasamos este ángulo de radianes a revoluciones (ovueltas):

c) Cuando la rueda comienza a frenar, la velocidadangular es la inicial, ω0. La componente normalde la aceleración para un punto de la periferiaserá:

La aceleración tangencial será:

2. CAUSAS DEL MOVIMIENTO

15. Si dejamos caer una piedra desde cierta altura, la Tierraejerce sobre ella una fuerza: la fuerza de la gravedad.Como esta fuerza no se ve compensada, la fuerza resultan-te sobre la piedra no es nula. Como resultado, y tal comoindica la segunda ley de Newton, la piedra adquiere unaaceleración proporcional a la fuerza que actúa sobre ella.

16. Datos:

a Rrad

sm

m

st = = − ⋅ = −απ

, ,40

0 25 0 022 2

a Rrad

sm

m

sn = =

⎝⎜

⎠⎟ ⋅ =ω π0

22

20 25 2 5, ,

20ππ

rad1 vuelta

rad210= vueltas

ϕ ϕ ω α

ϕ π

0 0 ( ) ( )= + − + −

= ⋅ − ⋅

t t t t

ss

0 021

2

4012

rad ππ40

πrad

ss

2240 20⋅ =( ) rad

ω ω α π α

απ π

0 ;= + + ⋅

= − = −

t s

s

0 40

40

rad/s

rad/s440 2

rad

s

ωπ

π0 , ,= = ⋅ =0 5 0 52rev

srevs

radrev

rads

m = 20 kg μc = 0,1

�N

�F

�p

�Fr

X

F2 = 52 N

F1 = 34 N

m = 10 kg

�F

m2

a b

m1 m�v2

�v1�v

�p

�p2

�p1

� �v m s i

m kg1

1

3

2

= −

=

/ � �v m s i

m kg2

2

4

5

=

=

/ m m m kg

p p p

= + =

= +1 2

1 2

7� � �

Page 16: 9523 gl fis_tx2_cas

19

Por lo tanto, si el bloque parte del reposo, no semoverá. Si, en cambio, inclinamos la cuerda cuan-do el bloque ya se estaba moviendo, éste se move-rá con movimiento rectilíneo desacelerado, conaceleración:

20. No, no siempre es cierto. El valor μe N indica la fuerzade rozamiento estática máxima entre un cuerpo y unasuperficie. Superado este valor, el cuerpo comienza adeslizarse, pero mientras el cuerpo está en reposo lafuerza de rozamiento no tiene por qué alcanzar este va-lor máximo. En general, su módulo tiene exactamenteel mismo valor que la compomente tangencial de lafuerza aplicada.

Ejemplo:

21. Si la caja baja a velocidad constante, la aceleración esnula y sabemos que la fuerza resultante es cero.

Eje tangencial: pt − Fr = m a = 0 ⇒ pt = Fr

m g sen α = μc N

Eje normal: N − pn = 0 ⇒ N = pn = p cos α

N = m g cos α

Sustituyendo esta expresión de N en la ecuación deleje tangencial:

22. Supondremos que el sistema se mueve hacia la iz-quierda. Es decir, que el cuerpo 1 desciende por elplano, mientras el cuerpo 2 asciende. Si la aceleraciónresultante fuera negativa, deberíamos repetir el pro-blema cambiando el sentido del movimiento.

m g sen m g

sentg

cos

cos,

α μ α

μαα

α

=

= =

c

c 0 255

aRm

F Fm

N Nkg

m

sx r= =

−=

−= −

4 2 5 13 5

0 26 2

, ,,

,

19. Datos: m = 3,5 kg; T = 6 N

a)

b) Para que la velocidad sea constante, es necesarioque la fuerza resultante sea nula:

R = F − Fr = 0 ⇒ Fr = F = 6 N

A partir de la fuerza de rozamiento, calculamos elcoeficiente cinético de rozamiento:

c)

Si la cuerda se inclina 45°, la fuerza se podrá des-componer en dos componentes y aparecerá unanueva componente vertical que antes no existía:

— Componente horizontal: Fx = F cos 45o = 4,2 N

— Componente vertical: Fy = F sen 45o = 4,4 N

Como la componente vertical es menor que elpeso, el bloque sólo puede moverse horizontal-mente:

p = m g = 3,5 kg ⋅ 9,8 m/s2 = 34,3 N > Fy = 4,2 N

Pero a causa de esta nueva componente vertical, lafuerza normal es menor que en el caso anterior.Teniendo en cuenta que el bloque no se mueveverticalmente y que, por tanto, la resultante en eleje vertical es cero:

Ry = N + Fy − p = 0 ⇒ N = p − Fy = 30,1 N

Entonces, la fuerza de rozamiento será más peque-ña que con la cuerda horizontal:

Fr = μc N = 0,17 ⋅ 30,1 N = 5,1 N < 6 N

Pero ahora la componente horizontal de la fuerzaejercida por la cuerda, Fx, también es menor queen los apartados anteriores. Además, Fx es más pe-queña que la fuerza de rozamiento:

Fx = 4,2 N < 5,1 N = Fr

F N p m g

Fm g

N

kg m

r

r

= = =

= =⋅

μ μ μ

μ

c c c

c , ,

6

3 5 9 8 ⋅⋅=

s2 0 17,

1. Dinámica de traslación y de rotación

m = 3,5 kg

�N

�F

�Fr

� �p m g=

m = 3,5 kg

Fx

Fy

�N

�Fr

�F

� �p m g=

m = 25 kg

F = 50 N

Fr = 98 NFr = 50 N

Fr = 24,5 N

Reposo Reposo

Desplazamiento

F = 98 N

F = 100 N

μe = 0,4

μc = 0,1

μe = 0,4

pn

p

α = 14o

α

�N

�Fr

�p

Page 17: 9523 gl fis_tx2_cas

20

y la sustituimos en la primera:

Despejamos la velocidad:

Teniendo en cuenta que R = 1 sen α:

Sustituyendo los valores del problema,

24. Una bola que gira verticalmente atada a una cuerdano cae en el punto más alto porque la fuerza del pesose emplea en cambiar la dirección del movimiento dela bola y no en hacerla caer al suelo. Si no actuara so-bre la bola ninguna fuerza, ésta no seguiría una trayec-toria circular, sino recta. La fuerza del peso de la bolacontribuye, junto con la tensión de la cuerda, a apor-tar la fuerza centrípeta necesaria para que la bolalleve a cabo un movimiento circular.

25. Datos: m = 150 g = 0,15 kg; R = 80 cm = 0,8 m; Tmáx = 10 N

a) La cuerda se romperá en el punto inferior de latrayectoria. En este punto la fuerza centrípeta esigual a la tensión de la cuerda menos el peso dela piedra.

b) La cuerda se romperá donde la tensión es máxi-ma. Esto sucede en el punto inferior de la trayec-toria, donde la fuerza del peso actúa en sentidocontrario a la tensión.

3. MOVIMIENTO DE ROTACIÓN

26. Un cuerpo sometido a una fuerza resultante nula y aun momento no nulo tendrá un movimiento de rota-ción debido al momento. Si inicialmente estaba en re-poso, no se trasladará. Si, en cambio, inicialmente es-taba en traslación, seguirá moviéndose con velocidadconstante y en trayectoria rectilínea.

Si está sometido a una fuerza resultante no nula, ten-drá un movimiento de traslación acelerado. Además,si el momento no es nulo, tendrá un movimiento derotación.

T pm v

Rv

R T m gm

vm N

máxmáx− = =

=−

;( )

, (

2

0 8 10 0,, , ),

,15 9 8

0 156 7

2kg m skg

ms

⋅=

/

vm

sm sen tg m s= ⋅ ⋅ ⋅ =9 8 0 5 11 5 11 5 0 452, , , , ,° ° /

v g R tg g sen tg= =α α α1

v g R tg= α

m gsen

m vR

m g tg mvR

cos αα

α

=

=

2

2

Representamos todas las fuerzas que actúan sobre cadacuerpo y calculamos la aceleración:

Cuerpo 1: p1t − Fr − T = m1 a

Cuerpo 2: T − p2 = m2 a

Sumando las dos ecuaciones:

Despejamos la tensión de la ecuación del cuerpo 2:

T = m2a + p2 = m2 a + m2 g = m2 (a + g)

T = 4 kg ⋅ (2,1 m/s2 + 9,8 m/s2) = 47,6 N

23. Datos:

Aplicamos la segunda ley de Newton, teniendo encuenta que la fuerza resultante en la dirección radialtiene que ser la fuerza centrípeta:

Eje X:

Eje Y:

Despejamos la tensión de la segunda ecuación:

Tm g

=cos α

T F T senm v

RT p T m g

x c

y

= =

= =

;

; cos

α

α

2

p F p m m a

ap F p

m mm g sen

t r

t r

1 2 1 2

1 2

1 2

1

− − = +

=− −

+=

( )

cos

, ,

α μ α

°

− −

+

=⋅ ⋅ −

cm g m gm m

asen

1 2

1 2

8 9 8 60 0 1 ⋅⋅ ⋅ ⋅ − ⋅

+

=

8 9 8 60 4 9 88 4

2 1 2

, cos ,

,

°

a m s/

60o

60o

p1t

m1 = 8 kgμc = 0,1

p1n

m2 = 4 kg

�N

�Fr

�T �

T

�p1

�p2

XTx

Ty

1 = 0,5 m

α = 11,5o

α

Y

�p

�T

Page 18: 9523 gl fis_tx2_cas

21

30. Datos: m = 1 kg; R = 0,25 m

Cilindro. Utilizaremos la expresión del momento deinercia para un cilindro macizo que aparece en la pá-gina 53 del libro del alumno:

Esfera. Aplicamos la fórmula correspondiente a la es-fera maciza que aparece en la página 53 del libro delalumno.

31. Para comprobar que al dividir las unidades del mo-mento de la fuerza entre las del momento de inerciase obtienen las de la aceleración angular, es necesariotener presente que las unidades de fuerza, los newton(N), son equivalentes a kg ⋅ m ⋅ s −2. Entonces:

32. La aceleración angular se relaciona con el momentoresultante sobre el cuerpo mediante la ecuación fun-damental de la dinámica de rotación:

Por su definición, el momento de inercia es siemprepositivo. Por tanto, la aceleración angular tendrá siem-pre la misma dirección y sentido que el momento re-sultante.

Nota para el profesor/a: en la página 41 del libro del alum-no se aclara que esta ecuación sólo es realmente válida si eleje de rotación es un eje de simetría del cuerpo que permane-ce fijo o siempre paralelo a sí mismo.

33. Datos:

Calculamos primero el momento de inercia y el mo-mento de la fuerza, para aplicar después la ecuaciónfundamental de la dinámica de traslación.

� �M I= α

�M

IN M

kg m

kg m s m

kg ms

⎡⎣ ⎤⎦⎡⎣ ⎤⎦

=⋅

⋅=

⋅ ⋅ ⋅

⋅= = ⎡

−−

2

2

22 α⎣⎣ ⎤⎦

I m R kg m kg m= = ⋅ ⋅ = ⋅25

25

1 0 25 0 0252 2 2( , ) ,

I m R kg m kg m= = ⋅ ⋅ = ⋅12

12

1 0 25 0 0312 2 2( , ) ,

r a m r r

I m r ma kg mb i i

1 2 3

2 2 2

3 0

1 3

= = = =

= = = ⋅ =

,

( ) 99 2kg mi

⋅∑

27. Datos:

Calculamos el momento de la fuerza como el produc-to vectorial de con la fuerza :

28. El momento de inercia Ii de una partícula, definido co-mo Ii = mi ri

2, depende del eje respecto al cual lo calcu-lemos, ya que variará la distancia ri de la partícula aleje. Del mismo modo, el momento de inercia de un sis-tema discreto de partículas depende del eje que escoja-mos, pues cambiarán las distancias ri de todas las par-tículas.

29. Datos: m1 = m2 = m3 = m = 1 kg; a = 3 m; b = 4 m

Calculamos el momento de inercia a partir de su defi-nición. Teniendo en cuenta que dos de las masas estánsobre el eje de rotación, sólo contribuirá al momentode inercia la tercera masa.

— Si gira en torno al primer cateto (a): r1 = r2 = 0, r3 = b = 4 m

— Si gira en torno al segundo cateto (b):

I m r mb kg m kg ma i ii

= = = ⋅ = ⋅∑ ( )2 2 2 21 4 16

� � �

� � �M r F m x N

M i j

= ⋅ =

= +

( , , ) ( , , )

(

1 2 1 3 5 1

2 ++ + +⎡⎣ ⎤⎦ ⋅

= − + −

� � � �

� � � �k x i j k N m

M i j

) ( )

(

3 5

3 2 kk N m) ⋅

�F

�r

� �F N r m= =( , , ) ; ( , , )3 5 1 1 2 1

1. Dinámica de traslación y de rotación

m

m

mm

a = 3 m

mm

m = 1 kg

b = 4 m

b = 4 m

a = 3 m

F = 1,5 N

M = 100 g

R = 30 cm

�k

Page 19: 9523 gl fis_tx2_cas

22

Tenemos en cuenta la orientación de los ejes para es-cribir el vector:

NOTA: La solución de este ejercicio depende de la elección delos ejes. Una variación de la colocación de los ejes dará unarespuesta diferente pero igualmente correcta. La solución queacompaña los ejercicios del libro del alumno se correspondecon la resolución que aparece en este solucionario.

36. Datos: M = 7,35 ⋅ 1022 kg; R = 1,74 ⋅ 106 m;

ω = 1 rev cada 28 días.

Expresamos la velocidad angular en el SI:

Hallamos el momento de inercia a partir de la expre-sión para una esfera maciza de la página 42 del librodel alumno:

Calculamos el momento angular:

Tenemos en cuenta la orientación de los ejes y quegira hacia el Este:

NOTA: La solución de este ejercicio depende de la elección delos ejes. Una variación de la colocación de los ejes dará unarespuesta diferente pero igualmente correcta. La solución queacompaña los ejercicios del libro del alumno se correspondecon la resolución que aparece en este solucionario.

37. Datos:

� �L k kg m s= ⋅ ⋅2 31 1029 2, /

L I kg m rad s

L

= = ⋅ ⋅ ⋅ ⋅

= ⋅

−ω 8 9 10 2 6 10

2 31 10

34 2 6, ,

,

/229 2kg m s⋅ /

I MR kg m

I

= = ⋅ ⋅ ⋅ ⋅

=

25

25

7 35 10 1 74 10

8 9

2 22 6 2, ( , )

, ⋅⋅ ⋅1034 2kg m

ωπ

= ⋅ ⋅ ⋅1

281

241

3 60021

vueltad

dh

hs v

raduuelta

rads

= ⋅ −2 6 10 6,

� �L k m= ⋅6 2π kg /s

Aplicamos la fórmula de la página 42 del libro delalumno para calcular el momento de inercia de un dis-co macizo:

Para calcular el momento de la fuerza, tenemos encuenta que y son perpendiculares. Entonces:

Aplicamos la ecuación fundamental de la dinámica derotación:

NOTA: La solución de este ejercicio depende de la elección delos ejes. Una variación de la colocación de los ejes dará unarespuesta diferente pero igualmente correcta. La solución queacompaña los ejercicios del libro del alumno se correspondecon la resolución que aparece en este solucionario.

34. Si el eje de rotación es fijo, todas las partículas del sóli-do rígido giran con velocidad angular de la misma di-rección y sentido. Lo que variará entre una partícula yotra será el momento de inercia. Pero como éste es unescalar, el momento angular y la velocidad angular decada partícula son paralelos y del mismo sentido. Si to-das las partículas tienen velocidades angulares de lamisma dirección y sentido, todos los momentos angu-lares serán paralelos.

35. Datos: m1 = m2 = 1,5 kg; d = 1 m; ω = 4 rev/s

Escribimos primero la velocidad angular en unidadesdel SI:

Calculamos el momento de inercia del sistema. La dis-

tancia de cada masa al eje de giro será

Por tanto:

Entonces, el momento angular será:

L I kg m m= = ⋅ ⋅ = ⋅,ω π π0 75 8 62 2rad/s kg /s

I m r kg m kg mi i= = ⋅ ⋅ = ⋅∑ , ( , ) ,2 2 22 1 5 0 5 0 75

rd

m= =2

0 5, .

ωπ

π= ⋅ =421

8revs

radrev

rads

� � �� �

M IMI

k N m

kg m= = =

⋅=

,

,α, α

0 45

0 00451002

�k

rad

s2

� �

� �

� �M F R N m

M N m M

= ⋅ = ⋅

= ⋅ =

1 5 0 3

0 45 0 4

, ,

, ; , 55�k N m⋅

�R

�F

I MR kg m kg m= = ⋅ ⋅ = ⋅12

12

0 1 0 3 0 00452 2 2, ( , ) ,

m = 1,5 kg m = 1,5 kg

d = 1 m

�k

�k

R

R = 1,74 · 106 m

M = 7,35 · 1022 kg

M = 200 g = 0,2 kg

R = 0,4 m

ω = 45 rpm

Page 20: 9523 gl fis_tx2_cas

23

Aplicamos ahora la conservación del momento angu-lar para hallar la velocidad angular final:

40. Datos: ω0 = 30 rpm; r0 = 70 cm = 0,7 m;

m = 350 g = 0,35 kg; Iplat = 120 kg/m2;

r = 53 cm = 0,35 m

Aplicamos el principio de conservación del momentoangular, teniendo en cuenta que los momentos deinercia de la niña en la plataforma y de las masas quesostiene con sus manos se suman:

41. Una experiencia sencilla para observar la conserva-ción del momento angular es la siguiente:

— Montamos un dispositivo como el de la figura: enuna barra vertical (puede servir, por ejemplo, lapata de una mesa) atamos una cuerda con unabola o cualquier objeto un poco pesado en el ex-tremo.

— Damos impulso a la bola y la hacemos girar alre-dedor de la barra. Al irse enroscando la cuerdaen la barra, la distancia de la bola a la barra (ejede rotación) irá disminuyendo. Por tanto, dismi-

L L I I

mr mr0 0 0

02 22 2

= =

+ = +

;

) )plat plat

ω ω

(Ι ω (Ι0 ωω

ω(Ι ω

Ι

ω

0=+

+

=

)

(

plat

plat

/

2

2

120

02

2

mr

mr

kg mm kg m rpm

m

2 2

2

2 0 35 0 7 30

120 2 0

+ ⋅ ⋅

+ ⋅

, ( , ) )

kg/ ,, ( , )

,

,min

35 0 35

30 06

30 06

2kg ⋅

=

=

m

rpm

rev

ω

ω ⋅⋅ ⋅ =160

21

3 15min

,s rev

rads

π rad

L LL

I I

m s

m

= =+

=⋅

⋅ +

00

1 22

2

0 108

0 018

;

,

,

ω

ωπ kg /

kg 0,004 kg ⋅=

= ⋅

m

rads

rads

vuelta

2 4 9

15 412

,

,

π

ωππ rad

= 2 45,vueltas

s

Expresamos primero la velocidad angular en unidadesdel SI:

Aplicamos la fórmula de la página 42 del libro delalumno para calcular el momento de inercia de un dis-co macizo:

Calculamos el momento angular:

Teniendo en cuenta la orientación de los ejes y el sen-tido de giro indicado en la figura, escribimos el mo-mento angular en forma vectorial:

NOTA: La solución de este ejercicio depende de la elección delos ejes. Una variación de la colocación de los ejes dará unarespuesta diferente pero igualmente correcta. La solución queacompaña los ejercicios del libro del alumno se correspondecon la resolución que aparece en este solucionario.

38. Si una persona situada sobre una plataforma circular enrotación se desplaza hacia su centro, la distancia de lapersona al eje disminuirá. Por tanto, su momento deinercia también será menor. Por la conservación del mo-mento angular, la velocidad angular de la plataformaaumentará y girará más rápido.

39. Datos: R1 = 30 cm = 0,3 m; M1 = 0,4 kg; ω0 = 3 rev/s; R2 = 20 cm = 0,2 m; M2 = 0,2 kg

Aplicamos la expresión que aparece en la página 42 dellibro del alumno para calcular el momento de inerciade un disco macizo.

Expresamos la velocidad angular inicial del primer dis-co en unidades del SI:

Inicialmente, todo el momento angular del sistema esdebido al primer disco:

Al final, cuando los dos discos giran unidos, el momen-to de inercia será la suma de los dos, y por tanto, su mo-mento angular:

L I I= +( )1 2 ω

L L I

L kg m0 1 1

020 018 6 0 108

= =

= ⋅ ⋅ =, ,

ω

π π

0

rad/s kgg m /s2⋅

ωπ

π0 = ⋅ =321

6revs rev

rads

rad

I M R kg m kg m1 1 12 2 21

212

0 4 0 3 0 018= = ⋅ ⋅ = ⋅, ( , ) ,

II M R kg m kg m2 2 22 21

212

0 2 0 2 0 004= = ⋅ ⋅ = ⋅, ( , ) , 22

L k kg= ⋅0 024, π�

m /s2

L I kg m kg= = ⋅ ⋅ = ⋅ω π π0 016 1 5 0 0242, , ,rad/s m /s2

I MR kg m kg m= = ⋅ = ⋅12

12

0 2 0 4 0 0162 2 2, ( , ) ,

ωπ

π= ⋅ ⋅ =45160

21

1 5min

min,

revs rev

rads

rad

1. Dinámica de traslación y de rotación

mr

Page 21: 9523 gl fis_tx2_cas

24

b) Hallamos primero el momento de inercia del ci-lindro, utilizando la expresión correspondientede la página 42 del libro del alumno. A continua-ción, aplicamos la ecuación fundamental de la di-námica de rotación para calcular la aceleraciónangular:

c) Aplicamos la ecuación del MCUA:

43. Datos: Mp = 1 kg; R = 25 cm = 0,25 m; mc = 2 kg

Se trata de una combinación del movimiento de rota-ción de la polea con el de traslación del cuerpo col-gado.

Planteamos las ecuaciones fundamentales de la diná-mica de traslación y de rotación, y las relaciones entreaceleración angular y tangencial.

a) La tensión de la cuerda valdrá lo mismo sobre elcuerpo que sobre la polea, y ejercerá un momen-to sobre ésta. Como actúa en la periferia y es per-pendicular al radio:

M = R T

Este momento provoca una aceleración angularde la polea. Hallamos el momento de inercia deésta a partir de la fórmula de la página 42 del librodel alumno y aplicamos la ley fundamental de ladinámica de rotación:

Por otro lado, la aceleración lineal del cuerpo col-gado se relaciona con la aceleración angular de lapolea:

b) Aplicamos la ley fundamental de la dinámica detraslación al cuerpo:

Con las cuatro ecuaciones anteriores, tenemos unsistema que resolvemos:

M RT

M IRT I

IR

a T m a

p m gc

c

=

=

⎫⎬⎭

= =

= − =

=

αα; Τ

α

α ;R p

== ⋅ =

− = =+

2 9 8 19 6

2

, ,

;

kg N kg N

pIR

m RpR

m Rc

c

/

αα

11

19 6 0 25

2 0 25 0 0312 2α =

⋅ + ⋅

, ,

( , ) ,

N m

kg m kg m== 31 4

2,

rad

s

F p T m ac= − =

a at= = αR

I MR kg m kg mpolea = = ⋅ = ⋅12

12

1 0 25 0 0312 2 2( , ) ,

MM I= α

ω α= = ⋅ =t / /10 67 180 1 9212, rad s s rad s

I MR kg m kg m

M

= = ⋅ ⋅ = ⋅

=

12

12

5 0 75 1 42 2 2( , ) ,

� ��

IIMI

N mkg m

ra,

,� � � �� �� ��

α α; = =⋅⋅

=15

1 410 67

dd

s2

nuirá su momento de inercia. Como consecuen-cia de la conservación del momento angular, ob-servaremos que la bola gira cada vez con mayorvelocidad.

FÍSICA Y SOCIEDAD

a) Se trata de un movimiento vertical:

El cuerpo llega al suelo cuando y = 0. Si la veloci-dad inicial es cero y t0 = 0:

La velocidad en este instante será, en módulo:

Calculamos la velocidad del impacto:

— Desde la estatua de la Libertad (y0 = 92 m):

— Del Taj Majal (y0 = 95 m):

— Del segundo piso de la torre Eiffel (y0 = 116 m):

b) Respuesta sugerida:

Los excesos de velocidad causan alrededor del30% de los accidentes en carretera y además agra-van las consecuencias de otros percances en losque no son la causa directa del accidente.

Fuente: Dirección General de Tráfico

RESOLUCIÓN DE EJERCICIOS Y PROBLEMAS

42. Datos: M = 5 kg; R = 0,75 m; F = 20 N;

t = 3 min = 180 s

a) Calculamos el momento de la fuerza, sabiendoque actúa en la periferia y es perpendicular al ra-dio:

� ��M F R N m N m= ⋅ = ⋅ = ⋅,20 0 75 15

� �

� �

v y g m m s

vms

= = ⋅ ⋅

= ⋅

,

,

2 2 116 9 8

47 71

02/

kkmm

sh

kmh1 000

3 6001

171 7,⋅ =

� �

� �

v y g m m s

vms

k

= = ⋅ ⋅

= ⋅

,

,

2 2 95 9 8

43 21

02/

mmm

sh

kmh1 000

3 6001

155 5,⋅ =

� �

� �

v y g m m s m s

v

= = ⋅ ⋅ =

=

, ,2 2 92 9 8 42 5

4

02/ /

22 51

1 0003 600

1153,

ms

kmm

sh

kmh

⋅ ⋅ =

� �v gt y g= = 2 0

y y g t tyg

= = − ⇒ =012

20

2 0

y y v t t g t t v v g t t= + − − − = − −0 0 0 02

0 012

( ) ( ) ; ( )

Page 22: 9523 gl fis_tx2_cas

25

b)

El rozamiento, Fr, que se opone al desplazamientode una mesa es ejercido por el suelo sobre ésta. Seaplica en la superficie de contacto entre la mesay el suelo. Su reacción es la fuerza F’r, aplicada so-bre el suelo.

c)

La tensión que ejerce la cuerda sobre el bloque, T,se aplica sobre el bloque, y su reacción, T,, la ejer-ce el bloque sobre la cuerda.

47.

El ciclista no cae porque la componente verticalde la normal equilibra el peso, y la componentehorizontal de la normal se emplea en hacerlo gi-rar. La componente horizontal de la normal coin-cide con la fuerza centrípeta.

48. a) Si la cantidad de movimiento de una masa pun-tual que describe una trayectoria circular se redu-ce a la mitad, se reduce también a la mitad su ve-locidad angular. Por tanto, su nuevo momento an-gular será también la mitad.

b) Si el radio del círculo se triplica manteniendoconstante la velocidad lineal, el momento de iner-cia aumentará, y también lo hará el momento an-gular. Concretamente, podemos relacionar el mo-

c) Aplicamos la ecuación del MRUA para hallar la ve-locidad del cuerpo a los 20 s de dejarlo libre:

EJERCICIOS Y PROBLEMAS

44. El vector desplazamiento es la diferencia entre el vec-tor de posición final y el vector de posición inicial,por tanto es una magnitud vectorial. Su módulo re-presenta la distancia (en línea recta) entre la posicióninicial y la posición final. En cambio, la distancia re-corrida es una magnitud escalar y se mide sobre latrayectoria, desde la posición inicial hasta la posiciónfinal.

— El módulo del vector desplazamiento y la distanciarecorrida sólo coinciden en caso de que la trayec-toria sea una recta y no exista inversión del movi-miento.

45. Si la aceleración es constante en módulo y perpendi-cular a la trayectoria en todo momento, se trata deun movimiento circular uniforme. La aceleración só-lo tiene componente normal, siendo nula la compo-nente tangencial. Además, la componente normal esconstante, por lo que el módulo de la velocidad li-neal es constante, y también es constante la velocidadangular.

46. a) Representamos la normal y su reacción en el casode un cuerpo, como un televisor, apoyado sobreuna mesa.

La normal es la fuerza de contacto que ejerce lamesa sobre el televisor. Es la reacción de otra fuer-za de contacto que ejerce el televisor sobre lamesa, N’. La normal no es la reacción del peso. Elpeso es ejercido por la Tierra sobre el televisor, ysu reacción es ejercida por el televisor sobre laTierra. La reacción del peso se aplica, por tanto,en el centro de la Tierra. La normal, en cambio, seaplica en la superficie de contacto entre el televi-sor y la mesa.

v v at v at m s s m s= + = = ⋅ =027 8 20 157; , / /

= = ⋅ =31 4 0 25 7 82, , ,a rad s m m sαR / / 22

2 20 031 31 40 25

3 9TIR

kg m rad sm

= =⋅ ⋅

=α , ,

,,

/NN

1. Dinámica de traslación y de rotación

�N

�N

� �p m g=

�F

�Fr

�Fr

'

�Fr

'�Fr

�Fr

'

�Fr

'

�Fr

�Fr

�N

�T

� �p m g=

� �p m g=

�T′

�T

�T′

N cos α

N sen α = Fc

α

α

�N

� �p m g=

Page 23: 9523 gl fis_tx2_cas

26

El vector de posición se halla integrando el vector ve-locidad:

51. Datos: y10 = 0 m; v20 = 0 m/s; v10 = 30 m/s; y20 = 20 m

Escribimos primero las ecuaciones de la posición decada piedra:

Las dos piedras se encontrarán cuando y1 = y2.Igualando las dos posiciones, obtenemos el momentoen que se encuentran:

Sustituyendo este valor del tiempo en la ecuación dela posición de la primera piedra obtenemos la altura aque se encuentran:

52 Datos: vy = 810 km/h; vx = 144 km/h

Expresamos las velocidades en unidades del SI:

Escribimos las ecuaciones del movimiento en cada eje:

a) Para obtener el tiempo que tarda el avión en avan-zar 1 km en dirección Norte, imponemos y = 1 km =1 000 m:

b) En la dirección Norte avanza 1 km. Calculamoscuánto avanza en la dirección Este:

La distancia recorrida sobre la Tierra será la com-posición de los dos desplazamientos:

� �Δr x y m m m= + = + =2 2 2 2176 1 000 1 015 4( ) ( ) ,

xms

tms

s m= = ⋅ =40 40 4 4 176,

y mms

t

tm

m ss

= =

= =

1 000 225

1 000225

4 4,/

x x v t xms

t

y y v t yms

t

o x

o y

= + =

= + =

;

;

40

224

vkmh

hs

mkm

ms

v

x

y

= ⋅ ⋅ =

=

1441

3 6001 000

140

810kmh

hs

mkm

ms

⋅ ⋅ =1

3 6001 000

1225

yms

sm

ss m= ⋅ − =30 0 67 4 9 0 67 17 82

2, , ( , ) ,

y y t t t

t

t s

1 22 230 4 9 20 4 9

30 20

2030

0 67

= ⇒ − = −

=

= =

, ,

, s

y y v t gt yms

tm

st

y

1 10 102

1 22

2

12

3012

9 8= + − = −

=

; ,

yy v t gt y mm

st20 20

22 2

212

2012

9 8+ − = −; ,

� � � � �r t r v t dt i t i d

t

t t( ) ( ) ( )= + = + −⌠

⌡⎮ ⌠

⌡⎮0

0

2

0

9 3 8 tt

r t t t i SI� �( ) ( ) ( )= − +3 8 9

mento angular y la velocidad lineal, teniendo encuenta que:

Por tanto, si el radio se triplica, el momento angu-lar se triplica.

49. Datos: en unidades SI

a) Para hallar la velocidad media, primero calculamoslos vectores de posición en los instantes inicial y fi-nal y el vector desplazamiento:

Aplicamos la definición de velocidad media:

b) La velocidad instantánea para cualquier instantede tiempo t se obtiene derivando respecto al tiem-po la ecuación del movimiento:

en unidades SI

c) Para hallar la aceleración media, primero calcula-mos la velocidad en los instantes t = 1 s y t = 2 s:

Aplicamos la definición de aceleración media:

d) La aceleración instantánea para cualquier instantede tiempo t se obtiene derivando respecto al tiem-po la ecuación de la velocidad:

50. Datos:

Obtendremos el vector velocidad integrando la acelera-ción:

� � � � �

v t v a t dt i t i dt

v

t

t t

( ) ( )

(

= + = − +⌠⌡⎮

⌠⌡⎮0

00

8 6

tt it

i SI t i SI) ( ) ( ) ( )= − +⎛

⎝⎜

⎠⎟ = −8

62

3 82

2� � �

� � � � � �a t i v i m s r i m= = − =6 8 90 0, ;/

�� � �

a td vdt

i jm

s( ) ( )= = +2 4 2

�� � � � �

avt

v s v ss s

i jm = =

−=

Δ

( ) ( ) ( )2 12 1

2 4 mm s

a i j m sm

/1 s

/� � �

= +( )2 4 2

� � � � �

�v s i j i j m s

v s

( ) ( ) ( )

(

1 2 1 3 4 1 4

2

= ⋅ − + ⋅ = − + /

)) ( ) ( )= ⋅ − + ⋅ = +2 2 3 4 2 8� � � �i j i j m s/

�� � �

v td rdt

t i t j( ) ( ) ,= = − +2 3 4

�� � � �

vrt

r s r ss s

j msm = =

−=

Δ

Δ

( ) ( ) ( )2 12 1

61

== ( )6�j

ms

� � � � �

�r s i j i j m( ) ( ) ( ) ( )1 1 3 1 2 1 4 2 62 2= − ⋅ + ⋅ + = − +

rr s i j i j m( ) ( ) ( ) ( )2 2 3 2 2 2 4 2 122 2= − ⋅ + ⋅ + = − +� � � �

ΔΔ� � � � � � �r r s r s i j m i= − = − + − − +( ) ( ) ( ) (2 1 2 6 2 12 jj m

r j m

)

( )Δ� �

= 6

� � �r t t t i t j( ) ( ) ( ) ,= − + +2 23 2 4

L I

vr

L Ivr

mrvr

mvr=

=

⎬⎪

⎭⎪= = =

ω

ω2

Page 24: 9523 gl fis_tx2_cas

27

c) Calculamos el ángulo descrito en 10 s utilizando laecuación del MCU:

El número de revoluciones (o vueltas) será:

55. Datos:

Si no actúa ninguna fuerza externa sobre el sistema, elmomento lineal o cantidad de movimiento se conserva.

Inicialmente el sistema está en reposo, de forma que lacantidad de movimiento es nula. Cuando se disparala bala, la cantidad de movimiento total tiene que ser lamisma. Por tanto:

Despejando la velocidad de la escopeta:

56. Datos:

El sistema se moverá hacia la derecha. Escribimos lasegunda ley de Newton para cada cuerpo:

Cuerpo 1:

Cuerpo 2:

Sumamos las dos ecuaciones:

Despejamos la aceleración:

Sustituimos la aceleración en la ecuación del cuerpo 2para obtener la tensión:

m g T m a T m g a

T kg m s2 2 2

24 9 8 5 9

( )

( , ,

− = ⇒ = −

= ⋅ −/ mm s

T N

/ 2

15 6

)

,=

am g m g

m m

akg m s kg

c=−

+

=⋅ − ⋅

2 1

1 224 9 8 0 2 2, ,

μ

/ ⋅⋅

+=

9 82 4

5 92

2

,,

m skg kg

m

s

/

m g m g m m ac2 1 1 2( )− = +μ

p T m a m g T m a2 2 2 2− = − =;

T m g m ac− =μ 1 1

T F m a T N m ar c− = − =1 1; μ

�� �

�v

m vM

kg i m skg

im

eb= − = −

⋅= −

0 01 1502

0 75,

,/

ss

v m se� � = 0 75, /

� � �p m v M vb e− = +0

M kg m g kg v i m sb= = = =2 10 0 01 150; , ; � �/

20 120

3 3π

3 π,rad

vueltarad

vueltas⋅ =

ϕ ωπ

3

π

3= = ⋅ =t

rads

s rad2

1020

c) Hallamos la ecuación de la trayectoria a partir delas ecuaciones paramétricas:

La trayectoria es una recta.

53. Datos: y0 = 20 m ; v = 10 m/s; α = 45o

Ecuaciones del movimiento:

a) La pelota llegará al suelo cuando y = 0:

La solución positiva de esta ecuación es:

b) Hallamos la ecuación de la trayectoria a partir delas ecuaciones paramétricas:

La trayectoria es una parábola.

c) Calculamos, a partir de la ecuación de la trayectoria, laaltura de la pelota cuando llega a la pared (x = 20 m):

La pelota caerá al suelo justo en la base de la paredy no llegará a chocar.

54. Datos: R = 20 cm = 0,2 m; ω = 20 rpm

a) Expresamos la velocidad angular en rad/s:

b) La velocidad de los puntos de la periferia (R = 0,2 m)será:

Para los puntos situados a 5 cm del centro, R = 5 cm= 0,05 m. Por tanto:

v Rrad

sm

ms

= ⋅ = ⋅ =ωπ

3

20 05 0 10, ,

v Rrad

sm

ms

= ⋅ = ⋅ =ωπ

3

20 2 0 42, ,

ωπ π

= = ⋅ ⋅ =20 20160

21

2min

minrpm

revs

radrev 33

rads

y x x

y m

= + − = + −

=

20 0 1 20 20 0 1 20

0

2 2, , ( )

x t

y sen t tt

x= °

= + ° −=

10 45

20 10 45 4 9 102

cos

, coss

cos,

45

20 1045

10 454 9

°

⎧⎨⎪

⎩⎪

= +°°

−yx sen xx

y x x

2

2 2

210 45

20 0 1cos

= + −

t s= 2 9,

y sen t t

t t

= = + ° −

− − =

0 20 10 45 4 9

4 9 7 1 20 0

2

2

,

, ,

x x v t x v tms

t

y y v

x= + = ⋅ = °

= +

0 0 0

0

10 45; cos cosα

0021

2

20 10 4512

9 8

y t gt

y mms

sen tm

s,

= + ° − 222t

x t

y tt

xy

x x=

=

⎧⎨⎩

= = ⋅ =40

225 40225

4045

8; ;

1. Dinámica de traslación y de rotación

N

T

Fr

T

p2

p1 m2 = 4 kg

m1 = 2 kg

Page 25: 9523 gl fis_tx2_cas

28

a) Cuando no existe rozamiento, la fuerza centrípetaes igual a la componente horizontal de la normal.Aplicamos la segunda ley de Newton en cada eje:

Eje X:

Eje Y:

Dividimos las dos ecuaciones entre sí:

Sustituimos la expresión de la fuerza centrípeta ydespejamos la velocidad:

b) Convertimos la velocidad a unidades del SI:

Calculamos el ángulo de peralte a partir de la expre-sión de tg α encontrada en el apartado anterior:

61. a) Hay dos posibles fórmulas para relacionar el al-cance (a) con el ángulo de tiro (α), el módulo dela velocidad inicial (v0) y g:

b) La programación puede ser:

α =1

arc senagv0

22

avg

= 0 2sen , de donde:α

α = ± −⎛

⎝⎜⎞

⎠⎟arc

agv

sen12

12

1 42 0

2

2

a =2vg

sen ·cos ,de donde:0 α α

tgvgR

tgm s

m

sm

;( , )

,α α

α

= =⋅

=

=

2 2

2

22 2

9 8 501

4

/

55°

vkmh

hs

mkm

msmáx = ⋅ ⋅ =80

13 600

1 0001

22 2,

tgm

vR

m gvgR

v gR tgm

sm tg,

α

α

= =

= = ⋅ ⋅

2

2

29 8 50 330 16 8° = ,ms

N senN

Fm g

tgF

m gc c

cos;

αα

α= =

N m g N m gcos ; cosα α− = =0

N sen Fcα =

57. Datos: M = 60 g = 0,06 kg; R = 8 cm = 0,08 m

Calculamos el momento de inercia a partir de la expre-sión que aparece en la página 42 del libro del alumno:

58. Datos: m = 150 g = 0,15 kg; ω = 3 rev/s;

r0 = 20 cm = 0,2 m; r = 5 cm = 0,05 m

Aplicamos la conservación del momento angular:

59. Datos: d = 2 km = 2 000 m; aA = 2 m/s2, v0A = 0;

aB = 0, vB = − 72 km/h

Expresamos la velocidad del segundo automóvil en m/s:

Tomamos como origen de la posición y del tiempo lasalida de A. Entonces, las ecuaciones del movimientode cada automóvil son:

Se encontrarán cuando coincidan sus posiciones.Igualando xA = xB obtenemos el tiempo que tardan enencontrarse desde la partida de A:

Sustituyendo este tiempo en la ecuación de A, hallamosla posición en que se encuentran, medida desde A:

60. Datos:

xm

st

m

smA = = ⋅ =1 1 36 3 1314 82

22

2( , ) ,

x x t t

t t

t s

A B= ⋅ = −

+ − =

=

;

,

1 2 000 20

20 2 000 0

36 3

2

2

A x v t a t

a tm

st

B

A A A

A

: x

x

: x

A

A

B

= + +

= =

0 02

22

2

12

12

1

== +

= + = −

x v t

d v t mms

t

B B

B

0

2 000 20xB

vkmh

hs

mkm

msB = − ⋅ ⋅ = −72

13 600

1 0001

20

L L I II

Imr vueltas s

mr

0 0 00 0

02 3

= = =

=⋅

;

/

ω ω; ωω

ω 22

2

2

0 2 3

0 05

48

=⋅

=

( , )

( , )

m vueltas s

mvuelta

/

ωss

srad

s= 96π

I MR kg m kg= = ⋅ ⋅ = ⋅ ⋅−23

23

0 06 0 08 2 56 102 2 4, ( , ) , mm2

�N

� �p m g=

N cos α

N sen α = Fc

α

α

Page 26: 9523 gl fis_tx2_cas

29

Las ecuaciones del movimiento son:

a) La altura máxima se alcanza cuando la componen-te vertical de la velocidad es nula. Imponiendovy = 0 obtenemos el tiempo en que el proyectil al-canza la altura máxima:

Sustituimos este tiempo en la ecuación de la posi-ción vertical para hallar la altura máxima:

b) Calculamos la posición 3 s después del lanzamien-to sustituyendo t = 3 s en las ecuaciones de cadacomponente de la posición:

c) El momento en que el proyectil llega al suelo seobtiene imponiendo que la coordenada y seacero.

La solución positiva de la ecuación es:

Introduciendo este tiempo en la ecuación de x ha-llamos el alcance:

xms

s

x m

= ⋅ ° ⋅

=

100 40 13 3

1 017 2

cos ,

,

t s= 13 3,

y mms

sen tm

st

t

= = + ⋅ ° −

0 10 100 40 4 9

4 9 64

22

2

,

, ,33 10 0t − =

x sms

s m

y s

( ) cos ,

( )

3 100 40 3 229 8

3 10

= ⋅ ° ⋅ =

= mmms

sen sm

ss

y s

+ ⋅ ° ⋅ −

=

100 40 3 4 9 3

3

22, ( )

( ) 1158 8

3 229 8 158 8

,

( ) ( , , , )

m

r s m�

=

y mms

sen sm

ss= + ⋅ ° ⋅ −10 100 40 6 5 4 9 6 52, , ( , )22

220 9y m= ,

v v sen gt

tv sen

gm s sen

y = − =

= =

0 0

100 40

0

; α

α / 00

9 86 52

°=

,,

m ss

/

v v gt v v sen gt

vm

y y y

y

= − = −

= 100

0 0; α

sssen

m

st⋅ ° − ,40 9 8

2

x x v t x v tms

t

v v

x

x

= + = = ⋅ ° ⋅

+

0 0 0

0

100 40; αcos cos

xx x

y

v vms

y y v t gt

= = ⋅ °

= + −

0

0 0

100 40

12

cos cosα;

220 0

212

10 100 40

; y y v sen t gt

y mms

sen

= + −

= + ⋅ °

α

ttm

st− 4 9

22,

vkmh

hs

mkm

ms0 360

13 600

1 0001

100= ⋅ ⋅ =c) Así, por ejemplo, el resultado para v(0)=6 y a=3

será:

62. Deben obtenerse después de la animación imágenesdel tipo:

EVALUACIÓN

1. Datos:

Integrando la aceleración, se halla la velocidad en fun-ción del tiempo:

Para hallar la ecuación de la posición integramos laecuación de la velocidad que acabamos de obtener:

2. Datos: y0 = 10 m; v0 = 360 km/h; α = 40o

Expresamos la velocidad en m/s:

� � � � �r r v dt i t t i d

t

t t

= + = − + − +⌠⌡⎮ ⌠

⌡⎮0

0

3 2

0

5 4 3 5( ) tt

r i t t t ir t t t i un

� � �

� �= − + − +

= − + −

5 55 5

4 3

4 3( )

( ) ( iidades SI)

� � � � �

�v v a dt i t t i dt

vt

t t

= + = + −⌠⌡⎮ ⌠

⌡⎮0

0

2

0

5 12 6( )

== + −= − −

5 4 34 3 5

3 2

3 2

� �

� �i t t iv t t i unidades

( )( ) ( )SI

� � � � � �a t t i v i m s r i m= − = = −( ) ; ;12 6 5 52

0 0/

1. Dinámica de traslación y de rotación

Page 27: 9523 gl fis_tx2_cas

30

b) Escribimos la segunda ley de Newton para cadacuerpo:

Cuerpo 1:

Cuerpo 2:

Hallamos la tensión de la cuerda a partir de laecuación del cuerpo 2:

Sustituimos la tensión en la ecuación del cuerpo 1y despejamos la velocidad lineal:

La componente tangencial de la aceleración es nulapor ser un MCU. La componente normal es debida ala tensión. Calculamos la componente normal:

7. Un buen ejemplo de conservación angular es un pati-nador dando vueltas, realizando una pirueta.Inicialmente, el patinador extiende los brazos y a ve-ces la pierna, y gira con cierta velocidad angular.Como sobre él no actúa ningún momento, al bajar lapierna y acercar los brazos al eje de rotación, porejemplo, estirándolos hacia arriba, su velocidad angu-lar aumenta. Por eso a menudo vemos a los patinado-res acabar sus piruetas girando a gran velocidad, sinque ello les suponga un esfuerzo adicional.

8. Datos: M = 6 ⋅ 1024 kg; R = 1,5 ⋅ 108 km = 1,5 ⋅ 1011 m

Determinamos el momento de inercia a partir de sudefinición:

9. Datos: R = 0,5 m; I = 1 kg⋅m2; F = 2 N

a) Calculamos el momento de la fuerza y aplicamos laecuación fundamental de la dinámica de traslación:

b) Determinamos el ángulo descrito por el disco apartir de la ecuación del MCUA y de aquí la longi-tud de la cuerda desenrollada:

φ α

φ

= = ⋅ =

= ⋅ =

12

12

1 10 50

0

2 2 2( )t rad s s rad

I R

/

,,5 50 25m rad m⋅ =

� �

� � � � � �� �

� � ��

M F R N m

M IMI

= ⋅ = ⋅

= = =

1

1;α α

NN m

kg m

rad

s

⋅=

112 2

I MR kg m

I kg

= = ⋅ ⋅ ⋅

= ⋅

2 24 11 2

47

6 10 1 5 10

1 35 10

( , )

, ⋅⋅ m2

avR

m sm

m

sn = = =2 2

2

4 40 2

96 8( , )

,,

/

m g mvR

vmm

Rg

vkgkg

m

2 1

22

1

1 50 15

0 2

;

,,

,

= =

= ⋅ ⋅ 99 8 4 42, ,m sms

/ =

T m g= 2

T m g− =2 0

T F T mvRc= =; 1

2

3. Datos: ω0 = 60 rev/min; α = − 2 rad/s2

La velocidad angular en rad/s es:

La ecuación de la velocidad angular para MCUA es:

Imponemos ω = 0 para hallar el tiempo que tarda eldisco en parar:

4. Las fuerzas de acción y reacción aparecen siempre porparejas. Si un cuerpo 1 ejerce una fuerza sobre uncuerpo 2 (acción), este cuerpo 2 ejercerá otra fuerzasobre el cuerpo 1 (reacción). Las fuerzas de acción yreacción tienen el mismo módulo y dirección, y senti-dos opuestos.

5. Datos:

Aplicamos el teorema de la conservación de la canti-dad de movimiento. Inicialmente, el patinador y el dis-co están en reposo. Por lo tanto, la cantidad de movi-miento total es cero.

Entonces:

El módulo de la velocidad del patinador es:

6. Datos: m1 = 150 g = 0,15 kg; R = 20 cm = 0,2 m;m2 = 1,5 kg

a)

v m sp = − ⋅4 8 102, /

� � �

��

p M v m v

vm v

Mkgkg

p d

pd

= + =

= − = −

0 0

0 375

12

;

, ��

� � �

ims

v i m s i m sp = − = − ⋅ −0 048 4 8 10 2, ,/ /

M kg m g kg v i m sd= = = =75 300 0 3 12; , ;�

/

ω ω α

ω

α

π0

0

= = +

= − = −−

=

0

2

23 14

2

t

trad s

rad ss,

/

/

ω ω α0= + t

ωπ

π0 = ⋅ ⋅ =60160

21

2min

minrevs

radrev

rads

m1

m2

�N

�T

�p1

�T

�p2

� �� �p m g

p m g1 1

2 2

=

=

m1 = 150 g m2 = 1,5 kg

Page 28: 9523 gl fis_tx2_cas

31

Campo gravitatorio2

Calculamos la energía cinética:

• Datos: M = 10 kg; h = 40 m

Determinamos la energía potencial gravitatoria:

• Datos: Dx = 10 m; F = 40 N; = 30°

Como la fuerza es constante, podemos determinar eltrabajo como el producto escalar de la fuerza por el vec-tor desplazamiento:

• Datos: v0 = 24,5 km/h; W = 1 738 J; M = 100 kg

Si suponemos despreciable el rozamiento, todo el traba-jo efectuado por el ciclista será empleado en incremen-tar su energía cinética.

Determinamos la energía cinética inicial y le sumamosel trabajo para determinar la energía cinética final y ha-llar la velocidad:

1. LA TIERRA EN EL UNIVERSO

1. Según la escuela aristotélica, los cuerpos celestes gira-ban alrededor de la Tierra, que ocupaba el centro deluniverso. Los cuerpos celestes describían movimientoscirculares uniformes, la forma de movimiento consi-derada perfecta en la Antigüedad.

vkmh

hs

mkm

ms

Ec

0

0

24 51

3 6001 000

16 8= ⋅ ⋅ =

=

, ,

112

12

100 6 8 2 315 802 2( , / ) ,M v kg m s J

W Ec

= ⋅ ⋅ =

= 00 0

1738 2 315 8 4 053 8

− = +

= + =

Ec Ec W Ec

Ec J J J

Ec

;

, ,

== = =⋅

=

=

12

2 2 4 053 8100

9

9

2;,

M v vECM

Jkg

ms

vmss

ms

kmm

sh

kmh

= ⋅ ⋅ =91

1 0003 600

132 4,

W F r F x N m

W

= = = ⋅ ⋅ °

=

� �cos cos

,

Δ Δ φ 40 10 30

346 44 J

Ep M g h kg N kg m J= = ⋅ ⋅ =, /10 9 8 40 3 920

Ec M v kgms

Ec

= = ⋅ ⋅⎛

⎝⎜

⎠⎟

= ⋅

12

12

850 22 2

2 1 1

22

,

, 00 2105 J kJ=

PREPARACIÓN DE LA UNIDAD

• Datos:

Hallamos primero las componentes de cada uno de losvectores para realizar después la suma vectorial:

• El vector que tiene la misma dirección pero sentido con-trario a tendrá las mismas componentes pe-ro con signo contrario:

Para obtener un vector unitario, calculamos el módulode , y dividimos sus componentes por el módulo:

• Datos:

Calculamos el producto escalar:

• Datos: M = 850 kg; v = 80 km/h

Pasamos primero la velocidad a unidades del SI:

vkmh s

mkm

ms

= ⋅ ⋅ =801

3 6001 000

122 2,

� � � �v v v v1 2 1 2 10 20 45⋅ = ⋅ ⋅ = ⋅ ⋅ °cos cos� � � � φ == 141 4,

� � � �� �v v1 210 20 45= = = °; ;

� �

� �

���

� �v

uvv

i j

′ = − + − =

=′′

= − − =

( ) ( )3 4 5

35

45

2 2

−− −0 6 0 8, ,� �i j

�v′

� � �v i j′ = − −3 4

� � �v i j= +3 4

� � � � � �

�v v i sen j i j1 1 4 3 2 5= + = +(cos ) , ,� � φ φ1 1

vv v i sen j i j

v

2 2 2 2

3

5 8 7= + = +(cos ) ,� �� � � � �

�φ φ

== + = − +

+

(cos ) ,� �� � � � �

�v i sen j i j

v

3 3 3

1

6 9 4φ φ�� � � � � � �v v i j i j i2 3 4 3 2 5 5 8 7 6 9+ = + + + + −( , , ) ( , ) ( , ++

+ + = + − + + +

4

4 3 5 6 9 2 5 8 7 41 2 3

� � � �j

v v v i

)

( , , ) ( , , )��

� � � � �j

v v v i j1 2 3 2 4 15 2+ + = +, ,

2. Campo gravitatorio

Y

X

φ3 = 150o

φ1 = 30o

φ2 = 60o

� � �v v v1 2 3+ +

� ��v2 10=

� ��v1 5=� ��v3 8=

Page 29: 9523 gl fis_tx2_cas

32

muy grande, la fuerza de atracción es inapreciable y,por tanto, predominan fácilmente otras fuerzas porencima de la gravitatoria.

8. Datos: m = 250 g = 0,25 kg; r = 10 cm = 0,1 m

Aplicamos la ley de la gravitación universal:

9. Datos: F = 10−10 N; r = 0,5 m

Despejamos la masa de la ley de la gravitación univer-sal, teniendo en cuenta que los dos objetos son igua-les:

10. Datos: m = 2 kg; F = 10−7 N

Despejamos la distancia de la ley de la gravitación uni-versal:

11. Datos:

Determinamos en primer lugar el módulo y la dire-ción del vector que une las dos masas:

El vector unitario en la dirección de la recta que unela masa 1 con la masa 2 y sentido de m1 a m2 será:

El vector unitario con sentido de m2 a m1 será opues-to a :

a) Calculamos la fuerza con que m1 atrae a m2:

b) Calculamos la fuerza con que m2 atrae a m1:�F21

� �

F Gm m

ru

FN m

kg

121 2

2 1

1211

2

26 67 10

= −

= − ⋅⋅−,

33 1 5

8 60 8 0 6

0 3

2

12

,

( , )( , , )

( ,

kg kg

mi j

F

⋅−

= −

�33 0 23 10 11� �

i j N+ ⋅ −, )

�F12

u u i j2 1 0 8 0 6= − = − −� � �

, ,

�u1

�� � �

� �u

rr

i j mm

i j112 7 5

8 60 8 0 6= =

−= −

( ),

, ,

� � � � �

�r r r i j m

r12 2 1

12

5 2 1 4

7

= − = − − + − −

=

[( ( )) ( ) ]

(�� �

�i j m

r r m

= = + − =

5

7 5 8 6122 2

)

( ) ,� �

m kg r m m kg

r1 1 2

2

3 2 4 1 5

5

= = − =

= −

; ( , ) ; , ;

( ,

��

11) m

F Gm

rr

G mF

F

N m

kgkg

= =

=

⋅⋅−

;

, ( )

2

2

2

112

226 67 10 2

10−−−= ⋅

725 2 10

Nm,

F Gm

rm

F rG

N m

N= = =

⋅⋅

;( , )

,

2

2

2 10 2

11

10 0 5

6 67 10mm

kg

kg2

2

0 6= ,

F Gm

r

N m

kg

kg= = ⋅

⋅⋅−,( , )

( ,

2

211

2

2

2

6 67 100 25

0 1 mm

F N

)

,

2

104 2 10= ⋅ −

2. El modelo de universo de Ptolomeo se diferencia del deCopérnico en que el primero es geocéntrico. Suponeque todos los cuerpos celestes giran alrededor de laTierra; en cambio, el segundo sitúa el Sol en el centro, yla Tierra, junto con los otros planetas, en órbita circularen torno a él. Actualmente se acepta el modelo heliocen-trista o copernicano, si bien se han sustituido las órbitascirculares de Copérnico por órbitas elípticas.

3. Identificamos las etapas del método científico en laevolución de los modelos de universo:

— Observación de la realidad: observación del firma-mento.

— Formulación de hipótesis: aparecen en este caso dis-tintas hipótesis:

• Modelo de universo geocéntrico de Aristóteles yde Ptolomeo.

• Modelo heliocéntrico de Copérnico, propuestoanteriormente por Aristarco de Samos.

— Experimentación: en el caso de la astronomía, debe-ríamos hablar más bien de observaciones rigurosasy toma de datos:

• Primeras observaciones con telescopio de Gali-leo.

• Catálogo de Tycho Brahe y observaciones deJ. Kepler.

— Organización de los datos experimentales: elaboraciónde un catálogo estelar por Tycho Brahe y estudios deJ. Kepler sobre las observaciones anteriores.

— Extracción de conclusiones y formulación de leyes: lastres leyes del movimiento de los planetas de Kepler.

— Elaboración de una teoría: teoría de la gravitación deIsaac Newton.

4. La física determinista dice que si se conocen todos losfactores que pueden influir en un sistema físico, sepuede determinar exactamente su comportamiento.

5. Ningún fenómeno puede superar la velocidad de laluz. Por tanto, tenemos un límite insuperable en el co-nocimiento de los factores que pueden influir en unsistema. En consecuencia, no se puede predecir conexactitud el futuro de un sistema físico.

6. En 1964, los físicos A. Penzias y R. Wilson desarrollabanuna antena muy sensible para los laboratorios Bell.Esto les permitió descubrir un misterioso ruido de fon-do que no podían suprimir y cuyo origen desconocían.Acababan de detectar por primera vez la radiación defondo de microondas, que es la prueba de que el ori-gen de nuestro universo es el big bang.

2. FUERZAS GRAVITATORIAS

7. Para determinar las unidades de G, despejamos la cons-tante de la ley de la gravitación universal:

El valor de G es 6,67 ⋅ 10−11 N ⋅ m2 ⋅ kg−2. Este valor estan pequeño que, a menos que alguna de las masas sea

F Gm m

rG

F rm m

GF L

M

N= =

⋅=

⋅; ; [ ]

[ ] [ ]

[ ]1 2

2

2

1 2

2

2

mm

kg

2

2

Page 30: 9523 gl fis_tx2_cas

33

17. Datos:

a) Calculamos el campo gravitatorio debido a cadamasa. Como las dos masas son iguales y están a lamisma distancia del punto donde calculamos elcampo, los módulos de los campos debidos a cadamasa serán iguales:

De donde

Sumamos vectorialmente los dos campos, tenien-do en cuenta la elección de los ejes:

El módulo del campo es:

b) Hallamos la fuerza sobre la masa: m = 10 g = 0,01 kg.

18. Al acercar dos masas, su energía potencial gravitatoriadisminuye. El desplazamiento tiene lugar en el mismosentido en que actúa la fuerza gravitatoria. El trabajorealizado por el campo es, entonces, positivo.

— Cuando alejamos dos masas, en cambio, la energíapotencial aumenta. Estamos realizando un trabajocontra la fuerza gravitatoria. Por tanto, el trabajo rea-lizado por el campo es negativo.

19. Si la única fuerza que actúa es la gravitatoria, la masa semoverá hacia potenciales menores. Esto corresponde aun trabajo positivo y una disminución de la energía po-tencial.

— La masa perderá energía potencial gravitatoria.

20. Datos: EpA = 100 J; EpB = −500 J

Relacionamos el trabajo con la variación de energíapotencial:

F m g kg N kg N= = ⋅ ⋅ = ⋅− −, , ,0 01 2 1 10 2 1 1011 13/

g N kg N kg= + ⋅ = ⋅− −( ) ( , ) ,15 1 5 10 2 1 102 2 11 11/ /

� � � �

� � � �u i u j

g g g i1 2

1 2111 5 10 1 5

= − = −

= + = ⋅ +−

;

( , , ⋅⋅ −10 11 )�j N kg/

� �g u N kg211

21 5 10= − ⋅ −, /

� �g u N kg111

11 5 10= − ⋅ −, ;/

� �

� �

g GM

rG

M

b

gN m

kg

22

22 2

211

2

26 67 102

= =

= ⋅⋅−,

( ),

kg

m

Nkg3

1 5 10211= ⋅ −

� �

� �

g GM

rG

M

a

gN m

kg

11

12 2

111

2

26 67 102

= =

= ⋅⋅−,

( ),

kg

m

Nkg3

1 5 10211= ⋅ −

c) El módulo de las dos fuerzas será igual debido aque son fuerzas de acción y reacción:

3. CONCEPTO DE CAMPO

12. Decimos que existe un campo en cierta región cuandoen ella hay una perturbación, real o ficticia, caracteriza-da por el valor de una magnitud en cada punto. Unejemplo de campo escalar es la densidad de un conta-minante vertido en un lago. Un campo vectorial es elcampo de velocidades de las partículas de un fluido enmovimiento.

13. En un campo uniforme, la magnitud característica delcampo (puede ser la fuerza) es constante en todos lospuntos del espacio. En cambio, en un campo central, lamagnitud característica depende únicamente de la dis-tancia al centro del campo y todos los vectores fuerza con-vergen en un punto llamado centro del campo. Ejemplos:

Campo uniforme: campo eléctrico entre las dos placasde un condensador plano.

Campo central: campo gravitatorio de un objeto puntual.

14. Los campos conservativos se caracterizan por que eltrabajo que realizan las fuerzas del campo no dependedel camino seguido, sino sólo del punto inicial y final.Como consecuencia, el trabajo realizado en una trayec-toria cerrada es nulo.

— A los campos conservativos se asocia una magnitudllamada energía potencial. El trabajo realizado porlas fuerzas conservativas es igual al incremento deenergía potencial.

— Ejemplos de campos conservativos: campo gravita-torio, campo eléctrico.

4. ESTUDIO DEL CAMPO GRAVITATORIO

15. La intensidad del campo gravitatorio disminuye a me-dida que nos alejamos de la masa que lo crea. En con-creto, de la ley de la gravitación universal se deriva queel campo creado por una masa puntual disminuye con elcuadrado de la distancia.

16. Datos: M = 3 kg; r = 5 m

Calculamos la intensidad del campo gravitatorio:

g GM

r

N m

kg

kg

m

N= = ⋅

⋅= ⋅− −

211

2

2 2126 67 10

3

58 10,

( ) kkg

� � � �

� � � �

� �

� �F F

F F

12 21

12 2120 33 0

=

= = − +( , ) ( ,, )23 10

4 10

2 11

12 2112

= = ⋅

N

F F N� � � �� �

� �

F Gm m

ru

FN m

kg

211 2

2 2

2111

2

26 67 10

= −

= − ⋅⋅

⋅−,11 5 3

8 60 8 0 6

0

2

21

,

( , )( , , )

( ,

kg kg

mi j

F

⋅− −

= −

�333 0 23 10 11� �

i j N− ⋅ −, )

2. Campo gravitatorio

M2 = 2 kg

M1 = 2 kga = 3 m

b = 3 m

Y

X

�g2

�g

�u1�u2

�g1

Page 31: 9523 gl fis_tx2_cas

34

punto hasta el segundo como la variación de ener-gía potencial:

24. Las líneas de campo y las superficies equipotencialesrepresentan de forma gráfica el campo gravitatorio.Las líneas de campo nos indican la dirección, el senti-do y la intensidad relativa del campo en cada punto.Por otra parte, las superficies equipotenciales nosmuestran las regiones del espacio con el mismo poten-cial. Si una masa se mueve manteniéndose por la mis-ma superficie, mantendrá constante su energía poten-cial. Por tanto, el campo gravitatorio no realiza traba-jo sobre ella.

25. Cerca de las masas dibujaremos más juntas las líneasde campo, ya que en esta zona es más intenso el cam-po gravitatorio.

26.

27.

Superficies equipotenciales

W Ep Ep m V m V

W m V V kg

= − = − = −

= − = ⋅

ΔΕp 1 2 1 2

1 2 2 5( ) , (−− − − ⋅

= − ⋅

8 3 3 10

1 2 10

3

2

( , ))

,

J kg

W J

/

La respuesta correcta es la b.

21. Para que en un punto del espacio exista un potencialgravitatorio diferente de cero no es necesario que exis-ta una masa en ese punto. Habrá un potencial distintode cero si en una zona cercana al punto hay alguna ma-sa, siempre que no tomemos ese preciso lugar comoorigen de la energía potencial.

22. Datos: M1 = 3,6 ⋅ 109 kg; M2 = 9,8 ⋅ 109 kg;

OP1 = (−3, −4) m; OP2 = (8, −4) m

a) Determinamos el potencial gravitatorio creado porM1 en P (−1, 5) m:

Determinamos el potencial gravitatorio debido aM2 en P:

El potencial total será la suma algebraica de V1 y V2:

b) Determinamos la energía potencial de una masade 140 g situada en el punto P:

23. Datos: M = 1,2 ⋅ 1012 kg; r1 = 10 km = 1 ⋅ 104 m;

r2 = 24 km = 2,4 ⋅ 104 m

a) Hallamos el potencial gravitatorio creado por M1

en cada punto:

b) Determinamos el trabajo realizado por el campopara llevar una masa m = 2,5 kg desde el primer

V GMr

N m

kg

kg1

1

1

112

2

12

6 67 101 2 10

1 1= − = − ⋅

⋅⋅

⋅−,

,

00

8 10

6 67 10

4

13

2 211,

m

V J kg

V GM

r

N m

= − ⋅

= − = − ⋅⋅

/22

2

12

4

23

1 2 10

2 4 10

3 3 10

kg

kg

m

V J kg

⋅⋅

= − ⋅ −

,

,

, /

Ep m V kg J kg J= = ⋅ − ⋅ = − ⋅− −, ( , ) ,0 14 7 7 10 1 1 102 2/

V V V J kg J kg

V

= + = − ⋅ − ⋅

= − ⋅

− −1 2

2 22 6 10 5 1 10

7 7

, ,

,

/ /

110 2− J kg/

� � ��� � ���r OP OP m m2 2 1 5 8 4 9 9= − = − − − = −( , ) ( , ) ( , ))

,

,

m

r m

V GMr

N m2

2 2

22

2

11

9 9 12 7

6 67 10

= + =

= − = − ⋅⋅−

22

2

9

22

9 8 1012 7

5 1 10

kg

kgm

V J kg

⋅⋅

= − ⋅ −

,,

, /

V GMr

N m

kg

kg1

1

1

112

2

9

6 67 103 6 10

9 2= − = − ⋅

⋅⋅

⋅−,,

, mm

V J kg122 6 10= − ⋅ −, /

OP m

r OP OP m

� ���

� � ��� � ���= −

= − = − −

( , )

( , )

1 5

1 51 1 (( , ) ( , )

,

− − =

= + =

3 4 2 9

2 9 9 212 2

m m

r m

W Ep Ep J J JB A= − = − − = − − − =ΔΕp ( ) ( )500 100 600

b) M = 40 kgLíneas de campo

Superficies equipotenciales

Campo de fuerzas uniforme

Líneas de campo

Superficiesequipotenciales

a) M = 10 kg

Líneas de campo

Superficies equipotenciales

Page 32: 9523 gl fis_tx2_cas

35

32. Datos: M = 3 000 kg; R = 3 m; r = 10

Escogemos como superficie gaussiana S una esferaconcéntrica a la esfera y de radio r = 10 m.

Calculamos el flujo a través de la superficie, teniendoen cuenta que será constante en toda la superficie, y

que y son paralelos y de sentido opuesto en ca-da punto de la superficie:

Aplicamos el teorema de Gauss y despejamos el cam-po gravitatorio:

Calculamos el potencial a partir del campo:

En este caso, como nos interesamos en un punto situa-do en el exterior de la distribución de masa, el campoy el potencial son los mismos que crearía una masapuntual situada en el centro de la esfera.

FÍSICA Y SOCIEDAD

a) Respuesta sugerida

Avión

Inventor: El alemán Otto Lilienthal creó el primer pla-neador en 1891 y efectuó con él más de 2 000 vuelos.Sin embargo, el primer vuelo controlado con motor lollevaron a cabo los hermanos Wright en 1903, en unaeroplano (el Flyer) que ellos mismos diseñaron yconstruyeron.

V g dr GM

ru dr G

M

rd

r r r= ⋅ = − ⋅ = −⌠

⌡⎮ ⌠

⌡⎮ ⌠

⌡⎮

∞ ∞ ∞� � � � �2 2 rr

V GMr

N m

kg

kgm

V

= − = − ⋅⋅

= −

−6 67 103 000

10

2

112

2,

⋅⋅ −10 8 J kg/

Φ π π π= − = − = =

= ⋅ −

4 4

6 67 10

2

11

GM; g 4 r GM;2 g GM

r

gN

,⋅⋅

⋅ = ⋅ −m

kg

kg

m

Nkg

2

2 293

102 10

000

( )

Φ = ⋅ = − ° = − = −⌠⌡⎮ ⌠

⌡⎮ ⌠

⌡⎮

S S Sg dS g dS g dS g� �

cos 180 4 πr2

dS��

g

�g

28. El flujo de un campo uniforme a través de una superfi-cie cerrada es nulo. Los vectores intensidad de un cam-po uniforme son paralelos y del mismo sentido en to-dos los puntos del espacio. Entonces, el número de lí-neas de campo que entran en una superficie cerrada esigual al número de líneas de campo que salen de ella,y el flujo total es cero.

29. El flujo gravitatorio a través de una superficie esféricaque encierra masa es siempre negativo. El vectorsuperficie apunta, por convenio, hacia el exteriorde la superficie cerrada. En cambio, el vector inten-sidad de campo gravitatorio apunta hacia la masa quelo crea, es decir, hacia el interior de S. Como el flujoa través de una superficie dS es el producto escalar de

por

el flujo es siempre negativo.

30. El teorema de Gauss es útil para calcular la intensidaddel campo gravitatorio en distribuciones de masa conuna geometría sencilla. En los casos donde el módulode es constante para toda la superficie S, y su di-rección es perpendicular a dicha superficie en cadapunto:

31. Datos: M = 4,5 ⋅ 108 kg; R = 150

Aplicamos el teorema de Gauss para hallar el flujo, te-niendo en cuenta que la distancia a la masa central esconstante en toda la superficie esférica:

Φ π

Φ π

Φ

= −

= − ⋅ ⋅⋅

⋅ ⋅−

4

4 6 67 10 4 5 10112

28

GM

, ,N m

kgkg

== − ⋅0 38 2, N m kg/

Φ = ⋅ = − = − = −⌠⌡⎮ ⌠

⌡⎮ ⌠

⌡⎮

S S Sg dS g dS g dS g S� �

�g

� �g dS g dS g dS⋅ = ° = −cos ,180

dS�,�

g

dS�

2. Campo gravitatorio

M = 3 000 kg

r = 10 mR = 3 m

dS�

�g

R = 150 km

M �g

dS�

Page 33: 9523 gl fis_tx2_cas

36

Juan de la Cierva solucionó el problema de la estabi-lidad y construyó el autogiro, un vehículo controla-ble.

Modelos: VS 300. 1940. Desarrollado por I. Sikorskij,fue el primer helicóptero capaz de transportar unacarga útil con total maniobrabilidad.

Sea King. Helicóptero británico utilizado actualmenteen misiones de rescate.

MI-26. Helicóptero ruso actual.

Sikorsky S-64. Helicóptero pesado norteamericano ca-paz de transportar grandes pesos, como camiones.

Boeing CH-47. Helicóptero pesado norteamericano.

Usos industriales o sociales: La gran maniobrabilidad delhelicóptero lo hace especialmente útil en actividadesque el avión no puede desempeñar, como vuelos ra-santes sobre el mar o misiones de rescate.

Globo aerostático

Inventor: El jesuita italiano Francisco de Lana propu-so en 1670 una nave que se sustentara mediante esfe-ras en las que se había aspirado el aire. El sacerdotebrasileño Bartolomeu de Gusmão demostró, en pre-sencia del rey Juan V de Portugal, el primer modelode globo de aire caliente. En 1783 se eleva en París elprimer globo con pasajeros. Fue construido por loshermanos Montgolfier y usado por Pilâstre de Roziery el marqués de Arlandes.

Usos industriales o sociales: El globo fue el primer siste-ma utilizado para elevarse en el aire. Hoy, debido a lasmayores prestaciones del avión, ha quedado en desu-so y se utiliza casi exclusivamente en actividades de-portivas, de recreo o aventura.

Modelos

Double Eagle II. Con este globo se llevó a cabo conéxito la primera travesía del Atlántico Norte en1978.

Breitling Orbiter 3. Permitió a B. Piccard y B. Jonescompletar la primera vuelta al mundo en globo en1999

b) Se recomienda que previamente a la organización delcoloquio, los alumnos estudien los contenidos desa-rrollados en esta página y completen su informacióncon una labor de investigación bibliográfica.

A continuación se elegirán los encargados de las di-ferentes funciones: moderador, participantes y pú-blico.

El moderador presentará a los participantes, introdu-cirá el tema citando los principales medios de trans-porte aéreo en la sociedad y las principales aplicacio-nes de cada uno, y formulará la primera pregunta aalguno de los participantes. Al final del coloquio, elpúblico podrá exponer sus opiniones acerca de la im-portancia de los medios de transporte aéreo en la so-ciedad y formular preguntas a los diferentes partici-pantes.

Modelos: Algunos de los modelos más representativosen la historia de la aviación han sido los siguientes:

Boeing B-247. 1933. Fue el primer avión de transportetotalmente metálico.

Boeing B-747 (Jumbo). 1970. Avión de pasajeros de grancapacidad (490 pasajeros) ampliamente utilizado.

Douglas DC-10. 1971. Avión de pasajeros de gran capa-cidad (375 personas).

Concorde. 1976. Primer avión de pasajeros supersónico.

Usos industriales o sociales: Actualmente el avión es el me-dio de transporte de pasajeros y mercancías más rápidoy más utilizado en largos recorridos. También se em-plea habitualmente en labores humanitarias, como ladistribución de medicinas y alimentos.

Cohete

Inventor: Las bases teóricas del vuelo de los cohetes fue-ron establecidas por el ruso K. E. Tsiolkovski en 1903.

Modelos

V2. 1942. Misil balístico alemán considerado el primercohete moderno. Se usó durante la Segunda GuerraMundial para bombardear la ciudad de Londres (1944).

R-7. 1957. Misil intercontinental soviético que, modifica-do, permitió el lanzamiento del primer satélite artificialde la historia, el Sputnik I (1957), así como el vuelo delprimer cosmonauta, el ruso Yuri Gagarin (1961).

Atlas D. 1962. Lanzador que permitió la puesta en órbitadel primer astronauta estadounidense, John Glenn.

Saturno V. 1967. Lanzador estadounidense que fue la ba-se de las misiones Apollo (conquista de la Luna por el serhumano; Armstrong y Collins, 1969) y la estación Skylab.

Arianne I. 1979. Primer lanzador de la serie europeaArianne.

Space Shuttle. 1981. Flota de transbordadores espacialesque ha permitido, entre otros hechos, el ensamblaje dela estación espacial internacional ISS (InternacionalSpace Station), en colaboración con Europa, Rusia,Japón y Canadá.

Larga Marcha II-F. 1992. Lanzador que ha permitido aChina la realización de misiones espaciales tripuladas apartir de 2003.

Arianne V. 1997. Principal lanzador de la AgenciaEspacial Europea (ESA) base de todas sus operaciones es-paciales de lanzamientos de satélites y apoyo a la estaciónespacial ISS.

Usos industriales o sociales: Permite propulsar vehículosaéreos espaciales; tripulados o no. Gracias a los cohe-tes, el ser humano ha conseguido viajar más allá de suplaneta natural, la Tierra.

Helicóptero

Inventor: Leonardo da Vinci diseñó el primer vehícu-lo de hélices giratorias movidas por un mecanismo derelojería. El primer helicóptero fue construido en1784 por los franceses Launoy y Bienvenu. El español

Page 34: 9523 gl fis_tx2_cas

37

El campo gravitatorio resultante es la suma vecto-rial de los cuatro:

Su módulo es

b) Calculamos la fuerza que actuaría sobre la masade 100 kg:

c) Calculamos el potencial gravitatorio debido a cadauna de las cuatro masas:

El potencial gravitatorio resultante será la suma delos cuatro:

d) Calculamos la energía potencial gravitatoria queadquiriría una masa de 100 kg sometida a este po-tencial:

34. Datos: R = 0,5 m; M = 1 000 kg

a) r = 1,5 m

Este punto es exterior a la corteza. Por tanto, apli-camos las expresiones del campo y el potencialgravitatorios creados por una corteza esférica enun punto exterior.

Ep m V kg J kg

Ep

= = ⋅ − ⋅

= − ⋅

( , )

,

100 8 1 10

8 1 10

9

7

/

JJ

V V V V V J kg= + + + = − ⋅ −1 2 3 4

98 1 10, /

V GMr

VN m

kg

kgm

33

311

2

26 67 102 00049 5

= −

= − ⋅⋅

=−,,

−− ⋅

= −

= − ⋅⋅

2 7 10

6 67 102

9

44

411

2

2

,

,

Jkg

V GMr

VN m

kg

000049 5

2 7 10 9

,,

kgm

Jkg

= − ⋅ −

V GMr

VN m

kg

kgm

11

111

2

26 67 101 00049 5

= −

= − ⋅⋅

=−,,

−− ⋅

= −

= − ⋅⋅

1 3 10

6 67 101

9

22

211

2

2

,

,

Jkg

V GMr

VN m

kg

000049 5

1 3 10 9

,,

kgm

Jkg

= − ⋅ −

� � �

�F m g kg j N kg

F

= = ⋅ − ⋅

= − ⋅

−( , )

,

100 3 8 10

3 8

11 /

110

3 8 10

9

9

−= ⋅,

�j N

F N

g N kg= − ⋅ −3 8 10 11, /

� � � � �

� � � �g g g g g

g i j i

= + + +

= ⋅ − + + +−

1 2 3 4111 9 10, ( ) (

��

� � � �j

i j i j N

)

, ( ) ( )

⎡⎣ ⎤⎦ +

+ ⋅ − − + −⎡⎣ ⎤⎦−3 8 10 11 //

/

kg

g j N kg� �

= − ⋅ −3 8 10 11,

RESOLUCIÓN DE EJERCICIOS Y PROBLEMAS

33. Datos:

Hallamos primero la distancia de cada masa al punto P,que será la misma para las cuatro masas:

Determinamos los vectores unitarios de las direccionesde cada masa:

a) Determinamos el campo creado en P por cada unade las cuatro masas:

�g

N m

kg

kg

m3

112

2 26 67 10

2 000

49 5

1

2= − ⋅

⋅⋅ ⋅−,( , )

(�� �

� � �

i j

g i j N kg

g GM

r

+

= ⋅ − −

= −

)

, ( )311

44

2

3 8 10 /

��

u

gN m

kg

kg

m

4

411

2

2 26 67 10

2 000

49 5= − ⋅

⋅⋅−,( , )

⋅⋅ − +

= ⋅ −−

1

2

3 8 10411

( )

, ( )

� �

� � �

i j

g i j N kg/

�g

N m

kg

kg

m1

112

2 26 67 10

1 000

49 5

1

2= − ⋅

⋅⋅ ⋅−,( , )

(�� �

� � �

i j

g i j N kg

g GM

r

= ⋅ − +

= −

)

, ( )111

22

2

1 9 10 /

��

u

gN m

kg

kg

m

2

211

2

2 26 67 10

1 000

49 5= − ⋅

⋅⋅−,( , )

⋅⋅ −

= ⋅ +

= −

1

2

1 9 10211

3

( )

, ( )

� �

� � �

i j

g i j N kg

g GM

/

332 3

ru�

� �g GM

ru1

12 1= −

� � � � � �

� � � �

u i j u i j

u i j u

1 3

2 4

1

2

1

21

2

= − = +

= − −

( ) ( )

( ) == − +1

2( )

� �i j

rm

m=⎛⎝⎜

⎞⎠⎟

+⎛⎝⎜

⎞⎠⎟

= = = =12

12

12

1

2

70

249 5

2 2 2

,

2. Campo gravitatorio

M1

P

M3

M2

l = 70 ml = 70 m

l = 70 m

l = 70 mM4

�g1

�g2

�u3�u4

�u2�u1

�g3

�g �

g4

Page 35: 9523 gl fis_tx2_cas

38

Por tanto, el vector campo gravitatorio es, para cadalado de la lámina,

por encima de la lámina:

por debajo de la lámina:

Calculamos el potencial a partir del campo gravitatorio:

Si escogemos como origen de potencial la lámina pla-na, obtenemos:

EJERCICIOS Y PROBLEMAS

36. Modelo de Ptolomeo (geocéntrico): este modelo considera-ba la Tierra en el centro del universo, con la Luna, elSol y los planetas describiendo órbitas circulares o epi-ciclos alrededor de puntos que, a su vez, orbitaban al-rededor de la Tierra. Este modelo explicaba los com-plejos movimientos de los planetas, algo que no habíaconseguido Aristóteles.

Modelo de Copérnico (heliocéntrico): según Copérnico,el Sol se situaba en el centro del sistema, y la Tierra,con los otros planetas, giraba alrededor del Sol.La Luna era el único objeto en órbita alrededor dela Tierra, mientras que esta última ya no era el cen-tro del universo, sino que también estaba en movi-miento.

37. Leyes de Kepler:

1. Todos los planetas describen órbitas elípticas con elSol situado en uno de sus focos.

2. La recta que une el Sol con un planeta barre áreasiguales en tiempos iguales.

3. El cuadrado del período de la órbita de un planetaes directamente proporcional al cubo de la distan-cia media del Sol al planeta,

T2 = C R3

38. Ley de la gravitación universal: dos partículas materiales seatraen mutuamente con una fuerza directamente pro-porcional al producto de sus masas e inversamente pro-porcional al cuadrado de la distancia que las separa.

Características principales de las fuerzas gravitatorias:

— La dirección del vector fuerza es la de la recta queune las dos partículas.

— Las fuerzas gravitatorias son siempre atractivas.

— Son fuerzas que actúan a distancia.

V G z= 2π σ

V V g dr G GA BA

B

A

B

− = ⋅ = − = − −⌠⌡⎮ ⌠

⌡⎮

� �2 2π σ π σdz (zB zz

V V G z

A

A B A

)

)− = −2π σ (zB

� �g kB = 2π σG

� �g kA = −2π σG

Φ π π σ; π σ

π σ

= − = − − = −

=

4 4 2

2

GM GS g S 4 GS

g G

b) r = 0,25 m

En este caso, el punto es interior a la corteza. Por tan-to,

La superficie gaussiana es cilíndrica, de caras para-lelas a la lámina, y está cortada por ésta.

35.

Determinamos el flujo a través de la superficie. En lasdos caras paralelas a la lámina, Sa y Sb, el campo gravi-tatorio es perpendicular a la superficie y de móduloconstante:

En cambio, en la superficie cilíndrica Sc, el campo esparalelo a la superficie, de forma que el ángulo entre

y es de 90°. Por tanto,

Entonces, el flujo total a través de la superficie, si lla-mamos S a la superficie de cada cara de la superficie ci-líndrica, será:

Aplicamos el teorema de Gauss para hallar el campogravitatorio, teniendo en cuenta que la masa interior ala superficie será M = S ⋅ σ:

Φ

Φ

= ⋅ + ⋅ + ⋅

= −

⌠⌡⎮ ⌠

⌡⎮ ⌠

⌡⎮

S S SA B C

g dS g dS g dS

g S

� � � � � �

AA B A Bg S g S S g S− + = − + = −( )0 2

� �g dS g dS⋅ = ° =cos 90 0

dS��

g

� �g dS g dS g dS⋅ = ° = −cos 180

g N kg V GMr

VN m

kg

kg

= = −

= − ⋅⋅−

0

6 67 101 000

011

2

2

;

,

/

,,,

51 3 10 7

mJ

kg= − ⋅ −

g GM

r

gN m

kg

kg

m

=

= ⋅⋅

⋅ =−

2

112

2 26 67 10

1 000

1 53,

( , )⋅⋅

= −

= − ⋅⋅

10

6 67 101 000

8

112

2,

Nkg

V GMr

VN m

kg

kggm

Jkg1 5

4 4 10 8

,,= − ⋅ −

g

g

g

Z

X Y

SC

SB

SA

dS�

dS�

�g

�g

dS�

dS�

dS��

g

dS�

Page 36: 9523 gl fis_tx2_cas

39

Es decir, el trabajo es igual al producto de la masa porla diferencia de potencial gravitatorio.

43. Al separar dos masas, su energía potencial gravitatoriaaumenta. Las masas se desplazan por acción de unafuerza exterior al campo gravitatorio, por tanto el tra-bajo realizado por el campo es negativo.

44. Para representar un campo de fuerzas como el gravi-tatorio, se utilizan líneas de campo y superficies equi-potenciales. Las líneas de campo son tangentes en ca-da punto al vector intensidad de campo, mientras quelas superficies equipotenciales unen todos los puntoscon el mismo potencial gravitatorio.

45. El flujo gravitatorio es una medida del número de lí-neas de campo gravitatorio que atraviesan una deter-minada superficie.

46. Datos: m1 = 150 g = 0,15 kg; m2 = 200 g = 0,20 kg; d = 10 cm = 0,1 m

Calculamos la fuerza a partir de la ley de la gravitaciónuniversal:

47. Datos: m1 = m2 = m = 10 kg; F = 10−5 N

Determinamos la distancia despejándola de la ley de lagravitación universal:

48. Datos: m1 = m2 = m; F = 10−4 N; r = 3 mm = 3 ⋅ 10−3 m

Determinamos la masa despejándola de la ley de lagravitación universal:

49. Datos: m = 2 kg; d = 50 m = 0,5 m

Determinamos la intensidad del campo gravitatoriocreado por la masa puntual:

F Gm

rm

F rG

mN m

= =

=⋅ ⋅

− −

2

2

2

4 3 210 3 10

6 67 10

;

( )

, 1112

2

3 7N m

kg

kg⋅

= ,

F Gm m

rG

m

rr

G mF

r

N m

kg

= = =

=

⋅⋅−

1 22

2

2

2

112

26 67 10

;

, (( )

,

10

102 6 10

2

52

kg

Nm−

−= ⋅

F Gm m

r

FN m

kg

kg k

=

= ⋅⋅ ⋅−

1 22

112

26 67 100 15 0 20

,, , gg

m

F N

( , )0 1

2 10

2

10= ⋅ −

W Ep Ep mV mV m V VA B A B A b= = − = − = −Δ Ep ( )— Se presentan siempre a pares, ya que si una partícu-la 1 atrae a otra partícula 2, la segunda tambiénatrae a la partícula 1, con una fuerza igual en mó-dulo y dirección pero de sentido contrario. Sonfuerzas de acción y reacción.

— La constante de gravitación universal G tiene un va-lor muy pequeño, de forma que la fuerza será ina-preciable a menos que una de las masas sea muygrande.

39. Recibe el nombre de campo de fuerzas una perturba-ción del espacio tal que, si situamos en esa región uncuerpo de prueba, éste se ve sometido a una fuerza.

— Ejemplos: campo gravitatorio creado por una placainfinita de densidad uniforme (campo uniforme)campo eléctrico creado por una carga puntual(campo central).

40. Cuando una partícula se mueve en un campo defuerzas conservativo sometida a la acción de la fuerzadel campo, la suma de la energía potencial más laenergía cinética (denominada energía mecánica), esconstante.

Demostración:

— El trabajo realizado por las fuerzas de un campoconservativo no depende del camino seguido. Portanto, se puede expresar como la variación decierta magnitud, que llamamos energía potencial:

— Por otro lado, el teorema de las fuerzas vivas estable-ce que el trabajo realizado sobre un cuerpo por lafuerza resultante se invierte en variar su energía ci-nética:

Entonces,

41. La intensidad del campo gravitatorio en un punto delespacio es la fuerza que experimentaría la unidad demasa situada en ese punto. Es una magnitud vectorial.En cambio, el potencial gravitatorio en un punto delespacio es la energía potencial gravitatoria que tendríauna masa unidad colocada en ese punto. Es una mag-nitud escalar.

42. El trabajo realizado por la fuerza gravitatoria para des-plazar un cuerpo de masa m desde un punto A a unpunto B es igual a la diferencia de energía potencialentre los dos puntos. La energía potencial puede calcu-larse como el producto del potencial por la masa, deforma que:

W Ec Ec

W Ep EpEc Ep Ec EpB A

A BA A B B

= −

= −

⎫⎬⎭

+ = +

W F dr m a dr mdvdt

dr

W

At

At

A

A

= = =

=

⌡⎮

⌡⎮

⌡⎮

Β Β Β

ΒΒ Β⌠

⌡⎮

⌡⎮= − = −m dv

drdt

mv dv m v m v

WA

B A12

12

2 2

== − =

=

Ec Ec

WB A Δ ;

Δ

Ec

Ec

W Ep EpA B= − = −ΔEp

2. Campo gravitatorio

Page 37: 9523 gl fis_tx2_cas

40

El campo total será la suma vectorial:

51. Datos: m1 = 0,5 kg; m2 = 0,75 kg; r = 2 m

Determinamos la energía potencial gravitatoria:

52. Datos: m1 = 450 g = 0,45 kg; d = 50 cm = 0,5 m; m2 = 3 g = 3 ⋅ 10−3 kg

Hallamos el potencial gravitatorio creado por m1 a 50 cm:

Determinamos la energía potencial multiplicando elpotencial por m2:

53. Datos: V = −5 ⋅ 10−9 J/kg; r = 2 m

Determinamos la masa despejándola de la expresióndel potencial creado por una masa puntual:

54. Datos: M1 = 2 ⋅ 106 kg; M 2 = 4 ⋅ 106 kg; r1 = 3 ⋅ 103 m; r2 = 4 ⋅ 103 m

Calculamos el potencial gravitatorio en el punto Pcomo la suma algebraica del potencial creado por cadauna de las masas:

V GMr

VN m

kg

kg

11

1

111

2

2

6

6 67 102 10

3 10

= −

= − ⋅⋅

⋅⋅

⋅−,

338

22

2

211

4 4 10

6 67 10

,

,

m

Jkg

V GMr

VN m

= − ⋅

= −

= − ⋅⋅

−22

2

6

38

1 2

4 10

4 106 67 10

kg

kg

m

Jkg

V V V

⋅⋅

⋅= − ⋅

= + =

−,

−− ⋅ −1 1 10 7, J kg/

V GMr

MVrG

MJ kg m

N

= − = −

=− ⋅ ⋅

⋅⋅

;

,

5 10 2

6 67 10

9

11

/

mm

kg

kg2

2

149 9= ,

Ep m V kg N kg J= = − ⋅ ⋅ ⋅ = − ⋅− − −2

3 11 133 10 6 10 1 8 10/ ,

V Gmr

VN m

kg

kgm

= −

= − ⋅⋅

⋅ = −−

1

112

26 67 100 450 5

6,,,

⋅⋅ −10 11 Jkg

Ep Gm m

r

EpN m

kg

kg

= −

= − ⋅⋅

⋅⋅−

1 2

112

26 67 10

0 5 0 7,

, , 552

1 2 10 10,

kgm

Ep J= − ⋅ −

� � � � �g g g u u N kg

g

= + = − − ⋅

=

−1 2 1 2

101 2 2 4 10( , , ) /

�� ��g N kg

g

= − + − ⋅

= ⋅

−( , ) ( , )

,

1 2 2 4 10

2 8 10

2 2 10 /−−10 N kg/Calculamos el potencial gravitatorio:

50. Datos:

— Punto P: cada masa está a una distancia r = 0,53 m.

Determinamos el campo creado por cada una delas masas:

El campo resultante será la suma vectorial:

— Punto Q: cada una de las masas está a una distan-cia r = 0,75 m.

Determinamos el campo creado por cada una delas masas:

� �

g Gm

ru

gN m

kg

kg

11

2 1

111

2

26 67 101

0

= −

= − ⋅⋅

⋅−,( ,775

1 2 10

2 1

110

1

22

2

)

,

mu

g u N kg

g Gm

r

� �

= − ⋅

= −

− /

��

� �

u

gN m

kg

kg

mu

2

211

2

2 26 67 102

10 75= − ⋅

⋅⋅−,( , ) 22

210

22 4 10� �g u N kg= − ⋅ −, /

� � � �

�g g g i N kg

g g

= + = ⋅

= = ⋅

1 2102 3 10

2 3 10

,

,

/

� � 110 N kg/

� �

g Gm

ru

gN m

kg

kg

11

2 1

111

2

26 67 101

0

= −

= − ⋅⋅

⋅−,( ,553

2 4 10

2

110

22

2

)

,

mi

g i N kg

g Gm

ru

� �

� �

= − ⋅

= −

− /

22

211

2

2 26 67 102

0 53� �g

N m

kg

kg

mi= − ⋅

⋅⋅ −−,( , )

( )

�� �g i N kg2

104 7 10= ⋅ −, /

V Gmr

VN m

kg

kgm

=

= − ⋅⋅

⋅ = − ⋅−6 67 102

0 52 7 1011

2

2,

,, −−10 J

kg

g Gm

r

gN m

kg

kg

m

=

= ⋅⋅

⋅ = ⋅−

2

112

2 26 67 10

2

0 55 3,

( , ), 110 10− N

kg

P

1,06 m

m1 = 1 kg m2 = 2 kg

0,75 m

Q

0,75 m

�g1

�gp

�g2

�gQ

�g2

�u1�u2

�g1

Page 38: 9523 gl fis_tx2_cas

41

57. Datos:

a) Determinamos el campo gravitatorio debido a ca-da una de las masas. Para ello, calculamos la dis-tancia de los vértices al centro del cuadrado:

El campo en el centro del cuadrado será la sumavectorial de las cuatro contribuciones:

b) Calculamos el campo gravitatorio debido a cadapartícula en un vértice del cuadrado:

El vector campo gravitatorio en cada vértice será lasuma vectorial de los campos de tres partículas:

� � � �

� � �g g g g

g i

= + +

= − ⋅ − ⋅− −

1 2 411 112 3 10 2 3 10( , , jj N kg

g i j N kg

)

, ( )

/

/� � �

= − + ⋅ −2 3 10 11

� �

g Gm

ru

gN m

kg

kg

11

12 1

111

2

26 67 10

1

2

= −

= − ⋅⋅

⋅−,( )

,

,

mi i

Nkg

g j N

211

411

1 7 10

1 7 10

� �

� �

= − ⋅

= − ⋅

− //kg

g Gm

ru

gN m

kg

� �

33

32 3

311

2

26 67 10

1

= −

= − ⋅⋅

⋅−,kkg

mi j

g i j

2 2

1

2

5 9 10

2

312

( )

, ( )

( )⋅ +

= − ⋅ +−

� �

� � �N kg/

� � � � �g g g g g N kg= + + + =1 2 3 4 0 /

rm

m

g Gm

r

=⎛

⎝⎜

⎠⎟ +

⎝⎜

⎠⎟ = = =

= −

12

12

1

2

2

21 4

2 2

1

,

�22 1

111

2

2 2 16 67 101

1 4

� �

u

gN m

kg

kg

mu= − ⋅

⋅⋅−,( , )

�� �

� �g u N kg

g u1

111

211

2

3 4 10

3 4 10

= − ⋅

= − ⋅

,

,

/

NN kg

g u N kg

g u

/

/� �

� �3

112

411

3 4 10

3 4 10

= ⋅

= ⋅

,

, 11 N kg/

— Si colocamos una masa de 500 kg en el punto, suenergía potencial será el producto de la masa porel potencial en el punto P:

55. Datos: M = 2 kg; r1 = 1 m; r2 = 40 cm = 0,4 m; m = 500 g = 0,5 kg

a) Determinamos el potencial creado por una masapuntual a las dos distancias:

b) El trabajo realizado por el campo para trasladar lamasa desde el primer punto hasta el segundo serála variación de energía potencial:

56. Datos: R = 500 m; M = 6 000 kg; d = 300 m

Tomamos como superficie gaussiana S, una esfera con-céntrica con la distribución de masa. Por simetría, elvector campo gravitatorio es perpendicular a la superfi-cie S en todos los puntos y de valor constante:

Entonces, el flujo a través de S es:

Aplicamos el teorema de Gauss para determinar la in-tensidad del campo gravitatorio:

Calculamos el potencial a partir del campo gravitatorio:

V g dr GM

R d

VN m

kg

r= ⋅ = −

+

= − ⋅⋅

⌠⌡

∞ � �

6 67 106 0011

2

2,00

500 3005 10 10

( )kg

mJ

kg+= − ⋅ −

Φ π π π= − − + = −

=+

=

4 4

6 67

2

2

GM; g 4 (R GM

(R

d

g GM

d

g

)

)

, ⋅⋅⋅

⋅+

= ⋅

−106 000

500 300

6 3 1

112

2 2

N m

kg

kg

m m

g

( )

, 00 13− N kg/

Φ π (= ⋅ = − = − = − +⌠⌡⎮ ⌠

⌡⎮ ⌠

⌡⎮

� �g dS g dS g dS g d)4 2R

� �g dS g dS g dS⋅ = − ⋅ ° = −cos 180

W m V V

W kg J kg

= −

= ⋅ − − − ⋅ −

( )

, [ , ( , )]1 2

100 5 1 3 3 3 10 /

WW J= −10 10

V GMr

VN m

kg

kgm

11

111

2

26 67 1021

1 3

= −

= − ⋅⋅

⋅ = − ⋅−, , 110

6 67 102

10

22

211

2

2

= −

= − ⋅⋅

Jkg

V GMr

VN m

kg

kg,

00 43 3 10 10

,,

mJ

kg= − ⋅ −

Ep m V kg J kg J= = − ⋅ ⋅ = − ⋅− −, ,500 1 1 10 5 5 107 5/

2. Campo gravitatorio

m1 = 1 kg

l = 2 m

m3 = 1 kg

m2 = 1 kg

m4 = 1 kg

Y

X

g

�g1

�g2�

g3

�g2

�g4

�u1

�g1�u2

�g3

Page 39: 9523 gl fis_tx2_cas

42

El trabajo realizado será el incremento de la energíapotencial elástica cambiado de signo.

Por tanto, la energía potencial elástica vendrá dadapor:

La constante C viene determinada por la elección delorigen de energías. Si escogemos como origen de po-tencia la posición de equilibrio (x = 0) obtenemos C =0 y la energía potencial elástica resulta:

59. Es fácil encontrar imágenes de líneas de fuerza enwebs como:

www.sociedadelainformacion.com/departfqtobarra/gravitacion/energia/intensidadenerg.htm

www.iesnicolascopernico.org/FQ/2BACH/FIS%20campos%20cuestiones.pdf

www.fisica-fa-cil.com/Temario/Electrostatica/Teorico/Campo/centro.htm

www.hiru.com/es/fisika/fisika_02200.html

www.fisicanet.com.ar/fisica/dinamica/ap10_campo_gravitatorio.php

60. Mediante un programa de representación de gráficoses fácil obtener:

a)

b)

Ep =12

2Kx

Ep = +12

2Kx C

W Ep Ep

Kx Kx Ep Ep

A B

A B A B

= − = −

= − = −

ΔEp

W12

12

2 2

Determinamos su módulo:

Y el módulo de la fuerza que experimenta cadapartícula es:

c) Calculamos el potencial debido a cada partícula enun vértice:

El potencial total será la suma algebraica:

Entonces, la energía potencial gravitatoria será elproducto del potencial por la masa:

58. La fuerza elástica que un muelle de constante elásticaigual a K ejerce cuando está comprimido una distanciax viene dada por:

donde es el vector unitario en la dirección de las x po-sitivas.

La fuerza será conservativa si el trabajo que realiza pa-ra trasladar una partícula depende sólo de las posicio-nes inicial y final:

Por tanto, como el resultado no depende del camino se-guido, la fuerza es conservativa. Podemos comprobar queel trabajo a lo largo de una trayectoria cerrada es cero:

W F dr

W F dr F dr

C

CA

B

A

B

A

B

=

= + +

⌠⌡⎮

⌠⌡⎮

⌠⌡⎮

⌠⌡

○� �

� � � �⎮⎮

⌠⌡⎮−

⎣⎢⎢

⎦⎥⎥+ −

⎣⎢⎢

⎦⎥⎥

=−

Kx dx Kx dx

W K x

A

B

C12 BB A A BK x K x K x2 2 2 21

212

12

0+ − + =

W F dr Kx i dr K x dx

W

A

B

A

B

A

B

= = − = −

=

⌠⌡⎮ ⌠

⌡⎮ ⌠

⌡⎮

� � � �

112

12

12

12

2 2 2 2K x K x K x K xB A A B+ = −

�i

� �F Kx i= −

Ep mV kg J kg J= = − ⋅ ⋅ = − ⋅− −1 8 9 10 8 9 1011 11, ,/

V V V V

V J kg J

= + +

= ⋅ − ⋅ − ⋅− −

1 3 411 112 3 3 10 2 3 10( , ) ,/ //

/

kg

V J kg= − ⋅ −8 9 10 11,

V V Gmr

V VN m

kg

kgm

1 41

1 411

2

26 67 1012

= = −

= = − ⋅⋅

⋅−, == − ⋅

= −

= − ⋅⋅

3 3 10

6 67 10

11

33

112

2

,

,

Jkg

V Gmr

N m

kg

11

2 22 3 10 11,

kg

m

Jkg

= − ⋅ −

F m g N= = ⋅ −,3 2 10 11

� ��g g N kg

g

= = − + − ⋅

= ⋅

( , ) ( , )

,

2 3 2 3 10

3 2 10

2 2 11 /111 N kg/

Page 40: 9523 gl fis_tx2_cas

43

a) Determinamos la distancia del punto al origen de coordenadas, donde se encuentra la masa:

Calculamos la intensidad del campo gravitatorio:

El vector unitario es:

Por tanto el campo gravitatorio también se puedeexpresar:

b) La fuerza que actuaría sobre una masa m = 20 kgserá el producto de la masa por el campo:

c) Calculamos el potencial gravitatorio en el punto (3, 4) m.

d) La energía potencial gravitatoria de la masa m = 20 kgserá:

6. Datos: ML = 7,47 ⋅ 1022 kg; RL = 1 740 km;RL = 1,74 ⋅ 106 m

Determinamos el campo en la superficie teniendo encuenta que, para una distribución de masa esférica, elcampo fuera de la distribución es el mismo que el crea-do por una masa puntual situada en el centro de la dis-tribución. Entonces:

g GM

R

gN m

kg

kg

L

L

= −

= ⋅⋅

⋅⋅−

2

112

2

22

6 67 107 47 10

1,

,

( ,, ),

74 101 6

6 2⋅=

m

Nkg

Ep m V kg J kg

Ep J

= = − ⋅ ⋅

= − ⋅

,

,

20 6 7 10

1 3 10

10

8

/

V GMr

VN m

kg

kgm

= −

= − ⋅⋅

⋅ = − ⋅−6 67 10505

6 7 10112

2, , −−10 Jkg

� � � �

�F m g kg i j N kg

F

= = ⋅ − − ⋅

=

−( , , )20 0 78 1 4 10 10 /

(( , , )

,

− −

= = ⋅ −

15 6 20 8

2 6 10 9

� �

�i j N

F F N� �

� � � �

�g g u N kg i j

g

= − = − ⋅ ⋅ +

=

−, ( , , )

(

1 3 10 0 6 0 8

0

10 /

,, , )78 1 4 10 10� �i j N kg− ⋅ − /

� � � � �u i j i j= + = +

15

3 4 0 6 0 8( ) , ,

�u

� �

g GM

ru

gN m

kg

kg

m

= −

= − ⋅⋅

⋅−

2

112

2 26 67 1050

5,

( )��

� �

u

g u N kg= − ⋅ −1 3 10 10, /

r m m m= + =( ) ( )3 4 52 2

c)

(Donde y representa el potencial)

EVALUACIÓN

1. Que un campo de fuerzas sea conservativo significaque el trabajo realizado por las fuerzas del campo paradesplazar una partícula de un punto a otro depende delas posiciones inicial y final, pero no del camino segui-do. El campo gravitatorio es conservativo.

2. El potencial gravitatorio y la energía potencial gravita-toria son negativos, porque tomamos el origen de ener-gía en el infinito. El trabajo del campo supone siempreuna disminución de la energía potencial y del poten-cial. Por tanto, si consideramos que una partícula libre(o en el infinito) tiene energía potencial nula, cuandoesté sometida al campo gravitatorio su energía poten-cial será negativa.

3. El potencial gravitatorio en un punto es la energía po-tencial que tendría una masa unidad situada en esepunto por el hecho de encontrarse sometida al cam-po gravitatorio. Por otro lado, la diferencia de poten-cial entre dos puntos A y B es el trabajo realizado porel campo sobre la masa unidad para desplazarla de Aa B.

4. Es posible que dos observadores den para el mismocuerpo energías potenciales diferentes, ya que puedenhaber considerado distintos orígenes de energía poten-cial. La magnitud que resulta relevante es la diferenciade energía potencial entre dos puntos. Esta diferen-cia tiene el mismo valor para ambos observadores, pe-ro el origen escogido es arbitrario.

5. Datos:

2. Campo gravitatorio

Y

(3, 4) m4

3

m = 50 kg

g

X

�u

Page 41: 9523 gl fis_tx2_cas

44

b) Calculamos la energía potencial de una masam = 12 kg en el cuarto vértice. Tendremos encuenta que M1 = M2 = M3 = M = 12 kg.

Calculamos la energía potencial de la masa m enel centro del cuadrado. Tendremos en cuentaque M1 = M2 = M3 = M = 12 kg y que estas tres

masas distan una distancia del centro del cua-drado.

El trabajo realizado por el campo será la variaciónde energía potencial:

9. Datos: EpA = −80 J; EpB = −160 J

a) El trabajo realizado por el campo es la variaciónde energía potencial:

Por tanto la respuesta correcta es: b) 80 J

W Ep Ep J J JA B= − = − = − − − =ΔΕp 80 160 80( )

W Ep Ep

W JA B= − = −

= − − − ⋅ = ⋅−

ΔΕp

[ , ( , )] ,5 2 8 3 10 3 1 109 −−9 J

V J kg

Ep m V JB

B

= − ⋅

= = − ⋅

6 9 10

8 3 10

10

9

,

,

/

r mm

V VB B

212

12

1

2

5

23 5

2 2

1

=⎛⎝⎜

⎞⎠⎟

+⎛⎝⎜

⎞⎠⎟

= = =

= +

,

VV V GMr

V NN m

kg

B B

B

2 3

112

2

3

2

3 6 67 1012

+ = −

= − ⋅ ⋅⋅

⋅−,kkgm3 5,

r2

V V V V

V GM

GM

GM

A A A A

A

= + +

= − −+

= − +⎛

⎝⎜

1 2 3

2 22

1 1 1

21

1

1 2

⎞⎞

⎠⎟

= − ⋅⋅

⋅ ⋅ +⋅

−VN m

kgkg

m mA 6 67 10 12

25

1

5 211

2

2,

⎛⎛

⎝⎜

⎠⎟

= − ⋅

= = − ⋅

V J kg

Ep m VA

A A

4 3 10

5 2 10

10

9

,

,

/

J

1 1 10311, (g i= − ⋅ −� �

++

= + +

= − ⋅ −−

� � � �

� �

j N kg

g g g g

g j

)

[ , ,

/

1 2 3

113 2 10 3 22 10 1 1 10

4 3

11 11⋅ − ⋅ +

= −

− −� � �

� �i i j N kg

g

, ( )]

( ,

/

ii j N kg

g g N k

− ⋅

= = ⋅

4 3 10

6 1 10

11

11

, )

,

� �/

/� � gg

7. Datos: m1 = 12 g = 0,012 kg; m2 = 18 g = 0,018 kg; d = 1 cm = 0,01 m

En el punto medio, la distancia a cada masa es d/2 == 5 ⋅ 10−3 m. Determinamos el potencial debido a cadamasa y los sumamos algebraicamente para determinarel potencial total.

8. Datos:

a) Calculamos el campo gravitatorio creado por cadamasa en el cuarto vértice, y los sumamos vectorial-mente para determinar al campo total:

� �

g GM

ru

gN m

kg

kg

33

2 3

311

2

26 67 1012

= −

= − ⋅⋅

⋅−,(55 5

1

22 22

) ( )( )

m mi j

+( )⋅ +

� �

� �

g GM

lu

gN m

kg

kg

11

2 1

111

2

26 67 1012

5

= −

= − ⋅⋅

⋅−,( )

,

mj

g j N kg

g GM

lu

g

2

111

22

2 2

3 2 10

� �

� �

= − ⋅

= −

− /

2211

2

2 2

2

6 67 1012

5

3 2

= − ⋅⋅

= −

−,( )

,

N m

kg

kg

mi

g

�⋅⋅ −10 11 �

i N kg/

V Gm

d

VN m

kg

112

111

2

2

2

6 67 100 012

=⎛

⎝⎜

⎠⎟

= − ⋅⋅

⋅−,, kkg

m

Jkg

V Gm

d

V

5 101 6 10

2

310

222

⋅= − ⋅

= −⎛

⎝⎜

⎠⎟

−−,

2211

2

2 36 67 10

0 018

5 102 4= − ⋅

⋅⋅

⋅= −−

−,

,,

N m

kg

kg

m⋅⋅

= + = − ⋅

10

4 0 10

10

1 210,

Jkg

V V V J kg/

g

m2 = 12 kg

m1 = 12 kg

l = 5 m

m3 = 12 kg l

l

rl l

X

Y

�g2

�j

�i�

g3 �g1

Page 42: 9523 gl fis_tx2_cas

45

Gravitación en el universo3

b) El trabajo que realiza el campo para llevar una masadel punto 1 al punto 2 es igual a la diferencia deenergía potencial entre los dos puntos.

Como podemos escribir la energía potencial en tér-minos del potencial gravitatorio:

Ep = m V

el trabajo será:

1. CAMPO GRAVITATORIO DE LA TIERRA

1. Existe un campo gravitatorio alrededor de la Tierra de-bido a la masa de ésta. Todos los cuerpos, por el hechode tener masa, crean a su alrededor un campo gravitato-rio. En el caso de la Tierra, como en el de todos los pla-netas y estrellas, al ser su masa muy grande, el campo esmás importante que el generado por otros cuerpos.

— La intensidad del campo gravitatorio terrestre enun punto del espacio representa la fuerza con quela Tierra atraería un objeto de masa unidad situa-do en ese punto.

2. La masa de un cuerpo es una propiedad intrínseca deéste e independiente del lugar donde se encuentra.Por tanto, aunque el cuerpo se aleje de la superficieterrestre, su masa no cambia, es la misma que en cual-quier otro lugar.

Su peso, por el contrario, es la fuerza con que la Tierralo atrae. Esta fuerza es inversamente proporcional a la distancia al centro de la Tierra. Por lo tanto, si elcuerpo se aleja de la superficie (asciende), su peso dis-minuye.

3. Datos: h = 200 km = 2 ⋅ 105 m; RT = 6,37 ⋅ 106 m;

Hallamos el módulo del campo gravitatorio terrestre auna distancia del centro de la Tierra r = RT + h:

g GM

R hr

T

=+( )2

GN m

kgM kgr= ⋅

⋅= ⋅−6 67 10 5 98 1011

2

224, ; ,

W m V m V m V V

W kg

= − = − = −

= ⋅ −

Ε Εp p1 2 ( )

[( ,

1 2 1 2

75 11 122 2 22

667 5

( , )]

,

J kg J kg

W J

/ /− −

= −

PREPARACIÓN DE LA UNIDAD

• a) 0,000003 km = 3 ⋅ 10−6 km

• b) 25 000 000 mg = 2,5 ⋅ 107 mg

• c) 4 537 000 kg = 4,537 ⋅ 106 kg

• d) 12 425,65 s = 1,242565 ⋅ 104 s

• Datos: m = 25 kg

Llamamos peso a la fuerza gravitatoria con la que la Tierraatrae a los cuerpos. Esta fuerza se expresa como p = m g,y es un vector dirigido hacia el centro de la Tierra. En la su-perficie terrestre, g = 9,8 m/s2. Por tanto:

p = m g = 25 kg ⋅ 9,8 m/s2 = 245 N

• Datos: m = 2 kg; v = 20 m/s; y0 = 0 m

a) Por la conservación de la energía mecánica, la ener-gía potencial gravitatoria que adquirirá será igual a laenergía cinética inicial:

b) A partir de la expresión para la energía potencial enpuntos cercanos a la superficie terrestre, podemoscalcular la altura a la que llegará el cuerpo:

• Datos: M = 5 ⋅ 1014 kg; r1 = 3 000 m; r2 = 15 000 m;

a) Calculamos el potencial gravitatorio a las dos distan-cias:

V GMr

N m

kg

kg1

1

112

2

14

6 67 105 103 000

= − = − ⋅⋅

⋅⋅−,

mm

VJ

kg

V GMr

N m

kg

1

22

112

2

11 12

6 67 10

= −

= − = − ⋅⋅−

,

, ⋅⋅⋅

= −

5 1015 000

2 22

14

2 ,

kgm

VJ

kg

m kg GN m

kg= = ⋅

⋅−75 6 67 10 112

2; ,

Εp

/

= =

=⋅

=

m g h hEp

m g

hJ

kg m sm

;

,,

400

2 9 820 42

Εp /= = = ⋅ ⋅ =Ec m v kg m s J02 21

212

2 20 400( )

3. Gravitación en el universo

Page 43: 9523 gl fis_tx2_cas

46

8. Ep = m g h. Esta expresión es válida sólo para puntospróximos a la superficie terrestre y para variaciones dealtura pequeñas comparadas con el radio terrestre. Elorigen de la energía potencial se toma en la superficiede la Tierra.

Esta expresión es la más general y

es válida para cualquier punto del espacio. El origende energía potencial está, en este caso, en el infinito.

9. Datos: m = 500 kg; h = 2 000 km = 2 ⋅ 106 m;

RT = 6,37 ⋅ 106 m; MT = 5,98 ⋅ 1024 kg;

A una altura de 2 000 km, la expresión de la energíapotencial para cuerpos situados cerca de la superficieya no es válida.

Si tomamos el origen de la energía potencial en el in-finito:

10. Datos: hA = 4 200 km = 4,2 ⋅ 106 m;

hB = 5 800 km = 5,8 ⋅ 106 m; RT = 6,37 ⋅ 106 m;

Si tomamos el origen del potencial en el infinito, elpotencial gravitatorio creado por la Tierra en cadauno de los dos puntos será:

— El trabajo realizado por el campo es igual a la varia-ción de energía potencial. Podemos expresar laenergía potencial gravitatoria como el producto dela masa del satélite por el potencial. Por tanto:

W EP EP m V V

W kg J kgA B A B= − = −

= ⋅ − ⋅ −

( )

[ ,7 500 3 77 107 / (( , )]

,

− ⋅

= − ⋅

3 28 10

3 68 10

7

10

J kg

W J

/

V GM

R h

VN m

kg

AT

T A

A

= −+

= − ⋅⋅

⋅⋅−6 67 10

5 98 10112

2

24

,,

, ,

,

kg

m m

VJ

kg

V

A

B

6 37 10 4 2 10

3 77 10

6 6

7

⋅ + ⋅

= − ⋅

= −−+

= − ⋅⋅

⋅⋅−

GM

R h

VN m

kg

k

T

T B

B 6 67 105 98 1011

2

2

24

,, gg

m m

VJ

kgB

6 37 10 5 8 10

3 28 10

6 6

7

, ,

,

⋅ + ⋅

= − ⋅

M kg m kg GN m

kgT = ⋅ = = ⋅

⋅−5 98 10 7 500 6 67 1024 112

2, ; ; ,

Ep GM mR h

EpN m

kg

T

T

= −+

= − ⋅⋅

⋅⋅−,

,6 67 10

5 98 10112

2

224

6 6

10

500

6 37 10 2 10

2 38 10

,

,

kg kg

m m

Ep

⋅ + ⋅

= ⋅ J

GN m

kg= ⋅

⋅−6 67 10 112

2,

Ep GM mR h

.T

T

== −+

4. Datos: m = 4 500 kg; h = 10 000 km; RT = 6,37 ⋅ 106 m

a) En la superficie terrestre, el peso del avión p0 será:

b) Hallamos el peso a una altura h = 10 000 km = 107 m,mediante la expresión de la variación del peso conla altura:

5. Datos: m = 4 kg; MM = 6,45 ⋅ 1023 kg;

a) La aceleración con que caen los cuerpos en caídalibre coincide con la intensidad del campo gravita-torio. Por tanto, en la superficie de Marte:

b) El peso de un objeto de m = 4 kg será el producto desu masa por la intensidad del campo gravitatorio:

6. Datos: RT = 6,37 ⋅ 106 m

Hallamos la altura a la cual el peso se reduce a la cuarta

parte, a partir de la expresión:

7. a) Los astronautas en órbita alrededor de la Tierraestán en estado de ingravidez porque su peso, esdecir, la fuerza con que la Tierra los atrae, es lafuerza que necesitan para describir su órbita circu-lar. La intensidad de campo gravitatorio en su ór-bita coincide con la aceleración centrípeta de sumovimiento circular.

b) Los planetas no caen sobre el Sol ni las lunas sobresus respectivos planetas por la misma razón que losastronautas están en estado de ingravidez. La fuer-za gravitatoria que actúa sobre ellos se emplea enhacerles describir su trayectoria circular.

pp

hR

ph

R

hR

T

T

T

=

+⎛

⎝⎜⎞

⎠⎟

= +⎛

⎝⎜⎞

⎠⎟=

+ =

02 0

2

1

14

1 4

1 2

;

;; ( ) ; ,h R R h R mT T T= − = = = ⋅2 1 6 37 106

p p=14 0,

p m g kg N kg NM= = ⋅ =, ,4 3 8 15 2/

a g GM

R

N m

kg

kgM

M

M

= = = ⋅⋅

⋅⋅−

211

2

2

23

6 67 106 45 10

,,

(( , )

,

3 38 10

3 8

6 2⋅

=

m

gNkgM

R km m GN m

kgM = = ⋅ = ⋅⋅−3 380 3 38 10 6 67 106 11

2

2, ; ,

pp

hR

N

m

mT

=

+⎛

⎝⎜⎞

⎠⎟

=

+⋅

02 7

61

44 100

110

6 37 10,⎜⎜⎞

⎠⎟

=2 6 678 N

p m g kg N kg N0 4 500 9 8 44 100= = ⋅ =, /

gN m

kg

kg= ⋅

⋅⋅

⋅ +−6 67 10

5 98 10

6 37 1011

2

2

24

6,,

( , 22 10

9 24

5 2 2⋅

=

)

,

m

gNkg

Page 44: 9523 gl fis_tx2_cas

47

En ausencia de rozamiento, la energía mecánica seconserva: Ec0 + Ep0 = Ec + Ep;

17. Datos: h = 2 000 km; MT = 5,98 ⋅ 1024 kg;

La distancia al centro de la Tierra es:

r = h + RT = 2 ⋅ 106 m + 6,37 ⋅ 106 m = 8,37 ⋅ 106 m

Calculamos la correspondiente velocidad de escape:

18. La órbita de los planetas tiene forma elíptica, con elSol en uno de sus focos. La órbita de los satélites esigualmente elíptica, con el planeta en uno de losfocos.

19. La velocidad de un planeta es mayor cerca del Solque lejos de éste. Teniendo en cuenta la segunda leyde Kepler, la velocidad será máxima cuando la distan-cia al Sol sea mínima, ya que con menos radio tieneque barrer la misma área que en los otros puntos dela órbita.

20. La existencia de los planetas puede predecirse a partirde su interacción gravitatoria con otros cuerpos celes-tes conocidos.

— Las masas de los planetas se determinan a partirdel radio y el período de alguno de sus satélites.Gracias a la tercera ley de Kepler, podemos rela-cionar el período y el radio de la órbita del saté-lite con la masa del objeto alrededor del cual or-bitan.

21. Datos: T = 16,7 días = 1,44 ⋅ 106 s; r = 1,88 ⋅ 109 m;

Hallamos la masa de Júpiter a partir de la tercera leyde Kepler:

TG M

r Mr

G T

Mm

22

32 3

2

9 3

4 4

4 1 88 10

= =

=⋅ ⋅

π π

π2

;

( , )

66 67 10 1 44 10

1 9 1011

2

26 2

27

, ( , )

,

⋅⋅

⋅ ⋅

= ⋅− N m

kgs

kkg

GN m

kg= ⋅

⋅−6 67 10 112

2,

vG M

r

N m

kgkg

eT= =

⋅ ⋅⋅

⋅ ⋅−

22 6 67 10 5 98 1011

2

224, ,

88 37 10

9 8 10

6

3

,

,

= ⋅

m

ve m/s

R m GN m

kgT = ⋅ = ⋅⋅−6 37 10 6 67 106 11

2

2, ; ,

12

2

2

02m v G

M mR

GM mR h

hG M R

G M

T

T

T

T

T T

− = −+

=TT T

TR vR

h

−−

=⋅ ⋅ ⋅ ⋅ ⋅ ⋅−, , ,

02

11 242 6 67 10 5 98 10 6 37 100

2 6 67 10 5 98 10 6 37 10 1 000

6

11 24 6 2⋅ ⋅ ⋅ ⋅ − ⋅−−, , , ( )

−− ⋅ = ⋅6 37 10 5 12 106 4, ,m m

2. MOVIMIENTO DE PLANETAS Y SATÉLITES

11. Cuesta más situar en órbita un satélite pesado que uno li-gero. Aunque una vez en órbita ambos tendrán la mismavelocidad, tanto la energía potencial como la energía ci-nética de cada uno será proporcional a su masa. Por tan-to, cuanto más pesado sea, más energía se necesita parasituarlo a determinada altura y darle la velocidad corres-pondiente a esa órbita.

12. La altura sobre el ecuador de un satélite geoestaciona-rio es fija e invariable. Su período debe ser igual al pe-ríodo orbital de la Tierra (24 h). Esta condición estable-ce una única velocidad y altura posibles para el satélite.Estas características de la órbita se han calculado en elejemplo 7.

13. Datos: r = 8 500 km; MT = 5,98 ⋅ 1024 kg;

Calculamos la velocidad orbital del satélite:

Hallamos el período de revolución:

14. Datos: v = 2,52 ⋅104 km/h = 7 000 m/s;

MT = 5,98 ⋅ 1024 kg; RT = 6,37 ⋅ 106 m;

a) Despejamos el radio de la órbita de la ecuación dela velocidad orbital:

b) Determinamos el período de revolución de la órbi-ta:

15. La energía mecánica de un satélite en órbita alrededorde la Tierra es siempre negativa, ya que el satélite está li-gado al campo gravitatorio terrestre. Si su energía mecá-nica no fuera negativa, el satélite escaparía de la órbita.

16. Datos: v0 = 1 000 m/s; MT = 5,98 ⋅ 1024 kg;

R m GN m

kgT = ⋅ = ⋅⋅−6 37 10 6 67 106 11

2

2, ; ,

Tv

mm s

s= =⋅ ⋅

= ⋅2 2 8 14 10

7 0007 3 10

63π πr

/,

,

vG M

rr

G M

v

r

N m

kg

T T= =

=

⋅⋅

⋅ ⋅−

;

, ,

2

112

26 67 10 5 98 10224

23

7 0008 14 10

kg

m sm

( ),

/= ⋅

GN m

kg= ⋅

⋅−6 67 10 112

2,

Tv

m

m ss= =

⋅ ⋅

⋅= ⋅

2 2 8 5 10

6 85 107 8 10

6

33π πr

/

,

,,

vG M

r

N m

kgkg

T= =

⋅⋅

⋅ ⋅

−, ,

,

6 67 10 5 98 10

8 5

112

224

110

6 85 10

6

3,

m

v = ⋅ m/s

GN m

kg= ⋅

⋅−6 67 10 112

2,

3. Gravitación en el universo

Page 45: 9523 gl fis_tx2_cas

48

Como consecuencia de esta inclinación, en di-ferentes puntos de la órbita el ángulo con queinciden los rayos de luz solares en los dos hemis-ferios y la superficie de éstos iluminada cam-bian.

Cuando la Tierra muestra al Sol uno de los hemis-ferios, la superficie de éste iluminada es mayor,los rayos inciden más perpendiculares y calientanmás; estamos en verano. Al mismo tiempo, en elotro hemisferio es invierno.

En épocas en las que los dos hemisferios están ex-puestos por igual a la radiación solar, hablamosde primavera y de otoño.

Las estaciones del año son claramente distingui-bles en las latitudes medias (zonas templadas). Enla zona ecuatorial no se distinguen estaciones,pues los rayos del Sol inciden siempre muy per-pendiculares, justo lo contrario de lo que ocurreen las zonas polares.

— La Luna no emite luz propia, sino que refleja laluz proveniente del Sol. Los eclipses de Luna seproducen cuando ésta entra en la zona de sombrade la Tierra.

Al dejar de estar iluminada por el Sol, veremoscómo se oscurece, produciéndose un eclipse lu-nar. La Tierra se interpone entre el Sol y laLuna.

23. Respuesta sugerida:

Las mareas consisten en el ascenso y descenso sucesivodel nivel del agua del mar por efecto de la atraccióngravitatoria de la Luna y el Sol.

— En las mareas vivas, la Tierra, la Luna y el Sol es-tán alineados. Las atracciones gravitatorias del Soly la Luna se suman.

— En las mareas muertas, la Luna, la Tierra y el Solforman ángulo recto con la Tierra en el vértice.Las atracciones gravitatorias del Sol y la Luna serestan.

24. — Las estaciones del año son debidas a la inclinacióndel eje de rotación terrestre respecto al plano de suórbita.

Sol

Tierra

Luna

Sol

Tierra

Luna

22. Éstos son los planetas del Sistema Solar y los datos de sus órbitas alrededor del Sol:

Nombre del planetaDistancia media al sol

(⋅106 km)Período de revolución

Velocidad orbital(⋅103 m/s)

Masa (⋅1024 kg)

Mercurio 58 88 días 47,93 0,36

Venus 108 225 días 34,91 4,84

Tierra 150 1 año 29,89 5,98

Marte 228 1,9 años 23,91 0,65

Júpiter 778 11,9 años 13,02 1 900,98

Saturno 1 427 29,5 años 9,64 568,94

Urano 2 870 84 años 6,81 86,83

Neptuno 4 497 164,8 años 5,44 103,16

Primavera en el H.N.Otoño en el H.S.

Verano en el H.N.Invierno en el H.S.

Invierno en el H.N.Verano en el H.S.

Otoño en el H.N.Primavera en el H.S.

Page 46: 9523 gl fis_tx2_cas

49

FÍSICA Y SOCIEDAD

a) La última expedición tripulada a la Luna fue la delApolo XVII, lanzado el 7 de diciembre de 1972. Alunizócinco días más tarde. El comandante de la misión fueEugene Cernan y estuvo acompañado por RolandEvans, piloto del módulo de mando, y HarrisonSchmitt, piloto del módulo lunar y primer científicotripulante de una misión Apolo. Fue considerada la mi-sión más cara del proyecto.

Entre los objetivos del Apolo XVII destacan el estudio dela composición de la corteza lunar, la investigaciónde las ondas de gravedad y la detección de posibles sig-nos de existencia de agua en la Luna. Realizaron tressalidas para estudiar la superficie y el subsuelo luna-res, recogiendo 150 kg de piedras y polvo lunar; ins-talaron una nueva estación transmisora de datos y uti-lizaron un detector de minerales por debajo de los1 300 m de profundidad.

La nave amaró en el Pacífico el 19 de diciembre, obte-niendo así un récord de permanencia en el espacio yponiendo fin al proyecto que llevó al hombre a laLuna.

RESOLUCIÓN DE EJERCICIOS Y PROBLEMAS

25. Datos: h = 500 km; ML = 7,47 ⋅ 1022 kg;

a) Determinamos la intensidad del campo gravitato-rio a 500 km de la superficie:

b) El valor de la aceleración de la gravedad coin-cide con el de la intensidad del campo gravita-torio:

g = 1,0 m/s2

c) La fuerza con que la Luna atrae a un objeto es elpeso de éste en la Luna:

p = m g = 200 kg ⋅ 1,0 N/kg = 200 N

26. Datos: m = 4 800 kg; h = 3 400 km;

MT = 5,98 ⋅ 1024 kg; RT = 6,37 ⋅ 106 m;

a) Calculamos el potencial gravitatorio a 3 400 km dela superficie terrestre:

GN m

kg= ⋅

⋅−6 67 10 112

2,

g GM

R h

gN m

kg

L

L

=+

= ⋅⋅

⋅⋅−

( )

,,

2

112

2

22

6 67 107 47 10 kkg

m m

g N kg

( , )

,

1 74 10 5 10

1 0

6 5 2⋅ + ⋅

= /

R m m kg GN m

kgL = ⋅ = = ⋅⋅−1 74 10 200 6 67 106 11

2

2, ; ; ,

En el caso de los eclipses de Sol, es la Luna la quese interpone entre el Sol y la Tierra. La Luna pasapor delante del Sol y proyecta su sombra sobre laTierra.

En los dos casos, el fenómeno no se produce encada órbita. El plano de la órbita de la Luna estáinclinado respecto al plano de la órbita de laTierra.

Ello es la causa de que la orientación relativa de lostres cuerpos vaya variando con el tiempo. En los mo-mentos en que coinciden los tres cuerpos alineadosy además la Luna pasa por delante o por detrás dela Tierra, se produce un eclipse de Sol o de Luna,según el caso.

— La Luna tiene distintas fases según la orientaciónde su cara iluminada respecto a la Tierra.

A medida que nuestro satélite describe su órbitaentorno a la Tierra, va orientando su cara ilumina-da en distintas direcciones. Cuando muestra su carailuminada a la Tierra, vemos la Luna llena, mientrasque si nos muestra la cara en sombra, estamos enLuna nueva. Las otras dos fases son posiciones in-termedias.

3. Gravitación en el universo

Eclipse de Luna

Tierra Luna

Eclipse de Sol

Zona de la Tierra en quees visible el eclipse de Sol

TierraLuna

No hay eclipse

Sí hay eclipseTierra

Tierra

Luna

Luna

Fases de la LunaLuna llena.

Vista desde la Tierra

Luna nueva.Vista desde la Tierra

Cuarto menguante.Visto desde la Tierra

Cuarto creciente.Visto desde la Tierra

Page 47: 9523 gl fis_tx2_cas

50

Calculamos su período de revolución:

29. Datos: p0 = 735 N (en la Tierra); h = 50 km;

ML = 0,01 MT; RL = 0,25 RT; MT = 5,98 ⋅ 1024 kg;

a) A partir del peso del cuerpo en la superficie te-rrestre, determinamos su masa:

Calculamos el peso del cuerpo cerca de la superfi-cie lunar, aprovechando que conocemos su pesoen la Tierra (p0) y las relaciones entre los radios ylas masas de ambos cuerpos celestes:

b) Aplicamos el principio de la conservación de laenergía mecánica para determinar la velocidaddel cuerpo, que cae desde una altura de 50 km,cuando llegue a la superficie de la Luna:

30. Datos: v0 = 750 km/h = 208,3 m/s; MS = 324 440 MT;

RS = 108 RT; MT = 5,98 ⋅ 1024 kg; RT = 6,37 ⋅ 106 m;

a) Calculamos la relación entre el peso del cuerpo enel Sol y en la Tierra:

p GM m

RG

M m

R

p GM m

R

SS

S

T

T

TT

= =

=

( )2 2

324 440

108

TT2

⎪⎪⎪

⎪⎪

= = =( )

, ; ,pp

p pS

TS T

324 440

10827 8 27 82

GN m

kg= ⋅

⋅−6 67 10 112

2,

Ec Ep Ec Ep

G M mR h

m vG M

A A B B

L

LB

L

+ = +

−+

= −

;

012

2 mmR

v G MR h R

G M R h RR

L

B LL L

L L L

L

= −+

+⎛

⎝⎜

⎠⎟ =

+ −2

1 1 2 ( )( )

( ),

,

R h

vG M h

R R hG M h

L

BL

L L

T

+

=+

=2 2 0 01

0 225 0 25

390 5

( , )

,

R R h

v

T T

B

+

= m/s

p GM m

RG

M m

R

p m g m GM

L

L

T

T

T

= =

= =

,

( , )2 2

0

0 01

0 25

TT

TR

p p

p

2

2 00 01

0 25

0

,

( , )

⎪⎪

⎪⎪

=

=,,

( , ),

01

0 25735 117 62 ⋅ =N N

p m g mpg

NNkg

kgTT

00 735

9 875= = = =;

,

R m GN m

kgT = ⋅ = ⋅⋅−6 37 10 6 67 106 11

2

2, ; ,

Tv

m

m ss= =

⋅ ⋅

⋅= ⋅

2 2 6 37 10

7 9 105 1 10

6

33π πr

/

,

,,

b) Hallamos la energía potencial gravitatoria de la nave:

Ep = m V = 4 800 kg ⋅ (−4,1 ⋅ 107 J/kg)

Ep = −1,97 ⋅ 1011 J

27. Datos: p0 = 8 330 N; r = 1,5 RT; MT = 5,98 ⋅ 1024 kg;

a)

b) Calculamos la velocidad orbital:

c) Determinamos el peso en la órbita a partir de supeso en la superficie terrestre mediante la expre-sión de la variación del peso con la altura:

28. Datos: r = RT; MT = 5,98 ⋅ 1024 kg; RT = 6,37 ⋅ 106 m;

Determinamos la velocidad orbital de un satélite a r = RT,o primera velocidad cósmica:

vG M

R

N m

kgkg

T

T

= =

⋅⋅

⋅ ⋅−, ,

,

6 67 10 5 98 10

6 3

112

224

77 10

7 9 10

6

3

= ⋅,

m

v m/s

GN m

kg= ⋅

⋅−6 67 10 112

2,

pp

hR

pN

R RRT

T T

T

=

+⎛

⎝⎜⎞

⎠⎟

=

+−⎛

⎝⎜⎞

02

1

8 330

11 5

;,

⎟⎟

=+

=

2

2

8 330

1 0 53 702 2p

NN

( , ),

vG M

r

N m

kgkg

T= =

⋅⋅

⋅ ⋅

−, ,

,

6 67 10 5 98 10

1 5

112

224

66 37 10

6 5 10

6

3

,

,

= ⋅

m

v m/s

R m GN m

kgT = ⋅ = ⋅⋅−6 37 10 6 67 106 11

2

2, ; ,

V GM

R h

VN m

kg

kg

T

T

=+

= − ⋅⋅

⋅⋅−6 67 10

5 98 10112

2

24

,,

66 37 10 3 4 10

4 1 10

6 6

7

, ,

,

⋅ + ⋅

= − ⋅

m m

V J kg/

Tierra

F GM m

rT s=

2

Page 48: 9523 gl fis_tx2_cas

51

nes gravitatorias, por una parte, y, por otra, la resis-tencia que opone a ser acelerado bajo la acción deuna fuerza.

34.

35. Cuando un cuerpo se eleva a cierta altura sobre la su-perficie de la Tierra, gana energía potencial. Si se dejacaer el cuerpo desde esta altura, la ganancia de energíapotencial implica que llegará a la superficie con mayorvelocidad.

— La pérdida (o ganancia) de energía potencial sig-nifica que el cuerpo queda más (o menos) ligadoal campo gravitatorio terrestre.

36. Lo consigue describiendo un movimiento con el mis-mo período que el período de rotación de la Tierra,24 horas. Para ello debe describir una órbita con unavelocidad y altura concretas. Así, su velocidad angularcoincide con la de giro de nuestro planeta.

37.

— Para que un cuerpo abandone el campo gravitato-rio terrestre, es necesario que su energía mecáni-ca sea igual o superior a cero. Esto se conseguirási se lanza desde la superficie a una velocidadigual o superior a la velocidad de escape.

38. La trayectoria de los planetas del Sistema Solar debeser plana por la conservación del momento angular.

El momento angular es una magnitud vectorial per-pendicular a los vectore Como sobre los plane-tas no actúa ningún momento de fuerzas, el momentoangular debe conservarse. Así, tendrá la misma direc-ción en cualquier punto de la órbita, y estaránsiempre en el mismo plano perpendicular a

39. Datos: h = 450 km; MT = 5,98 ⋅ 1024 kg;

R m GN m

kgT = ⋅ = ⋅⋅−6 37 10 6 67 106 11

2

2, ; ,

�L.

� �r y v

� �r y v.

Energía mecánica Tipo de órbita

E > 0 Abierta: hipérbola

E = 0 Abierta: parábola

E < 0 Cerrada: circular o elíptica

b) Determinamos la altura máxima alcanzada por elproyectil aplicando el principio de conservaciónde la energía mecánica:

EJERCICIOS Y PROBLEMAS

31. — La aceleración de la gravedad varía con la altura ala superficie de la Tierra porque varía con la dis-tancia al centro de la Tierra. La aceleración de lagravedad es la intensidad del campo gravitatorio,es decir, la fuerza con que la Tierra atraería uncuerpo de masa unidad situado en ese punto.Como la fuerza gravitatoria es inversamente pro-porcional al cuadrado de la distancia, la acelera-ción de la gravedad disminuye con la altura de lamisma manera.

— El peso de un cuerpo no tiene el mismo valor en laTierra que en la Luna. El peso es la fuerza con quela Tierra o la Luna atraen al objeto, y es proporcio-nal a la masa del planeta o del satélite. Por tanto,no tienen el mismo peso.

32. La intensidad del campo gravitatorio y la aceleraciónde la gravedad coinciden en cada punto debido a quela masa inercial y la masa gravitatoria de cualquiercuerpo son iguales. La intensidad del campo g es lafuerza por unidad de masa gravitatoria que la Tierraejerce sobre todo cuerpo. Entonces, un cuerpo de ma-sa gravitatoria mg siente una fuerza (peso):

p = mg g

Por otro lado, debido a esta fuerza, el cuerpo experi-mentará una aceleración a, proporcional a su masainercial mi:

p = mi a

Como la fuerza es la misma y la masa inercial coinci-de con la gravitatoria, mi = mg, la aceleración del cuerpocoincide con la intensidad del campo gravitatorio enese punto.

33. El peso es la fuerza con que la Tierra atrae a los ob-jetos en su superficie por el hecho de tener masa. Esinversamente proporcional a la distancia al centrode la Tierra, de modo que no es una magnitud cons-tante.

La masa, en cambio, es una propiedad inherente alos cuerpos. Es fija e invariable. Representa la inten-sidad con que el cuerpo participa en las interaccio-

Ec Ep Ec Ep

m v GM m

RG

M mR h

h

S

S

S

S

0 0

021

2

2

+ = +

− = −+

=G M R

G M R vR

hG M

S S

S SS

T

2

2 324 440 100

2−−

=⋅ ⋅ 88

2 324 440 108108

2 6 60

2

,

R

G M R vR

h

T

T TT⋅ −

=⋅ 77 10 324 440 5 98 10 108 6 37 10

2 6 6

11 24 6⋅ ⋅ ⋅ ⋅ ⋅ ⋅ ⋅

− , ,

, 77 10 324 440 5 98 10 108 6 37 10 20811 24 6⋅ ⋅ ⋅ ⋅ − ⋅ ⋅ ⋅− , , ( ,, )

,

3

108 6 37 10 79

2

6

− ⋅ ⋅ =m m

3. Gravitación en el universo

Líneas de campo

Superficiesequipotenciales

Page 49: 9523 gl fis_tx2_cas

52

43. Datos: m = 600 kg; r = 10 000 km;

Calculamos la energía potencial del satélite a 10 000 kmdel centro de la Tierra:

44. Datos: V = −2 ⋅ 107 J/kg; MT = 5,98 ⋅ 1024 kg;

Despejamos de la expresión general del potencial gra-vitatorio la distancia r al centro de la Tierra:

Por tanto, la distancia a la superficie terrestre será:

h = r − RT = 1,99 ⋅ 107 m − 6,37 ⋅106 m = 1,35 ⋅ 107 m

45. Datos: m = 2 500 kg; r1 = 8 000 km; r2 = 10 000 km;

MT = 5,98 ⋅ 1024 kg; RT = 6,37 ⋅ 106 m;

El trabajo necesario para trasladar el satélite coincidi-rá con la variación de su energía potencial:

46. Datos: r = 7 000 km; MT = 5,98 ⋅ 1024 kg;

Calculamos la velocidad orbital en una órbita de7 000 km de radio:

vG M

r

N m

kgkg

T= =

⋅⋅

⋅ ⋅

−, ,6 67 10 5 98 10

7 10

112

224

66

37 5 10,

m

v m s= ⋅ /

GN m

kg= ⋅

⋅−,6 67 10 112

2

W Ep Ep m V m V m V V

W m GMr

GMr

T T

= − = − = −

= −

1 2 1 2 1 2

1 2

( )

⎛⎛

⎝⎜⎜

⎠⎟⎟ = −

⎝⎜⎜

⎠⎟⎟

= ⋅ ,

m G Mr r

W

T1 1

2 500 6 67

1 2

⋅⋅ ⋅ ⋅ ⋅⋅

−⎛

⎝⎜

⎠⎟

= ⋅

−10 5 98 101

8 10

1

10

2 5 10

11 246 7

,

,W 110 J

GN m

kg= ⋅

⋅−6 67 10 112

2,

V GM

rr G

MV

rN m

kg

T T= − = −

= − ⋅⋅

⋅⋅−

;

,,

6 67 105 98 111

2

2

00

2 101 99 10

24

77

( ),

kg

J kgm

− ⋅= − ⋅

/

R m GN m

kgT = ⋅ = ⋅⋅−6 37 10 6 67 106 11

2

2, ; ,

Ep GM m

r

EpN m

kg

T= −

= − ⋅⋅

⋅⋅−,

,6 67 10

5 98 10112

2

24 kkg kg

m

Ep J

= − ⋅ −

600

10

2 39 10

7

10,

M kg GN m

kgT = ⋅ = ⋅⋅−5 98 10 6 67 1024 11

2

2, ; ,

Calculamos la intensidad del campo gravitatorio terres-tre a 450 km de la superficie:

40. Datos: m = 25 kg; g0 = 9,8 m/s2; h = 3 000 km;

RT = 6,37 ⋅ 106 m

a) Determinamos el peso del cuerpo en la superficieterrestre, donde conocemos el valor de la intensi-dad del campo gravitatorio:

p0 = m g0 = 25 kg ⋅ 9,8 N/kg = 245 N

b) A 3 000 km de altura ya no es válida la expresiónutilizada en el problema anterior. Para calcular elpeso utilizaremos la fórmula de la variación del pe-so con la altura:

41. Datos: p0 = 19,6 N (en la Tierra); ML = 7,47 ⋅ 1022 kg;

a) Calculamos la masa del objeto a partir de su pesoen la Tierra y de la intensidad del campo gravitato-rio en la superficie terrestre:

b) El valor de la masa en la Luna será el mismo que enla Tierra y que en cualquier otro lugar, m = 2 kg.

El peso en la superficie lunar es la fuerza gravitato-ria con que la Luna atrae el objeto:

42. Datos: g0 = 9,8 m/s2; RT = 6,37 ⋅ 106 m

Para determinar el punto sobre la superficie terrestredonde la gravedad es dos tercios de g0, despejamos h de la expresión de la variación de la gravedad con la al-tura:

g gg

hR

hR

hR

T

T

T

= =

+⎛

⎝⎜⎞

⎠⎟

= +⎛

⎝⎜⎞

⎠⎟

+ =

23

1

32

1

1

00

2

2

;

332

32

1 6 37 1032

1

1

6; ,h R m

h

T= −⎛

⎝⎜

⎠⎟ = ⋅ −

⎝⎜

⎠⎟

= ,,43 106⋅ m

P GM m

R

PN m

kg

LL

L

L

=

= ⋅⋅

⋅⋅−,

,

2

112

2

22

6 67 107 47 10 kkg kg

mN

⋅=

2

1 74 103 36 2( , ),

p m g

mpg

NNkg

kg

0 0

0

0

19 6

9 82

=

= = =,

,

R m GN m

kgg N kgT = ⋅ = ⋅

⋅=−1 74 10 6 67 10 9 86 11

2

2 0, ; , ; , /

pp

hR

pN

m

mT

=

+⎛

⎝⎜⎞

⎠⎟

=

+⋅

02 6

61

245

13 10

6 37 10

;

,

⎛⎛

⎝⎜

⎠⎟

=2 113 2, N

gG M

R h

gN m

kg

T

T

=+

= ⋅⋅

⋅⋅−

( )

,,

2

112

2

24

6 67 105 98 10

( , , )

,

kg

m m

g N kg

6 37 10 4 5 10

8 57

6 5 2⋅ + ⋅

= /

Page 50: 9523 gl fis_tx2_cas

53

Calculamos la velocidad de escape desde la superficiede la Luna:

50. Datos: r = 6,7 ⋅ 105 km; MJ = 318,4 MT;

Calculamos el período de revolución de Europa a par-tir de la tercera ley de Kepler:

51. Datos: T = 1 día = 24 h = 8,64 ⋅ 104 s;

Para determinar el radio de la órbita aplicamos la ter-cera ley de Kepler:

52. Datos: R = 1,25 RT; g0 = 14,7 m/s2 (en el planeta);

a) Para calcular la relación entre las masas de laTierra y el planeta, escribimos las expresiones delcampo gravitatorio en la superficie de cada unode ellos y las dividimos:

g en la Tierra GM

R

m

s

g en el planeta

T

T0 2 2

0

9 8( ) ,

( )

= =

== = =GM

RG

M

R

m

sT2 2 21 25

14 7( , )

,

GN m

kg= ⋅

⋅−6 67 10 112

2,

TG M

r

rG M T

r

N m

k

T

T

22

3

2

23

112

4

4

6 67 10

=

=

=

⋅⋅−

π

π

,gg

kg s

r

224 4 2

2

3

7

5 98 10 8 64 10

4

4 22 10

⋅ ⋅ ⋅ ⋅

= ⋅

, ( , )

,

π

mm

M kg GN m

kgT = ⋅ = ⋅

⋅−5 98 10 6 67 1024 112

2, ; ,

TG M

r

TG M

rG M

r

T

J

J T

22

3

23

23

4

4 4318 4

4

=

= =⋅

=

π

π π,

ππ2 8 3

112

2

6 7 10

6 67 10 318 4 5

⋅ ⋅

⋅⋅

⋅ ⋅−

( , )

, , ,

m

N m

kg998 10

3 0 10

24

5

= ⋅,

kg

T s

M kg GN m

kgT = ⋅ = ⋅⋅−5 98 10 6 67 1024 11

2

2, ; ,

vG MR

v

N m

kg

eL

L

e

=

=

⋅ ⋅⋅

⋅ ⋅−

2

2 6 67 10 7 47 10112

222, , kkg

m

ms1 74 10

2 4 106

3

,,

⋅= ⋅

47. Datos: m = 1 250 kg; h = 1 400 km; MT = 5,98 ⋅ 1024 kg;

a) Determinamos su energía potencial:

b) Para determinar la energía cinética del satélite, cal-culamos primero su velocidad orbital:

c) Hallamos el período de revolución a partir de la ve-locidad y el radio de la órbita:

48. Datos: h = 5 000 km; MT = 5,98 ⋅ 1024 kg;

Para que el satélite llegue a 5 000 km de altura, es ne-cesario lanzarlo con una velocidad tal que su energíamecánica inicial sea igual a la energía potencial quetendrá a esa altura, a donde llegaría con velocidadnula:

49. Datos: ML = 7,47 ⋅ 1022 kg; RL = 1,74 ⋅ 106 m;

GN m

kg= ⋅

⋅−6 67 10 112

2,

12

2 1 1

02

02

m v GM m

RG

M mR h

vm

G M mR

T

T

T

T

TT

− = −+

= −RR h

v G MR h RR R h

v G M

T

TT T

T TT

+

⎝⎜⎞

⎠⎟

=+ −

+=0

202 2

( );

hhR R h

vN m

kg

T T( )

, ,

+

= ⋅ ⋅⋅

⋅ ⋅−0

112

2242 6 67 10 5 98 10 kkg

m

m m m

⋅⋅

⋅ ⋅ ⋅ + ⋅, ( ,

5 10

6 37 10 6 37 10 5 10

6

6 6 6 )),= ⋅7 4 103 m/s

R m GN m

kgT = ⋅ = ⋅⋅−6 37 10 6 67 106 11

2

2, ; ,

Tr

vR h

v

Tm

T= =+

=⋅ ⋅ + ⋅

2 2

2 6 37 10 1 4 106 6

π π

π

( )

( , , mm

m ss

)

,,

7 2 106 8 103

3

⋅= ⋅

/

vG MR h

N m

kgkg

T

T

=+

=

⋅⋅

⋅ ⋅, ,

,

6 67 10 5 98 10

6

112

224

337 10 1 4 10

7 2 10

12

12

6 6

3

2

⋅ + ⋅

= ⋅

= = ⋅

,

,

m m

v

Ec m v

m/s

11 250 7 2 10 3 24 103 2 10kg m s J( , ) ,⋅ = ⋅/

Ep GMR h

EpN m

kg

T m

T

= −+

= − ⋅⋅

⋅⋅−,

,6 67 10

5 98 10112

2

244

6 6

1 250

6 37 10 1 4 10

6 42 10

, ,

,

kg kg

m m

Ep

⋅ + ⋅

= − ⋅ 110 J

R m GN m

kgT = ⋅ = ⋅

⋅−6 37 10 6 67 106 112

2, ; ,

3. Gravitación en el universo

Page 51: 9523 gl fis_tx2_cas

54

d) Calculamos la aceleración centrípeta, que debecoincidir con la aceleración de la gravedad a esaaltura, pues el campo gravitatorio es el responsa-ble de que el satélite describa una órbita circular:

54. Datos: rUmbriel = 2,67 ⋅ 108 m; TUmbriel = 3, 58 ⋅ 105 s;

a) Determinamos la masa de Urano a partir de la ter-cera ley de Kepler, aplicada a su satélite Umbriel:

b) Conocida la masa, aplicamos la misma ley para de-terminar el período de revolución de Oberón apartir de su distancia al centro del planeta:

55. Las gráficas deben ser similares a éstas:

a) Para la Tierra

TG M

r

T

OberónUrano

Oberón

Oberón

=

=⋅

4

4 5

23

2

π

π ( ,886 10

6 67 10 8 79 10

1 18 3

112

225

⋅⋅

⋅ ⋅

=−

m

N m

kgkg

)

, ,

, 66 106⋅ s

TG M

r

MG

UmbrielUrano

Umbriel

Urano

22

3

2

4

4

=

=

π

π

TTr

Mm

UmbrielUmbriel

Urano( , )

3

2 8 34 2 67 10

6

=⋅ ⋅π

,, ( , )

,

67 10 3 58 10

8 79 1011

2

25 2

25

⋅⋅

⋅ ⋅

= ⋅− N m

kgs

kg

r m GN m

kgOberón = ⋅ = ⋅⋅−5 86 10 6 67 108 11

2

2, ; ,

avr

vR h

am s

m

cT

c

= =+

=⋅

⋅ + ⋅

2 2

3 2

6

7 6 10

6 37 10 5

( , )

,

/

1108 4

6 67 10

52

2

11

,

( )

,

mm s

gG M

R h

gN m

T

T

=

=+

= ⋅⋅

/

22

2

24

6 6 2

5 98 10

6 37 10 0 5 10kg

kg

m m

g

⋅⋅

⋅ + ⋅

=

,

( , , )

88 4 2, m s/

E

N m

kgkg k

= − ⋅

⋅⋅

⋅ ⋅ ⋅12

6 67 10 5 98 10 1 500112

224, , gg

m m

E J

6 37 10 5 10

4 35 10

6 5

10

,

,

⋅ + ⋅

= − ⋅

b) A 275 m sobre la superficie, podemos escribir laenergía potencial como Ep = m g h.

Aplicamos la conservación de la energía mecánicapara calcular la velocidad con que el objeto llega-ría a la superficie:

Imponemos que las velocidades sean las mismasen los dos planetas para determinar la altura des-de la cual debemos soltar el objeto en el otro pla-neta:

53. Datos: m = 1 500 kg; h = 500 km;

MT = 5,98 ⋅ 1024 kg; RT = 6,37 ⋅ 106 m;

a) Calculamos la velocidad orbital:

b) Determinamos el período orbital a partir de la ve-locidad y el radio de la órbita:

c) Hallamos la energía mecánica de traslación delsatélite:

EG M m

rG M mR h

T T

T

= − = −+

12

12

Tr

vR hv

Tm m

T= =+

=⋅ ⋅ + ⋅

2 2

2 6 37 10 5 106 5

π π

π

( )

( , )

77 6 105 7 10

33

,,

⋅= ⋅

m ss

/

vG M

rG MR h

v

N m

kg

T T

T

= =+

=

⋅⋅

⋅ ⋅, ,6 67 10 5 98 10112

2244

6 53

6 37 10 5 107 6 10

kg

m mm s

,,

⋅ + ⋅= ⋅ /

GN m

kg= ⋅

⋅−6 67 10 112

2,

v v

g h g h

hgg

h h

m

sm

s

t

T T

TT

=

=

= = ⋅

2 2

9 8

14 727

2

2

;,

,55 183 3,m =

Ec Ep Ec Ep m g h m v

v g h

0 021

2

2

+ = + =

=

;

g en el planetag en la Tierra

GM

0

0

1 25( )( )

( ,=

)

( , )

( , )(

R

GM

R

M

M

MR

g en el

T

T

T

T

T

2

2

2

2 0

1 25

1 25

=

= ⋅pplaneta

g en la Tierra

m

sm

)( )

( , ),

,0

2 21 25

14 7

9 8= ⋅

ss2

2 34= ,

Page 52: 9523 gl fis_tx2_cas

55

EVALUACIÓN

1. El peso de un cuerpo es la fuerza con que éste es atraí-do por la Tierra o por el planeta sobre el que se en-cuentre.

Depende directamente de la masa del cuerpo y dela masa del planeta, y es inversamente proporcional a ladistancia al centro del planeta al cuadrado.

2. Para hallar la expresión de la velocidad de escape,imponemos que su energía mecánica final sea iguala cero. Por tanto, por la conservación de la energíamecánica:

3. Leyes de Kepler:

1. Todos los planetas describen órbitas elípticas con elSol situado en uno de sus focos.

2. La recta que une un planeta con el Sol barre áreasiguales en tiempos iguales.

3. El cuadrado del período de la órbita de un planetaes directamente proporcional al cubo de la distan-cia media del planeta al Sol:

T2 = C R3

— Para demostrar la tercera ley de Kepler partimosde las expresiones para la velocidad orbital y parael período:

Sustituimos la expresión de v en T y elevamos alcuadrado:

4. Datos: h = 275 km; g0 = 9,8 N/kg; RT = 6,37 ⋅ 106 m;

Calculamos la intensidad del campo gravitatorio:

gN m

kg= ⋅

⋅⋅

⋅−,,11

2

2

24

6 67 105 98 10

( , , )

,

kg

m m

g N kg

6 37 10 2 75 10

9 03

6 5 2⋅ + ⋅

= /

gG M

R hT

T

=+( )2

M kg GN m

kgT = ⋅ = ⋅⋅−5 98 10 6 67 1024 11

2

2, ; ,

TG M

r

TG M

rG M

r= = =2 4 42

2 2 23π π πr r

;

vG M

rT

v= =; 2 2π r

12

0

12

2

2

2

m v GM m

r

v GMr

vG Mre

− =

=

p m g m GM

r= = 2

b) Para Marte:

56. Si se realiza la simulación con Interactive Physics 2000,aparte de introducir los datos con cuidado, para que lasimulación funcione razonablemente hay que tener encuenta las siguientes configuraciones en el programa:

— En relación al tamaño.

— En relación a la velocidad de animación:

— En relación a la ubicación de los objetos en el sis-tema de coordenadas se propone:

3. Gravitación en el universo

Page 53: 9523 gl fis_tx2_cas

56

a) Calculamos el radio de la órbita mediante la terce-ra ley de Kepler:

b) Su aceleración normal coincide con la aceleraciónde la gravedad en la órbita:

c) Determinamos la energía potencial gravitato-ria:

8. Datos: r = 9,4 ⋅ 106 m; T = 460 min = 27 600 s;

Para determinar la masa de Marte aplicamos la terce-ra ley de Kepler:

TG M

r MG T

r

Mm

MM

M

22

32

23

2 6

4 4

4 9 4 10

= =

=⋅ ⋅

π π

π

;

( , )33

112

22

23

6 67 10 27 600

6 45 10

, ( )

,

⋅⋅

= ⋅− N m

kgs

kgg

GN m

kg= ⋅

⋅−6 67 10 112

2,

Ep GM m

r

EpN m

kg

T= −

= − ⋅⋅

⋅⋅−,

,6 67 10

5 98 10112

2

24 kkg kg

m

Ep J

= − ⋅

1 000

6 71 10

5 94 10

7

9

,

,

a g GM

r

gN m

kg

kg

nT= =

= ⋅⋅

⋅⋅−

2

112

2

24

6 67 105 98 10

,,

(( , )

, ,

6 71 10

0 09 0 09

7 2

2

= =

m

g N kg m s/ /

TG M

r rG M T

r

N m

T

T22

32

3

112

4

4

6 67 10

= =

=

⋅⋅−

π

π2;

,kkg

kg s

r

224 5 2

2

35 98 10 1 73 10

4

6 71 1

⋅ ⋅ ⋅ ⋅

= ⋅

, ( , )

,

π

007 m

GN m

kg= ⋅

⋅−6 67 10 112

2,— Determinamos la altura en que g = g0 − 0,15 g0 =

= 0,85 g0 a partir de la expresión de la variación de lagravedad con la altura:

5. Datos: t = 3 s; ML = 7,47 ⋅ 1022 kg; RL = 1,74 ⋅ 106 m;

La aceleración de una partícula en caída libre coincidecon la intensidad del campo gravitatorio en ese punto.Determinamos, pues, el campo gravitatorio de la Lunacerca de su superficie:

Para determinar la distancia que recorre la partículaen tres segundos, aplicamos la ecuación correspon-diente del MRUA:

6. Datos: m = 5 kg

Si la balanza se equilibra en la Tierra con pesas por valorde 5 kg, la masa del cuerpo es de 5 kg. La balanza estáequilibrada porque el peso en los dos platillos es el mis-mo: es el producto de la gravedad en la superficie terres-tre por la masa en los platillos. Como la gravedad es lamisma en los lados de la balanza, ésta se equilibra conmasas iguales.

En la Luna, lo único que cambia es la intensidad delcampo gravitatorio o gravedad en la superficie. Comoen los dos platillos la gravedad que actúa es la misma,la balanza se equilibrará también con masas iguales.Por tanto, en la Luna necesitaremos 5 kg de pesas.

7. Datos: m = 1 000 kg; T = 2 días = 48 h = 1,73 ⋅ 105 s

MT = 5,98 ⋅ 1024 kg; RT = 6,37 ⋅ 106 m;

x a t g t m s s m= = = ⋅ ⋅ =12

12

12

1 65 3 7 42 2 2 2, ( ) ,/

gG M

R

gN m

kg

kg

L

L

=

= ⋅⋅

⋅⋅−,

,

(

2

112

2

22

6 67 107 47 10

11 74 101 656 2

2

, ),

⋅=

mm s/

GN m

kg= ⋅

⋅−6 67 10 112

2,

g gg

hR

hR

h R

T

T

T

= =

+⎛

⎝⎜

⎠⎟

+ =

=

0 85

1

11

0 85

10

00

2, ;

,

,,,

,851 6 37 10

10 85

16−⎛

⎝⎜⎜

⎠⎟⎟ = ⋅ ⋅ −

⎝⎜⎜

⎠⎟⎟

=

m

h 55 4 105, ⋅ m

Page 54: 9523 gl fis_tx2_cas

57

Movimientos vibratorios4

6. Datos: MAS con 15 vibraciones cada 40 segundos.

a) La frecuencia es el número de vibraciones por se-gundo. Por tanto:

b) Calculamos el período a partir de la frecuencia:

c) Determinamos la pulsación:

7. Datos: MAS; ϕ0 = 0; f = 50 Hz; A = 3 cm = 0,03 m

a) Calculamos el período como el inverso de la fre-cuencia:

b) Determinamos la pulsación a partir de la frecuencia:

c) Escribimos la ecuación de la elongación del movi-miento armónico simple:

8. Datos: MAS; ϕ0 = π/4; f = 60 Hz; A = 2 m

a) Calculamos el período como el inverso de la fre-cuencia:

b) Determinamos la pulsación a partir de la frecuencia:

c) Escribimos la ecuación de la elongación:

9. Datos: MAS; A = 0,05 m; T = 4 s; t0 = 0; x0 = 0; v0 > 0

a) Si la partícula se encuentra en el origen en eltiempo inicial, su fase inicial es cero:

b) Determinamos la pulsación a partir del período:

ωπ

Τ

π

= = =2 2

2srad

s

ϕ0 = 0

x A sen t sen t= + = +⎛⎝⎜

⎞⎠⎟

( )ω ϕ ππ

0 2 1204

ω π π π= = ⋅ =2 2 60 120f Hz rad s/

Tf Hz

s= = =1 1

600 017,

x A sen t sen t= + =( ) , ( )ω ϕ π0 0 03 100

ω π π π= = ⋅ =2 2 50 100f Hz rad s/

Tf Hz

s= = =1 1

500 02,

ω π π= = ⋅ =2 2 0 375 2 36, ,f Hz rad s/

Tf Hz

s= = =1 1

0 3752 67

,,

fvibraciones

sHz= =

1540

0 375,

1. MOVIMIENTO VIBRATORIO ARMÓNICO SIMPLE

1. En un movimiento periódico las variables posición, ve-locidad y aceleración de la partícula o del cuerpo to-man los mismos valores después de cada intervalo detiempo denominado período.

En un movimiento oscilatorio la partícula se desplazasucesivamente a un lado y a otro de la posición de equi-librio, repitiendo a intervalos regulares de tiempo losvalores de sus variables cinemáticas.

El movimiento armónico simple es el movimiento osci-latorio sobre una recta, de un cuerpo sometido a unafuerza de atracción proporcional a la distancia al pun-to de equilibrio o centro de oscilaciones, y de sentidoopuesto al vector posición del cuerpo respecto a dichopunto.

2. Un movimiento periódico no tiene por qué ser oscila-torio. Pueden repetirse los valores de las magnitudes ci-nemáticas cada cierto intervalo de tiempo sin que elcuerpo se desplace a un lado y a otro de un punto deequilibrio. Cualquier movimiento circular uniforme esperiódico sin ser oscilatorio. Por ejemplo, el movimien-to de los planetas en torno al Sol.

3. No todos los movimientos oscilatorios son armónicos.Son oscilatorios todos los movimientos periódicos deun lado a otro de un punto de equilibrio, pero no tie-nen por qué tener lugar a lo largo de una recta ni estarcausados por una fuerza de atracción proporcional a ladistancia al punto de equilibrio.

4. Las oscilaciones de los extremos del diapasón se deno-minan vibraciones porque son muy rápidas. Su perío-do es muy corto, y el valor exacto del período determi-na el tono del sonido que emite el diapasón.

5. a) Falso. La elongación no es el valor máximo de laamplitud, sino que la amplitud es el valor máximode la elongación.

b) Falso. x = ± A cuando sen (ωt + ϕ0) = 1

c) Cierto, siempre que utilicemos la ecuación delMAS en la forma x = A sen (ωt + ϕ0)

d) Cierto.

e) Falso. La aceleración es nula cuando x = 0, y es má-xima para x = ± A.

f) Falso. La partícula se halla en el centro de oscila-ción cuando x = 0 y a = 0, pero en este punto la ve-locidad toma su valor máximo.

4 Movimientos vibratorios

Page 55: 9523 gl fis_tx2_cas

58

Para t = 10 s:

x (t = 10 s) = 0,05 sen (40 π) = 0 m

v (t = 10 s) = 0,2 π cos (40 π) = 0,2 π m/s

a (t = 10 s) = −0,8 π2 sen (40 π) = 0 m/s2

b) El cuerpo se encuentra en el origen de coordena-das, que es el punto de equilibrio o centro de osci-lación del movimiento. En este punto, la velocidaddel cuerpo es máxima y la aceleración es nula.

13. Datos: MAS; A = 0,2 m; T = 4 s; ϕ0 = π/3

Calculamos la pulsación del movimiento a partir delperíodo:

Escribimos las ecuaciones para la elongación, la velo-cidad y la aceleración del movimiento:

14. Datos: MAS; f = 50 Hz; x = −0,001 m

Determinamos la pulsación:

Hallamos la aceleración a partir de su relación con laelongación:

15. En el MAS sólo existe una componente de la acelera-ción, ya que es un movimiento unidimensional, sobreuna recta. La aceleración es siempre tangente a la velo-cidad, ya que son paralelas y, por lo tanto, no existe ace-leración normal.

16. a) La coordenada x tiene el mismo valor para la par-tícula que describe el MCU que para su proyec-ción. Lo que diferencia el MCU del MAS es que enel primero varía, además de la coordenada x, lacoordenada y.

b) No, las dos partículas no tienen siempre la mismavelocidad. La del MAS tiene una velocidad igual a laproyección en el eje horizontal de la velocidad delMCU. Tampoco tienen la misma aceleración, puesen el MCU la aceleración es normal, mientrasque en el MAS es la proyección en el eje x de la ace-leración anterior.

17. Datos: MCU; R = 0,20 m; T = 2 s; x0 = 0; ϕ0 = π/2

El movimiento de la proyección sobre el eje de absci-sas será un MAS.

a rad/

/

= − = − ⋅ −

=

ω π

π

2 2

2 2

100 0 001

10

x s m

a m s

( ) ( , )

ω π π π= = ⋅ =2 2 50 100f rad/sHz

x t

v

= + = +⎛

⎝⎜

⎠⎟

=

A sen sen t

A

( ) ,

co

ω ϕπ π

ω

0 0 22 3

ss ( ) , cosω ϕ ππ π

ω

0t

a

+ = +⎛

⎝⎜

⎠⎟

= −

0 12 3

2

t

A sen (( ) ,ω ϕ ππ π

0t + = − +⎛

⎝⎜

⎠⎟0 05

2 32 sen t

ωπ π π

= = =2 2

4 2T srad

s

a t= − + = − ⋅

= −

A sen sen t

a

ω ω ϕ π π02 20 05 4 4( ) , ( ) ( )

00 8 42, ( )π πsen t

c) Escribimos la ecuación de la elongación:

d) Calculamos el valor de la elongación en t = 1 s:

10. Datos: MAS; A = 0,03 m; f = 150 Hz; x0 = A; t0= 0

Calculamos la pulsación a partir de la frecuencia:

Escribimos la ecuación del MAS, dejando la fase inicialpor determinar:

Determinamos la fase inicial a partir de los datos delproblema:

Entonces:

11. Datos: MAS; A = 3 cm = 0,03 m; f = 5 Hz; ϕ0 = 3π/2

a) Calculamos la pulsación, para escribir la ecuaciónde la elongación:

b) Escribimos la ecuación de la velocidad:

c) La ecuación de la aceleración es:

12. Datos: MAS; T = 0,5 s; A = 0,05 m; ϕ0 = 0

a) Determinamos la pulsación y escribimos las ecuacio-nes de la elongación, la velocidad y la aceleración:

ωπ π

4 π

ω ϕ0

= = =

= + =

2 20 5

0 05T s

rads

A sen se,

( ) ,x t nn t

A t

v

( )

cos ( ) , cos ( )

4

0 05 4 4

π

ω ω ϕ π π0v t= + = ⋅

== 0 2 4, cos ( )π πt

a A sen

a sen t

= − +

= − ⋅ +

ω ω ϕ

π) π

02

20 03 10 103

( )

, (

t

ππ

π ππ

2

3 1032

⎛⎝⎜

⎞⎠⎟

= − +⎛⎝⎜

⎞⎠⎟

a sen t

v A t= + = ⋅ +⎛⎝⎜

⎞⎠

ω ω ϕ π ππ

0cos ( ) , cost 0 03 10 1032 ⎟⎟

= +⎛⎝⎜

⎞⎠⎟

v t0 3 1032

, cosπ ππ

ω π π π

ω ϕ0

= = ⋅ =

= + =

2 2 5 10

0 0

f rad/s

t( ) ,

Hz

x A sen 33 102

sen tππ

+⎛⎝⎜

⎞⎠⎟

x sen t= +⎛⎝⎜

⎞⎠⎟

0 03 3002

, ππ

x A sen sen0 0 03 0 03 1

2

= = = =

=

, , ; ) ;ϕ (ϕ

ϕπ

0 0

0

x A sen t sen t= + = +( ) , ( )ω ϕ π ϕ0 00 03 300

ω π π π= = ⋅ =2 2 150 300f Hz rad s/

x sen m= ⋅⎛⎝⎜

⎞⎠⎟

=0 052

1 0 05, ,π

x A sen t sen t= + =⎛⎝⎜

⎞⎠⎟

( ) ,ω ϕπ

0 0 052

Page 56: 9523 gl fis_tx2_cas

59

20. Datos: m = 200 g = 0,2 kg; K = 25 N/m

a) La amplitud será igual a la distancia desde el pun-to del que soltamos el cuerpo hasta el punto deequilibrio. El punto de equilibrio para el sistemaformado por el resorte y la masa se encuentra, res-pecto a la longitud natural del resorte, a una dis-tancia tal que el peso del cuerpo y la fuerza del re-sorte son iguales:

b) Determinamos el período del movimiento, que esindependiente de la amplitud:

El período del movimiento es el mismo que en elcaso en que el resorte está horizontal. La presen-cia de la gravedad no altera el período.

21. Datos: m = 2,0 kg; Fmáx = 8,0 N; A = 20 cm = 0,2 m

a) Hallamos la constante elástica a partir de la fuerzaque realiza el resorte cuando la elongación es má-xima:

b) Calculamos el período del movimiento:

22. Datos: m = 50 g = 0,05 kg; T = 1,5 s

Hallamos la constante recuperadora del resorte a par-tir de la expresión del período:

23. Datos: m = 2 kg; K = 65 N/m; A = 0,3 m

a) Inicialmente el cuerpo está en reposo. Por tanto,su energía potencial inicial coincide con su ener-gía mecánica:

b) La velocidad máxima se alcanzará cuando la ener-gía potencial sea nula. Entonces, toda la energíamecánica es energía cinética, Ec = E:

Ep E K ANm

m J= = = ⋅ ⋅ =12

12

65 0 3 2 9252 2( , ) ,

TmK

TmK

Km

T

Kkg

= = =

=

2 4 4

40 05

1 5

2 2 22

2

π π π

π

; ;

,

( , ss

Nm)

,2 0 88=

TmK

kgNm

s= = =2 22

401 4π π ,

KA

Nm

Nm

= = =Fmáx 8 0

0 240

,,

TmK

TkgNm

s= = =2 20 2

250 56π π;

,,

mg Kx xm gK

xkg

Nkg

Nm

m A

= =

=

= =

;

, ,, ; ,

0 2 9 8

250 08 0 008 m

La amplitud coincidirá con el radio de la circunferen-cia, A = R = 0,20 m.

El período será el mismo que el del MCU, T = 2 s.Entonces, la pulsación es:

Teniendo en cuenta que en el instante inicial la proyec-ción de la posición de la partícula coincide con el ori-gen, si escribimos la ecuación en seno, la fase inicialserá nula:

Podemos también escribir la ecuación en función delcoseno, y la fase inicial será entonces ϕ0 = π/2:

2. OSCILADOR ARMÓNICO SIMPLE

18. a) En el oscilador armónico simple, el valor de lafuerza recuperadora es F = −Kx. Por lo tanto, dela 2ª ley de Newton tenemos que:

— En x = 0, la aceleración es nula.

— En x = A, el valor de la aceleración es con di-

rección y sentido hacia la posición de equilibrio.

— En x = −A, el valor de la aceleración es tam-

bién con dirección y sentido hacia la posición deequilibrio.

b) La aceleración es máxima para x = ± A.

19. Determinamos cómo varía la pulsación o frecuenciaangular si duplicamos la masa del cuerpo:

Hallamos la variación de la frecuencia y del perío-do:

La velocidad máxima será:

Y la aceleración máxima:

a amáx 0 máx= ± = ±⎛

⎝⎜⎞

⎠⎟= ± =A Aω ω Αω0

22

21

2

12

120

v vmáx 0 máx= ± = ± =A Aωω0

2

1

2

f = = ⋅ =

= = =

ωπ

ω

π

π

ω

π

ω

0

0

2 21

2

1

22 2

2 2

0

0

f

T T

ω ω ω ωω

0 002

0

2

0

2

212 2

= = = = =K

mKm

Km

; ;

Km

A

−Km

A

F m a Kx aKm

x= = − = −;

x A t t= + = +⎛⎝⎜

⎞⎠⎟

cos ( ) , cosω ϕ ππ

0 0 22

x A sen t sen t= + =( ) , ( )ω ϕ π0 0 2

ωπ

Τ

ππ= = =

2 22 s

rads

4. Movimientos vibratorios

Page 57: 9523 gl fis_tx2_cas

60

ca será la energía mecánica menos la potencial.Determinamos la energía potencial del resorte enx = 2 cm = 0,02 m:

Entonces, la energía cinética en este punto es:

d) Determinamos la velocidad en este punto a partirde la energía cinética:

26. Si tenemos un reloj de péndulo que adelanta, hemosde aumentar la longitud del péndulo. De esta forma,el período de oscilación será más largo y las manecillasdel reloj avanzarán más lentamente.

— Si un péndulo simple tiene un período de T = 2 scon L = 1 m, otro con T = 5 s tendrá una longitudmayor, ya que el período es directamente propor-cional a la raíz cuadrada de la longitud del pén-dulo. Concretamente, la longitud del segundopéndulo será de 6,25 m. Para encontrarla, despe-jamos el valor de la gravedad en el lugar del expe-rimento a partir de la longitud y el período delprimer péndulo:

Utilizamos este valor de g para hallar la longituddel segundo péndulo:

27. a) Datos: L = 0,556 m; g = 9,75 m/s2

Determinamos el período del péndulo en este lu-gar:

b) Datos: gL = 1,96 m/s2; TT = 2 s; gT = 9,8 m/s2

Con los datos del péndulo en la Tierra, determina-mos su longitud:

Conociendo la longitud, hallamos el período en laLuna:

TL

gT

mm

s

sLL

L= = =2 20 99

1 964 47

2

π π;,

,,

TL

gL

g T

L

m

ss

TT

T T= =

=⋅

=

24

9 8 2

40 99

2

2

22

2

ππ

π

;

, ( ), mm

TLg

Tm

m

s

s= = =2 20 556

9 751 5

2

π π;,

,,

Lg T

L m= =; ,2

246 25

π

TLg

gL

Tg

m

s= = =2 4 9 872

2 2π π; ; ,

Ec m v vEcm

vJ

kgms

= = =⋅

=12

2 2 0 0490 5

0 4432; ;,

,,

Ec E Ep Ec J J J= − = − =; , , ,0 088 0 039 0 0049

Ep K x EpNm

m J= = ⋅ ⋅ =12

12

196 0 02 0 0392 2; ( , ) ,

24. Datos: m = 1,5 kg; K = 1,5 N/m; vmáx = ± 3 m/s

a) Por la conservación de la energía mecánica, laenergía del bloque parado es igual a su energíaen cualquier otro instante de tiempo. Cuando lavelocidad es máxima, la energía potencial es ceroy la energía mecánica coincide con la energía ci-nética.

b) Despejamos la amplitud de la expresión de la ener-gía mecánica:

c) Calculamos la pulsación y determinamos la acele-ración máxima:

25. Datos: x1 = 5 cm = 0,05 m; m1 = 1,0 kg;

m2 = 500 g = 0,5 kg; A = 3 cm = 0,03 m

a) Calculamos la constante de recuperación del resor-te a partir de los datos para la primera masa que col-gamos. La posición de equilibrio x1 es aquélla parala cual el peso del cuerpo que colgamos y la fuerzadel muelle son iguales y de sentido contrario:

b) Calculamos la energía potencial del resorte en elpunto de máxima deformación (x = A), dondetoda la energía será energía potencial:

c) La energía mecánica se conserva, por lo que esigual a la energía potencial en el punto de máximadeformación, donde el cuerpo está en reposo. Enotra posición, como en x = 2 cm, la energía cinéti-

Ep E K A

ENm

m J

= =

= ⋅ ⋅ =

12

12

196 0 03 0 088

2

2( , ) ,

m g Kx Km g

x

Kkg

Nkg

mN

1 11

1

1 9 8

0 05196

; ;

,

,

= =

=

=mm

ω

ω

= = =

= ± = ±

Km

Nmkg

rads

a A arad

s

1 5

1 51

12

,

,

;máx máx⎛⎛⎝⎜

⎞⎠⎟

⋅ = ±2

23 3mm

s

E K A AE

KA

JNm

m= = =⋅

=12

2 2 6 75

1 532; ;

,

,

E Ec m v kgms

= = = ⋅ ⋅ ±⎛⎝⎜

⎞⎠⎟

=máx máx12

12

1 5 3 622

, ,775 J

E Ec m v vE

m

vJ

kgms

= = = ±

= ±⋅

= ±

12

2

2 2 9252

1 7

2;

,,

Page 58: 9523 gl fis_tx2_cas

61

Otro ejemplo de resonancia poco visible pero muy útiles el microondas. En este caso, los osciladores son lasmoléculas de agua que todos los alimentos contienen.Como la temperatura de un material es consecuenciade las vibraciones de sus átomos y moléculas, si conse-guimos hacer vibrar con mayor amplitud las moléculasde agua de los alimentos, conseguiremos que su tem-peratura aumente. Para hacer oscilar la molécula deagua, el microondas emite radiación electromagnéticade una frecuencia igual o parecida a la frecuencia pro-pia de la molécula de agua, la cual entra en resonan-cia y vibra cada vez con más amplitud.

En la industria y en la construcción es necesario preve-nir el fenómeno de resonancia y tomar medidas paraevitarlo. Por ejemplo, en los puertos, la distancia entrelos diques no debe ser un múltiplo de la longitud de on-da de las olas de los temporales más frecuentes en esacosta. Si lo fuera, las oscilaciones del agua dentro delpuerto entrarían en resonancia con las olas del tempo-ral y su altura iría en aumento. Otro ejemplo es la cons-trucción de grandes edificios. Algunos rascacielos dispo-nen de un sistema amortiguador para reducir las oscila-ciones cuando soplan fuertes vientos. El sistema oscilacon la misma frecuencia que el edificio pero con un des-fase de 180°, de modo que amortigua las oscilaciones.

RESOLUCIÓN DE EJERCICIOS Y PROBLEMAS

30. Datos: x (t = 5 s) = 3,36 m; v (t = 5 s) = 0,216 m/s;

ω = 0,1 rad/s

a) Determinamos la frecuencia a partir de la pulsa-ción:

b) Determinamos la amplitud a partir de la relaciónentre la elongación y la velocidad para t = 5 s:

c) Escribimos la ecuación de la elongación en t = 5 s,cuando la velocidad es positiva, para determinar lafase inicial:

Como la velocidad, que varía con el coseno del án-gulo de fase, es positiva:

( , ) ; ( , ) ,0 5 1 1 0 5 0 5+ = = − =ϕ ϕ0 0rad rad rad

senm

marcsen

( , ),

,

, ,

0 53 36

40 84

0 5 0

+ = =

+ =

ϕ

ϕ

0

0 884 1 1= −( )rad o radπ

x A sen

sen

= +

= ⋅ +

( )

, ( , )

ω ϕ

ϕ0

0

t

3 36 4 0 1 5

v = ± − = − = +

= +

ω ωω

ω

A x v A x Av

x

Av

x

2 2 2 2 2 2 22

22

2

22

; ( );

; AA

ms

rads

m=

⎛⎝⎜

⎞⎠⎟

⎛⎝⎜

⎞⎠⎟

+ =0 216

0 1

3 36 4

2

22

,

,

( , ) m

ω πωπ

0,1

π π= = = =2

2 20 05

f; f

rads f Hz;

,

3. OTROS MOVIMIENTOS OSCILATORIOS

28. La frecuencia de un oscilador amortiguado permanececonstante, no disminuye. Lo único que disminuye es laamplitud. La frecuencia no puede variar porque es unamagnitud propia del oscilador, esté o no amortiguado.El amortiguamiento depende de las características deloscilador y del agente amortiguador, y de la velocidaddel oscilador en cada momento.

29. Un oscilador entra en resonancia cuando actúa sobreél una fuerza externa periódica de frecuencia igual a lafrecuencia propia del oscilador. Si además de esta fuer-za periódica existe otra fuerza amortiguadora que disi-pe más energía que la suministrada por la fuerza perió-dica, las oscilaciones serán amortiguadas. En cambio, sila energía disipada es inferior a la suministrada por lafuerza periódica, la amplitud de las oscilaciones, en vezde amortiguarse, aumentará.

FÍSICA Y SOCIEDAD

a) Resonancia, osciladores forzados y osciladores amorti-guados.

Un oscilador sobre el que actúe una fuerza disipativarealizará oscilaciones amortiguadas. En cada ciclo, eloscilador irá perdiendo energía, y por ello la amplituddel movimiento irá disminuyendo. El movimiento seráperiódico, con la misma frecuencia natural del oscila-dor, pero, a diferencia del MAS, la amplitud no es cons-tante, sino que decrece.

Si además de la fuerza disipativa existe alguna otrafuerza externa que proporcione energía al oscilador,hablamos de oscilaciones forzadas. En caso de que laenergía suministrada compense exactamente la que eloscilador pierde a causa de la amortiguación, el movi-miento tiene el mismo período natural y amplitudconstante, como el MAS.

Una manera de introducir esta energía es mediante unafuerza periódica. La absorción de energía por parte deloscilador será máxima cuando esta fuerza tenga un pe-ríodo igual o casi igual al período natural del oscilador.En este caso, no sólo se mantendrán las oscilaciones sindisminuir su amplitud, sino que la amplitud del movi-miento irá en aumento, llegando incluso a sobrepasar loslímites de resistencia de la estructura del oscilador. Se tra-ta de un fenómeno de resonancia. El período del movi-miento es el natural del sistema, pero en este caso la am-plitud tampoco es constante, sino que aumenta.

b) Tres ejemplos de resonancia:

Cuando nos impulsamos en un columpio, estamos for-zando las oscilaciones. Si nos impulsamos en el mo-mento adecuado en cada ciclo (es decir, con la fre-cuencia natural del columpio), la amplitud del movi-miento va creciendo.

La mayoría de los instrumentos musicales tiene lo quese llama una caja de resonancia. Así, por ejemplo, laforma de una guitarra o de un violín es la adecuada pa-ra que el aire de su interior entre en resonancia con lasnotas producidas por la vibración de las cuerdas. De es-ta forma, se amplifica la intensidad del sonido.

4. Movimientos vibratorios

Page 59: 9523 gl fis_tx2_cas

62

c) Utilizamos la relación entre la velocidad y la elon-gación para determinar la velocidad de la par-tícula cuando su elongación es de x = 1,2 mm == 1,2 ⋅ 10−3 m:

d) Determinamos la fase inicial a partir de la veloci-dad para t = 2 s:

Escribimos las ecuaciones de la elongación, la ve-locidad y la aceleración:

32. Datos: m = 0,6 kg; K = 10 N/m; A = 5 cm = 0,05 m

Hay que tener en cuenta que se trata de un MAS deenergía constante, ya que no existe rozamiento.

a) Calculamos la energía total del sistema:

b) El cuerpo tendrá velocidad máxima cuando todasu energía sea cinética, lo que sucede en el puntode equilibrio:

c) Determinamos la energía potencial del cuerpo en x = 2 cm = 0,02 m:

La energía cinética es la energía total menos la po-tencial:

33. Datos: m = 0,5 kg; A = 10 cm = 0,1 m; T = 2 s

a) Determinamos la energía total del sistema, quecoincidirá con la energía potencial máxima enlos extremos de la trayectoria, y con la cinéticamáxima en el punto de equilibrio. Para ello, cal-

E Ep Ec

Ec E Ep Ec J J J

= +

= − = − ⋅ =−; , ,0 012 2 10 0 013

Ep K x EpNm

m J= = ⋅ = ⋅ −12

12

10 0 02 2 102 2 3; ( , )

Ec E m v vE

m

vJ

máx máx máx

máx

= = = ±

= ±⋅

12

2

2 0 0120

2;

,,,

,6

0 2kg

ms

= ±

E K A ENm

m J= = ⋅ =12

12

10 0 05 0 0122 2; ( , ) ,

x A sen sen t

v A

= + = ⋅ ⋅ − ⋅

=

( ) ( )ω ϕ0t 2 10 2 10 4 103 3 3

ωω ω ϕ

ω

0cos t

s

( ) cos ( )+ = ⋅ − ⋅

= −

4 2 10 4 103 3

2

t

a A een t( ) ( )ω ϕ0+ = − ⋅ ⋅ − ⋅8 10 2 10 4 103 3 3sen t

v A= +

= ⋅ ⋅ ⋅ ⋅ ⋅ +

ω ω ϕ0cos ( t )

cos (4 2 10 2 10 2 10 23 3 3 )

cos ( ); cos ( )

(

ϕ

ϕ ϕ0 0

0

4 4 4 10 4 10 13 3= ⋅ + ⋅ + =

44 10 0 4 103 3⋅ + = = − ⋅) ;ϕ ϕ0 0 rad

v A x

v rad s m

= ± −

= ± ⋅ ⋅ − ⋅ −

ω 2 2

3 3 2 32 10 2 10 1 2 10( ) ( ,/ )

,

m

v m s

2

3 2= ± /

d) La aceleración en t = 5 s será:

e) Determinamos la elongación, la velocidad y la ace-leración para t = 0 s:

f) Las expresiones de la elongación, la velocidad y laaceleración en función del tiempo son:

g) Para representar la elongación en función deltiempo, hallamos varios puntos y los representa-mos. Escogemos los puntos de elongación máxima,mínima y cero:

31. Datos: m = 1 g = 0,001 kg; f = 103/π Hz;

amáx = ± 8,0· 103 m/s2

a) Determinamos la pulsación a partir de la frecuencia:

b) Hallamos la amplitud a partir de la aceleración enel extremo del recorrido, punto donde la elonga-ción coincide con la amplitud del movimiento:

� �� �

a A Aa

Am s

máxmáx /

= = =⋅

⋅ω

ω2

2; ;,

(

8 0 10

2 10

3 2

3 )rad s

A m mm

/ 2

32 10 2= ⋅ =−

ω π ππ

= = ⋅ = ⋅2 210

2 103

3f /Hz rad s

t(s) x(m)

0 1,9

10,7 A = 4

26,4 0

42,1 −A = −4

57,8 0

60 0,86

x A sen sen t

v A

= + = +

=

( ) ( , , )

(

ω ϕ

ω ω0t

cos

4 0 1 0 5

tt

sen t

+ = +

= − +

ϕ

ω ω ϕ

0

0

) , cos ( , , )

( )

0 4 0 1 0 52

t

a A == − +0 04 0 1 0 5, cos ( , , )t

x A sen sen m

v A

= + = ⋅ + =

=

( ) ( , , ) ,ω ϕ

ω0t 4 0 1 0 0 5 1 9

cos t( ) , cos ( , , )

,

ω ϕ0+ = ⋅ ⋅ +

=

4 0 1 0 1 0 0 5

0 35v m//

/ /

s

a x rad s m m s= − = − ⋅ = −ω2 2 20 1 1 9 0 02( , ) , ,

a A sen x

a rad s

= − + = −

= − ⋅

ω ω ϕ ω02 2

20 1 3 36

( )

( , ) ,

t

/ ,m m s= −0 03 2/

x (m)

t(s)

4

2

15 30 45 60

–2

–4

Page 60: 9523 gl fis_tx2_cas

63

c) La fuerza que tiende a llevarlo a la posición deequilibrio es máxima en los puntos de máximaelongación:

EJERCICIOS Y PROBLEMAS

36. a) La aceleración de un oscilador armónico, en elpunto donde la velocidad es máxima, es nula (es-to ocurre en el punto de equilibrio, de elongaciónnula).

b) Cuando la elongación es máxima, la aceleraciónes también máxima, mientras que la velocidad esnula.

37. x = A sen (ωt + ϕ0)

a) Si el movimiento comienza en el centro de la osci-lación, x = 0 para t = 0. Por tanto:

b) Si el movimiento empieza en el punto extremo delas elongaciones positivas:

c) En el caso que comience en el extremo de laselongaciones negativas:

38. Una partícula que describe un MAS en un períodorecorre una distancia igual al doble de la amplitud,x = 2A.

— Si sabemos que en un instante la velocidad de unapartícula en una MAS es nula, podemos decir quela partícula está en uno de los puntos extremos,pero no podemos saber si está en x = A o en x = −A.Tampoco podemos conocer el sentido de su des-plazamiento.

39. La proyección sobre un diámetro de un MCU co-rresponde a la elongación de un MAS. El radio delMCU es igual a la amplitud del MAS. La frecuenciay el período del MCU y del MAS son los mismos. Lavelocidad del MAS corresponde a la proyección dela velocidad lineal del MCU, y la aceleración delMAS es la proyección de la aceleración normaldel MCU.

− = = − =A A sen sen; ;ϕ ϕ ϕ3π

0 0 012

A A sen sen= = =; ;ϕ ϕ ϕπ

0 0 012

0 0 0= = =A sen sen ó; ;ϕ ϕ ϕ π0 0 0

� �F Kx F KAm g

LA

Fkg

m

= = ± =

= ±⋅

;

, ,

máx

máx

0 005 9 79ss

mm N

2

0 2480 078 0 015

,, ,⋅ = ±

Α α

ωπ

Τ

π

1

= = ⋅ =

= = =

L rad m m

s

0 314 0 248 0 078

2 22

, , ,

ππ

ω π

rads

v A m s m smáx rad/ /= ± = ± ⋅ = ±0 078 2 0 49, ,

culamos primero la pulsación a partir del pe-ríodo:

b) La velocidad del cuerpo será máxima cuando seamáxima su energía cinética. Por tanto:

c) En este sistema, la energía sólo depende de las ca-racterísticas del muelle (K) y del movimiento (A).Es independiente de la masa del cuerpo. Por tanto,con un cuerpo de m = 2 kg tendríamos los mismosresultados.

34. Datos: m = 0,5 kg; L = 1 m; α = 8° = 0,14 rad; g = 9,8 m/s2

a) Hallamos la pulsación y la amplitud para determi-nar la energía potencial máxima, Ep:

b) Calculamos la velocidad máxima a partir de laenergía cinética máxima:

35. Datos: L = 0,248 m; T = 1 s; α = 18° = 0,314 rad;

m = 5 g = 0,005 kg

a) Determinamos g en ese punto a partir del períododel movimiento:

b) Hallamos la amplitud y la pulsación del movimien-to para calcular la velocidad máxima:

TLg

gL

T

gm

sg m s

= =

= =

2 4

40 248

19 79

22

22

π π

π

;

,

( ); , / 22

Ec m v Ec Ep vEcm

v

= = = ±

= ±⋅

12

2

2

2; ;máx máx máxmáx

máx00 05

0 50 45

,,

,J

kgm s= ± /

ω ω

α

= = =

= = ⋅

Lg

m

sm

rad s

A rad

;,

,

,

9 8

13 1

0 14 1

2/

L mm m

Ep E K A m A

Ep

=

= = =

=

0 14

12

12

12

2 2 2

,

máx

máx

ω

⋅⋅ ⋅⎛⎝⎜

⎞⎠⎟

⋅ =0 5 0 14 0 052

2, ( , ) ,kgrad

sm J3,1

Ec m v vEm

vJ

kg

= = ±

= ±⋅

=

12

2

2 0 020 5

2;

,,

máxmáx

máx ±±0 3, m s/

ωπ

Τω

ππ= = =

= = = =

2 22

12

12

2

;s

rads

Ec Ep E K Amáx máx mm A

E kgrad

sm

Ec

, ( , )

ω

π

2 2

221

20 5 0 1= ⋅ ⋅

⎛⎝⎜

⎞⎠⎟

mmáx máx= = =Ep E J0 02,

4. Movimientos vibratorios

Page 61: 9523 gl fis_tx2_cas

64

Como en t = 0 es v > 0, tenemos:

v = A cos ϕ0 > 0

Por lo tanto, ϕ0 = 0:

46. Datos: MAS; x = 0,20 sen (10t + π/2), unidades SI

De la expresión de la elongación deducimos que A = 0,20 m, ω = 10 rad/s y ϕ0 = π/2.

Determinamos la velocidad máxima a partir de laecuación de la velocidad:

— Cuando la velocidad es máxima, sabemos que lapartícula está exactamente en x = 0. Sin más infor-mación, no podemos saber el sentido del movi-miento.

47. a) Datos: MAS; a = −90 m/s2 cuando x = 0,10 m

Determinamos la pulsación a partir de la relaciónentre la aceleración y la elongación:

Hallamos el período:

b) Datos: MAS; x = −0,01 m; f = 5 Hz

Calculamos la pulsación y relacionamos la acelera-ción con la elongación:

c) Datos: MAS; a = −2x; A = 0,01 m

De la relación entre a y x hallamos la pulsación yel período:

Con ello, podemos escribir la ecuación de la elonga-ción, excepto la fase inicial, que supondremos nula:

48. Datos: MAS; a = −16 π2 x; A = 0,04 m

A partir de la elongación máxima, encontramos el va-lor de la aceleración máxima:

a Arad

sm

m

smáx = ± = ± ⋅ = ±ω π π2 22

22

216 0 04 0 64, ,

x A sen t

x sen t

= +

= ( )( )

,

ω ϕ0

0 01 2

a x x rad s

Trad s

s

= − = − =

= = =

ω2 /

/

2 2

2 2

22

; ω

π

ω

ππ

ω π π

ω

= = ⋅ =

= − = −

2 2 5 10

10

f /2

;

; (

Hz rad s

a x a r

ω π

π aad s m m s/ /) ( , )2 2 20 01⋅ − = π

ωπ

ΤΤ

π π= = = =

2 230

0 21; ,ω

2/rad s

s

a xa

x

m

sm

rads

= − =−

=− −

⎝⎜

⎠⎟

=ω ω ω22

90

0 1030; ;

,

v A v rad s m v

vmáx máx máx

máx

/= ± = ± ⋅

= ±

ω ; , ;10 0 20

2 m s/

x sen t= 0 05 300, ( )π

40. La fuerza que aplicamos para estirar el extremo de unmuelle debe ser igual pero de signo opuesto a la fuerzaelástica del resorte en el punto en que lo soltamos. No esla fuerza elástica, porque la aplicamos nosotros y no elmuelle, pero debe ser igual en módulo y de sentidoopuesto.

41. El período de oscilación de un resorte depende de la ma-sa colgada en el extremo y de la constante del muelle.Por tanto, si cambiamos la masa, cambiará el período.

— En particular, si duplicamos la masa, m’ = 2 m:

42. Como la energía total del sistema es la cinética más la po-tencial, los puntos de la trayectoria de un MAS en los quela energía potencial y la cinética sean iguales verifi-

carán que

43. Como el período de un péndulo es inversamente pro-porcional a la raíz cuadrada de la gravedad, en la Luna,donde la gravedad es menor, el péndulo oscilará máslentamente que en la Tierra; es decir, el período serámás largo. En cambio, en Júpiter, donde la gravedad esmucho mayor, el período del péndulo será mucho máspequeño y oscilará más rápido.

44. Un sistema real, con fuerzas de rozamiento, sólo podrátener un MAS si existe algún dispositivo que fuerce lasoscilaciones, proporcionando al sistema justamente laenergía que disipa la fuerza de rozamiento.

45. Datos: MAS; f = 150 Hz; A = 5 cm = 0,05 m

a) Hallamos el período a partir de la frecuencia:

b) Calculamos la pulsación:

c) Escribimos la ecuación de la elongación:

Si en t = 0 la elongación es x = 0, podemos deter-minar la fase inicial:

0 0 05 300 0 0 00 0 0= ⋅ + = =, ( ); ;sen sen óπ ϕ ϕ ϕ π

x A sen t sen t= + = +( ) , ( )ω ϕ π ϕ0 00 05 300

ω π ω π ω π= = ⋅2 2 150 300; ;f Hz rad s/

Tf

THz

s= = = ⋅ −1 1150

6 7 10 3; ,

Ep K x E K A K A

K

= = =⎛⎝⎜

⎞⎠⎟

=12

12

12

12

14

12

2 2 2

; ;x K A x A x A2 2 2 214

12

1

2= = = ±

Ep Ec E= =12

:

TmK

TmK

mK

mK

T T

=

= = = ⋅ =

2

2 22

2 2 2

π

′ π′

π π ′;

P van

avx

Rωt + ϕ0

x = R cos (ωt + ϕ0) = A sen (ωt + ϕ’0)

ωt + ϕ0

Page 62: 9523 gl fis_tx2_cas

65

b) Calculamos la posición, la velocidad y la acelera-ción para t = 1 s:

53. Datos: MAS; t1 = 0,75 s, x1 = 2 m; t2 = 3,75 s; v2 = 0 m/s;

T = 6 s

a) Determinamos primero la pulsación a partir delperíodo, para escribir la ecuación de la elonga-ción para t1 y la ecuación de la velocidad para t2:

A partir de la segunda ecuación determinamosparcialmente la fase inicial:

Como la elongación en t1 es positiva:

sen (0,25 π + ϕ0) > 0 y ϕ0 = −1,75 π rad

Hallamos ahora la amplitud despejándola de laecuación de la elongación para t1:

b) En t2 = 3,75 s, como sabemos que la velocidad esnula, la partícula se encuentra en uno de los extre-mos de su movimiento y la aceleración es máxima.Por tanto:

c) Calculamos la velocidad máxima:

d) Escribimos las ecuaciones de la elongación, la ve-locidad y la aceleración en función del tiempo:

x sen t

v t

= −⎛

⎝⎜

⎠⎟

= −⎛

⎝⎜

23

1 75

23 3

1 75

,

cos ,

ππ

π ππ

⎞⎞

⎠⎟

= − −⎛

⎝⎜

⎠⎟a sen t

29 3

1 752π π

π,

v Arad

sm

msmáx = ± = ±

⎛⎝⎜

⎞⎠⎟

⋅ = ±ωπ3

2 2 1,

� �amáx máx; ,= =⎛⎝⎜

⎞⎠⎟

⋅ =ωπ2

2

32 2 2A a

rads

mm

ss2

2 0 25 1 75 1 5

2

= − = − =

=

A sen A sen A

A m

( , , ) ( , )π π π

03

1 25 1 25 0

1 25

0 0= + + =

+

π ϕ π ϕ

π

cos( , ); cos( , )

, ϕϕπ

ϕ π ϕ π

0

0 0

20 75 1 75

= ±

= − = −, ,rad o rad

ωπ π π

πϕ

= = =

= +⎛⎝⎜

⎞⎠⎟

2 26 3

31 1 0

T srad

s

x A sen t ; ( , )

cos

2 0 25

3 3

0

2 2 0

= +

= +⎛⎝⎜

⎞⎠⎟

A sen

v A t

π ϕ

π πϕ ;; cos( , )0

31 25 0= +A

ππ ϕ

x s m

v

= ⋅ +⎛⎝⎜

⎞⎠⎟

= = −

= −

4 14

454

2 83

4

cos cos ,ππ π

ππ ππ

ππ

,sen s senms

a

⋅ +⎛⎝⎜

⎞⎠⎟

= − =

= −

14

454

8 89

44 14

454

27 92 22π π

ππ

πcos cos ,⋅ +

⎛⎝⎜

⎞⎠⎟

= − =sm

s

Calculamos la velocidad máxima:

49. Datos: K = 20 N/m; m = 300 g = 0,3 kg

La posición de equilibrio del cuerpo será aquélla enque la fuerza recuperadora del muelle y el peso delcuerpo sean iguales:

a) La amplitud de las oscilaciones será la distancia des-de el punto en que lo soltamos (longitud natural delmuelle) hasta el punto de equilibrio, A = 0,15 m. Esdecir, la amplitud coincidirá con el alargamiento delmuelle. Entonces, la posición más baja estará a −2Ade donde lo hemos soltado, a una distancia x = −Adel punto de equilibrio y centro de las oscilaciones.Así, la posición más baja será x = −0,15 m respecto ala posición de equilibrio.

b) Determinamos el período del movimiento:

50. Datos: m = 1,8 kg; K = 20 N/m; A = 30 cm = 0,3 m

En la posición de equilibrio, la energía cinética y la velo-cidad son máximas, mientras que la energía potencial esnula. Calculamos la energía total del sistema, que coinci-dirá con la energía cinética en la posición de equilibrio:

Hallamos la velocidad:

51. Datos: T = 10 s

Despejamos la longitud del péndulo de la expresióndel período:

52. Datos: MAS; x = 4 cos (πt + π/4), unidades SI

a) La pulsación del movimiento es ω = π rad/s. Portanto, el período y la frecuencia serán:

T s fT

Hz= = = = =2 2

21

0 5π

ω

π

π; ,

TLg

TLg

LT g

Ls

m

= = =

=⋅

2 44

10 9 8

2 22

2

2

π ππ

; ;

( ) ,ss m

2

2424 8

π= ,

Ec m v vEcm

vJ

kgv= = ± = ±

⋅= ±

12

2 2 0 91 8

1 02; ;,

,,;

ms

Ec E K A ENm

m J

Ec E

= = = ⋅ ⋅ =

=

12

12

20 0 3 0 92 2; ( , ) ,

== 0 9, J

TmK

Tkg

N ms

=

= =

2

20 3

200 77

π

π,

,/

m g Kx

xm gK

xkg

Nkg

Nm

, ,,

= −

= =

= −;0 3 9 8

200 155 m

v A Am

s

msmáx = ± = ± = ± ⋅ = ±ω π π π16 4 0 04 0 162

2, ,

4. Movimientos vibratorios

Page 63: 9523 gl fis_tx2_cas

66

La velocidad y el desplazamiento tienen el mismosentido en el tramo del centro de las oscilaciones alos extremos, pero tienen sentido opuesto en el mo-vimiento de vuelta desde los extremos al punto deequilibrio.

2. Escribimos primero la ecuación de la velocidad y deter-

minamos el valor de la fase cuando

Conocida la fase, hallamos la elongación correspon-diente:

3. a) Si un oscilador lineal duplica su amplitud, su ener-gía total se cuadruplicará, ya que si A1 = 2A:

b) Si se duplica la frecuencia, la energía también secuadruplica, ya que es proporcional a la constanterecuperadora K, y K = mω2. Así, si ω1 = 2ω:

c) Si se duplica la amplitud y se reduce la frecuenciaa la mitad, la energía permanece constante, ya quesi A1 = 2A y ω1 = ω/2:

4. El período de un péndulo simple de longitud L queoscila en un lugar con gravedad g es, independiente-mente de la masa colgada:

Para otro péndulo de longitud L’ = 2L, tendremos:

5. Datos: MAS; x0 = −A; T/4 = 0,1 s; 2A = 0,2 m

a) El tiempo que tarda en ir de un extremo de la tra-yectoria al centro es una cuarta parte del período:

T = 4 ⋅ 0,1 s = 0,4 s

b) Hallamos la pulsación a partir del período:

ωπ

ωπ

π= = =2 2

0 45

T srad

s;

,

TLg

Lg

Lg

T

T T

′ =′

= = =

′ =

2 22

2 2 2

2

π π π

TLg

= 2π

E K A m A m A

E m A

1 1 12

12

12

22

12 2

12

12

12 4

4

12

= = = ⋅

=

ωω

ω == =12

2K A E

E K A m A m A E1 12

12 2 2 21

212

412

4= = = ⋅ =ω ω

E K A K A K A E1 12 2 21

212

2 412

4= = = ⋅ =( )

x A sen t A sen A= + = =( ) ,ω ϕπ

0 30 86

v v A A t

t

= = = +

= +

12

12

12

0

0

máx ω ω ω ϕ

ω ϕ

cos( )

cos( ); ωω ϕπ

t + =0 3

v v=12 máx:

54. Datos: Δx = 10 cm = 0,1 m; m = 2 kg; A = 3 cm = 0,03 m

a) Determinamos la constante del resorte a partir de lanueva posición de equilibrio para la masa de 2 kg:

Hallamos la pulsación y el período del MAS del re-sorte:

b) La velocidad máxima es:

c) Determinamos la energía mecánica, que será cons-tante para todo el movimiento:

55. La construcción deberá ser como la siguiente:

Para que funcione la animación correctamente pue-den tomarse los ajustes estándar del programa.

El resumen del análisis cualitativo es que para una ve-locidad inicial fija (que puede ser nula):

— Al disminuir el valor de la constante de elasticidad,disminuye la frecuencia de oscilación.

— Al aumentar la diferencia entre la longitud inicialy la longitud en reposo aumenta la amplitud de os-cilación y se altera el punto de equilibrio.

EVALUACIÓN

1. En un MAS el vector posición y la aceleración nuncatienen el mismo sentido, ya que la aceleración siempreapunta hacia el punto de equilibrio, y este mismo pun-to es el origen de las posiciones.

La aceleración y la velocidad, en cambio, sí pueden te-ner el mismo sentido, pero no lo tienen siempre. Sólotienen el mismo sentido cuando el oscilador se despla-za desde uno de los extremos hasta el punto de equili-brio. Pero mientras se mueve del centro a los extremos,la velocidad y la aceleración tienen sentidos opuestos.

E K A ENm

m J= = ⋅ ⋅ =12

12

196 0 03 0 0882 2; ( , ) ,

v A m rad s

v m smáx

máx

/

/

= ± = ± ⋅

= ±

ω 0 03 9 90

0 30

, ,

,

ω ω

π

ω

π

= = =

= =

Km

Nm

kgrad

s

Tra

196

29 90

2 2

9 90

; ,

,dd

s

T s; ,= 0 63

m g K x Km g

x

kgNkg

mNm

;,

,= = =

=ΔΔ

2 9 8

0 1196

Resorte 12Constante del

resorte

Resorte 12Longitud en

reposo

30.00 1.00

v

Page 64: 9523 gl fis_tx2_cas

67

b) La aceleración de la masa es nula en la posición deequilibrio, para x = 0.

c) La energía total del oscilador es constante a partirdel momento en que soltamos la masa:

En los puntos extremos de la trayectoria, la masaqueda momentáneamente en reposo, y toda laenergía es potencial elástica, debida a la compre-sión o alargamiento del resorte.

En el punto de equilibrio, la energía potencialelástica es nula, y toda la energía del sistema estáen forma de energía cinética.

En este problema, como los desplazamientos sonpequeños, podemos considerar que la energíapotencial gravitatoria es constante en todo mo-mento. Escogemos el origen de forma que seanula.

E K ANm

m

E J

= = ⋅ ⋅

=

12

12

500 0 1

2 5

2 2( , )

,

− = ⋅ +

= − =

=

0 1 0 1 10 0

132

0

0

0 0

, , ( )

;

,

sen

sen

x

ϕ

ϕ ϕπ

11 1032

sen t +⎛⎝⎜

⎞⎠⎟

π

c) Escribimos la ecuación de la elongación:

Sabiendo que en t = 0 la posición es x0 = −A, deter-minamos la fase inicial:

Hallamos la posición para t = 1 s:

6. Datos: m1 = 5 kg; K = 500 N/m; A = 10 cm = 0,1m

a) Calculamos la pulsación del resorte:

Escribimos la ecuación de la elongación tomandocomo origen de posiciones el punto de equilibrio,después de haber colgado la masa, y el sentido po-sitivo hacia arriba:

Teniendo en cuenta que la posición inicial (parat = 0) es x0 = −A, determinamos la fase inicial:

x A sen t x sen t= + = +( ); , ( )ω ϕ ϕ0 00 1 10

ω = = =Km

Nm

kgrad

s

500

510

x sen s sen= ⋅ −⎛⎝⎜

⎞⎠⎟

=⎛⎝⎜

⎞⎠⎟

=0 1 5 12

0 192

0, ,ππ π

,,1 m

− = ⋅ + = −

= −

A A sen sen( );5 0 1

2

0 0

0

π ϕ ϕ

ϕπ

x A sen t sen t= + = +( ) , ( )ω ϕ π ϕ0 00 1 5

4. Movimientos vibratorios

Page 65: 9523 gl fis_tx2_cas

69

Movimiento ondulatorio5

b) Si en t = 0, x = A sen ϕ0 = A sen = A. Por

tanto, en el instante inicial la elongación debe sermáxima y positiva.

• Que el logaritmo en base b de a valga c (logb a = c) sig-nifica que b elevado a c vale a (bc = a).

• La unidad de presión en el SI es el Pa y la de densidades el kg/m3.

1. ONDAS

1. La perturbación que se propaga en las ondas forma-das en la superficie del agua es la elevación de algunaspartículas y el hundimiento de otras. De hecho, se estátransmitiendo energía a través de las ondas, cosa quepermite la elevación del agua.

2. a) Cuando la onda generada por nuestra sacudida enla cuerda alcance el cuerpo que cuelga de ella, elcuerpo se elevará y volverá a bajar.

b) Este hecho demuestra que las ondas transportanenergía, ya que el cuerpo, al elevarse, tiene ma-yor energía potencial que antes, y esta energía nose puede crear de la nada. Es la energía que lellega a través de la onda.

2. ONDAS MECÁNICAS

3. Respuesta sugerida:

Un fenómeno típicamente ondulatorio son las ondasque se crean en la superficie de un estanque cuandodejamos caer una piedra. Las ondas que se propaganpor el agua son ondas mecánicas, ya que precisan lapresencia del agua para propagarse.

4. Se define como onda transversal aquel movimientoondulatorio en el que la dirección de propagaciónde la perturbación y la dirección de oscilación de laspartículas del medio perturbado son perpendicula-res.

Se define como onda longitudinal aquel movimientoondulatorio en el que la dirección de propagación dela perturbación y la dirección de oscilación de las par-tículas del medio perturbado son paralelas.

5. Las ondas transmitidas por una cuerda son transversa-les porque la propagación de la onda a lo largo de lacuerda y la dirección de vibración de la cuerda (su-biendo y bajando) son perpendiculares.

π2

ϕπ

0 2= ;

PREPARACIÓN DE LA UNIDAD

• a) Movimiento periódico: La característica que defineun movimiento periódico es que se repite cada cier-to intervalo de tiempo, que denominamos período.

b) Movimiento oscilatorio periódico: Se caracteriza porrepetirse cada cierto intervalo de tiempo (es periódi-co) sucesivamente a un lado y a otro de un punto deequilibrio.

c) Movimiento oscilatorio armónico simple: Se trata deun movimiento periódico y oscilatorio, en el que la ace-leración en cada momento es proporcional a la elonga-ción y de sentido contrario.

• Datos: MAS; x = 0,3 sen (10 πt) en unidades SI

a) Si comparamos con la expresión general de un MAS,x = A sen (ωt + ϕo), vemos que la fase inicial es ϕ0 = 0.

b) La pulsación es ω = 10 π rad/s.

c) La amplitud es A = 0,3 m.

d) Calculamos el período a partir de la pulsación:

e) Hallamos la frecuencia, que es la inversa del período:

f) Calculamos la elongación para t = 0:

g) La elongación para t = 2 s es:

h) Para hallar la velocidad, derivamos la elongación:

• Consideramos un MAS con ecuación para la elongaciónx = A sen (ωt + ϕ0).

a) Si ϕ0 = 0, se cumple que en t = 0:

x = A sen ϕ0 = A sen 0 = 0

Por tanto, la elongación en el instante inicial debeser nula.

vdxdt

t t

v s

= = ⋅ =0 3 10 10 3 10

5

, cos( ) cos( )

( )

π π π π

== = ⋅

=

3 10 3 10 0 5

5 3

π π π π

π

cos( ) cos( , )

( ) cos

t

v s (( )5 3π π= −

x sen t sen m= = =0 3 10 0 3 20 0, ( ) , ( )π π

x sen t sen m= = =0 3 10 0 3 0 0, ( ) ,π

fT s

Hz= = =1 1

0 25

,

ωπ π

ω

π

π= = = =

2 2 210

0 2T

T s; ,

5. Movimiento ondulatorio

Page 66: 9523 gl fis_tx2_cas

70

tud de onda al duplicar la frecuencia mediante la re-lación entre la longitud de onda, la velocidad y la fre-cuencia:

Por tanto, la longitud de onda se reducirá a la mitad.

13. Datos: λ = 20 cm = 0,2 m; f = 1 750 Hz

Calculamos la velocidad de la onda:

v = λ f = 0,2 m ⋅ 1 750 Hz = 350 m⋅s−1

14. En la ecuación general de las ondas armónicas, ϕ0es la fase inicial. En el instante inicial, t = 0 s, ϕ0 de-termina el estado de vibración de cada punto x.En concreto, para x = 0, la elongación inicial seráy0 = A sen ϕ0.

15. Datos: y = 0,03 sen (3,5t − 2,2x), en unidades SI

Comparamos la ecuación dada con la expresión gene-ral de la función de onda y = A sen (ωt − kx), para de-terminar:

ω = 3,5 rad/s; k = 2,2 m−1; A = 0,03 m

a) Hallamos la longitud de onda a partir del númerode ondas:

b) Determinamos el período a partir de la pulsación:

c) Calculamos la velocidad de propagación de la on-da a partir de la longitud de onda y el período:

d) Hallamos la velocidad derivando la función deonda:

La velocidad es máxima cuando el coseno vale 1:

16. Datos: x = λ/6; t = T/4; A = 2 cm; y(0, 0) = 0

Con los datos del problema, hallamos la elongación,escribiendo primero la función de onda:

y A sen t kx

y A sen

= − −

= ⇒ = =

( )

( , ) ;

ω ϕ

ϕ ϕ0

0 00 0 0 0 00

21 1

21

41

y A senT

t x A senT

T= −

⎛⎝⎜

⎞⎠⎟

⎣⎢

⎦⎥ = −π

λπ

λλλ

π

6

214

16

⎛⎝⎜

⎞⎠⎟

⎣⎢

⎦⎥

= −⎛⎝⎜

⎞⎠⎟

⎣⎢

⎦⎥ =y A sen AA sen A sen2

112 6

ππ⎛

⎝⎜⎞⎠⎟

=

vmsmáx = 0 105,

vdydt

t x

v

= = ⋅ −

=

0 03 3 5 3 5 2 2

0 105 3

, , cos( , , )

, cos( ,, , )5 2 2t x−

vT

ms

ms

= = =λ 2 86

1 81 6

,,

,

ωπ π

ω

π= = = =

2 2 2

3 51 8

TT

rads

s;,

,

kk m

m= = = =−

2 2 2

2 22 861

π

λλ

π π;

,,

v fvf

si f fvf

vf

= = ′ = ⇒ ′ =′

= =λ λ λ λ; ; 22

12

6. Las ondas de compresión y expansión transmitidaspor un resorte son longitudinales porque la direc-ción de propagación de las ondas y la dirección delmovimiento oscilatorio coinciden, son paralelas al re-sorte.

— Podemos establecer una onda transversal en un re-sorte dando un golpe en uno de sus extremos en ladirección perpendicular al resorte.

7. La velocidad de propagación de una onda mecánica de-pende de las propiedades del medio en el que se trans-mite. Para las ondas longitudinales, por ejemplo, en elcaso de sólidos, estas propiedades son la densidad y laconstante elástica; mientras que en los fluidos la veloci-dad depende de la densidad y del módulo de compre-sibilidad.

8. a) En los fluidos sólo pueden propagarse las ondasmecánicas longitudinales, ya que carecen de lasfuerzas recuperadoras necesarias para la transmi-sión de ondas mecánicas transversales.

b) En los sólidos pueden transmitirse tanto las ondasmecánicas transversales como las longitudinales.

9. Datos: L = 50 m; t = 90 s

La velocidad de propagación de la onda es:

3. ONDAS ARMÓNICAS

10. Una onda armónica se define como una onda que tie-ne su origen en las perturbaciones periódicas produci-das en un medio elástico por un movimiento armónicosimple.

Las magnitudes que la caracterizan son:

Amplitud de la onda. Es el valor máximo de la elonga-ción, la máxima distancia al punto de equilibrio.

Longitud de onda, λ. Es la distancia mínima entre dospuntos consecutivos que se hallan en el mismo estadode oscilación.

Período, T. Es el tiempo que tarda un punto cualquie-ra en efectuar una oscilación completa, o bien, eltiempo que emplea la onda en avanzar una longitudde onda.

Frecuencia, f. Es el número de ondas que pasan por unpunto dado por unidad de tiempo. Coincide tambiéncon el número de oscilaciones que efectúa un puntodel medio por unidad de tiempo.

Las magnitudes de una onda transversal y de una ondalongitudinal se definen de la misma forma.

11. Si sacudimos el extremo de una cuerda tensa tres vecespor segundo, el período será:

12. Si no variamos la tensión de la cuerda, la velocidad dela onda será la misma. Hallamos el valor de la longi-

Ts

oscilacioness= =

13

0 33,

vLt

ms

ms

= = =5090

0 56,

Page 67: 9523 gl fis_tx2_cas

71

c) La velocidad y la aceleración de cualquier puntode la cuerda serán:

Por tanto:

vmáx = 1,5 π = 4,7 m/s; amáx = 75 π2 = 740 m/s2

20. Datos: y = 0,3 sen (4 πt − 8 πx), en unidades SI

Si comparamos con la expresión general de la funciónde onda, obtenemos:

A = 0,3 m; ω = 4 π rad/s; k = 8 π m−1

a) Estarán en fase con el punto que se encuentra enx = 3 m todos los que disten de él un número en-tero de longitudes de onda. Determinamos, pues,la longitud de onda del movimiento a partir delnúmero de ondas:

Por tanto, estarán en fase con el punto en x = 3 m lospuntos situados en x = (3 + n 0,25) m, con n ∈ Z.

b) El estado de vibración será el mismo que para t = 2 spara todos los instantes separados de él por un nú-mero entero de períodos. Hallamos el período apartir de la pulsación:

Por tanto, el estado de vibración será el mismoque en t = 2 s para t = (2 + n 0,5) s, con n ∈ Z.

21. Si duplicamos la amplitud de una onda armónica, co-mo la potencia es proporcional a la intensidad y éstaes proporcional al cuadrado de la amplitud, la poten-cia se cuadruplicará. Por tanto, para duplicar la ampli-tud de una onda es necesario cuadruplicar la potencianecesaria para generarla.

22. La intensidad se define como la potencia por unidad desuperficie perpendicular a la dirección de propagación.La potencia es proporcional a la energía, y ésta se con-serva.

Si consideramos un foco emisor puntual, la intensidada cierta distancia será la potencia emitida (constantepara todas las distancias) dividida por una superficieesférica de radio R, igual a la distancia al foco. Comola superficie de una esfera vale 4 πR2, la intensidad se-rá inversamente proporcional al cuadrado de la dis-tancia.

IPS

Et

S= = Δ

ωπ π

ω

π

π= = = =

2 2 2

40 5

TT

rads

s; ,

kk m

m= = = =−

2 2 2

80 251

π

λλ

π π

π; ,

vd x

dtt x

v

= = ⋅ +

=

( ), cos ( )

, co

Δ0 03 50 50 10

1 5

π π π

π ss ( )

, (

50 10

1 5 50 50 102

π π

π π π

t x

advdt

sen t

+

= = − ⋅ + xx

a sen t x

)

( )= − +75 50 102π π π

17. Datos: v = 8 m/s; y = 0,3 sen (16 πt + kx), en unidades SI

a) Si comparamos la ecuación dada con la expresióngeneral de la función de onda, obtenemos:

A = 0,3 m; ω = 16 π rad/s

Calculamos la frecuencia a partir de la pulsación:

El signo positivo del término kx indica que la ondase mueve en el sentido negativo del eje X.

b) Determinamos λ y k a partir de los valores de la fre-cuencia y la velocidad:

c) La velocidad de cualquier punto de la cuerda será:

Para x = 0,5 m y t = 60 s:

18. El hecho de que la función de onda sea periódica res-pecto a la posición significa que cada cierta distan-cia, denominada longitud de onda, encontramos pun-tos del sistema en el mismo estado de vibración. El hechode que sea periódica en el tiempo significa que cadacierto intervalo de tiempo, denominado período, todoel sistema vuelve a estar en el mismo estado de vibra-ción.

19. Datos: λ = 20 cm = 0,2 m; f = 25 Hz; A = 3 cm = 0,03 m

a) Determinamos la velocidad a partir de la frecuen-cia y la longitud de onda:

b) Escribimos la ecuación de una onda longitu-dinal que se propaga en el sentido negativo deleje X:

Hemos supuesto que en el instante inicial, t = 0, elpunto en el origen, x = 0, tiene una elongación nu-la, por lo que la fase inicial ϕ0 será nula.

Δx A sen t kx

f Hzrad

s

k

= +

= = ⋅ =

=

( )ω

ω π π π

π

λ

2 2 25 50

2== =

= + =

−20 2

10

0 03

1ππ

ω,

( ) , sen (m

m

x A sen t kxΔ 550 10 )π πt x+

v f m Hz m s= = ⋅ =λ 0 2 25 5, /

v t x= + = ⋅ + ⋅4 8 16 2 4 8 16 60 2 0 5, cos( ) , cos( ,π π π π π π ))

, cos( ) , cos ,v m s= = = − ⋅ −4 8 961 4 8 15 1 1π π π π

vdydt

t x

v t

= = ⋅ +

=

0 3 16 16 2

4 8 16

, cos( )

, cos(

π π π

π π ++ 2πx)

v fvf

ms

Hzm k

mm= = = = = = = −λ λ

π

λ

ππ; ;

8

81

2 21

2 1

ω πωπ

π

π= = = =2

2

16

28f f

rads Hz;

yA

= = =2

22

1cm

cm

5. Movimiento ondulatorio

Page 68: 9523 gl fis_tx2_cas

72

31. Las ondas sonoras se forman y se propagan mediantesucesivas compresiones y dilataciones del medio pro-ducidas por un foco en movimiento vibratorio.Podemos esquematizar la formación y propagaciónmediante tres figuras:

Disponemos de un émbolo vibratorio situado en el ex-tremo de un cilindro estrecho de longitud indefinidaque contiene un gas. Al empujar el émbolo hacia laderecha, el gas se comprime en la región más próximaal émbolo, aumentando la presión y la densidad delgas. Se forma así un pulso de compresión que viaja ha-cia la derecha.

Al retroceder el émbolo hacia la izquierda, el gas pró-ximo a éste se expansiona, disminuyendo así su pre-sión y densidad. Se produce un pulso de enrareci-miento que se propaga por el cilindro, siguiendo elanterior pulso de compresión.

Al hacer oscilar el émbolo rápida y periódicamente,viaja por el cilindro un tren de sucesivas compresionesy enrarecimientos. La onda longitudinal se propagapor el tubo, siendo λ la distancia entre dos compresio-nes o dos enrarecimientos consecutivos.

32. La diferencia de fase existente entre el desplazamien-

to y la presión de una onda sonora es de

33. Respuesta sugerida:

La principal aplicación médica de los ultrasonidos esla ecografía. Además de utilizarse para otros estudios,se usa para examinar el feto durante el embarazo. Lasonda articulada que se desliza por encima del vientrede la madre emite ondas sonoras de alta frecuencia,ultrasonidos. Las ondas se reflejan en los tejidos cor-porales del feto, siendo esta reflexión de mayor o me-

π2

.rad

23. La intensidad de la luz decrece con el cuadrado de ladistancia, debido a que la misma energía emitida porel foco debe repartirse por un área mayor cuanto ma-yor es la distancia. La energía y la potencia que cruzanuna superficie esférica cerrada centrada en el foco sonlas mismas para cualquier distancia, ya que la energíase conserva. Pero como la intensidad se define como laenergía por unidad de superficie, cuanto más grandesea la superficie, menor es la intensidad.

24. Datos: P = 4 W

a) Hallamos la intensidad a 2 m de la fuente:

b) Al duplicar la distancia, R2 = 4 m:

Por tanto, la intensidad disminuye en:

ΔI = I2 − I1= −0,060 W ⋅ m−2

c) Utilizamos la relación general entre amplitudes ydistancias:

4. ONDAS SONORAS

25. El sonido es una vibración o perturbación mecánica dealgún cuerpo que se propaga a través de cualquier me-dio material elástico. Algunos ejemplos de ondas sono-ras son las generadas por voces humanas, por altavocesde aparatos de audio o por el televisor, las generadaspor los instrumentos musicales…

26. Decimos que las ondas sonoras son longitudinales por-que la perturbación que se propaga es una compresióny dilatación del medio. El movimiento generado por laperturbación se realiza, pues, en la misma dirección enque se propaga la onda.

27. Los límites de frecuencia para que una onda sonorasea audible por el oído humano son 20 Hz y 20 000 Hz.

28. Para los sonidos con frecuencias inferiores a 20 Hz ofrecuencias superiores a 20 000 Hz todos los humanossomos sordos. El oído humano no es capaz de percibiresos sonidos.

29. Las ondas ultrasónicas tienen una frecuencia mayorque la máxima frecuencia audible, mayor que 20000 Hz,mientras que las infrasónicas tienen una frecuenciamenor que la mínima frecuencia audible, inferior a20 Hz. Por tanto, las ondas ultrasónicas tienen mayorfrecuencia que las infrasónicas.

30. La energía de un movimiento ondulatorio es propor-cional al cuadrado de la frecuencia. Por tanto, una on-da ultrasónica tiene mayor energía que una onda infra-sónica, ya que su frecuencia es mayor.

AA

RR

mm

A A1

2

2

11 2

42

2 2= = = =;

IP

R

W

m

W

m22

2 2 24

4

4 40 020= = =

π π ( ),

IP

R

W

m

W

m112 2 24

4

4 20 080= = =

π π ( ),

Compresión

Compresión

Enrarecimiento

Enrarecimiento

λ

Page 69: 9523 gl fis_tx2_cas

73

38. Datos: t = 10 s

Si se oye el trueno 10 s después de verse el relámpago:

39. Datos: vsonido (20 °C) = 5 130 m ⋅ s−1 ; λ = 5,1 m

Determinamos el período y la frecuencia de la ondasonora:

40. Las tres cualidades del sonido son la intensidad, el to-no y el timbre.

Intensidad. Se definen la intensidad física (poten-cia transmitida por unidad de superficie) y la intensi-dad subjetiva (sensación sonora más o menos intensa).Ambas se relacionan según una escala logarítmica. Es loque comúnmente llamamos volumen del sonido.

Tono. El tono permite distinguir sonidos de distintasfrecuencias. Llamamos sonidos agudos a los de alta fre-cuencia y graves a los de frecuencias bajas. Esta cuali-dad permite distinguir las notas musicales y crear lamúsica.

Timbre. El timbre viene determinado por la forma dela onda sonora, que es el resultado de varios movi-mientos periódicos superpuestos a la onda fundamen-tal. Según el timbre, somos capaces de distinguir unmismo tono con la misma intensidad emitido por di-ferentes instrumentos musicales.

41. Datos: I0 = 1,0 ⋅ 10−12 W ⋅ m−2; I = 1,0 W ⋅ m−2

Determinamos los niveles de intensidad sonora corres-pondientes al umbral de audición, β0, y al umbral deldolor, β:

42. Datos: β = 100 dB; λ1 = 30 m; λ2 = 12 m; λ3 = 0,003 m; vsonido = 340 m/s

El nivel de intensidad es superior al umbral de audi-ción e inferior al umbral del dolor. Por lo tanto, por loque a la intensidad se refiere, podemos captar el soni-do. Pero falta determinar si las frecuencias de cadauno están dentro de los límites audibles (somos capa-ces de oír ondas sonoras entre 20 Hz y 20 000 Hz).

β

β

00

0

0

10 10 1 0

10 10

= = =

= =

log log

log

II

dB

II

log,

,log

1 0

1 0 1010 10

2

12 212W m

W m

⋅ ⋅=

=

− −

β 1120 dB

v fT

Tv

mms

s

fv

= =

= = = ⋅

= =

λλ

λ

λ

,,

5 1

5 1309 9 10

5 13

4

00

5 11 005 9

,,

ms

mHz=

x v t m s s m km= = ⋅ = = ,340 10 3 400 3 4/

nor intensidad según las características del tejido. Losecos son registrados y convertidos electrónicamente enuna imagen en una pantalla.

34. Los indios ponían el oído en tierra para determinar lapresencia de soldados en su territorio porque el sonidoviaja más rápidamente y a mayor distancia en la tierraque en el aire. En general, las ondas sonoras se propagana mayor velocidad en los sólidos que en los gases, debidoa que el módulo de Young de los sólidos es mayor que elmódulo de compresibilidad en los fluidos. Como las on-das viajan más rápidamente y más lejos en los sólidos queen el aire, los indios podían percibir la presencia de sol-dados escuchando en tierra antes de oírlos normalmen-te por el aire.

35. a) Si colocamos el despertador en el extremo de unaviga de 100 m, podremos oír el tictac del reloj po-niendo el oído en el otro extremo de la viga, ya queel sonido se transmitirá por el hierro mejor que loharía por el aire.

b) Por el mismo motivo, es posible que no podamosoír el despertador a través del aire a la misma dis-tancia a la que sí podemos percibirlo a través de laviga.

36. Para conocer la distancia en kilómetros a la que cayóun rayo, se puede contar los segundos desde que se vioel relámpago hasta que se oye el trueno y dividirlos portres. Lo que hacemos es contar el tiempo que empleael sonido en llegar hasta nosotros. Como la velocidadde la luz es muy grande (3 ⋅ 108 m/s), podemos consi-derar que la luz del relámpago nos llega instantánea-mente y en el mismo momento en que se produce eltrueno. Si contamos el tiempo que tarda en llegar el so-nido del trueno en segundos, t(s), y lo multiplicamospor la velocidad en metros por segundo, tendremos ladistancia en metros. Pasando los metros a kilómetros:

37. Datos: Maire = 28,8 ⋅ 10−3 kg ⋅ mol−1 ;

R = 8,314 J ⋅ K−1 ⋅ mol−1;

γaire = 1,4; T1 = 0 °C = 273 K; T2 = 30 °C = 303 K

a) Hallamos la velocidad de las ondas sonoras en elaire a 0 °C:

b) Si la temperatura es de 30 °C:

vRT

M

vJ K mol

aireaire

aire

aire

=

=⋅ ⋅ ⋅−

γ

1 4 8 314 1, , −−

− −

⋅ ⋅=

1

3 1

303

28 8 10350

,

K

kg mol

ms

vRT

M

vJ K mol

aireaire

aire

aire

=

=⋅ ⋅ ⋅−

γ

1 4 8 314 1, , −−

− −

⋅ ⋅=

1

3 1

273

28 8 10332

,

K

kg mol

ms

x m v t s m s t s

x kmkm

m

( ) ( ) ( )

( )

= = ⋅

= ⋅

340

11 000

340

/

( )( )m

st s

t skm⋅ ≅

3

5. Movimiento ondulatorio

Page 70: 9523 gl fis_tx2_cas

74

45. Datos: β = 45 dB

Si tenemos dos aparatos de radio, tendremos el doblede intensidad de sonido, pero no de nivel de intensi-dad sonora. Calculamos el nuevo nivel de intensidadsonora:

46. La legislación a nivel local, las ordenanzas municipa-les, recogen o deberían recoger una serie de valores lí-mite para los niveles de ruido según los criterios de:

— Zona: es decir, si se trata de una zona industrial,residencial, de servidumbre (por el paso de im-portantes infraestructuras), etc.

— Horario: diurno o nocturno.

— Si el ruido se mide en exteriores, por ejemplo lacalle, o interiores, como una vivienda.

— Si la medida es del ruido que procede de un fococoncreto o del ruido total ambiente.

Para conseguir una mayor calidad acústica en nuestrasvidas, existen otras regulaciones locales, autonómicas,estatales e incluso europeas que afectan a conceptoscomo:

— La emisión sonora de actividades realizadas en elexterior, de vehículos de motor, de maquinaria in-dustrial, de instalaciones (ascensores, calefacción,climatización...) en los edificios de viviendas y lo-cales públicos.

— Horarios de realización de obras, de cierre de lo-cales nocturnos...

— Requisitos acústicos (emisión de ruidos) y de ais-lamiento que son necesarios para obtener una li-cencia para comercios, locales, industrias... Porejemplo, la solicitud de licencias para actividadesclasificadas suele incluir un estudio del impactoacústico de la actividad, o el proyecto de una nuevaedificación debe justificar el cumplimiento de lanormativa sobre aislamiento acústico.

— Limitaciones de emisiones sonoras en los avio-nes.

— Protección de los trabajadores contra los riesgosdebidos a la exposición al ruido en su puesto detrabajo.

Además, los ayuntamientos realizan medidas periódi-cas del nivel de ruido en el ambiente y publican los re-sultados. También es su misión, en lo concerniente ala vía pública:

— Colocar apantallamiento acústico donde sea nece-sario.

— Pavimentar con materiales absorbentes del ruido.

— Instalar equipos urbanos de baja emisión sonora.

β

β

′ = = +⎛

⎝⎜⎞

⎠⎟

′ =

102

10 2

10

0 0

log log logI

III

llog log log

( )

2 10 10 2

3 3 45

0

+ = +

′ = + = +

II

d

β

β β BB dB= 48

Calculamos a qué longitudes de onda corresponden es-tas frecuencias en el aire:

Por tanto, λ2 será audible para el oído humano. Peroλ1 y λ3 quedan fuera del rango que nuestro órgano au-ditivo es capaz de registrar.

43. Datos: fLa = 440 Hz; fDo = 264 Hz; vsonido = 340 m/s

Determinamos las longitudes de onda correspondien-tes:

44. Datos: P = 1 mW = 1 ⋅ 10–3 W; R = 4 m

a) A una distancia de 4 m, si suponemos que la poten-cia de la onda sonora se distribuye uniformementepor una semiesfera, la intensidad de la onda sono-ra será:

Calculamos el nivel de intensidad sonora:

b) Si ladran tres perros a la vez, la potencia emitida se-rá el triple, P = 3 ⋅ 10−3 W:

Calculamos el nivel de intensidad sonora:

La intensidad del sonido, que es proporcional a lapotencia, se ha multiplicado por 3. En cambio, elnivel de intensidad sonora, como está en relaciónlogarítmica con la intensidad, y por tanto con lapotencia, no se ha multiplicado por 3, sino por1,067.

β = =⋅ ⋅

− −

−10 10

2 98 10

1 0 100

5 2

12log log

,

,

II

W m

WW m

dB

=

−2

74 7β ,

IPS

P

R

W

m

I W

= = =⋅

= ⋅ ⋅

12

4

3 10

2 4

2 98 10

2

3

2

5

π π ( )

, mm−2

β = =⋅ ⋅

− −

−10 109 9 10

1 0 100

6 2

12log log,

,

II

W m

W ⋅⋅=−m

dB2 70

IPS

P

R

W

mW m= = =

⋅= ⋅ ⋅

−−

12

4

1 10

2 49 9 10

2

3

26

( ),

π π−−2

λ

λ

Lasonido

La

Dosoni

vf

ms

Hzm

v

= = =

=

340

4400 77,

ddo

Dof

ms

Hzm= =

340

2641 29,

λ

λ

λ

=

= =

=

vf

ms

Hzm

msH

máx

mín

340

2017

340

20 000 zzm= 0 017,

Page 71: 9523 gl fis_tx2_cas

75

c) Determinamos la aceleración:

48. Datos:

ω = 2 π/5 rad ⋅ s−1; x = 4 m; t = 8 s

a) El número de ondas es k = π/4 m−1. Calculamos lalongitud de onda:

b) Calculamos la velocidad de vibración:

Para x = 4 m y t = 8 s:

c) Determinamos la aceleración:

Para t = 8 s y x = 4 m:

49. Datos: A = 0,12 m; λ = 3 m; f = 5 Hz

a) Determinamos el período y la velocidad de propa-gación:

b) Calculamos la pulsación y el número de ondas pa-ra escribir la ecuación de onda. Suponemos que lafase inicial es cero y que la onda se propaga en elsentido positivo del eje X:

ω π π π

π

λ

π π

= = ⋅ =

= = = −

2 2 5 10

2 23

23

1

f Hzrad

s

km

m

y == −⎛⎝⎜

⎞⎠⎟

0 12 1023

, sen t xππ

Tf Hz

s

v f m Hz m s

= = =

= = ⋅ =

1 15

0 2

3 5 15

,

λ /

a sen sen= − ⋅ − ⋅⎛⎝⎜

⎞⎠⎟

= −825

25

84

4825

162 2π π π π πππ

5

1 86 2

−⎛⎝⎜

⎞⎠⎟

= −am

s,

advdt

sen t x

a

= = − ⋅ −⎛⎝⎜

⎞⎠⎟

= −

45

25

25 4

825

2

π π π π

πssen t x

25 4π π

−⎛⎝⎜

⎞⎠⎟

v = ⋅ − ⋅⎛⎝⎜

⎞⎠⎟

= −⎛⎝

45

25

84

445

165

π π π π ππcos cos ⎜⎜

⎞⎠⎟

=vms

2 03,

vdydt

t x

v

= = ⋅ −⎛⎝⎜

⎞⎠⎟

=

225

25 4

45

2

π π π

π π

cos

cos55 4

t x−⎛⎝⎜

⎞⎠⎟

π

kk m

m= = = =−

2 2 2

4

81

π

λλ

π ππ

;

y sen t x A m k m= −⎛⎝⎜

⎞⎠⎟

= = −225 4

2 4 1; ; ;π π

π/

advdt

sen t x

a sen

= = − ⋅ −

= −

12 60 60 50

720 2

π π π π

π

( )

(660 50 720 2 2π π πt x a m s− =); máx /

— Gestionar el tráfico adecuadamente para conseguirlos menores niveles de ruido posibles.

Finalmente, es potestad de los ayuntamientos efectuarlas labores de inspección y sanción. Las sanciones pue-den consistir en una multa, la suspensión temporal dela actividad, la retirada temporal o definitiva de la li-cencia, o la clausura del local o negocio. En cuanto alos vehículos, puede comportar multas y la inmoviliza-ción de éstos.

FÍSICA Y SOCIEDAD

a) Paul Langevin (1872-1946) utilizó el fenómeno de lapiezoelectricidad, descubierto anteriormente porCurie, para generar ultrasonidos. Dicho fenómenoconsiste en que si se somete un cristal de cuarzo a unavariación alterna de potencial eléctrico de elevada fre-cuencia, el cristal vibra con una frecuencia tan alta quees capaz de generar ondas ultrasónicas.

Langevin pretendía desarrollar un invento basado enlos ultrasonidos que localizara objetos submarinos me-diante la detección de los ultrasonidos reflejados enellos. Su invento fue la base del sonar moderno.

b)

RESOLUCIÓN DE EJERCICIOS Y PROBLEMAS

47. Datos: y = 0,2 sen (60 πt − 50 πx); A = 0,2 m; k = 50 π m−1;

ω = 60 π rad ⋅ s−1; x = 2 m; t = 10 s

a) Calculamos la velocidad de propagación de la onda:

b) Calculamos la velocidad de vibración:

Para x = 2 m y t = 10 s:

vdydt

= = ⋅ − ⋅ =12 60 10 50 2 12 500π π π π πcos ( ) cos ( ))

cos ( )v m s= =12 2 12π π π /

vdydt

t x

v

= = ⋅ −

=

0 2 60 60 50

12 60

, cos ( )

cos (

π π π

π ππ πt x− 50 )

v f vk k

rads

m

ms

= = ⋅ = = =−λ

π ωπ

ω π

π; ,

22

60

501 21

5. Movimiento ondulatorio

Aplicación de los ultrasonidosFrecuencia

(kHz)Intensidad(W/cm2)

Ecografía 16-20 0,01

Señales submarinas y detección de objetos

16-20 —

Soldadura ósea 16-20 3-30

Soldadura de termoplásticos 20 1000

Limpieza ultrasónica 20-25 0-5

Terapia ultrasónica 100-1000 1

Page 72: 9523 gl fis_tx2_cas

76

Por tanto, la ecuación del MAS que describe lapartícula situada en el punto x = −0,1 m es:

b) En primer lugar hallamos el número de ondas:

La ecuación de onda se puede expresar:

y (t, x) = A sen (ωt + kx + δ0)

y (t, x) = 0,5 sen (100πt + 10πx + δ0) en unidades del SI

Y la fase inicial se obtiene de la condición inicialdel enunciado:

Con esto la función de onda resulta:

en unidades del SI

53. Datos: T = 2 s

a) Hallamos el número de ondas a partir de la dife-rencia de fase.

Calculamos la pulsación.

b) Hallamos la diferencia de fase entre dos oscilacio-nes de un mismo punto separadas 1 s.

54. Datos: v = 45 m/s Δx = 1 m ⇒ Δϕ = π rad

a) De la diferencia de fase determinamos el númerode ondas y, a partir de éste, la longitud de onda yla frecuencia.

kx

radm

m

k mm

fv

= = =

= = =

= =

Δ

Δ

ϕ ππ

λπ π

π

λ

1

2 22

1

1

4452

22 5,m sm

Hz/

=

Δϕ ω π π= − = ⋅ =( )t trad

ss rad2 1 1

ωπ π

π= = =2 2

2T srad

s

kx m

m= = = −Δ

Δ

ϕπ

π20 1

5 1

,

Δ Δx m rad= ⇒ = =0 12

, ϕπ

y t x sen t x( , ) ,= + +⎛⎝⎜

⎞⎠⎟

0 5 100 1032

π ππ

y t x m m

m sen sen

( , , ) ,

, , ( ) (

= = − =

= − + ⇒ −

0 0 1 0 5

0 5 0 5 0π δ ππ δ

π δπ

δπ

+ =

− + = =

0

0 0

1

232

)

;

kf

vs

m sm= = =

⋅=

−−2 2 2 50

1010

1

11π

λ

π ππ

y t sen t( ) ,= +⎛⎝⎜

⎞⎠⎟

0 5 1002

ππ

c) La distancia mínima entre dos puntos en oposiciónde fase es la mitad de la longitud de onda:

50. Datos: λ = 0,2 m; f = 10 Hz; A = 0,12 m

a) Determinamos el período y la velocidad de laonda:

b) Los puntos que están en fase con el que ocupa la po-sición x = 3 m son los que distan de él un número en-tero de longitudes de onda. Por tanto, son los que ve-rifican:

x = (3 + λn) m = (3 + 0,2n) m, con n ∈ Z

51. Datos: A = 0,05 m; v = 12,5 m/s; T = 0,08 s

y (t = 0, x = 0) = 0,05 m

a) Hallamos el número de ondas y la pulsación apli-cando la definición.

b) La ecuación de onda es:

y (t, x) = A sen (ω t − kx + ϕo)

La fase inicial se obtiene de la condición inicial delenunciado.

Por tanto, la función de onda resulta:

en unidades del SI

52. Datos: A = 0,5 m; f = 50 Hz; v = 10 m/s

y (t = 0, x = −0,1 m) = 0,5 m; ω = 2 πf = 100π s−1

a) La amplitud y la pulsación del MAS son conocidas.La fase inicial se obtiene de la condición inicial delenunciado.

y t A sen t

y t x m m

( ) ( )

( , , ) ,

,

= +

= = − =

ω ϕ0

0 0 1 0 5

0 5 , ;m m sen sen rad= ⇒ = =0 5 120 0 0ϕ ϕ ϕπ

y t x sen t x( , ) ,= ⋅ − +⎛⎝⎜

⎞⎠⎟

0 05 25 252

π ππ

y t x m

m m sen sen

( , ) ,

, ,

= = =

= ⇒

0 0 0 05

0 05 0 05 0ϕ ϕϕ

ϕπ

0

0

1

2

=

= rad

λ

π

λ

ππ

= ⋅ = ⋅ =

= = = −

v T m s s m

km

m

12 5 0 08 1

2 21

2 1

, ,/

ωωπ π

π= = = −2 20 08

25 1

T ss

,

Tf Hz

s

v f m Hz m s

= = =

= = ⋅ =

1 110

0 1

0 2 10 2

,

,λ /

dm

m= = =λ2

32

1 5,

Page 73: 9523 gl fis_tx2_cas

77

60. La función de onda, y, representa matemáticamente elvalor de la elongación para cada punto del medio enfunción del tiempo.

El hecho de que la función de onda sea periódica res-pecto a la posición significa que cada cierta distan-cia, denominada longitud de onda, encontramos pun-tos del sistema en el mismo estado de vibración. Elhecho de que sea periódica en el tiempo significa quecada cierto intervalo de tiempo, denominado período,todo el sistema vuelve a estar en el mismo estado de vi-bración.

61. Dos partículas de un medio por el que se propaga unaonda están en fase cuando se encuentran en el mismoestado de vibración, con la misma elongación, veloci-dad y aceleración.

62. Energía mecánica de una onda armónica:

a) Se deduce calculando la energía cinética en elpunto de equilibrio, posición en que toda la ener-gía es energía cinética.

b) La unidad de la energía mecánica total es la de laenergía, el julio, J, en el SI.

63. La potencia de una onda, energía transmitida por uni-dad de tiempo, es proporcional al cuadrado de la am-plitud y al cuadrado de la frecuencia. Por tanto, si seduplica la potencia de un movimiento ondulatorio ymantenemos constante la frecuencia, la amplitud au-mentará en un factor Si por el contrario, se man-tiene constante la amplitud, la frecuencia será la que

aumentará en un factor

64. La amplitud de una onda está relacionada con la in-tensidad. La intensidad es proporcional al cuadradode la amplitud. La frecuencia, la longitud de onda y elperíodo se relacionan entre sí, pero son independien-tes de la amplitud.

65. La energía de la onda disminuye a medida que ésta sealeja del foco emisor. Ello se debe a que:

— La energía total disponible debe repartirse sobreuna superficie de onda mayor cuanto más nos ale-jamos del foco. Por tanto, la energía por unidadde superficie disminuye, lo que recibe el nombre deatenuación.

— El rozamiento de las partículas del medio y otrascausas producen una absorción de energía de laonda. La absorción depende de la naturaleza delmedio.

66. Datos: fmurc. = 120 000 Hz; fdelf. = 200 000 Hz;

vaire = 340 m/s; vagua = 1 435 m/s

a) Determinamos la longitud de onda correspondien-te al murciélago:

b) Determinamos la longitud de onda correspondien-te al delfín:

λmurcaire

murc

vf

msHz.

.

,= = = ⋅ −340

120 0002 83 10 33 m

2 .

2 .

E m A f m A= =212

2 2 2 2 2π ω

b) En primer lugar hallamos la pulsación:

ω = 2 πf = 2 π ⋅ 22,5 Hz = 45 π rad/s

Hallamos la diferencia de fase entre dos oscilacionesde un mismo punto separadas 1 s.

EJERCICIOS Y PROBLEMAS

55. Ondas mecánicas: se caracterizan por propagarse nece-sariamente a través de un medio material. La perturba-ción que transmiten es de tipo mecánico.

Ondas transversales: la dirección de propagación de laonda y la dirección del movimiento vibratorio genera-do por la perturbación son perpendiculares.

Ondas longitudinales: en este caso, la dirección de pro-pagación de la onda y la dirección del movimiento vi-bratorio generado por la perturbación son paralelas.

Ondas superficiales: se caracterizan por ser ondas bidi-mensionales, que se propagan en dos dimensiones a lolargo de una superficie.

Ondas armónicas: se caracterizan por ser generadaspor las perturbaciones producidas en el medio por unmovimiento armónico simple.

56. En las ondas armónicas, la velocidad de propagación, la frecuencia y la longitud de onda se relacionan porv = λ f.

57. Datos: f = 500 Hz; vhierro = 4 500 m/s; vaire = 340 m/s

a) La relación entre velocidad, longitud de onda y fre-cuencia, v = λ f, nos indica que la longitud de onda

es proporcional a la velocidad, . Por tanto, la

longitud de onda será mayor para el hierro que pa-ra el aire, ya que en el hierro es mayor la velocidadde propagación.

b) Calculamos cuántas veces es mayor en el hierroque en el aire:

58. Si se duplica el período de una onda armónica y la ve-locidad de propagación permanece constante, la longi-tud de onda también se duplica:

59. La velocidad de una onda en una cuerda tensa sólo de-pende de la tensión de la cuerda, T, y de su masa por

unidad de longitud, Por tanto, si modi-

ficamos la frecuencia pero mantenemos constante latensión de la cuerda, la velocidad permanece cons-tante.

μμ

: v =T

.

λ λ λ= ′ = ′ = ⋅ =v t v T v T; 2 2

λ

λhierro

aire

hierro

aire

hierro

aire

vf

vf

vv

= = =4 5000

34013 2,

ms

ms

=

λ =vf

Δϕ ω π π= − = ⋅ =( )t trad

ss rad2 1 45 1 45

5. Movimiento ondulatorio

Page 74: 9523 gl fis_tx2_cas

78

72. Datos: y = sen(0,5x − 200t + 2,5), en unidades SI

Si comparamos la ecuación dada con la expresión ge-neral de la función de onda, obtenemos:

A = 1 m; k = 0,5 m−1; ω = 200 rad/s; ϕ0 = 2,5 rad

Determinamos la frecuencia a partir de la pulsación:

Hallamos la longitud de onda a partir del número de on-das:

Calculamos la velocidad a partir de la longitud de on-da y la frecuencia:

73. Datos: y = 0,05 sen (1 992t − 6x), en unidades SI

Si comparamos la ecuación dada con la expresión ge-neral de la función de onda, obtenemos:

A = 0,05 m; ω = 1 992 rad/s; k = 6 m−1

a) La amplitud del movimiento es A = 0,05 m. Deter-minamos la frecuencia a partir de la pulsación delmovimiento:

De forma análoga, determinamos la longitud deonda a partir del número de ondas:

b) Para calcular la distancia recorrida por la onda en3 s determinamos primero la velocidad de propa-gación de la onda:

Entonces, la distancia recorrida en 3 s será:

c) La onda descrita por la función de onda se propa-ga en el sentido positivo del eje de las X, ya que eltérmino kx es negativo. La ecuación de una ondaidéntica pero que se propague en sentido contrarioserá y = 0,05 sen (1 992t + 6x).

74. Datos: y = 3 sen (8t − 0,5x)

Si comparamos la ecuación dada con la expresión ge-neral de la función de onda, obtenemos:

A = 3 m; ω = 8 rad/s; k = 0,5 m−1

x v tms

s m= = ⋅ =333 3 999

v f m Hz m s= = ⋅ =λ ,1 05 317 333 /

kk m

m= = = =−

2 2 2

61 051

π

λλ

π π; ,

ω πωπ π

= = = =22

1 992

2317f f

rads

Hz;

v f m Hz m s= = ⋅ =λ π ,4 31 8 400 /

kk m

m= = = =−

2 2 2

0 541

π

λλ

π ππ;

,

ω πωπ π

= = = =22

200

231 8f f

rads

Hz; ,

67. La potencia de un foco sonoro es la energía que emitepor unidad de tiempo. Su unidad en el SI es el J/s o elW. Se puede medir registrando la intensidad de un fo-co puntual a una distancia R. Como la intensidad es lapotencia por unidad de superficie, si multiplicamosla intensidad recibida por la superficie esférica de ra-dio R, obtenemos la potencia emitida.

68. El umbral de audición es la intensidad mínima que escapaz de captar el oído humano. El umbral de dolor es lamáxima intensidad de sonido que puede resistir el oí-do humano sin sensación dolorosa. Estos umbrales sonde intensidad y, por tanto, son independientes de lafrecuencia del sonido.

69. La intensidad de un violín es:

Un nivel de intensidad de 60 dB corresponde a una in-tensidad de sonido:

Por tanto, para tener un nivel de intensidad de 60 dBcon violines de 40 dB cada uno, necesitamos:

70. Calculamos el nivel de intensidad sonora si la intensi-dad se multiplica por 100; I’ = 100 I:

Por tanto, si la intensidad se multiplica por 100, el ni-vel de intensidad sonora aumenta en 20 dB.

71. Datos: L = 1,0 m; m = 10,0 g = 0,010 kg; T = 30 N

Determinamos la masa por unidad de longitud y, conella, la velocidad de propagación:

μ

μ

= = = ⋅

= =⋅

−mL

kgm

kgm

vT N

0 0101 0

1 0 10

30

1 0

2,,

,

, 11058 8

2−=

kgm

ms,

β′ =′

= = +10 10100

10 1000 0

log log log loII

II

gg

log log

II

II

0

0

10 2 20 10

⎝⎜⎞

⎠⎟

′ = +⎛

⎝⎜⎞

⎠⎟= +β

II0

20= + β

II

W

mW

m

violines1

62

82

1 0 10

1 0 10100=

⋅=

,

,

β = = =10 60 10 6

10

0 0 0

6

log ; log ; logII

II

II

== = = ⋅ ⋅

= ⋅

II

I IW

m

I

0

60

6 12210 10 1 0 10

1 0 10

; ,

, 662

W

m

β = = =10 40 10 41

0

1

0

1

0

log ; log ; logII

II

II

;;

, ,

10

10 10 1 0 10 1 0 1

4 1

0

14

04 12

2

=

= = ⋅ ⋅ = ⋅−

II

I IW

m00 8

2− W

m

λdelfagua

delf

v

f

msHz.

.

,= = = ⋅ −1 435

200 0007 2 10 33m

Page 75: 9523 gl fis_tx2_cas

79

Por tanto:

Si tenemos en cuenta que en el origen, x = 0, laelongación inicial es y0 = 0,5 y coincide con la am-

plitud, debe ser Por tanto,

la ecuación de la onda será:

b) Hallamos la diferencia de fase entre dos puntos se-parados 0,2 m:

77. Datos:

La relación entre amplitudes y distancias es:

78. Datos: T = 10 °C = 283 K; gaire = 1,4; R = 8,314 J ⋅K−1 ⋅mol−1;

Maire = 28,8 ⋅ 10−3 kg ⋅ mol−1

La velocidad del sonido en el aire a 10 °C es:

79. Datos: t = 5 s; v = 340 m/s

Calculamos la distancia a la que se encuentra la tor-menta, foco del trueno:

80. Datos: f = 5 Hz; vaire = 340 m/s; vagua = 1 435 m/s

Determinamos la longitud de onda en el aire:

λaireairevf

ms

Hzm= = =

340

568

x v t m s s m= = ⋅ =340 5 1700/

vRT

M

vJ K mol

aireaire

aire

aire

=

=⋅ ⋅ ⋅−

γ

1 4 8 314 1, , −−

− −

⋅ ⋅=

1

3 1

283

28 8 10338 2

,,

K

kg mol

ms

AA

RR

AA

R mR

R R m R m

1

2

2

1

1

2

1

1

1 1 1

53

3 5 2 5

=

=+

=

= + =

( )

;

RRm

m15

22 5= = ,

AA

R R m1

22 13 5= − =;

200 202

200 20 0 22

π ππ

π ππ

t x t x− +⎛⎝⎜

⎞⎠⎟

− − + +⎛⎝⎜

⎞( , )

⎠⎠⎟

⋅ =20 0 2 4π π, rad

y sen t x= − +⎛⎝⎜

⎞⎠⎟

0 5 200 202

, π ππ

sen .ϕ ϕπ

0 012

= ⇒ =

y sen t x= − +0 5 200 20 0, ( )π π ϕ

ω π π π

λ

= = ⋅ =

= =

2 2 100 200

10

100

f rad s

vf

msHz

Hz /

== = =

= −

0 12 2

0 1

20 1

, ;,

m km

m

π

λ

π

λ π

a) La amplitud del movimiento es A = 3 m. Deter-minamos la frecuencia a partir de la pulsación delmovimiento:

Calculamos el período como el inverso de la fre-

cuencia,

Determinamos la longitud de onda a partir del nú-mero de ondas:

b) Determinamos la velocidad de propagación de laonda a partir de la longitud de onda y la frecuencia:

Para x = 15 m y t = 4 s, la elongación será:

75. Datos: y = 0,4 cos (50t − 2x), en unidades SI

Si comparamos la ecuación dada con la expresión de lafunción de onda en coseno, y = A cos (ωt − kx), obtene-mos:

A = 0,4 m; ω = 50 rad/s; k = 2 m−1

a) Calculamos la velocidad de propagación de laonda:

b) Determinamos la elongación y la velocidad de vi-bración para t = 0,5 s y x = 20 cm = 0,2 m:

c) La elongación máxima es la amplitud del movi-miento, ymáx = 0,4 m. Determinamos la velocidadmáxima a partir de su expresión cuando el senovale −1:

76. Datos: f = 100 Hz; A = 0,5 m; v = 10 m/s; t0 = 0;

y0 = 0,5 m para x = 0

a) La ecuación de la onda es de la forma:

Determinamos la pulsación w y el número de on-das k a partir de la frecuencia f y de la velocidad v:

y A sen t kx= − +( )ω ϕ0

v sen t x v m s= − − =20 50 2 20( ); máx /

y t x

y

= − = ⋅ − ⋅

=

0 4 50 2 0 4 50 0 5 2 0 2

0

, cos( ) , cos( , , )

,3345

0 4 50 50 2 20 50, ( ) (

m

vdydt

sen t x sen t= = − ⋅ − = − − 22

20 50 0 5 2 0 2 10 16

x

v sen m s

)

( , , ) ,= − ⋅ − ⋅ = /

v f vk k

rads

m

ms

= = ⋅ = = =−

λπ ω

πω

;2

2

50

225

1

y sen t x sen

y m

= − = ⋅ − ⋅

= −

3 8 0 5 3 8 4 0 5 15

1 77

( , ) ( , )

,

v f m Hz m s= = ⋅ =λ , ,12 6 1 27 16 /

kk m

m m= = = = =−

2 2 2

0 54 12 61

π

λλ

π ππ;

,,

Tf Hz

s= = =1 1

1 270 79

,, .

ω πωπ π

= = = =22

8

21 27; ,f f

rads

Hz

5. Movimiento ondulatorio

Page 76: 9523 gl fis_tx2_cas

80

En esta gráfica se aprecia perfectamente que la fun-ción de onda representa una onda viajera.

84. Una manera de proceder puede ser:

— Definir la celda de entrada de la longitud de onda(C1).

— Definir la celda de entrada de la amplitud de on-da (C2).

— Colocar en una matriz columna valores de la varia-ble independiente, a intervalos de 0,1 s, entre 0 y10 s (B4:B104).

— Colocar en las celdas contiguas el cálculo de lafunción de onda en función de los valores intro-ducidos en las celdas de entrada de información.Así, por ejemplo, pondremos en C4 la fómula:

=C$2*SENO(2*PI()/C$1*B4)

— Traslado de la fórmula a la matriz (B4:B104) de latabla de valores.

La programación será:

— A continuación se define un gráfico asociado a latabla.

Para λ = 10 m y A = 4 m se obtiene:

EVALUACIÓN

1. Se define como onda transversal aquel movimiento on-dulatorio en el que la dirección de propagación de laperturbación y la dirección del movimiento de oscila-ción generado por la perturbación son perpendiculares.Por ejemplo, las ondas transmitidas por una cuerda sontransversales, ya que la propagación de la onda a lo lar-go de la cuerda y la dirección de vibración de la cuerda(subiendo y bajando) son perpendiculares.

Igualmente calculamos la longitud de onda en el agua:

81. Datos: fmín = 20 Hz; fmáx = 20 000 Hz; v = 340 m/s

Hallamos las correspondientes longitudes de onda:

82. Datos: y = 0,1 sen 2 π(2t − x/1,5), en unidades SI

Comparamos la ecuación dada con la expresión gene-

ral de la función de onda y = A sen 2 π paradeterminar:

a) Determinamos la frecuencia a partir del inversodel período:

La longitud de onda es λ = 1,5 m.Calculamos la velocidad de propagación de la on-da o velocidad de fase a partir de la longitud deonda y la frecuencia:

b) Los puntos que están en fase con el punto situadoen x = 2 m son aquellos que distan de él un núme-ro entero de longitudes de onda:

Los que están en oposición de fase distarán un nú-mero impar de medias de onda:

83. La representación sucesiva de las funciones:

— y = sen (x − 2 · 0)— y = sen (x − 2 · 0,2)— y = sen (x − 2 · 0,4)— y = sen (x − 2 · 0,6)— y = sen (x − 2 · 0,8)

da lugar a la gráfica:

x n m n m= + +⎛⎝⎜

⎞⎠⎟

= + +⎛⎝⎜

⎞⎠⎟

2 2 12

2 2 11 52

( ) ( ),

ccon n ∈ Ζ

x n m n m con n= + = + ∈( ) ( , ) ,2 2 1 5λ Ζ

v f m Hz= = ⋅ =λ ,1 5 2 3 m/s

fT s

Hz= = =1 1

12

2

A m T s m= = =0 112

1 5, ; ; ,λ

tT

x−

⎛⎝⎜

⎞⎠⎟λ

λ

λ

máxmín

mínmáx

= = =

= =

vf

ms

Hzm

vf

m

340

2017

340ssHz

m20 000

0 017,=

λaguaaguav

f

ms

Hzm= = =

1 435

5287

Page 77: 9523 gl fis_tx2_cas

81

Por tanto,

b) La longitud de onda es λ = 0,7 m. Podemos deter-minar el período como la inversa de la frecuencia,

La velocidad de

transmisión de la onda es v = 210 m/s.

c) El desplazamiento máximo coincide con la ampli-tud de la onda, ymáx = 0,1 m.

Para hallar la velocidad máxima, derivamos laecuación de la onda:

Para calcular la aceleración, derivamos la velocidad:

6. Datos: P = 5 W; R = 3 m

Calculamos la intensidad a 3 m, si suponemos que lapotencia se distribuye uniformemente sobre una su-perficie esférica:

7. Datos: β1 = 90 dB; 1 000 espectadores

Si 1 000 espectadores gritan a la vez, la intensidad delsonido emitido por cada uno de ellos se multiplicarápor 1 000. Debemos, pues, determinar la intensidad deun espectador a partir del nivel de intensidad sonora:

Por tanto, la intensidad de sonido total y el correspon-diente nivel de intensidad sonora son:

I I W m W mtot = = ⋅ ⋅ =

=

−1000 1000 1 0 10 1 0

1

13 2 2, / , /

β 00 101 0

1 0 1010

0

2

122

log log,

,lo

II

W

mW

m

tot =⋅

=−

gg 10

120

12

β = dB

β = = =

=

10 90 10 9

10

1

0

1

0

1

0

9

log ; log ; logII

II

II

I11

01

90

9 122

1

10 10 1 0 10

1 0 10

II I

W

m

I

; ,

,

= = ⋅ ⋅

= ⋅

−332

W

m

IR (3 m)2 2= = = =

PS

P W W

m4

5

40 044 2π π,

a =dydt

= t x

a =

− ⋅ −

60 600 600 9

36 000

π π πsen ( )

ππ π

π

2

2

sen t x

a m/smáx2

( )

,

600 9

36 000 3 6 104

= = ⋅ m/s2

v =dydt

= 0,1 600 cos t x

v = 60 cos

⋅ −π π

π

( )600 9

t x v 60 m/smáx( );600 9π π− =

Tf Hz

s= = = ⋅ −1 1300

3 3 10 3, .

y x= −0 1 600 9, sen ( ).π tSe define como onda longitudinal aquel movimiento ondulatorio en el que la dirección de propagación de la perturbación y la dirección del movimiento de oscila-ción generado por la perturbación son paralelas. Porejemplo, las ondas de compresión y expansión transmiti-das por un resorte son longitudinales, ya que la direcciónde propagación de las ondas y la dirección del movimien-to oscilatorio coinciden, son paralelas al resorte.

2. En la función de onda, la fase inicial ϕ0 determina el es-tado de vibración de cada punto x en el instante inicial,t = 0 s. En concreto, para x = 0, la elongación inicial se-rá y0 = A sen ϕ0.

3. Un vibrador produce ondas en la superficie de un es-tanque a intervalos regulares de tiempo. Si se ajusta elvibrador de modo que produzca un número triple deondas por segundo, en este caso, las ondas...

d) Tienen un tercio de la longitud de onda.

Como la velocidad de propagación se mantiene cons-tante, si se triplica la frecuencia, la longitud de ondadisminuye en un tercio.

4. Datos: y (x, t) = 0,02 sen π(20t + 2x), en unidades SI

Para calcular la aceleración, derivamos dos veces laelongación:

En x = −0,3 m:

5. Datos: L = 4,2 m; f = 300 Hz; A = 10 cm = 0,1 m;

Δt = 0,02 s

a) Suponemos que la fase inicial es cero y que las on-das se propagan en el sentido positivo de las X.Para escribir la ecuación de la onda necesitamos lapulsación y el número de ondas. Para determinar-los, calculamos la velocidad y la longitud de onda:

vLt

ms

m s

vf

ms

Hz

= = =

= = =

Δ4 20 02

210

210

300

,,

/

λ 00 7

2 20 7

9

2 2 300 6

1

,

,

m

km

m

Hz

= = =

= = ⋅ =

−π π

π πλ

ω f 000π /rad s

a = sen (20t + 2 )

a = sen

− ⋅ −

8 0 3

8

π π

π

2

2

( , )

(20t ) en unidades SIπ − 0 6,

v =dydt

= 0,02 20 cos (20t + 2x)

v = 0,4

⋅ π π

π ccos (20t + 2x)

a =d y

dt= 0,4 sen

2

2

π

π π π− ⋅ 20 (20t + 2x)

a = sen (20t + 2x)− 8 π π2

5. Movimiento ondulatorio

Page 78: 9523 gl fis_tx2_cas

83

Fenómenos ondulatorios6

Calculamos la velocidad de propagación de la onda apartir del número de ondas y de la pulsación:

1. FENÓMENOS BÁSICOS

1.

2. a) Falso. De hecho, es cierto que siempre se producedifracción al interceptar la propagación de unaonda. Pero sólo será apreciable cuando las dimen-siones del objeto u orificio que intercepta la ondasean iguales o menores que la longitud de onda dela perturbación.

b) Cierto. El principio de Huygens dice que todopunto de un frente de onda se convierte en uncentro puntual productor de ondas secunda-rias.

3.

vT T k m

ms

= = ⋅ = =⋅

=−

λ π λπ

ω22

200

306 7

1

1 ,rad s

PREPARACIÓN DE LA UNIDAD

• Las ondas mecánicas se definen como una perturbaciónque se propaga necesariamente a través de un medio mate-rial. La perturbación que transmiten es de tipo mecánico.

Se define como onda transversal aquel movimiento on-dulatorio en que la dirección de propagación de la per-turbación y la dirección del movimiento de oscilación ge-nerado por la perturbación son perpendiculares.

Se define como onda longitudinal aquel movimiento on-dulatorio en que la dirección de propagación de la per-turbación y la dirección del movimiento de oscilación ge-nerado por la perturbación son paralelas.

• La velocidad de propagación de una onda es la velocidada la que se desplaza la perturbación a través del medio,mientras que la velocidad de vibración de una partículaafectada por la onda corresponde a la velocidad con quecada punto del medio afectado por la onda se mueve res-pecto de su punto de equilibrio.

• La velocidad de una onda en una cuerda tensa sólo de-pende de la tensión de la cuerda, T, y de su masa por uni-

dad de longitud, μ:

• Datos: y = 0,5 sen (200t − 30x) (unidades SI)

Determinamos la amplitud de la onda por comparacióncon la ecuación general de una onda armónica, y = A sen(ωt − kx):

A = 0,5 m

De la misma manera, identificamos el número de ondas, k = 30 m−1, y la pulsación, ω = 200 rad ⋅ s−1.

Hallamos la longitud de onda a partir del número de ondas:

kk m

m= = = =−

2 2 2

300 211

π

λλ

π π; ,

vT

6. Fenómenos ondulatorios

Rayos del trende ondasparalelas

Frentes de onda paralelos

Frentes de ondacirculares

Ángulo ϕ sen ϕ cos ϕ

0 0 1

π2

1 0

π 0 −1

32π

−1 0

• Nuevos focos emisoresE

C

A

O

B

D

F

Rayos procedentes de O

Frente de onda

Ondas secundarias

Page 79: 9523 gl fis_tx2_cas

84

2. FENÓMENOS POR SUPERPOSICIÓN DE ONDAS

9. A pesar de que en el libro del alumno se ha estudiadoel caso de la interferencia de ondas armónicas, la in-terferencia es un fenómeno propio de todos los tiposde movimiento ondulatorio.

10. Cuando dos ondas interfieren constructiva o destruc-tivamente no se produce ninguna ganancia o pérdidade energía en el sistema, pero sí una redistribución dela energía. Los puntos donde las ondas interfierenconstructivamente tendrán más energía, provenientede los nodos o puntos de interferencia destructiva.

11. Datos: y = 0,25 cos [4 π(10t − x)] (SI); r = 1,5 m;r’= 1,75 m

Si comparamos la ecuación del enunciado con la ex-presión de una onda armónica, obtenemos:

A = 0,25 m; ω = 40 π rad⋅s−1; k = 4 π m−1

a) Deducimos la longitud de onda a partir del núme-ro de ondas:

b) Hallamos si la interferencia es constructiva o des-tructiva a partir del valor de la diferencia de reco-rridos:

Por tanto, como la diferencia de recorridos es unnúmero impar de semilongitudes de onda, la in-terferencia en este punto es destructiva.

12. Datos: f = 40 Hz; v = 20 cm ⋅ s−1 = 0,2 m ⋅ s−1

Calculamos la longitud de onda:

a) Determinamos el tipo de interferencia en el puntoA a partir del valor de la diferencia de recorridos:

Por tanto, como la diferencia de recorridos es unnúmero entero de longitudes de onda, la interfe-rencia en este punto es constructiva.

b) Determinamos el tipo de interferencia en el pun-to B a partir del valor de la diferencia de recorri-dos:

Por tanto, como la diferencia de recorridos es unnúmero impar de semilongitudes de onda, la in-terferencia en este punto es destructiva.

r r cm cm cm

r r cmc

− = − =

−=

8 6 25 1 75

1 750 5

, ,

,,λ mm

= =3 572

,

r r cm cm cm

r r cmcm

− = − =

−=

12 10 5 1 5

1 50 5

, ,

,,λ

== 3

λ = = = ⋅ =−vf

m m0 2

5 10 0 53,,

m/s40 Hz

r r m m m

r r mm

− = − =

−= =

1 75 1 5 0 25

0 250 5

1

, , ,

,,λ 22

λπ π

π= = =−

2 2

40 51k m

m,

4. Datos: n21 >1

— Como n21 es el cociente de velocidades, la velocidaden el nuevo medio es menor que en el medio 1:

— La frecuencia se mantiene constante.

— El período, que es el inverso de la frecuencia, serátambién constante.

— La longitud de onda se relaciona con la velocidady el período como λ = v T. Si T es constante y la ve-locidad v disminuye, también la longitud de ondaserá menor.

— El ángulo de refracción será menor que el ángulode incidencia, ya que:

5. Datos: v1 = 0,1 m⋅s−1; λ1 = 0,02 m; i = 30°; λ2 = 0,01 m

a) Hallamos la frecuencia de la onda a partir de suvelocidad y su longitud de onda en el primer me-dio:

b) Determinamos la velocidad de la onda en el se-gundo medio a partir de la frecuencia, que se man-tiene constante, y de la longitud de onda en estemedio:

c) Calculamos el seno del ángulo de refracción a par-tir de la ley de Snell:

6. Falso. Esta afirmación es verdadera para las ondas po-larizadas linealmente pero no en el caso de la polariza-ción circular o elíptica. En éstas, las partículas del me-dio no vibran todas en el mismo plano, como en lapolarización lineal, sino que cada una se mueve descri-biendo círculos en un plano perpendicular a la direc-ción de propagación.

7. La polarización es una característica de las ondas trans-versales porque consiste en limitar las direcciones de vi-bración de las partículas del medio. En el caso de lasondas longitudinales, la dirección de vibración estásiempre limitada, ya que coincide con la dirección depropagación. Por tanto, no tiene sentido hablar de po-larización en ondas longitudinales.

8. No es posible polarizar las ondas sonoras ya que se tra-ta de ondas longitudinales. Sólo es posible polarizarondas transversales.

sensen

sen sen,,

ir

vv

rvv

i

=

= ⋅ =

1

2

2

1

0 050 1

m/ssen ,

m/s⋅ ° =30 0 25

v f m Hz2 2 0 01 5 0 05= = ⋅ =λ , , m/s

fv

mHz= = =1

1

0 10 02

,,

m/s

sensen

; sen sen ;ir

n i r i r= > > >21 1

nvv

v v211

21 21= > >;

Page 80: 9523 gl fis_tx2_cas

85

17. Respuesta sugerida:

El período de las pulsaciones se relaciona con las fre-cuencias de los diapasones mediante la expresión:

Por lo tanto, cuanto más parecidas son ambas frecuen-cias, mayor será el período de las pulsaciones.

Al realizar la práctica se observa que cuanto más pare-cida es la posición de las piezas de modificación de fre-cuencia, el período de las pulsaciones es mayor. Así,podemos concluir que:

a) La frecuencia de los diapasones depende de la po-sición de las piezas de modificación.

b) Cuanto más parecida es la posición de estas piezasen ambos diapasones, más cercanos están los valoresde sus frecuencias y mayor es el período de las pul-saciones.

18. a) Para que se produzca una onda estacionaria es ne-cesaria la interferencia de dos ondas armónicas deamplitud y frecuencia iguales que se propaguenen la misma dirección y sentido contrario.

b) Todos los puntos de la cuerda en la que se produ-ce una onda estacionaria, excepto los nodos, osci-lan armónica y verticalmente y alcanzan a la vez laposición de equilibrio.

19. Datos: y1 = A sen (ωt + kx); y2 = A sen (ωt − kx + ϕ)

a) La superposición de estas dos ondas será:

Se trata de una onda armónica de igual frecuen-cia que las ondas componentes y con una ampli-tud Ar independiente del tiempo, donde la fasede la oscilación es independiente del punto con-siderado. Por tanto, todos los puntos de la ondaalcanzan a la vez la posición de equilibrio y los no-dos se encuentran siempre en reposo. En definiti-va, se trata de una onda estacionaria.

b) Si en x = 0 hay un nodo, la amplitud es siemprenula, para todo t. Por tanto:

cos coskx −⎛⎝⎜

⎞⎠⎟

= = ⇒ =ϕ2

ϕ2

ϕ0 π

y y y

y A t kx A t kx

y

r

r

r

= +

= + + − +

=

1 2

sen ( ) sen ( )ω ω ϕ

222

At kx t kx

t k

sen( ) ( )

cos(

ω ω

ω

+ + + +⎡

⎣⎢

⎦⎥ ⋅

⋅+

ϕ

xx t kx

y A tr

) ( )

sen co

− − +⎡

⎣⎢

⎦⎥

= +⎛⎝⎜

⎞⎠⎟

ω

ω

ϕ

ϕ2

2

2 ss

sen ;

kx

y A t Ar r

−⎛⎝⎜

⎞⎠⎟

= +⎛⎝⎜

⎞⎠⎟

ϕ2

ϕ2

ω rr A kx= −⎛⎝⎜

⎞⎠⎟

2 cosϕ2

Tf f

=−1

1 2

f f f f f f Hz Hz Hzp p= − = + = + =2 1 2 1 349 3 352;c) Determinamos el tipo de interferencia en el pun-to C a partir del valor de la diferencia de recorri-dos:

Por tanto, como la diferencia de recorridos es unnúmero entero de longitudes de onda, la interfe-rencia en este punto es constructiva.

13. Datos: r’ = 26 cm = 0,26 m; r = 25,8 cm = 0,258 m;

v = 1 200 m/s

La condición que cumple el primer nodo o mínimo deamplitud es:

Si tenemos en cuenta la relación entre la longitud deonda, la velocidad de propagación y la frecuencia, po-demos escribir la condición del primer mínimo en laforma:

14. a) Cierto.

b) Cierto. siempre que ambas tengan la misma veloci-dad de propagación, ya que en este caso tendrántambién distinta frecuencia.

c) Falso. No aparecerán pulsaciones a no ser que tam-bién tengan frecuencias próximas.

d) Falso. Si tienen velocidades distintas pero próxi-mas, y la misma frecuencia, no se producirán pul-saciones.

15. Datos: f1 = 430 Hz; f2 = 436 Hz

a) Calculamos la frecuencia de la onda resultantecomo el promedio de f1 y f2:

b) Hallamos la frecuencia de la pulsación y su período:

16. Datos: f1 = 349 Hz; 3 pulsaciones por segundo

La frecuencia correspondiente a 3 pulsaciones por se-gundo es de 3 Hz, fp = 3 Hz. Por tanto, si suponemosque la frecuencia conocida es la mayor:

Pero también es posible que la frecuencia conocida seala menor. En tal caso:

f f f f f f

f Hz Hz Hzp p= − = −

= − =1 2 2 1

2 349 3 346

;

f f f Hz Hz Hz

Tf Hz

p

p

= − = − =

= = =

2 1 436 430 6

1 16

0 17, s

ff f Hz Hz

Hz=+

=+

=1 2

2430 436

2433

r rvf

fv

r r

f

′′

− = =−

=

2 2

1 200

;( )

m/s2 (0,26 mm −

= ⋅0 258

3 105

, )mHz

r r′ − =λ2

r r cm cm cm

r r cmcm

− = − =

−= =

9 5 4

40 5

8,λ

6. Fenómenos ondulatorios

Page 81: 9523 gl fis_tx2_cas

86

Si la cuerda está fija sólo por un extremo, la longitud

de onda de las ondas estacionarias ha de ser

22. Datos: L = 2 m; A(x = 1 m) = 0,1 m; v = 4 m/s; cuerdasujeta por los dos extremos; n = 1

a) La longitud de onda de la onda estacionaria es:

b) Hallamos la frecuencia a partir de la velocidad depropagación y la longitud de onda:

c) Como la cuerda está fija en sus dos extremos, x = 0y x = L deben ser nodos de la onda estacionaria.Su ecuación es:

23. Datos: L = 1,2 m; v = 130 m ⋅ s−1

Las frecuencias de la serie armónica en una cuerdafija por los dos extremos vienen dadas por la expre-sión:

Calculamos las frecuencias correspondientes a los tresprimeros armónicos, n = 1, 2, 3:

24. Datos: L = 1 m; f1 = 430 Hz; cuerda fija por un ex-tremo

Hallamos la velocidad de propagación de las ondas enla cuerda a partir de la expresión del primer armóni-co para una cuerda fija por un extremo:

25. Las longitudes de onda de cada uno de los modos nor-males de vibración son las mismas en la cuerda fija porlos dos extremos que en el tubo de la misma longitud

abierto por los dos extremos: λ =2 Ln

fvL

v L f

v m Hz

1 144

4 1 430 1720

= =

= ⋅ ⋅ =

;

m/s

fvL m

Hz

fvL

1

2

12

11302 1 2

54 2

22

2

= ⋅ = ⋅⋅

=

= ⋅ =

,,

m/s

⋅⋅⋅

=

= ⋅ = ⋅

1302 1 2

108 3

32

3130

3

,,

m/s

m/s

mHz

fvL 22 1 2

162 5⋅

=,

,m

Hz

f nvL

n= =2

1 2 3, , ...

y A k x t

y A x

r

r

=

=⎛⎝⎜

⎞⎠⎟

2

22

sen ( ) cos ( )

sen

ω

π

λcos ( )

, sen cos (

2

0 124

2 1

π

ππ

f t

y xr =⎛⎝⎜

⎞⎠⎟

⋅ ⋅⋅

=⎛⎝⎜

⎞⎠⎟

t

y x t SIr

)

, sen cos ( ) ( )0 12

π

fv

mHz= = =

λ

44

1m/s

λ λ= =⋅

=2 2 2

14

Ln

mm;

λ =4 Ln

.

c) La elongación máxima en cualquier punto se da enel momento en que sen(ωt + π/2) = 1. Por tanto,en x = λ/4:

Derivamos la elongación respecto del tiempo parahallar la velocidad:

20. Datos: y = 0,3 cos (0,2x − 100t) (SI)

a) Si comparamos la ecuación del enunciado con laexpresión de una onda armónica, obtenemos:

A = 0,3 m; ω = 100 rad ⋅ s−1; k = 0,2 m−1

Hallamos la longitud de onda a partir del númerode ondas; la frecuencia, a partir de la pulsación; yla velocidad de propagación, a partir de la frecuen-cia y de la longitud de onda:

b) La onda que se propaga en el sentido contrario tie-ne por ecuación y = 0,3 cos (0,2 x + 100t). Por lotanto, la ecuación de la onda estacionaria resultan-te será:

c) La distancia entre dos nodos consecutivos será:

21. En una cuerda con sus dos extremos fijos sólo pue-den formarse ondas estacionarias de longitud de onda

con L la longitud de la cuerda y n un núme-

ro entero.

λ =2Ln

,

x x x xm

mn n n n− = − = =− −1 1210

25

λ ππ;

y y y

y x tr

r

= +

= − +1 2

0 3 0 2 100 0 3 0 2, cos ( , ) , cos ( , xx t

yx t x t

r

+

= ⋅− + +

100

2 0 30 2 100 0 2 100

)

, cos( , ) ( , ))

cos( , ) ( , )

2

0 2 100 0 2 1002

⎣⎢

⎦⎥ ⋅

⋅− − +⎡

x t x t⎢⎢

⎦⎥

= −

=

y x t

yr

r

0 6 0 2 100

0 6

, cos ( , ) cos ( )

, coss ( , ) cos ( ) ( )0 2 100x t SI

λπ π

π

ωπ π

= = =

= =⋅

=

2 2

0 210

2100

250

1

1

k mm

f

,

rad sππ

λ ππ

Hz

v f m Hzms

= = ⋅ =1050

500

vd ydt

A kx t= = −⎛

⎝⎜

⎠⎟ +

⎝⎜

⎠2

2 2cos cosω

πω

π⎟⎟

= = ⋅ −⎛

⎝⎜

⎠⎟ +

⎛v x A t( cos cosλ ω

π

λλ π

ωπ

/4) 22

4 2 2⎝⎝⎜

⎠⎟

= = ⋅ +⎛

⎝⎜

⎠⎟

=

v x A t

v x

( cos cos

(

λ ω ωπ

/4) 2 02

λλ ω ω/4)= −2 sen( )A t

y A k

y A

máx

máx

= −⎛⎝⎜

⎞⎠⎟

= ⋅ −

24 2

22

4

cos

cos

λ π

π

λλ ππ

22 0 2

⎛⎝⎜

⎞⎠⎟

= =cosA A

Page 82: 9523 gl fis_tx2_cas

87

3. FENÓMENOS DEBIDOS AL MOVIMIENTO DE LA FUENTE Y DEL RECEPTOR

29. El efecto Doppler es un fenómeno común a todas lasondas armónicas.

30. Si una fuente sonora y un observador se mueven conla misma velocidad, dirección y sentido, no habráefecto Doppler. El efecto Doppler sólo aparece cuan-do existe un movimiento relativo entre el observadory la fuente y en la dirección de la línea que los une. Sila velocidad relativa es nula, no aparece el efectoDoppler.

31. Datos: vF = 30 m ⋅ s−1; f = 600 Hz; v = 340 m ⋅ s−1

a) Si el automóvil se aproxima:

b) Si la fuente se aleja del receptor:

32. Datos: vR = 15 m ⋅ s−1; f = 1 000 Hz; v = 340 m ⋅ s−1

a) Si el automóvil se aproxima a la sirena:

b) Si el conductor se aleja de la fuente:

33. Datos: vF = 40 m ⋅ s−1; vR = 20 m⋅s−1; f = 980 Hz; v = 340 m ⋅ s−1

a) Si el automóvil se aproxima a la ambulancia:

b) Si el conductor y la fuente se alejan uno del otro:

f fv vv v

f Hz

RR

F

R

=−

+

= ⋅⋅ − ⋅− −

980340 20

340

1 1m s m s,

m s m s⋅ + ⋅=− −1 140

825 3 Hz

f fv vv v

f Hz

RR

F

R

=+

= ⋅⋅ + ⋅− −

980340 20

340

1 1m s m s

m s m s⋅ − ⋅=− −1 140

1176 Hz

f fv v

v

f Hz

RR

R

=−

= ⋅⋅ − ⋅− −

1000340 15

340

1 1m s m s

m ⋅⋅=−s 1 955 9, Hz

f fv v

v

f Hz

RR

R

=+

= ⋅⋅ + ⋅− −

1000340 15

340

1 1m s m s

m ⋅⋅=−s 1 1044 1, Hz

f fv

v v

f Hz

RF

R

=+

= ⋅⋅

⋅ + ⋅

−600340

340 30

1

1

m s

m s m ss− =1 551 4, Hz

f fv

v v

f Hz

RF

R

=−

= ⋅⋅

⋅ − ⋅

−600340

340 30

1

1

m s

m s m ss− =1 658 1, Hz

De la misma forma, la expresión de la longitud deonda de los modos normales de vibración de un tuboabierto por un extremo coincide con la de los de unacuerda fija por un extremo de la misma longitud:

26. Datos: fn = 300 Hz; fm = 425 Hz

a) fn y fm son armónicos de la misma frecuencia fun-damental f. Por tanto, fn = n f y fm = m f. Entonces,la frecuencia fundamental es el máximo comúndivisor de fn y fm:

b) Conocida la frecuencia fundamental, hallamos elorden de los armónicos:

27. Datos: L = 1,25 m; v = 342 m ⋅ s−1

a) Hallamos la frecuencia de los tres primeros armó-nicos si el tubo está abierto por sus dos extremos:

b) Calculamos la frecuencia de los tres primeros ar-mónicos si el tubo está abierto sólo por uno de susextremos:

28. El sonido amplificado de la voz humana manteniendocierto tono durante varios segundos es capaz de rom-per una copa de cristal debido a un fenómeno de reso-nancia. Este fenómeno sólo aparece en el caso de quela frecuencia correspondiente al tono de la voz huma-na coincida con la frecuencia de uno de los modos nor-males de vibración del cristal de la copa. En tal caso, lavoz es capaz de estimular una vibración en el cristalcuya amplitud irá en aumento. Si la amplitud de la vi-bración del cristal llega a superar los límites de elastici-dad de la estructura de la copa, ésta se romperá.

f nvL

n

fm

= =

= ⋅⋅

=

41 3 5

1342

1 251

; , , ...

,m/s

4668 4

3342

1 25205 2

53

2

3

,

,,

Hz

fm

Hz

f

= ⋅⋅

=

= ⋅

m/s4

4421 25

342,m/s

4 ⋅=

mHz

f nvL

n

fm

= =

= ⋅⋅

=

21 2 3

1342

1 251

; , , ...

,m/s

21136 8

2342

1 25273 6

3

2

3

,

,,

Hz

fm

Hz

f

= ⋅⋅

=

= ⋅

m/s23342

1 25410 4

,,

m/s2 ⋅

=m

Hz

nff

nHzHz

mff

m

n

m

= = =

= =

;

;

30025

12

425 HzHz25

17=

f Hz Hz

f Hz Hz

f

n

m

= = ⋅ ⋅

= = ⋅

=

300 2 3 5

425 5 17

25

2 2

2

HHz

λ =4 Ln

6. Fenómenos ondulatorios

Page 83: 9523 gl fis_tx2_cas

88

c) Calculamos la posición de los nodos:

d) La distancia entre dos nodos consecutivos será:

36. Datos:

a) La ecuación general de una onda estacionaria gene-rada por superposición de dos ondas iguales que semueven en sentido contrario es yr = 2A sen (kx) cos(ωt). Si comparamos con la ecuación del problema,vemos que la amplitud de las ondas que pueden ge-nerarla por superposición es A = 1 m. Deducimostambién que k = π/3 m−1 y ω = 40π rad ⋅ s−1. Por tan-to, la velocidad de propagación de las ondas genera-doras es:

b) La distancia entre dos nodos consecutivos será:

c) Hallamos la velocidad en cualquier punto de lacuerda derivando la ecuación de la elongación:

Para la partícula situada en x = 1,5 m cuandot = 1,125 s, la velocidad es:

37. Datos: vF = 33 m ⋅ s−1; vR = 25 m ⋅ s−1; f = 400 Hz; v = 340 m ⋅ s−1

Antes de ser adelantado, cuando la ambulancia seaproxima al automóvil y éste se aleja de ella:

f fv vv v

f Hz

RR

F

R

=−

= ⋅⋅ − ⋅− −

400340 25

340

1 1m s m s,

m s m s⋅ − ⋅=− −1 133

410 4 Hz

vm

= − ⋅⎛⎝⎜

⎞⎠⎟

⋅ ⋅ =803

1 5 40 1 125 0ππ

πsen , sen ( , )ss

vdydt

x t

v

= = − ⋅⎛⎝⎜

⎞⎠⎟

= −

2 403

40

80

ππ

πsen sen ( )

πππ

πsen sen ( ) ( )3

40x t SI⎛⎝⎜

⎞⎠⎟

x x

x xk m

m

n n

n n

− =

− = = =

−−

1

11

222

3

3

λ

π ππ

vT k m

ms

= = =⋅

=−

λ ω ππ

40

3

1201

1

rad s

y x t SIr =⎛⎝⎜

⎞⎠⎟

23

40sen cos ( ) ( )π

π

x x x xm

mn n n n− = − = =− −1 1220

210

λ ππ;

x n nm

x n m

= + = +

= +

( ) ( )

( )

2 14

2 120

42 1 5

λ π

π

cos( , ) ( , )100 0 1 100 0 1

2⋅

− − +⎡

⎣⎢

⎤t x t x

⎦⎦⎥

= −

=

y t x

yr

r

2 4 100 0 1

2 4

, cos ( ) cos ( , )

, cos (( , ) cos ( ) ( )0 1 100x t SI

34. Datos: v = 340 m ⋅ s−1; si el tren se acerca, fR = 704 Hz;si el tren se aleja, fR = 619 Hz

Si el tren se acerca, la frecuencia percibida por el re-ceptor es:

Si el tren se aleja, la frecuencia percibida es:

Tenemos así un sistema de dos ecuaciones y dos incóg-nitas. Si dividimos la primera ecuación entre la segun-da, obtenemos la velocidad de la fuente:

Hallamos la frecuencia a partir de la primera ecuación:

RESOLUCIÓN DE EJERCICIOS Y PROBLEMAS

35. Datos: y = 1,2 cos (100t − 0,1x) (SI)

a) Si comparamos la ecuación del enunciado con laexpresión de una onda armónica, obtenemos:

A = 1,2 m; ω = 100 rad ⋅ s−1; k = 0,1 m−1

Hallamos la longitud de onda a partir del númerode ondas; la frecuencia, a partir de la pulsación; yla velocidad de propagación, a partir de la frecuen-cia y de la longitud de onda:

b) La onda que se propaga en sentido contrario tienepor ecuación y = 1,2 cos (100t + 0,1x). Por lo tantola ecuación de la onda estacionaria resultante será:

y y y

y t xr

r

= +

= − +1 2

1 2 100 0 1 1 2 100, cos ( , ) , cos ( tt x

yt x t x

r

+

= ⋅− + +

0 1

2 1 2100 0 1 100 0 1

, )

, cos( , ) ( , ))

2⎡

⎣⎢

⎦⎥ ⋅

λπ π

π

ωπ π

= = =

= =⋅

=

2 2

0 120

2100

250

1

1

k mm

f

,

rad sππ

λ ππ

Hz

v f m Hzms

ms

= = ⋅ = =2050

1 000 103

704340

340 21 8

704

1

1 1,Hz f

Hz

=⋅

⋅ − ⋅

− −

m s

m s m s

⋅⋅ ⋅ = ⋅ ⋅

=⋅ ⋅

− −318 2 340

704 318 2

1 1,

,

m s m s

m s

f

fHz −−

−⋅=

1

1340659

m sHz

704619

340

340

704 3

1

1

(

HzHz

v

vF

F

=⋅ +

⋅ −

m s

m s

440 619 340

239 36

1 1 1) ( )m s m s m s⋅ − = ⋅ ⋅ +− − −v vF F

00 7 004 210 460 619

1323

1 1m s m s⋅ − = ⋅ +− −v v

vF F

F == ⋅

= ⋅

28 900

21 8

1

1,

m s

m sv F

f fv

v vHz f

vRF F

=+

=⋅

⋅ +

−; 619340

340

1

1

m s

m s

f fv

v vHz f

vRF F

=−

=⋅

⋅ −

−; 704340

340

1

1

m s

m s

Page 84: 9523 gl fis_tx2_cas

89

Las leyes de la refracción son:

1.a El rayo refractado, la normal a la superficie en elpunto de incidencia y el rayo incidente están enel mismo plano.

2.a La razón entre el seno del ángulo de incidencia yel del ángulo de refracción es una constante iguala la razón entre las respectivas velocidades de pro-pagación del movimiento ondulatorio:

43. La velocidad de propagación de las ondas sonoras en el agua es mayor que en el aire. Por tanto, el índice derefracción relativo del agua respecto del aire será

Como el cociente del seno del ángulo

incidente y el seno del ángulo refractado es igual al ín-dice de refracción relativo n21:

Por tanto, la onda refractada se alejará de la normal.

44. Una onda que viaja por una cuerda tensa, al reflejarseen una pared, sufre una inversión de fase. Por tanto, laonda reflejada está en oposición de fase con la onda in-cidente.

La inversión se debe a que la onda produce una fuer-za hacia arriba sobre la pared que, debido a la terceraley de Newton, provoca una fuerza de reacción haciaabajo sobre la cuerda, fuerza que genera la onda refle-jada en oposición de fase con la incidente.

Si la ecuación de la onda incidente es y = 0,02 sen (50t −− 3x), la onda reflejada, que viajará en sentido contra-rio y con un desfase de π/2, será:

y′ = 0,02 sen (50t + 3x − π/2) = 0,02 cos (50t + 3x)

45. Polarización: limitación en la dirección o direccionesde vibración de los puntos afectados por una ondatransversal.

Polarización rectilínea o lineal: una onda está polariza-da rectilínea o linealmente si la vibración tiene lugarsiempre siguiendo rectas con la misma dirección per-pendicular a la dirección de propagación.

Polarización circular: hablamos de polarización circu-lar cuando la vibración de un punto a lo largo deltiempo tiene lugar siguiendo círculos situados en pla-

sensen

; sen senir

n i r r i= < < ⇒ >21 1

nvv21

1

2

1= < .

sensen

ir

vv

n= =1

221

Después de ser adelantado, la fuente se aleja del recep-tor pero éste se dirige hacia ella:

38. Datos: vF = 30 m ⋅ s−1; f = 150 Hz; v = 340 m ⋅ s−1

a) El camionero que circula tras él con una velocidadde vR = 20 m⋅s−1 se acercaría a la fuente si ésta se en-contrara en reposo. Al mismo tiempo, la fuente sealeja del receptor. Por lo tanto:

b) El automovilista que circula en sentido contrariocon una velocidad de vR = 28 m ⋅ s−1 se acerca a lafuente. Por tanto:

EJERCICIOS Y PROBLEMAS

39. Principio de Huygens: todo punto de un frente de ondase convierte en un centro puntual productor de on-das elementales secundarias, de igual velocidad y fre-cuencia que la onda inicial, cuya superficie envolventeconstituye un nuevo frente de onda.

El principio de Huygens es aplicable a todas las ondas,incluidas las electromagnéticas.

40. La difracción consiste en la desviación en la propaga-ción rectilínea de las ondas cuando éstas atraviesan unaabertura o pasan próximas a un obstáculo. La difracciónse produce tanto en ondas longitudinales (por ejemplo,el sonido) como en ondas transversales (por ejemplo, laluz).

41. Podemos oír la conversación que mantienen unaspersonas al otro lado de la esquina de un edificio, sinque podamos verlas, porque las ondas sonoras se di-fractan en la esquina. Es decir, su trayectoria rectilí-nea se ve alterada y las ondas son capaces de «doblar»la esquina.

En cambio, si se sitúan detrás de una casa y delante dela fachada, la pared refleja todo el sonido. Si éste nollega a una esquina cercana, no se podrá difractar y lle-gar hasta nosotros.

42. El índice de refracción relativo de un medio respecto aotro se define como la razón entre las velocidades depropagación de un movimiento ondulatorio en los dosmedios.

f fv vv v

f Hz

RR

F

R

=+

= ⋅⋅ + ⋅− −

150340 28

340

1 1m s m s

m s m s⋅ − ⋅=− −1 130

178 Hz

f fv vv v

f Hz

RR

F

R

=+

+

= ⋅⋅ + ⋅− −

150340 20

340

1 1m s m s,

m s m s⋅ + ⋅=− −1 130

145 9 Hz

f fv vv v

f Hz

RR

F

R

=+

+

= ⋅⋅ + ⋅− −

400340 25

340

1 1m s m s,

m s m s⋅ + ⋅=− −1 133

391 4 Hz

6. Fenómenos ondulatorios

Onda incidente

Onda reflejada

Page 85: 9523 gl fis_tx2_cas

90

51. Un tubo abierto por los dos extremos tiene como fre-

cuencia fundamental mientras que un tubo

abierto sólo por un extremo tiene por frecuencia funda-

mental Si el segundo tubo tiene una longitud

L′= L/2, las frecuencias fundamentales coincidirán.

52. El efecto Doppler en la luz sólo tiene efectos aprecia-bles para movimientos con grandes velocidades. Ade-más, como la longitud de onda de la luz visible es muypequeña, los correspondientes desplazamientos Dop-pler son también pequeños.

Sin embargo, el efecto Doppler en la luz visible se pue-de apreciar en la luz de las estrellas, donde las veloci-dades implicadas son muy grandes.

53. Datos: t1 = 8 s; t2 = 12 s; v = 340 m ⋅ s−1

Si tardamos 8 segundos en oír la explosión proceden-te del barco:

Es decir, estamos a 2 720 m del barco.

Si tardamos 12 segundos en oír el eco procedente delos acantilados, como a los 8 segundos el sonido noshabía llegado desde el barco, las ondas han tardadot3 = 12 s − 8 s = 4 s en viajar de nuestra barca al acan-tilado y volver. En este tiempo han recorrido 2d2, don-de d2 es la distancia que nos separa del acantilado:

54. Datos: f = 225 Hz; v1 = 120 m ⋅ s−1; v2 = 210 m ⋅ s−1

a) Calculamos el índice de refracción del segundomedio respecto al primero:

b) Hallamos la longitud de onda en el primer medio,donde v1 = 120 m ⋅ s−1:

En el segundo medio:

55. Datos: f = 50 Hz; A = 2 cm; v = 100 cm ⋅ s−1; r = 5 cm;r′= 9 cm

Hallamos primero el número de ondas y la pulsaciónde estas ondas armónicas:

ω π π π

π

λ

π π

= = ⋅ = ⋅

= = =

−2 2 50 100

2 2 2

1f Hz

kf

rad s

v/f vv v

kc

cm

=

=⋅

⋅=

−−

ω

ππ

100

100

1

11rad s

m s

λ λ22

2

1210225

0 93= =⋅

=−v

f Hzm; ,

m s

λ λ11

1

1120225

0 53= =⋅

=−v

f Hzm; ,

m s

nvv21

1

2

1

1

120

2100 57= =

⋅=

− ,m s

m s

212

12

340 4 6802 3 2 31d v t d v t s= = = ⋅ ⋅ ⋅ =−; m s m

d v t s m1 11340 8 2 720= = ⋅ ⋅ =−m s

fvL1

′=

4.

fvL1 2

= ,

nos perpendiculares a la dirección de propagación dela onda.

Polarización elíptica: hablamos de polarización elípti-ca cuando la vibración de un punto a lo largo del tiem-po tiene lugar siguiendo elipses situadas en planos perpendiculares a la dirección de propagación de laonda.

Podemos conseguir una onda polarizada linealmentesi sacudimos arriba y abajo el extremo libre de unacuerda fija, de modo que sus puntos vibren siempre enel mismo plano. Si hacemos vibrar el extremo libre dela cuerda formando círculos o elipses, obtendremosuna onda polarizada circular o elípticamente.

46. a) Falso. La interferencia es constructiva en los pun-tos donde las ondas llegan en concordancia defase. Si las dos fuentes están separadas una distan-cia arbitraria, aunque emitan en fase, las ondas nollegarán a todos los puntos del espacio en fase, y lainterferencia no será siempre constructiva.

b) Falso. Como la diferencia de fase entre ambas semantiene constante, se trata de fuentes coherentes.

47.

48. Si dos violinistas separados 2 metros tocan la mismanota, existirán puntos de la habitación donde la inter-ferencia será destructiva. La nota no se oirá en esospuntos sólo si tocan los dos con la misma intensidad,de forma que las ondas tengan la misma amplitud.

49. Si una cuerda está vibrando con seis vientres, los nodospueden tocarse sin perturbar el movimiento, ya queson puntos de vibración nula.

Si la cuerda tiene seis vientres y está fijada por los ex-tremos (que también son nodos), habrá siete puntosen total que puedan tocarse sin perturbar el movimien-to. Si sólo está fija por un extremo, los nodos serán seis.

50. La cuerda vibrará con la frecuencia fundamental y lacorrespondiente longitud de onda, determinadas am-bas por la posición de los dedos del violinista (es decir,por la longitud efectiva de la cuerda). Pero, además, sesuperpondrán otros armónicos, que serán los responsa-bles del timbre característico del violín.

A

T

t

Tp

Page 86: 9523 gl fis_tx2_cas

91

a) Si comparamos la ecuación de la onda del proble-ma con la ecuación general de una onda estacio-naria, y = 2 A cos (kx) sen (ωt), obtenemos:

Calculamos la longitud de onda y la velocidad depropagación:

b) Calculamos la posición de los nodos:

La distancia entre dos nodos o dos vientres consecu-tivos es media longitud de onda. Por tanto, comoλ = 12 m, la distancia entre dos nodos o dos vien-tres es de 6 m. Pero como nodos y vientres estánalternados y equiespaciados, la distancia entre unnodo y el vientre siguiente será de un cuarto delongitud de onda, es decir, de 3 m.

c) Hallamos la velocidad en cualquier punto de lacuerda derivando la ecuación de la elongación:

Para la partícula situada en x = 6 m:

La velocidad máxima corresponde al instante enque sen (5 πt) = 1; por tanto, vmáx = 10 π m ⋅ s−1.

60. Datos: L = 1,2 m; f = 120 Hz; 4 vientres

a) Si la cuerda está sujeta por un extremo, se genera-rá un número impar de cuartos de longitud deonda, ya que el extremo sujeto será siempre unnodo y el libre, un vientre. Si vemos 4 vientres enla cuerda, se han generado 3 nodos además del ex-tremo fijo. Por tanto, la longitud de la cuerdaequivale a 7/4 de longitud de onda. Entonces:

L L m m= = = ⋅ =74

47

47

1 2 0 69λ λ; , ,

v t= − ⋅⎛⎝⎜

⎞⎠⎟

=106

6 5 10 5ππ

π π πcos sen ( ) sen ( tt) m s⋅ −1

vd ydt

x t

v

= = − ⋅⎛⎝⎜

⎞⎠⎟

= −

2 56

5

10

ππ

π

π

cos sen ( )

ccos sen ( ) ( )π

π6

5x t SI⎛⎝⎜

⎞⎠⎟

x n n

x nm

= + =

= + =

( ) ; , , ...

( ) (

2 14

0 1 2

2 112

43 2

λ

nn m n+ =1 0 1 2) ; , , ...

λπ

λπ

π

λ λωπ

= = =

= = =

2 212

2

1k mm

v f v

;

;

/6

1125

230m

ms

⋅ =π

π

rad/s

A m k m= = = ⋅− −16

51 1; ;π

ω π rad s

Entonces, la ecuación resultante de la superposiciónserá:

56. Datos: f = 100 Hz; r′ = 83,4 m; r = 80 m; v = 340 m ⋅ s−1

Únicamente no habrá sonido si en P se cumple la con-dición de interferencia destructiva:

Calculamos λ:

En el punto donde hemos situado el aparato registra-dor, r′ = 83,4 m y r = 80 m:

Por tanto, como este punto no verifica la condición paraque la amplitud resultante sea nula, el aparato registrarásonido.

57. Datos: f1 = 380 Hz; f2 = 374 Hz

Calculamos la frecuencia de la pulsación, fp, y la fre-cuencia, f, de la onda resultante de la interferencia:

58. Datos: y1 = 2 sen (1 500t − 250x) (SI);

y2 = 2 sen (1 500t + 250x) (SI)

a) La superposición de estas dos ondas será:

b) La distancia entre dos antinodos consecutivos será:

59. Datos: y x t SI=⎛⎝⎜

⎞⎠⎟

26

5cos cos ( ) ( )π

π

x xk

x xm

m

n n

n n

− = = =

− = =

− −

1

1 1

22

250 250

λ π π

π π

/k2

y y y

y t x tr

r

= +

= − + +1 2

2 1 500 250 2 1 500sen ( ) sen ( 2250

4 250 1 500

x

y x t SIr

)

cos ( ) sen ( ) ( )=

f f f Hz Hz Hz

ff f Hz

p = − = − =

=+

=+

1 2

1 2

380 374 6

2380 3374

2377

HzHz=

83 4 80 2 1 1 7

3 4 2 1 1 7

, ( ) ,

, ( ) ,

(

m m n m

m n m

− = + ⋅

= + ⋅

22 1 2 2 112

0 1n n n+ = = = ≠) ; ; , ...

λ = =⋅

=−v

f Hzm

340100

3 41

,m s

r r n n′ − = + =( ) ; , , ...2 12

0 1 2λ

y A kr r

t kr r

y

r =−⎛

⎝⎜⎞⎠⎟

−+⎛

⎝⎜⎞⎠⎟

22 2

cos sen′ ′

ω

rr cmcm cm

= ⋅ ⋅−⎛

⎝⎜⎞⎠⎟

−2 29 5

2

1

1cos

sen

π

0009 5

24 2

1π π

π

t cmcm cm

yr

− ⋅+⎛

⎝⎜⎞⎠⎟

=

cos ( ) ssen ( )

sen ( )

100 7

4 100 7

π π

π π

t

y t cmr

= −

6. Fenómenos ondulatorios

L

λ

λ/4

Page 87: 9523 gl fis_tx2_cas

92

b) Hallamos la longitud del tubo si tiene un solo ex-tremo abierto:

63. Datos: vF = 60 km/h = 16,7 m ⋅ s−1; f = 1 000 Hz; v = 340 m ⋅ s−1

a) Si el tren se aproxima:

b) Si la fuente se aleja del receptor:

64. Datos: L = 2 m; μ = 0,005 kg ⋅ m−1; f1 = 65 Hz

La velocidad de una onda en una cuerda tensa sólo de-pende de la tensión de la cuerda, T, y de su masa porunidad de longitud. Por tanto, si determinamos la ve-locidad a partir de la frecuencia del modo fundamen-tal de vibración y la longitud de la cuerda, hallaremosla tensión.

Calculamos la velocidad de propagación de las ondasen la cuerda:

Con ella, hallamos la tensión:

65. Datos: λ roja = 6,2 ⋅ 10−7 m; λ verde = 5,4 ⋅ 10−7 m; c = 3 ⋅ 108 m ⋅ s−1

Hallamos las frecuencias de la luz roja y la luz verde:

Calculamos la velocidad a la que debe circular el vehí-culo para que el conductor vea verde (fR = fverde) la luzroja del semáforo (f = froja):

f fc v

cc v c

ff

v cff

v

RR

RR

RR

R

=+

+ = = −⎡

⎣⎢⎤

⎦⎥

= ⋅

; ; 1

3 108mm sHz

Hz⋅

⋅−

⎣⎢

⎦⎥ = ⋅−1

14

14

5 56 10

4 84 101 4 46 1

,

,, 007 m

s

fC

fm s

mrojaroja

roja= =⋅ ⋅

⋅= ⋅

λ;

,,

3 10

6 2 104 84

8 1

7 110

3 10

5 4

14

8 1

;,

Hz

fC

fm s

verdeverde

verde= =⋅ ⋅ −

λ ⋅⋅= ⋅

105 56 107

14

mHz,

vT

T v kg m

T

= = = ⋅ ⋅ ⋅( )− −

μμ; ,2 1 1 2

0 005 260 m s

== 338 N

f nvL

vL fn

m Hz ms

= = =⋅ ⋅

=2

2 2 2 651

260;

f fv

v v

f Hz

RF

R

=+

= ⋅⋅

⋅ +

−1 000340

340 16 7

1

1 ,

m s

m s,

m s⋅=−1 953 2 Hz

f fv

v v

f Hz

RF

R

=−

= ⋅⋅

⋅ −

−1 000340

340 16 7

1

1 ,

m s

m s,

m s⋅=

−1 1 051 7 Hz

f nvL

L nvf Hz

= = = ⋅⋅

⋅=

4 41

3404 250

0 341

; ,m s

m

f nvL

L nvf Hz

= = = ⋅⋅

⋅=

2 21

3402 250

0 681

; ,m s

mb) Como los modos normales de vibración de la cuerda

fija en un extremo son de la forma la vibra-

ción corresponde al séptimo armónico, n = 7:

Por lo tanto:

61. Datos: L = 1 m; 9 nodos; Amáx = 2 cm = 0,02 m; v = 10 m ⋅ s−1

a) Si en la cuerda de guitarra, que está sujeta por losdos extremos, se forman 9 nodos, la longitud deonda corresponde a un cuarto de la longitud de la

cuerda:

Para escribir la ecuación de la onda estacionaria, de-terminamos primero el número de ondas y la pulsa-ción:

Como la cuerda está fija en sus dos extremos, x = 0y x = L deben ser nodos de la onda estacionaria.Por lo tanto, su ecuación es:

b) Calculamos la frecuencia fundamental de vibra-ción:

c) Hallamos la longitud de onda correspondiente a lafrecuencia fundamental:

o también:

62. Datos: f = 250 Hz; v = 340 m ⋅ s−1

a) Calculamos la longitud del tubo si tiene los dos ex-tremos abiertos:

λ = =⋅

=−v

f Hzm

105

21m s

λ λ= =⋅

=2 2 1

12

Ln

mm; ;

f nvL m

Hz= = ⋅⋅

⋅=

21

102 1

51m s

y A kx t

y xr

r

= ( )=

2

0 02 8

sen ( ) cos

, sen ( ) co

ω

π ss ( ) ( )8π t SI

km

m

fv

k v m

= = =

= = = = ⋅

2 20 25

8

2 2 8 10

1

1

π

λ

ππ

ω π πλ

π

,

m s

rad s

= ⋅

1

180ω π

λ = = =L m

m4

14

0 25, .

fvL

f ff Hz

Hz7 1 177

47

7120

717 1= ⋅ = = = =; ,

f nvL

fvL

= =4 41;

λ = 4Ln

,

λ

L

λ λ λ

Page 88: 9523 gl fis_tx2_cas

93

EVALUACIÓN

1. Principio de Huygens: todo punto de un frente de ondase convierte en un centro puntual productor de on-das elementales secundarias cuya superficie envolventeconstituye un nuevo frente de onda.

Consideremos un frente de onda (representado por elsegmento AB) que incide con un ángulo i sobre la su-perficie de separación de un medio, en que la onda sepropaga con velocidad v1, respecto de otro en que lohace con velocidad v2, siendo v2 menor que v1. Cadapunto de la superficie donde incide el primer frente deonda, por el principio de Huygens, se convierte en unnuevo foco emisor.

Debido a la menor velocidad de propagación de laonda en el segundo medio, las ondas secundarias re-corren una menor distancia en el mismo tiempo de laque recorrerían en el primer medio. Por ello, la ondase refracta, es decir, cambia de dirección. La envolven-te de las ondas secundarias es el nuevo frente de laonda refractada, DC.

El rayo correspondiente al punto B del frente deonda incidente alcanzará la superficie en un tiempot después de que lo haya hecho el rayo del mismofrente situado en A. Por tanto, para el triángulo ABCtenemos:

Por otro lado, cuando el rayo procedente de B incidasobre la superficie, el rayo refractado en A habrá avan-zado v2 t por el nuevo medio. Entonces, para el trián-gulo ACD podemos escribir:

Si igualamos los dos valores de AC, obtenemos la se-gunda ley de la refracción:

2. El eco consiste en la reflexión del sonido sobre una su-perficie suficientemente alejada como para que sea-mos capaces de diferenciar el sonido emitido del refle-jado. En el caso de la luz, al ser su velocidad de

v ti

v tr

ir

vv

n1 2 1

221sen sen

;sensen

= = =

sensen

rADAC

v tAC

ACv t

r= = ⇒ =2 2

sensen

iBCAC

v tAC

ACv t

i= = ⇒ =1 1

67. Introduciendo la información de la siguiente manera:Celda Fórmula

B1 λ(1)=B2 A(1)=

C1 0,6C2 4

D1 λ(1)=D2 A(2)

E1 0,66E2 4

B5 0C5 =C$2*SENO(2*PI()/C$1*B5)D5 =E$2*SENO(2*PI()/E$1*B5)E5 =C5+D5

Luego, sucesivamente se programa: B6 =B5+0,1C6 =C$2*SENO(2*PI()/C$1*B6)D6 =E$2*SENO(2*PI()/E$1*B6)E6 =C6+D6

B7 =B6+0,1C7 =C$2*SENO(2*PI()/C$1*B7)D7 =E$2*SENO(2*PI()/E$1*B7)E7 =C7+D7

...

Etcétera

La programación de las fórmulas correspondientes alas 30 primeras filas será la siguiente:

Es decir, programamos la interferencia entre dos ondasde amplitud AM y longitud de onda 0,6 m y 0,66 m, res-pectivamente. La onda que resulta es:

6. Fenómenos ondulatorios

Rayoincidente

Ondaincidente

Superficiede separación

Rayo refractado

Onda refractada

Normali

i

r

rA

D

B

CA’ A’’

Page 89: 9523 gl fis_tx2_cas

94

b) Si el tubo está abierto sólo por un extremo, sóloexisten armónicos impares:

5. Datos: vF = 14 m ⋅ s−1; vR = 18 m ⋅ s−1; f = 600 Hz; v = 340 m ⋅ s−1

a) Si el tren A y el tren B están en movimiento acer-cándose el uno al otro:

Si el pasajero y la sirena se alejan el uno del otro:

b) Si el pasajero del tren A está en reposo (vR = 0) yel tren B se aproxima:

Si la sirena, por el contrario, se aleja del receptor:

f fv

v v

f Hz

RF

R

=+

= ⋅⋅

⋅ + ⋅

−600340

340 14

1

1

m s

m s m ss− =1 576 3, Hz

f fv

v v

f Hz

RF

R

=−

= ⋅⋅

⋅ − ⋅

−600340

340 14

1

1

m s

m s m ss− =1 625 8, Hz

f fv vv v

f Hz

RR

F

R

=−

+

= ⋅⋅ − ⋅− −

600340 18

340

1 1m s m s,

m s m s⋅ + ⋅=− −1 114

545 8 Hz

f fv vv v

f Hz

RR

F

R

=+

= ⋅⋅ + ⋅− −

600340 18

340

1 1m s m s,

m s m s⋅ − ⋅=− −1 114

658 9 Hz

;m s

= = ⋅⋅ −

33 3340

n f11

5

1

4 2127 5

5 5340

4 2

⋅=

= = ⋅⋅

,

;

mHz

n fm s

,m

Hz= 212 5

f nvL

n

n f

= =

= = ⋅

41 3 5

1 1340

1

; , , ...

; ,m s⋅

⋅=

−1

4 242 5

mHz

f nvL

n

n f

= =

= = ⋅

21 2 3

1 1340

1

; , , ...

;

;

m s

m s

⋅=

= = ⋅⋅

1

2

1

2 285

2 2340

2

mHz

n f⋅⋅

=

= = ⋅⋅

⋅=

2170

3 3340

2 223

1

;

mHz

n fm

m s555 Hz

propagación muy grande, nuestra vista sólo sería capazde distinguir la reflexión de la emisión si la superficiereflejante estuviera muy lejos. En ese caso podríamosobservar, por ejemplo, que la imagen de nuestros mo-vimientos ante un espejo lleva retraso respecto al pro-pio movimiento real.

Para hacer una estimación de cuál sería la distancia ne-cesaria, supondremos que la vista puede distinguir se-ñales distanciadas por medio segundo. Entonces, si te-nemos en cuenta que la velocidad de propagación dela luz en el aire es de 3 ⋅ 108 m/s:

Es decir, la superficie debería encontrarse a x = d/2, x = 0,75 ⋅ 108 m = 75 000 km.

3. Datos: f = 50 Hz; A = 2 cm = 0,02 m; v = 1 m ⋅ s−1;ϕ = π/3; propagación en el sentido positivo del eje OX

Hallamos la pulsación y el número de ondas:

Escribimos la ecuación de cada una de las ondas:

La onda resultante de su interferencia será:

4. Datos: L = 2 m; v = 340 m ⋅ s−1

a) Si el tubo está abierto en sus dos extremos:

y y y A t kx A t kx

y

r

r

= + = − + − +

=

1 2

2

sen ( ) sen (ω ω ϕ)

AAt kx t kx

t

sen( ) (

cos(

ω ω

ω

− + − +⎡

⎣⎢

⎦⎥ ⋅

⋅−

ϕ)

2

kkx t kx

y A t kxr

) (

sen

− − +⎡

⎣⎢

⎦⎥

= − +⎛⎝⎜

ω

ω

ϕ)

ϕ2

2

2⎞⎞⎠⎟

−⎛⎝⎜

⎞⎠⎟

= − +⎛⎝⎜

⎞⎠⎟

cos

sen

ϕ2

ϕ2

y A t kxr 2 ω cos

, sen ( )

ϕ2

⎛⎝⎜

⎞⎠⎟

= − +⎡

⎣⎢⎤y t xr 0 04 100

π

⎦⎦⎥⎛⎝⎜

⎞⎠⎟

= − +⎡

cos

, sen ( )

π

ππ

6

0 035 1006

y t xr⎣⎣⎢

⎦⎥( )SI

y A t kx

y t x

= − +

= −

sen (

, sen [ ( )]

ω

π

ϕ)

1 0 02 100 ( )

, sen

SI

y t x2 0 02 1003

= − +⎛⎝⎜

⎞⎠⎟

⎣⎢

⎦⎥π

π( )SI

ω π π π

π

λ

π π

= = ⋅ = ⋅

= = =

−2 2 50 100

2 2 2

1

1f Hz

kv

f

rad s

m s⋅⋅ =−

−1

150 100Hz mπ

d v t s m= = ⋅ ⋅ ⋅ = ⋅− , ,3 10 0 5 1 5 108 1 8m s

Page 90: 9523 gl fis_tx2_cas

95

Campo eléctrico7

que C. A. Coulomb estableciera la primera relación mate-mática entre las cargas, la distancia entre ellas y las fuerzaseléctricas que experimentan. Durante el siglo XIX se desa-rrolló la mayor parte de la teoría electromagnética, conlos experimentos de Faraday sobre la inducción magnéti-ca y la teoría de Maxwell sobre la naturaleza electromag-nética de la luz, comprobada posteriormente con los ex-perimentos de Hertz.

1. FUERZAS ELÉCTRICAS

1. La carga eléctrica de los cuerpos es debida a la estructu-ra atómica de la materia. Algunos átomos tienen más fa-cilidad que otros en desprenderse de los electrones másexternos, quedando así cargados positivamente. Encambio, algunos átomos aceptan fácilmente los electro-nes que otro les cede, y quedan con carga negativa.

2. El plástico del peine, al frotarlo con el pelo, es capazde tomar algunos electrones de éste. Así, el peine que-da con carga negativa y atrae el pelo, que ha quedadocargado positivamente.

3. Respuesta sugerida:

La conservación de la carga se pone de manifiesto enfenómenos como el descrito en el ejercicio anterior.La carga no se crea, sino que los electrones pasan deun cuerpo a otro. La carga total de los dos cuerpos alfinal es la misma que teníamos inicialmente.

Otro ejemplo no tan cotidiano es el fenómeno de ani-quilación de partículas con antipartículas. La carga to-tal inicial y final es nula, ya que partícula y antipartícu-la tienen siempre cargas opuestas.

4. a) Datos: q = −39 C

Teniendo en cuenta que cada electrón tiene unacarga de e = −1,602 ⋅ 10−19 C:

b) Datos: 4 ⋅ 1020 e

5. Respuesta sugerida:

La electrización por influencia o inducción electrostá-tica consiste en cargar un cuerpo acercándolo a otroobjeto ya cargado, pero sin que entren en contacto.

4 101 602 10

164 120

19

⋅ ⋅− ⋅

= −−,

,eC

eC

− ⋅− ⋅

= − ⋅−391

1 602 102 43 1019

20

,,C

e

Ce

PREPARACIÓN DE LA UNIDAD

• Se dice que un cuerpo es neutro cuando su carga eléctri-ca total es cero. Esto no quiere decir que no tenga cargaspositivas y cargas negativas, sino que la suma de todas suscargas eléctricas es nula.

Se dice que está cargado positiva o negativamente si sucarga total es positiva o negativa.

• Una experiencia de electrización por frotamiento es elpéndulo eléctrico. Consiste en una pequeña bolita dematerial muy ligero colgada de un hilo. Si frotamos unbolígrafo de plástico y lo acercamos a la bolita, veremoscómo el bolígrafo la atrae. Si llega a tocarla, desde esemomento el bolígrafo repelerá la bola. La explicación esque el bolígrafo, al frotarlo, ha quedado cargado negati-vamente, porque ha arrancado cargas negativas del paño.Entonces es capaz de atraer la bolita. Pero cuando entranen contacto, parte de las cargas negativas del bolígrafopasan a la bola. Como los dos cuerpos quedan ahora car-gados con cargas del mismo signo, se repelen. En cam-bio, si frotamos un trozo de vidrio con un paño de seda ylo acercamos a la bola cargada negativamente, a diferen-cia del plástico, el vidrio atraerá la bola. Esto nos indicaque el vidrio, al frotarlo, adquiere cargas positivas.

• Decimos que un campo de fuerzas es uniforme cuando lafuerza que experimenta una partícula de prueba es igualen todos los puntos del espacio.

Decimos que un campo de fuerzas es central cuando lafuerza que experimenta una partícula de prueba en cual-quier punto está dirigida hacia el centro del campo y de-pende de la distancia a dicho centro.

• Los primeros en fijarse en los fenómenos eléctricos fueronlos griegos, quienes observaron que, si frotaban el ámbar,éste atraía pequeños pedacitos de tela. Precisamente la pa-labra electricidad procede del griego elektron, que significa‘ámbar’. En los siglos XVII y XVIII resurgió el interés por laelectricidad. Así, B. Franklin advirtió que existen dos tiposde cargas eléctricas, a los que dio los nombres de positiva ynegativa. Pero hubo que esperar hasta el año 1785 para

Materiales conductores Materiales aislantes

Cobre Vidrio

Hierro Plástico

Solución salina Madera

7. Campo eléctrico

Page 91: 9523 gl fis_tx2_cas

96

9. Datos: Q1 = +2 ⋅ 10−6 C; Q2 = + 4 ⋅ 10−6 C;

r12 = 90 cm = 0,9 m; Q3 = −3 ⋅ 10−6 C; r13 = 30 cm = 0,3 m

Calculamos primero la fuerza que ejerce Q1 sobre Q3:

Calculamos la fuerza que ejerce Q2 sobre Q3:

Teniendo en cuenta que Q 3 se encuentra entre lascargas Q1 y Q 2, las fuerzas

�F13 y

�F23 tienen sentidos

opuestos.

Aplicamos el principio de superposición:

Como hemos tomado el sentido positivo de Q1 a Q3, lafuerza sobre Q3 apunta hacia Q1.

Si la carga Q3 fuera positiva, las dos fuerzas�F13 y

�F23 ten-

drían el valor calculado anteriormente, pero sentidoopuesto. La fuerza resultante tendría el mismo valor,0,3 N, pero sentido opuesto; es decir, apuntaría hacia Q2.

2. ESTUDIO DEL CAMPO ELÉCTRICO

10. La intensidad del campo eléctrico creado por una car-ga puntual es inversamente proporcional al cuadradode la distancia, disminuyendo a medida que nos aleja-mos de la carga que genera el campo.

11. Datos: Q = +4μC; r = 50 cm = 0,5 m

a) En el vacío:

E

E

=

= ⋅⋅

⋅⋅

= ⋅−

KQ

rN m

C

C

m

2

92

2

6

29 104 10

0 51 4

( , ), 1105 N

C

� � �F F F

F F F N N N3 13 23

3 13 23 0 6 0 3 0 3

= +

= − = − =, , ,

F

F

23

23

=

= ⋅⋅

⋅⋅ ⋅ ⋅−

KQ Q

r

N m

C

C

2 3

232

92

2

6

9 104 10 3 100

0 6

0 3

6

2

=

( , )

,

C

m

N�F23

F

F

13

13

=

= ⋅⋅

⋅⋅ ⋅ ⋅−

KQ Q

r

N m

C

C

1 3

132

92

2

6

9 102 10 3 100

0 3

0 6

6

2

=

( , )

,

C

m

N�F13

Para ello ponemos en contacto dos objetos descargadosde material conductor; por ejemplo, dos esferas metáli-cas. Si acercamos a una de las esferas un cuerpo carga-do positivamente, éste atraerá las cargas negativas de lasdos esferas. Entonces, las cargas negativas acudirán ala esfera más cercana al cuerpo cargado, mientras que laesfera más alejada quedará cargada positivamente. Si eneste momento, y sin apartar el cuerpo cargado, separa-mos dos esferas, tendremos las dos con cargas iguales yopuestas.

Otro procedimiento de electrización por influencia esel que se explica en la página 93 del libro Física y Quí-mica 1.o Bachillerato, Ed. Edebé.

6. Respuesta sugerida:

Los camiones que transportan productos inflamablesarrastran una cadena metálica para evitar que el ca-mión, por el rozamiento con el aire, quede cargado.Como la cadena metálica es conductora, cualquier car-ga que adquiera el camión por rozamiento se descarga-rá a tierra por la cadena.

7. Ley de Coulomb:

La fuerza de atracción o repulsión entre dos cargas eléc-tricas puntuales es directamente proporcional al produc-to de las cargas e inversamente proporcional al cuadradode la distancia que las separa.

Las fuerzas eléctricas tienen las siguientes características:

— La fuerza está dirigida a lo largo de la recta queune las dos cargas.

— Es repulsiva para cargas del mismo signo y atracti-va para cargas de signo opuesto.

— Actúan a distancia sin necesidad de que exista nin-gún medio material entre ellas.

— Siempre se presentan a pares, siguiendo el princi-pio de acción y reacción.

— Verifican el principio de superposición.

8. Datos: Q1 = +3 ⋅ 10−6 C; Q2 = +1,2 ⋅ 10−5 C; r = 50 cm = 0,5 m

a) En el aire, la constante dieléctrica relativa es εr = 1.Por tanto:

b) En el agua, εr = 80. Entonces:

F

F

= ⋅

=⋅ ⋅ ⋅ ⋅− −

14

1

4 8 854 10

0

1 22

12 2 1

π

π

ε εr

Q Q

r

C N, mm

C C

m

− −

⋅⋅

⋅⋅ ⋅ ⋅

= ⋅

2

6 5

2

80

3 10 1 2 10

0 5

1 6

,

( , )

,�F 110 2− N

F

F

=

= ⋅⋅

⋅⋅ ⋅ ⋅ −

KQ Q

rN m

C

C C

1 22

92

2

6 6

9 103 10 12 10

(00 5

1 3

2, )

,

m

N�F =

E

r

E KQ

r=

2

Page 92: 9523 gl fis_tx2_cas

97

Aplicamos el principio de superposición teniendo encuenta que

�E1 y

�E2 tienen sentidos opuestos:

Por tanto, el vector intensidad de campo eléctricoapunta hacia Q1.

Si en dicho punto situamos una carga Q3= −0,5 μC, lafuerza que experimentará será:

La fuerza estará dirigida hacia Q2.

15. La energía potencial eléctrica de una carga q se rela-ciona con el potencial eléctrico V de esta manera:

Ep = q V

a) La carga se dirige hacia potenciales eléctricos me-nores. Al tratarse de una carga positiva, su energíapotencial eléctrica disminuye.

b) La carga se desplaza hacia potenciales eléctricosmayores. Por tanto, su energía potencial eléctri-ca aumenta.

c) El potencial eléctrico no varía. Por tanto, la ener-gía potencial eléctrica de la carga se mantieneconstante.

d) Si la carga vuelve al punto de partida, el potencialeléctrico final es igual al inicial. Por tanto, la varia-ción de la energía potencial eléctrica es nula.

16. Datos: Q = +4 ⋅ 10-8 C

a) Calculamos el potencial eléctrico a una distancia r = 5 cm = 0,05 m:

b) Una carga q = −1,5 ⋅ 10−9 C tendrá una energía po-tencial eléctrica:

17. Datos:

E q V C V Jp = = − ⋅ ⋅ = − ⋅− −, ,1 5 10 7 200 1 1 109 5

V KQr

N m

C

Cm

V= = ⋅⋅

⋅⋅

=−

9 104 100 05

7 20092

2

8

,

� � �F E N/C) i= = − ⋅ ⋅ − ⋅ =−Q C3

6 60 5 10 2 2 10 1 1, ( , ,��i N

� � �

� � �E E E

E N/C i N/C i1 2= +

= ⋅ − ⋅2 5 10 2 78 106 5, ,�� �E i N/C= − ⋅2 2 106,

b) En agua, εr = 80:

12. Datos: Q = 120 nC = 1,2 ⋅ 10−7 C; E = 6 750 N/C

Despejamos la distancia de la expresión de la intensi-dad del campo eléctrico:

13. Datos: Q1 = +3 μC; Q2 = −2 μC; d = 40 cm = 0,4 m

El punto medio del segmento que une las dos cargas está

situado a una distancia de ambas cargas.

El campo eléctrico en este punto será la superposiciónde los campos creados por cada una de las cargas:

Aplicamos el principio de superposición teniendo en

cuenta que�E1 y

�E2 tienen el mismo sentido:

14. Datos: Q1= +4 ⋅ 10−6 C; Q2 = +1 ⋅ 10−6 C;

d = 30 cm = 0,3 m; r1 = 12 cm = 0,12 m

Calculamos el campo creado por cada una de las car-gas en el punto que dista 12 cm de Q1 (r1 = 0,12 m) y18 cm de Q2 (r2 = 0,3 m − 0,12 m = 0,18 m), tomandocomo sentido positivo el que va de Q1 a Q2:

E KQ

r

EN m

C

C

m

11

12

19

2

2

6

29 104 10

0 122

=

= ⋅⋅

⋅⋅

=−

( , ),,

(

5 10

9 101 10

0

6

22

22

29

2

2

6

=

= ⋅⋅

⋅⋅ −

NC

E KQ

r

EN m

C

C

,, ),

182 78 102

5

m

NC

= ⋅

� � �

� � �E E E

E N/C i N/C i1 2= +

= ⋅ + ⋅6 75 10 4 5 105 5, ,�� �E i N/C= ⋅1 12 106,

E KQ

r

EN m

C

C

m

11

2

19

2

2

6

29 103 10

0 26 7

=

= ⋅⋅

⋅⋅

=−

( , ), 55 10

9 102 10

0 2

5

22

2

29

2

2

6

=

= ⋅⋅

⋅⋅ −

NC

E KQ

r

EN m

C

C

( , ),

m

NC2

54 5 10= ⋅

rd

m= =2

0 2,

E

r9

N/C

= ⇒ =

=⋅

⋅⋅ −

KQ

rr

K Q

E

N m

CC

2

92

2710 1 2 10

6 750

,== 0 4, m

E

E

=

=⋅ ⋅ ⋅ ⋅ ⋅− − −

14

1

4 8 854 10

02

12 2 1 2

π

π

ε εr

Q

r

C N m, 880

4 10

0 5

1 8 10

6

2

3

⋅⋅

= ⋅

( , )

,

C

m�E N/C

7. Campo eléctrico

12

12

Q Q Q Q Q C

cm m

d

= = = = = + ⋅

= =

=⎛⎝⎜

−1 2 3 4

65 10

1 40 0 4

12

,

⎞⎞⎠⎟

+⎛⎝⎜

⎞⎠⎟

=⎛⎝⎜

⎞⎠⎟

+⎛⎝⎜

⎞⎠⎟

2 2 2 212

0 42

0 42

, ,m m== 0 28, m

Q1+

A

Bl

l

d

+

+ +

Q2

Q3 Q4

Page 93: 9523 gl fis_tx2_cas

98

19. — Dos líneas de campo eléctrico no pueden cruzar-se, ya que en cada punto del espacio el vector in-tensidad del campo eléctrico tiene una sola direc-ción y sentido bien definidos.

— Para trasladar una carga eléctrica a través de unasuperficie equipotencial no es necesario realizarningún trabajo. En una superficie equipotencial,el potencial es constante. Como el trabajo es lacarga por la variación del potencial, y no hay va-riación del potencial, el trabajo es nulo.

20. El flujo eléctrico a través de una superficie es una me-dida del número de líneas de campo que atraviesan di-cha superficie. Como el número de líneas que entranen la esfera es igual al número de líneas que salen deella, el flujo eléctrico es nulo, es decir 0 V ⋅ m.

21.

++++

++

++

++

++

+

+ + + + + + + +

++

++

++

++

++++

Plano infinito cargado uniformemente

Alambre infinito cargado uniformemente

b

c

El trabajo necesario para trasladar una carga (porejemplo, Q1) del vértice al centro del cuadrado seráigual a la variación de la energía potencial eléctrica deQ1. Determinamos el potencial eléctrico creado por lasotras tres cargas en el vértice y en el centro:

Por el principio de superposición, el potencial eléctri-co en el vértice será:

En el centro del cuadrado:

Por el principio de superposición:

Entonces, el trabajo necesario para trasladar Q1 será:

18.

a

Esfera cargada uniformemente

W Q V V

W C VB A= −

= ⋅ ⋅ ⋅ − ⋅−1

6 55 10 4 821 10 3 053 1

( )

( , , 00

0 88

5 )

,

V

W J=

V V V V

V V VB B B B

B

= + +

= ⋅ + ⋅ +

2 3 44 41 607 10 1 607 10 1, , ,6607 10

4 821 10

4

5

= ⋅,

V

V VB

V KQd

N m

C

Cm

V

B

B

29

2

2

6

2

9 105 100 28

1 607

= = ⋅⋅

⋅⋅

=

,

, ⋅⋅

= = ⋅⋅

⋅⋅ −

10

9 105 100 28

5

39

2

2

6

3

,

V

V KQd

N m

C

Cm

V

B

BB

B

V

V KQd

N m

C

C

= ⋅

= = ⋅⋅

⋅⋅ −

1 607 10

9 105 100

5

49

2

2

6

,

,,

,

28

1 607 1045

m

V VB = ⋅

V V V V

V V VA A A A

A

= + +

= ⋅ + ⋅ +

2 3 45 51 25 10 1 125 10 8 0, , , 33 10

3 053 10

4

5

= ⋅,

V

V VA

V KQ N m

C

CmA2

92

2

65

19 10

5 100 4

1 125 10= = ⋅⋅

⋅⋅

= ⋅−

,,

,,

V

V KQ N m

C

CmA3

92

2

6

19 10

5 100 4

1 125= = ⋅⋅

⋅⋅

= ⋅−

110

29 10

5 102 0 28

5

49

2

2

6

,

V

V KQd

N m

C

CmA = = ⋅

⋅⋅

⋅⋅

== ⋅8 03 104, V

Si

Se

d�Si

�E

d�Se

�E

�E

Page 94: 9523 gl fis_tx2_cas

99

eléctrico interior de un conductor es nulo aun cuan-do se aplica otro campo eléctrico exterior.

27. Un dipolo eléctrico en un campo eléctrico uniformese orienta paralelo al campo, debido al par de fuerzasque actúa sobre él como consecuencia del campo y desu distribución de carga.

El campo eléctrico en el interior de un dieléctrico esinferior al exterior, debido a la distribución de cargasen el interior del material. El campo exterior

�E0 hace

aparecer cargas superficiales en las paredes del con-ductor de forma que generan un campo eléctrico

�E′

opuesto al exterior. Como consecuencia, el campoeléctrico resultante en el interior del dipolo

�Eint es in-

ferior al exterior.

28. Datos:

29. Un condensador está constituido por dos placas con-ductoras muy próximas entre sí, denominadas arma-duras, y un material dieléctrico entre ellas.

a) La principal característica de un condensador esel almacenamiento de carga eléctrica y de ener-gía. Por esta razón es muy utilizado para crearcampos eléctricos, para rectificar corrientes alter-nas y en el diseño de aparatos para transmitir y re-cibir señales electromagnéticas (radio, TV...).

b) La capacidad de un condensador es la constantede proporcionalidad entre la carga acumulada enel dispositivo y la diferencia de potencial aplicadaentre sus armaduras.

c) Montar condensadores en serie o en paralelo esútil para conseguir condensadores equivalentescon una capacidad determinada a partir de con-densadores de capacidades distintas a la deseada.

30. Datos: C = 12 μF = 1,2 ⋅ 10−5 F; V = 220 V

Determinamos la carga acumulada en el conden-sador:

La energía acumulada en el condensador será:

FÍSICA Y SOCIEDAD

a) Respuesta sugerida:

Desviar partículas cargadas mediante campos eléctricostiene muchísimas aplicaciones. Además de las citadasen el texto Aplicaciones de la electrostática, de la página187 del libro del alumno, un ejemplo muy común es elfuncionamiento de los televisores de tubo de rayos ca-tódicos, que van quedando en desuso. La pantalla del

E C V F V Jp = = ⋅ ⋅ =−12

12

1 2 10 220 0 32 5 2, ( ) ,

CQV

Q C V F V C= ⇒ = = ⋅ ⋅ = ⋅− −, ,1 2 10 220 2 64 105 3

��

EE N/C

int0= =

⋅= ⋅

−−

εr

NC

1 106

1 7 105

6,

εr = = ⋅ −6 1 10 5;�E N/C0

� � �E E Eint 0= + ′

Puntos interiores: Para determinar el campo en el inte-rior de la corteza, escogemos como superficie de Gaussuna superficie esférica concéntrica con la corteza Si yde radio menor. Como no hay carga en su interior:

Puntos exteriores: En este caso elegimos como superfi-cie de Gauss una esfera concéntrica a la corteza Se, perode radio mayor, r. Como en cada punto de la superficieel campo eléctrico y el vector superficie son paralelos,

Además, como la distancia de todoslos puntos de Se a la carga es igual, el campo será cons-tante en toda la superficie. Entonces, el flujo a travésde Se será:

Aplicando el teorema de Gauss:

3. COMPORTAMIENTO DE LA MATERIA EN CAMPOSELÉCTRICOS

22. Decimos que un conductor está en equilibrio electros-tático cuando sus cargas libres están en reposo. Enequilibrio electrostático, un conductor tiene todas suscargas eléctricas distribuidas en la superficie de modoque el campo eléctrico interior es nulo.

23. El potencial eléctrico de un conductor en equilibrioelectrostático es constante en todo el conductor por-que el campo eléctrico en su interior es cero. Como no hay campo en el interior, el trabajo para desplazaruna partícula de prueba desde un punto a otro del in-terior es nulo. Por lo tanto, el potencial eléctrico esconstante.

24. Datos: R = 10 cm = 0,1 m

Determinamos la capacidad de la esfera metálica:

25. Datos: R = 10 cm = 0,1 m; Q = +1 μC

La capacidad de la esfera será:

Entonces, el potencial es:

26. Los conductores se caracterizan por su capacidad, a di-ferencia de los dieléctricos, que se caracterizan por laconstante dieléctrica relativa. La constante dieléctricarelativa de un conductor es infinita, ya que el campo

VQC

C

FV= =

⋅= ⋅

1 10

1 1 109 0 10

6

114

,,

C R C N m m

C

= = ⋅ ⋅ ⋅ ⋅ ⋅

=

− − −4 4 8 854 10 0 1

10

12 2 1 2π πε , ,

,11 10 11⋅ − F

C R C N m m

C

= = ⋅ ⋅ ⋅ ⋅ ⋅

=

− − −4 4 8 854 10 0 1

10

12 2 1 2π πε , ,

,11 10 11⋅ − F

Φ = = =Q

E rQ

EQ

re eε ε ε0

2

0 024

4; ;π

π

Φ

Φ

= ⋅ = ⋅ =

=

⌠⌡⎮ ⌠

⌡⎮ ⌠

⌡⎮

Se See

Se

e

d d E d

E

� � �E S E S Se e e e e

S E re e= 4 2π

� �E Se e⋅ =d E dSe e.

Φ = ⋅ = = ⇒ =⌠⌡⎮

Sid

Q� � �E S Ei i iε0

0 0

7. Campo eléctrico

Page 95: 9523 gl fis_tx2_cas

100

Para la elaboración del informe se recomienda seguiresta estructura:

— Introducción. Plantea el objetivo del trabajo, lajustificación del método seguido y el comentariode las causas que han motivado la selección deltema.

— Cuerpo o desarrollo. Describe mediante una ex-posición lógica y coherente el contenido del estu-dio, organizado en capítulos, apartados y subapar-tados. Conviene incluir ejemplos, dibujos,fotografías, gráficos... que ilustren y completen laexposición del tema.

— Conclusión. Resume las principales ideas que sehan ido exponiendo e incluye las impresionespersonales y los juicios críticos oportunos.

— Bibliografía. Es la relación de los libros que sehan consultado, ordenados alfabéticamente. Sedeben indicar los apellidos y el nombre del autor,el título del libro, la editorial, y el lugar y la fechade edición.

RESOLUCIÓN DE EJERCICIOS Y PROBLEMAS

31. Datos:

a) Determinamos el campo eléctrico en el cuartovértice. Para ello, calculamos primero la contribu-ción de cada carga:

d m m m= + = + =1 1 0 4 0 4 0 572 2 2 2( , ) ( , ) ,

televisor emite luz porque desde el fondo del tubo seenvían partículas cargadas que, al chocar contra la pan-talla, hacen que ésta emita luz. La partículas cargadasson aceleradas y dirigidas a lo largo del tubo del televi-sor mediante campos eléctricos para así formar las imá-genes. El televisor recibe por la antena la informaciónde cómo deben variar estos campos para formar la ima-gen correcta en cada momento.

b) Los conductores eléctricos presentan una propiedad co-nocida como efecto de las puntas: el campo eléctrico exte-rior es más intenso en las zonas del conductor con me-nor radio de curvatura, pues la carga de un conductortiende a concentrarse en las partes más puntiagudas deéste. Por esta razón, cuando una nube con carga negati-va pasa sobre un cuerpo conductor puntiagudo, induceuna carga positiva en su punta. El rayo es absorbido porla punta y descargado a tierra a través del conductor.

Los pararrayos consisten en un conductor metálicoque une la parte más alta de un edificio a tierra. Esteconductor, acabado en punta, incentiva la formacióndel rayo y lo conduce a tierra.

c) Respuesta sugerida:

La xerografía es una técnica para la reproducción deimágenes en papel basada en la electrostática. Las prin-cipales etapas de este proceso son:

1. Sobre un sustrato metálico conectado a tierra secoloca una fina lámina de un material fotoconduc-tor. Este tipo de material es aislante de la electrici-dad en la oscuridad, pero conduce la corrienteeléctrica cuando se ilumina.

En la oscuridad, se cubre la superficie del fotocon-ductor con una carga eléctrica positiva uniforme.En la unión metal-fotoconductor se induce unacarga negativa y la lámina fotoconductora quedasometida a una gran diferencia de potencial.

2. La lámina fotoconductora se expone a la luz refle-jada en la imagen que se tiene que reproducir. Enla lámina se absorbe la luz y se generan pares elec-trón-agujero (carga positiva).

Por la acción del campo eléctrico, los electrones semueven hacia la superficie del fotoconductor, don-de neutralizan las cargas positivas, mientras que losagujeros se dirigen hacia la unión metal-fotoconduc-tor, donde neutralizan las cargas negativas. La ima-gen óptica del documento original queda registradaen una imagen electrostática sobre la lámina.

3. Se cubre la lámina con partículas de tóner. Se tra-ta de partículas pigmentadas con carga negativa,que son atraídas por la carga positiva de la superfi-cie. De esta manera la imagen electrostática se con-vierte en una imagen visible.

4. Se coloca una hoja de papel cargado positivamen-te sobre la lámina para que el tóner se le adhiera.A continuación se calienta el papel, con lo cual eltóner se funde y se fija al papel de una manera per-manente. Se ha obtenido una fotocopia.

Se limpia de tóner la lámina y se descarga expo-niéndola a la luz. Así, la lámina está preparadapara repetir nuevamente el proceso.

aCarga electrostática

Lámina fotoconductora

Sustrato metálico

Carga inducida

Pares electrón hueco

Luz

PapelPartículasde tóner

b

c d

Y

Xd

+

++

l = 0,4 m

Q1 = +1,25 · 10–4 C

Q3 = +1,25 · 10–4Q2 = 1,25 · 10–4

�E �

E2

�E3

�E1

Page 96: 9523 gl fis_tx2_cas

101

Por el principio de superposición:

Entonces, el trabajo necesario para trasladar q será:

32. Datos:

a) Determinamos la fuerza que ejerce cada carga so-bre q y aplicamos el principio de superposición:

El módulo de la fuerza será:

b) El cálculo del trabajo necesario para trasladar lacarga q = +1 μC desde A(0, 0) m hasta B(1, 1) mexige determinar primero el potencial en cadauno de los dos puntos a partir de la contribuciónde cada carga, Q1 y Q2:

Por el principio de superposición, el potencialeléctrico en A será:

V V V V V VA A A= + = − = −1 2 36 000 45 000 9 000

V KQr

N m

C

Cm

V

AA

A

11

1

92

2

6

1

9 104 10

1

36 00

= = ⋅⋅

⋅⋅

=

00

9 105 10

122

2

92

2

6( )

V

V KQr

N m

C

Cm

V

AA

= = ⋅⋅

⋅− ⋅ −

22 45 000A V=

F N N N= + = ⋅ −( , ) ( , ) ,0 045 0 036 5 7 102 2 2

� �

F KQ q

ru

FN m

C

CA

11

12 1

19

2

2

6

9 104 10

=

= − ⋅⋅

⋅⋅ ⋅− 11 10

10 036

6

2

22

22

⋅= −

=

( ),

C

mj j N

F KQ q

r A

� �

� ��

u

FN m

C

C C

2

29

2

2

6 6

9 105 10 1 10

1= − ⋅

⋅⋅

− ⋅ ⋅ ⋅− −( )

( )( )

,

( ,

mi

F i N

F F F

2

2

1 2

0 045

0 045

=

= + =

� �

� � � �ii j N− 0 036, )

W q V V

W C Vver cen= −

= − ⋅ ⋅ ⋅ −−

( )

( , ,1 10 7 63 10 1 205 6 66 10 447⋅ =)V J

V V V V

V Vcen cen cen cen

cen

= + +

= ⋅ + ⋅

1 2 364 02 10 4 02, , 110 4 02 10

1 206 10

6 6

7

,

,

V V

V Vcen

+ ⋅

= ⋅

El campo eléctrico total será la suma vectorial delos tres:

Su módulo es:

b) Para calcular el trabajo necesario para trasladaruna carga de q = −10 μC del cuarto vértice hasta elcentro del cuadrado debemos determinar primeroel potencial en cada uno de los dos puntos. Deter-minamos la contribución de cada carga al poten-cial en el vértice del cuadrado:

Por el principio de superposición, el potencialeléctrico en el vértice será:

Determinamos el potencial en el centro del cua-drado:

V KQd

N m

C

Cm

V

cen19

2

2

4

1

2

9 101 25 10

0 28= = ⋅

⋅⋅

⋅ −,,

ccen

cen

V

V KQd

N m

C

= ⋅

= = ⋅⋅

4 02 10

2

9 101 25

6

29

2

2

,

, ⋅⋅

= ⋅

= = ⋅

−100 28

4 02 10

2

9

4

26

3

,

,

Cm

V V

V KQd

cen

cen 1101 25 10

0 28

4 02 10

92

2

4

36

,,

,

N m

C

Cm

V cen

⋅⋅

= ⋅

V

V V V V

V Vver ver ver ver

ver

= + +

= ⋅ + ⋅

1 2 362 81 10 2 81 1, , 00 2 01 10

7 63 10

6 6

6

V V

V Vver

+ ⋅

= ⋅

,

,

V KQ N m

C

Cmver1

92

2

4

19 10

1 25 100 4

2 8= = ⋅⋅

⋅⋅

=−,

,, 11 10

19 10

1 25 100

6

29

2

2

4

= = ⋅⋅

⋅⋅ −

V

V KQ N m

C

Cver

,,44

2 81 10

19 10

1 25

6

39

2

2

,

,

mV

V KQ N m

Cver

= ⋅

= = ⋅⋅

⋅⋅110

0 562 01 10

46

= ⋅C

mV

,,

E = ⋅ − + = ⋅9 48 10 1 1 1 34 106 2 2 7, ( ) , N/C

� � � � � �E E E E i j= + + = ⋅ − +1 2 3

69 48 10, ( ) N/C

� �E u1 1= = − ⋅⋅

⋅⋅ −

KQ N m

C

C

m1

29

2

2

4

19 10

1 25 10

0 4

,

( , ))

,

2

6

22

9

7 03 10

19 10

� �

� �

i

E i N/C

E u

1

2 2

= − ⋅

= = ⋅KQ NN m

C

C

m

⋅⋅

= ⋅

−2

2

4

2

6

1 25 10

0 4

7 03 10

,

( , )

,

� �

j

E2 jj N/C

E u

E

3 3

3

� �

=

= ⋅⋅

⋅⋅ −

KQ

d

N m

C

32

92

2

4

9 101 25 10,

( , )(

, [

C

m0 57

22

2 45 10

2

6

− +⎡

⎣⎢⎢

⎦⎥⎥

= ⋅

� �

i j)

E3 ((− +� �i j)] N/C

7. Campo eléctrico

+

+

Y

X

r1

Q1 = +4 · 10–6 C

Q2 = –5 · 10–6 C

P1 (0, 1) m

P2 (1, 0) m

B (1, 1) m

A (0, 0) m

q = +1 · 10–6 C

r2

�F2

�F1 �

F

Page 97: 9523 gl fis_tx2_cas

102

su módulo es constante sobre esta superficie. Por tan-to, el flujo a través de la superficie gaussiana será:

Por tanto, el campo eléctrico en el interior del cilin-dro es nulo.

Puntos exteriores

Elegimos como superficie gaussiana un cilindro cual-quiera de radio r, r > R. En las tapas de este cilindro,

�E

es perpendicular a d�S, de forma que el flujo es nulo. En

la cara lateral del cilindro,�E es paralelo a d

�S y su mó-

dulo es constante sobre la superficie, de forma que:

Aplicamos el teorema de Gauss, teniendo en cuentaque la carga eléctrica dentro de la superficie esQ = 2πRhσ:

— Con los datos del problema, R = 20 cm = 0,2 m; σ = +5 ⋅ 10-6 C ⋅ m−2; r = 30 cm = 0,3 m:

35. Datos: S = 10 cm2 = 1 ⋅ 10−3 m2; d = 1 mm = 1 ⋅ 10−3 m;Q = 2,4 ⋅ 10−9 C

ER

rm C m

C N= =

⋅ ⋅ ⋅

⋅ ⋅

− −

σε0

6 2

12 2 1

0 2 5 10

8 854 10

,

, ⋅⋅ ⋅

= ⋅

−m m

ENC

2

5

0 3

3 8 10

,

,

Φ = = =Q

E rhRh

ER

rεπ

π σ

εσ

ε0 0 0

22

; ;

Φ = ⋅ = = =⌠⌡⎮ ⌠

⌡⎮

s sextdS E dS E S E rh

� �Eext ext ext 2π

Φ = ⋅ = = = ⇒ =⌠⌡⎮

sdS E S

QE

� �Eint int intε0

0 0

En B(1, 1) m:

Por el principio de superposición:

Entonces, el trabajo necesario para trasladar q des-de el punto A al punto B será:

El potencial eléctrico tiene el mismo valor en lospuntos A y B, pues sólo depende de las cargas y dela distancia entre ellas, de forma que no es necesa-rio realizar ningún trabajo para trasladar una car-ga entre dichos puntos.

33. Datos: λ = +30 μC ⋅ m−1 = +3 ⋅ 10−5 C ⋅m−1; r0 = 1 m;

r = 3 m

Determinamos el campo y el potencial eléctricos paralos datos del enunciado a partir de los resultados delejemplo 2 (página 250):

— Representamos el vector intensidad de campoeléctrico sobre una circunferencia de 3 m de radiocentrada en el hilo.

34. Aplicamos el teorema de Gauss para determinar elcampo eléctrico en el interior y en el exterior de un ci-lindro infinito hueco de radio R y cargado uniforme-mente con una densidad superficial de carga σ.

Puntos interiores

Elegimos como superficie gaussiana un cilindro de ra-dio r, r < R. Por simetría, en las tapas del cilindro elvector

�E es perpendicular a d

�S, de forma que el flujo es

nulo. En la cara lateral, el vector�E es paralelo a d

�S y

Er

C m

C N= =

+ ⋅ ⋅

⋅ ⋅ ⋅

− −

− −

λπ ε π2

3 10

2 8 854 100

5 1

12 2 1, ⋅⋅ ⋅

= ⋅

= − = −+ ⋅

−m m

E N C

V lnrr

2

5

0 0

3

1 8 10

23 10

, /

λπ ε

−− −

− − −

⋅ ⋅ ⋅ ⋅⋅

5 1

12 2 1 22 8 854 10

31

,

C m

C N m

lnm

π

,m

V= − ⋅5 9 105

W q V V C JA B= − = ⋅ ⋅ =−( ) 1 10 0 06

V V V V V VB B B= + = − = −1 2 36000 45000 9000

V KQr

N m

C

Cm

V

V

BB

11

1

92

2

6

9 104 10

136000= = ⋅

⋅⋅

⋅=

222

2

92

2

6

9 105 10

14500B

B

KQr

N m

C

Cm

= = ⋅⋅

⋅− ⋅

= −−( )

00 V

r = 3 m

R

r

+

++

+

+

+

+

+

++

σ

r σ

h

R

+ +

++

+

++

d�S

�E

�E

�E

d�S

Page 98: 9523 gl fis_tx2_cas

103

La capacidad equivalente total será:

EJERCICIOS Y PROBLEMAS

38. Conservación de la carga eléctrica

En los procesos físicos la carga eléctrica puede redis-tribuirse en un cuerpo, o pasar de un cuerpo a otro,pero en todo proceso la carga eléctrica total permane-ce constante.

Cuantización de la carga eléctrica

Cualquier carga eléctrica es un múltiplo entero deuna unidad elemental de carga, la carga del electrón,cuyo valor absoluto denotamos por e.

⎜e⎜ = 1,602 ⋅ 10−19 C

39. a) Si una carga positiva penetra en un campo eléc-trico uniforme con una velocidad inicial con ladirección y el sentido del campo, la aceleracióndebida al campo y la velocidad inicial serán para-lelas y del mismo sentido. Por tanto, la partículase acelerará e incrementará su velocidad. La car-ga describe un MRUA.

b) Si la velocidad inicial tiene sentido opuesto al cam-po, la aceleración será de sentido contrario a la velo-cidad inicial. La partícula irá frenando hasta parar-se. A continuación, empezará a moverse en sentidocontrario al inicial, incrementando uniformementesu velocidad. La carga describe un MRUA.

c) Si la velocidad forma cierto ángulo con el campoeléctrico, la carga adquiere una aceleración en ladirección y el sentido del campo eléctrico. La velo-cidad tiene dos componentes: una perpendicularal campo que se mantiene constante, y una parale-la al campo que varía uniformemente. La cargaeléctrica describe un movimiento parabólico.

40. Si en cierta región del espacio el potencial eléctrico esconstante, el campo eléctrico es nulo. La diferencia depotencial entre dos puntos es el trabajo que realiza elcampo para trasladar una partícula de carga unidad en-tre los dos puntos. Si el potencial es constante en unaregión del espacio, el trabajo realizado por el campopara desplazar la carga eléctrica unidad entre dos pun-tos cualesquiera de esta región es nulo. Esto sólo puedesuceder si el campo en dicha región es nulo.

41.

C C C C nF= + + =12 34 567 3 95,

a) Determinamos la capacidad del condensador:

b) Calculamos la diferencia de potencial:

c) Si introducimos un dieléctrico de εr = 6,8 entre lasarmaduras del condensador, la capacidad será:

Entonces:

36. Datos: Q = 1,8 ⋅ 10−11 C; V = 1,5 V; εr = 7,5

a) La nueva capacidad del condensador será:

Como el condensador se mantiene aislado, su car-ga no varía: Q’ = 1,8 ⋅ 10−11 C. Entonces, la diferen-cia de potencial es:

b) Si mantenemos la pila conectada, la carga eléctri-ca varía, pero la diferencia de potencial se mantie-ne constante. La capacidad variará de la mismamanera que en el apartado anterior. Por tanto:

37. a) Llamamos C23 a la capacidad equivalente de la aso-ciación en paralelo de C2 y C3, que será:

La capacidad equivalente total será:

b) Calculamos la capacidad equivalente de cada rama:

1 1 1 1 16

14

2 40

112 1 2 12

12

3

C C C CC nF

C

= + = + =; ; ,

44 3 4 3434

567

1 1 1 11

13

0 75

1

= + = + =C C C

C nF

C

; ; ,

== + + = + +

=

1 1 1 1 12

14

12

0 805 6 7 567

567

C C C C

C nF

;

,

1 1 1 1 16

19

16

2 251 23 4C C C C

C nF

= + + = + +

= ,

C C C nF nF nF23 2 3 4 5 9= + = + =

V V

Q C CV Q

Q Cr r

′ ′ ′

=

= = =

= ⋅ ⋅ −

1 5

7 5 1 8 10 11

,

, ,

V ε ε

== ⋅ −1 3 10 10, C

VQC

QC

V

VV

V

r r

′′′

= = =

= =

ε ε

1 57 5

0 2,

,,

CSd

Cr r′ = =ε ε ε0

VQC

C

FV= =

⋅=

2 4 10

59 8 1040

9

12

,

,

CSd

C

N m

mr= = ⋅

⋅⋅ ⋅

⋅−

ε ε012

2

2

3 2

8 854 10 6 81 10

1 1, ,

0059 8 10 59 8

3

12

−= ⋅ =, ,m

C F pF

VQC

C

FV= =

⋅=

2 4 10

8 8 10271 5

9

12

,

,,

CSd

C

N m

m

mC

= = ⋅⋅

⋅⋅

⋅−

−ε012

2

2

3 2

38 854 101 10

1 10,

== ⋅ =−8 8 10 8 812, ,F pF

7. Campo eléctrico

Superficies equipotenciales

�E

Page 99: 9523 gl fis_tx2_cas

104

cos creados por Q1 y Q2 en el punto medio delsegmento que une las dos cargas.

Aplicamos el principio de superposición:

Determinamos el potencial eléctrico en este puntocalculando las contribuciones de las dos cargas y apli-cando el principio de superposición:

b) Representamos los campos eléctricos creados porQ1 y Q2 en un punto equidistante 10 cm de ambascargas y calculamos su valor.

E KQ

ru

N m

C

C

m

�� �1

1

12 1

92

2

5

29 101 10

0 1= = ⋅

⋅⋅

⋅ −

( , )⋅⋅

⋅ +⎡

⎣⎢⎤

⎦⎥= ⋅ +

⎣⎢⎤

⎦⎥12

3 9 1012

36( ) ( )� � � �i j i j

NC

EE KQ

ru

N m

C

C�� �2

2

22 2

92

2

5

9 101 10

0 1= = ⋅

⋅⋅

− ⋅ −( )

( , )

( ) ( )

m

i j i j

2

612

3 9 1012

3

⋅ − +⎡

⎣⎢⎤

⎦⎥= ⋅ −

⎣⎢� � � � ⎤⎤

⎦⎥NC

� � � � �

u i j i j

u

o o1

2

60 3012

3= + = +

=

cos cos ( )

cos cos ( )120 3012

3o oi j i j� � � �

+ = − +

V KQr

N m

C

Cm1

1

1

92

2

569 10

1 100 05

1 8 10= = ⋅⋅

⋅⋅

= ⋅−

,,

( ),

V

V KQr

N m

C

Cm2

2

2

92

2

5

9 101 100 05

= = ⋅⋅

⋅− ⋅

= −−

11 8 10

1 8 10 1 8 10 0

6

1 26 6

,

, ,

= + = ⋅ − ⋅ =

V

V V V V V V

E E E u N C u N C�� �� �� � �

= + = ⋅ + ⋅1 27 73 6 10 3 6 10, / , /

EE u N C�� �

= ⋅7 2 107, /

r rd

m

E KQ

ru

N m

C

1 2

11

12

92

2

20 05

9 101

= = =

= = ⋅⋅

⋅⋅

,

�� � 110

0 05

3 6 10

5

2

17

2

= ⋅

=

( , )

, /

C

mu

E u N C

E K

�� �

�� QQ

ru

N m

C

C

m2

22

92

2

5

9 101 10

0 05( )

( )

( ,− = − ⋅

⋅⋅

− ⋅ −�))

, /

2

273 6 10

�� �

u

E u N C= ⋅

42. En un conductor, toda la carga eléctrica se distribuyepor la superficie de éste y de forma que el campo eléc-trico en el interior del conductor sea nulo. Si quere-mos proteger un aparato sensible de un campo eléctri-co, podemos introducirlo en una caja metálica (jaulade Faraday). Cuando exista cualquier campo eléctricoexterior, las cargas de la caja metálica se distribuiránpor la superficie de manera que el campo en el interiorsea nulo.

43. La capacidad de un condensador es el cociente entrela carga eléctrica que almacena y la diferencia de po-tencial aplicada entre sus bornes. Esta relación es cons-tante para cada condensador y depende de sus caracte-rísticas geométricas, de la separación entre las placas ydel aislante que existe entre ellas.

— En un condensador plano, la capacidad es directa-mente proporcional a la constante dieléctrica delmaterial interpuesto entre las armaduras.

44. Datos: Q1 = +4,0 ⋅ 10−9 C; Q2 = +2,0 ⋅ 10−9 C;

r = 6 cm = 0,06 m

Determinamos el módulo de la fuerza que ejercen lasdos cargas entre sí:

45. Datos: Q1 = +5 ⋅ 10−6 C; Q2 = −4 ⋅ 10−6 C;

Q3 = +2 ⋅ 10−6 C; r12 = 30 cm = 0,3 m;

r13 = 10 cm = 0,1 m

r23 = r12 − r13 = 0,3 m − 0,1 m = 0,2 m

Determinamos la fuerza que ejercen Q1 y Q2 por sepa-rado sobre Q3. Llamaremos �u al vector unitario en ladirección y el sentido de Q1 a Q2:

La fuerza total sobre Q3 será la suma de las dos:

Su módulo es F = 10,8 N.

46. Datos: Q1 = +1 ⋅ 10−5 C; Q2 = −1 ⋅ 10−5 C; d = 10 cm = 0,1 m

a) Llamamos �u al vector unitario en la dirección y elsentido de Q1 a Q2 y calculamos los campos eléctri-

� � � � � �F F F u N u N u N= + = + =12 23 9 1 8 10 8, ,

� �

F KQ Q

ru

FN m

C

C

131 3

132

139

2

2

6

9 105 10

=

= ⋅⋅

⋅⋅ ⋅− 22 10

0 1

9

6

2

13

232 3

2

=

=

( , )

C

mu

F u N

F KQ Q

r

� �

332

239

2

2

6

9 104 10 2 10

( )

( )

= − ⋅⋅

⋅− ⋅ ⋅ ⋅− −

u

FN m

C

C 66

2

23

0 2

1 8

( , )

,

C

mu

F u N

� �=

� �� � � �

� �

F KQ Q

r

FN m

C

=

= ⋅⋅

⋅⋅ −

1 22

92

2

9

9 104 10 CC C

m

F N

⋅ ⋅

= ⋅

2 10

0 06

2 10

9

2

5

( , )

� ��

Y

X

+ –

30°

60°

Q1 = +1 · 10–5 C Q2 = –1 · 10–5 C

r1 = 10 cm

d = 10 cm

r2 = 10 cm

�E

�E1

�E2

�u2�u1

Page 100: 9523 gl fis_tx2_cas

105

no es necesario realizar trabajo alguno para trasla-dar una carga entre dos puntos de una superficieequipotencial.

W = 0 J

49. Datos: r = 50 cm = 0,5 m; R = 30 cm = 0,3 m;

Q = 4,3 ⋅ 10−6 C

Para calcular el campo aplicaremos el teorema deGauss sobre una esfera de radio r concéntrica a la es-fera cargada. Por simetría, en cada punto de la super-ficie el campo y el vector superficie son parale-los, Además, el campo será constante entoda la superficie. Entonces, el flujo a través de la es-fera de radio r será:

Aplicamos el teorema de Gauss:

El campo creado por la esfera en puntos exteriores aella es el mismo que crearía una carga puntual situadaen el centro de la esfera. El potencial será tambiénequivalente al creado por una carga puntual:

50. Datos: ET = 110 N/C; RT = 6 370 km = 6,37 ⋅ 106 m

El campo eléctrico creado por una esfera cargada enpuntos exteriores a la esfera es equivalente al campocreado por una carga puntual situada en el centro de laesfera. Como el campo eléctrico está dirigido haciala Tierra, sabemos que la carga será negativa. La cargatotal de la Tierra será, pues:

Si consideramos esta carga uniformemente distribui-da, la densidad superficial de carga de la Tierra será:

51. Datos: d = 1 mm = 1 ⋅ 10−3 m; V = 1 000 V; εr = 2,3

Determinamos primero la capacidad del condensadorpor unidad de área:

CS d

C

N m mC

r= = ⋅⋅

⋅ ⋅⋅

−−ε ε0

122

2 3

18 854 10 2 3

1

1 10, ,

SSF

m= ⋅ −2 0 10 8

2,

σπ π

= =− ⋅

⋅= − ⋅

Q

R

C

m4

4 96 10

4 6 37 109 7 10

2

5

6 2

,

( , ), −−10

2

C

m

E KQ

rQ

ErK

QN C

� �� �

� �

= =

=⋅ ⋅

2

2

6110 6 37 10

;

/ ( , ),

,

m

N m

C

C

Q C

2

92

2

5

5

9 10

4 96 10

4 96 10

⋅⋅

= ⋅

= − ⋅

V KQr

N m

C

Cm

= = ⋅⋅

⋅⋅

= ⋅−

9 104 3 10

0 57 74 109

2

2

64,

,, VV

Φ = = =

=⋅ −

QE r

QE

Q

r

EC

επ

ε πε0

2

0 02

6

44

4 3 10

4

; ;

,

ππ , ( , )

,

8 854 10 0 05

1 55 1

12 2 1 2 2⋅ ⋅ ⋅

= ⋅

− − −C N m m

E 005 /N C

Φ = ⋅ = ⋅ = = =⌠⌡⎮ ⌠

⌡⎮ ⌠

⌡⎮

s s sd E dS E dS ES E r

� �E S 4 2π

� �E ⋅ = ⋅d E dSS .

Aplicamos el principio de superposición:

Determinamos el potencial eléctrico calculando lascontribuciones de las dos cargas y aplicando el princi-pio de superposición:

47. Datos: Q1 = +4 ⋅ 10−8 C; Q2 = −3 ⋅ 10−8 C; d = 10 cm = 0,1 m

a) El punto medio del segmento que une las cargas dis-

ta de cada una Calculamos

el potencial eléctrico debido a cada carga en dichopunto:

Aplicamos el principio de superposición:

b) En un punto situado a r1 = 8 cm = 0,08 m de la pri-mera carga y a r2 = 6 cm = 0,06 m de la segunda,los potenciales eléctricos son:

Aplicamos el principio de superposición:

c) La energía potencial de una partícula con carga q = +5 ⋅ 10−9 C en los puntos anteriores será:

48. Datos: q = +1 C

a) V1 = −25 V; V2 = +25 V

El trabajo necesario para trasladar la carga q de V1a V2 será:

b) Los puntos de una superficie equipotencial, por de-finición, tienen todos el mismo potencial. Por tanto,

W q V V C V V J= − = − − = −( ) ( )1 2 1 25 25 50

Ep qV C V J

Ep qVA A

B b

= = + ⋅ ⋅ = ⋅

= = +

− −5 10 1 800 9 10

5

9 6

⋅⋅ ⋅ =−10 0 09 C V J

V V V V V VB = + = − =1 2 4 500 4 500 0

V KQr

N m

C

Cm

V

V

11

1

92

2

8

9 104 100 08

4 500= = ⋅⋅

⋅⋅

=−

,

222

2

92

2

8

9 103 100 06

4 500= = ⋅⋅

⋅− ⋅

= −−

KQr

N m

C

Cm,

VV

V V V V V VA = + = − =1 2 7 200 5 400 1 800

V KQr

N m

C

Cm

V

V

11

1

92

2

8

9 104 100 05

7 200= = ⋅⋅

⋅⋅

=−

,

222

2

92

2

8

9 103 100 05

5 40= = ⋅⋅

⋅− ⋅

= −−

KQr

N m

C

Cm

( ),

00 V

r r d m1 212

0 05= = = , .

V KQr

N m

C

Cm

V

V

11

1

92

2

559 10

1 100 1

9 10= = ⋅⋅

⋅⋅

= ⋅−

,

222

2

92

2

5

9 101 100 1

9 10= = ⋅⋅

⋅− ⋅

= − ⋅−

KQr

N m

C

Cm

( ),

55

1 25 59 10 9 10 0

V

V V V V V V= + = ⋅ − ⋅ =

E E E i jNC

�� �� �� � �= + = ⋅ +

⎣⎢⎤

⎦⎥+

+ ⋅

1 299 10

12

3

9 1

( )

0012

3 9 10

9 10

6 6

6

⋅ −⎡

⎣⎢⎤

⎦⎥= ⋅

= ⋅

( )� � �i j

NC

iNC

ENC

7. Campo eléctrico

Page 101: 9523 gl fis_tx2_cas

106

Aplicamos el principio de superposición:

El módulo es:

53. Datos: r = 10 cm = 0,1 m; Q = 3 ⋅ 10-6 C; R = 5 cm;

R = 0,05 m

Campo eléctrico en el interior

Para aplicar el teorema de Gauss escogemos como su-perficie gaussiana una esfera de radio r1 < R concén-trica con la esfera metálica. Por simetría en cada pun-to de esta superficie, el campo eléctrico y el vectorsuperficie son paralelos y el campo eléctrico es cons-tante en toda la superficie.

Calculamos el flujo a través de S1:

Aplicamos el teorema de Gauss:

El campo eléctrico en el interior de la esfera metálicaes el mismo que crearía una carga eléctrica puntual Qsituada en el centro de la esfera.

Campo eléctrico en el exterior

Como la carga total del conductor es nula, en la super-ficie exterior de la corteza quedará una carga +Q, queserá la responsable del campo eléctrico exterior a laesfera. Así pues, en los puntos exteriores, el cálculoserá análogo al anterior, y el campo será:

— Sustituimos los datos del enunciado en la expre-sión obtenida para el campo eléctrico en el exte-rior de la esfera metálica:

EQ

rK

Q

r

N m

C

C

m2

0 22 2

92

2

6

49 10

3 10

0 1= = = ⋅

⋅⋅

⋅ −

πε ( , ))

, /

2

262 7 10E N C= ⋅

EQ

r20 2

24=

π ε

Φ = = =Q

E rQ

EQ

rεπ

ε π01 1

2

01

12

44

; ;

Φ = ⋅ = = =⌠⌡⎮ ⌠

⌡⎮ ⌠

⌡⎮

s s sdS E dS E dS E S

1 1 1

1 1 1 1 1 1 1

� �E1

ΦΦ = E r1 124 π

F N N N= + − = ⋅( , ) ( , ) ,1096 4 1096 4 1 55 102 2 3

� � � � � �

F F F F i j N

j

= + + = −⎡

⎣⎢

⎦⎥ −

1 2 3 4051

2

810

( )

, ( )N i N i j N+ = −810 1096 4� � �

Entonces, calculamos la carga de una armadura porunidad de área, que es igual a la carga inducida en lasuperficie del dieléctrico por unidad de área:

52. Datos:

a) Por simetría, el campo en el centro del cuadradodebe ser nulo. Teniendo en cuenta que todas lascargas distan del centro la misma distancia y quetienen el mismo valor Q, el módulo del campo crea-do por cada una de ellas será el mismo, E. Si suma-mos vectorialmente todos los campos:

b) Determinamos la fuerza eléctrica que experimentaQ4 en el cuarto vértice debido a las otras tres car-gas. Para ello, calculamos primero la contribuciónde cada carga:

� �F KQ Q

du

N m

CC

11 4

29

2

2

4

9 10

3 10 3 10

= = ⋅⋅

⋅⋅ ⋅ ⋅− −

′1

44

2

1

2

1

2

4051

2

( )( )

( )

C

mi j

F i j

⋅ −⎡

⎣⎢

⎦⎥

= −⎡

� �

� � �

⎣⎣⎢

⎦⎥

= = ⋅⋅

⋅⋅ −

N

F KQ Q

lu

N m

C

� �2

2 42

92

2

4

9 10

3 10

′2

( )( )

C C

mj j N

F KQ Q

⋅ ⋅⋅ − = −

=

−3 10

1810

4

2

33 4

� �

llu

N m

CC C

m

29

2

2

4 4

9 10

3 10 3 10

1

�′3 = ⋅

⋅⋅

⋅⋅ ⋅ ⋅− −

( )22

3 810

� �

i

F i N=

E E u u u u

E E i j i

�� � � � �

�� � � �= + + +

= − + − −

( )

( ) (

1 2 3 4

1

2

�� � � � �j i j i j) ( ) ( )+ + + − +⎡⎣ ⎤⎦ = 0

� � � � � �

� �

u i j d u i j

u

1 1

2

1

2

1

21

2

= − = + = −

= −

( ) ( )

(

l l2 2 ′

ii j d l m m m u j

u i

− = + = = −

= +

� � �

� �

) ( ) ( )

(

2 22

3

1 2

1

2

�� � � � � �j u i j u i) ( )4 3

1

2= − + =′

σ

σ

= = = ⋅ ⋅

= ⋅

QS

CS

VF

mV

C m

,

, /

2 0 10 1 000

2 0 10

82

5 2

Y

X d

+ +

Q2 = +3 · 10–4 C

Q4 = +3 · 10–4 C

Q3 = +3 · 10–4 C

Q1 = +3 · 10–4 C

l = 1 m

l = 1 m

+ +

�E1

�E4 �

E3

�E2 �u1

�u2

�u3

�u4

�u′1�u′2

�u′3

R

Q+

r1

r2

�E1

�E2

d�S1

d�S2

Page 102: 9523 gl fis_tx2_cas

107

Supongamos que la carga ligada al muelle es positivay que la carga fija, que es negativa, está situada a su de-recha.

Cuando se modifican los controles el comportamien-to del sistema se puede resumir como:

— Al aumentar la carga positiva, ésta oscila alrededordel punto de equilibrio y queda finalmente despla-zada hacia la derecha (al disminuirla queda des-plazada hacia la izquierda).

— Al aumentar la constante del resorte, la carga po-sitiva oscila hacia la izquierda (al disminuirla lohace hacia la derecha).

— Al aumentar la longitud del resorte, la carga posi-tiva oscila hacia la derecha (al disminuirla lo hacehacia la izquierda).

EVALUACIÓN

1. El hecho de que la carga eléctrica esté cuantizada sig-nifica que toda carga eléctrica que se encuentre en la naturaleza es múltiplo de una unidad elemental de carga, que coincide en módulo con la carga del elec-

trón,

2. Datos:

El campo se anulará en algún punto del segmento queva de P1 a P2. Dicho segmento es perpendicular al ejeX, con x = 1 constante.

Si llamamos (x, y) a las coordenadas del punto que bus-camos, observamos que x = 1 m. La distancia de dicho

punto a Q1 es y la distancia de dicho punto a

Q2 es

Para que el campo eléctrico se anule en P(x, y), loscampos creados por las cargas Q1 y Q2 deben tener elmismo módulo y signo opuesto.

0

016 8

1 2= = − = −−

= =+ −

E E E KQ

yK

Q

y

E KQy

� � � �

� �

2 2

2

(4 ) )

�� � � �� � � �

� �� �

y y

y y

KQy

y

⎝⎜

⎠⎟

=−

2

2 2

2

(4 )

016 8

;

(4 )2−

= ⇒ − = =

� �

� � � �

y

E y y0 16 8 0 2

d2 = − .4 � �y

d ,1 = � �y

� �e C= ⋅ −1 602 10 19, .

54. — Al frotar el objeto con un trozo de lana, el primeroadquiere una carga eléctrica negativa. Al acercareste objeto a la bolita del electroscopio se produceen ésta una redistribución de la carga eléctrica: lascargas positivas se acercan al objeto de plástico ylas cargas negativas se alejan de él. En consecuen-cia, la bolita es atraída por el objeto de plástico.

Al tocar la bolita con el objeto de plástico, la bolitaqueda cargada negativamente y sufre una repulsión.

— Al frotar el objeto de vidrio con un trozo de seda,el primero queda cargado positivamente. Por tan-to, la bolita (cargada negativamente) resulta atraí-da cuando la acercamos al objeto de vidrio.

55. Datos:

Llamamos C23 a la capacidad equivalente de la asocia-ción de C2 y C3 en paralelo:

Entonces, la capacidad equivalente de la rama superiorC1234 será:

Para la rama inferior la capacidad equivalente C567 será:

Entonces, la capacidad equivalente de todo el sistema será:

56. El montaje sugerido realizado con INTERACTIVEPHYSICS en condiciones de equilibrio tiene el siguien-te aspecto:

C C C F F F= + = + =1234 567 12 5 10 22 5, ,μ μ μ

1 1 1 1 160

120

130567 5 6 7

56

C C C C F F F

C

= + + = + +μ μ μ

77 10= μF

1 1 1 1 125

1150

1301234 1 23 4C C C C F F F

= + + = + +μ μ μ

CC F1234 12 5= , μ

C C C F F F23 2 3 75 75 150= + = + =μ μ μ

7. Campo eléctrico

C2 = 75 μF

C3 = 75 μF

C6 = 20 μF

C1 = 25 μF C4 = 30 μF

C7 = 30 μFC5 = 60 μF

Y

X–

P (x, y)

P2 (1, –4) m

P1 (1, 0) m

Q1 = Q = –3,5 · 10–6 F

Q2 = Q = –3,5 · 10–6 F

�E1

�E2

Resorte 3Longitud en reposo

Resorte 3Constante del resorte

Círculo 1Carga

Page 103: 9523 gl fis_tx2_cas

108

mismo punto el campo eléctrico (perpendicular a la su-perficie equipotencial correspondiente) podía tenerdos direcciones diferentes, lo cual no tiene sentido.

5. Datos: V = 15 V; E = 30 N/C; q = + 2,5 ⋅ 10−9 C;

m = 5 ⋅ 10−3 kg

a) Determinamos la separación entre placas, te-niendo en cuenta que el campo eléctrico es uni-forme:

b) La aceleración vendrá dada por la ecuación fun-damental de la dinámica:

c) La variación de la energía potencial será:

6. Datos: C1 = 14 μF; C2 = 21 μF; C3 = 12 μF

Determinamos la capacidad equivalente de la asocia-ción en serie de la rama superior:

Entonces, la capacidad equivalente total será:

C C C F F F= + = + =12 3 8 4 12 20 4, ,μ μ μ

1 1 1 114

121

8 412 1 2

12C C C F FC F= + = + =; ,

μ μμ

Δ ΔEp q V C V J= = ⋅ ⋅ = ⋅− −, ,2 5 10 15 3 7 109 8

F m a qE m a

aqEm

C N C

= =

= =⋅ ⋅−

;

, /,

2 5 10 300 00

9

551 5 10 5

2,kg

m

s= ⋅ −

V E dr E dr Ed

dVE

VN C

m

= = =

= = =

⌠⌡⎮ ⌠

⌡⎮

�� �

/,

1530

0 5

Por tanto, el punto donde se anula el campo es P(1, −2) m.

— El hecho de que el campo eléctrico sea nulo noquiere decir que el potencial sea cero. El potencialen P(x, y) será la suma algebraica de los poten-ciales creados por Q1 y Q2:

3. Datos: WA→∞ = 1,25 J; WB→∞ = 4,50 J

a) El trabajo realizado para desplazar la carga q de Aa B será:

Hemos utilizado la propiedad del campo eléctricode ser conservativo, por esta razón el trabajo reali-zado para desplazar una carga eléctrica entre dospuntos no depende del camino seguido.

b) Si q = −5 μC:

4. Las superficies equipotenciales de un campo eléctricono pueden cortarse, ya que esto significaría que en un

W qV W qV

VW

qJ

A A B B

AA

→ →

→−

= =

= =− ⋅

∞ ∞

;

,1 25

5 10 6 FFV

VW

qJ

FBB

= ⋅

= =− ⋅

= − ⋅→

2 5 10

4 50

5 109 10

5

65

,

,∞ V

W W W J J JAB A B= − = − = −→ →∞ ∞ 1 25 4 50 3 25, , ,

VN

= ⋅ 99 10⋅⋅

⋅ − ⋅ +⎛⎝⎜

⎞⎠⎟

= − ⋅

−m

CC

m m

V

2

263 5 10

12

12

3 15

( , )

, 1104 V

V V V KQd

KQd

KQd d

= + = + = +⎛

⎝⎜⎞

⎠⎟1 2

1

1

2

2 1 2

1 1

Page 104: 9523 gl fis_tx2_cas

109

Campo magnético8

• a) Datos:

Una masa de 40 kg en una campo gravitatorio de 1,6 N/kg experimenta una fuerza:

F = m g = 40 kg ⋅ 1,6 N/kg = 64 N

b) Datos: q = 125 μC = 1,25 ⋅ 10−4 C;

Una masa de 125 μC en una campo eléctrico de 3 ⋅ 10−6 N/C experimenta una fuerza:

F = q E = 1,25 ⋅ 10−4 C ⋅ 3 ⋅ 10−6 N/C = 3,75 ⋅ 10−10 N

• Respuesta sugerida:

— Un timbre está constituido por un circuito eléctricocon un interruptor, un electroimán, una armadurade hierro dulce, una varilla elástica, una campanillay un tornillo o elemento ruptor.

Un altavoz está constituido por un imán, una o dos bo-binas, una membrana vibratoria y un pabellón acústico.

— Timbre

Cuando se cierra el circuito eléctrico pulsando el in-terruptor, una corriente eléctrica circula a través delelectroimán, que atrae entonces a su armadura. Laarmadura está unida a la varilla elástica que, al des-plazarse, golpea la campanilla produciendo el soni-do. El elemento ruptor hace que el electroimán sedesactive cada vez que atrae a la varilla.

Altavoz

La corriente variable que circula por la bobina ha-ce variar el flujo magnético y, por tanto, tambiénhace variar la fuerza de atracción sobre la membrana,provocando las vibraciones de ésta. La vibración setransmite a las partículas de aire adyacentes y de estamanera se genera el sonido. El altavoz está provistode un pabellón para mejorar el rendimiento acústico.

� �E E= = ⋅ −3 10 6 N/C

m 40 kg; g g 1,6 N/kg= = =�

PREPARACIÓN DE LA UNIDAD

• Podemos identificar los polos norte y sur de los imanescon la ayuda de una brújula. La punta de la aguja que se-ñala el polo Norte terrestre constituye el polo norte de laaguja imantada. Por tanto, como los polos distintos seatraen, si acercamos la brújula a los imanes, la aguja se-ñalará el polo sur de éstos.

• Existen diversos métodos para obtener imanes artificiales.A continuación explicamos cuatro métodos de imantación.

Imantación por frotamiento. Consiste en frotar repetidasveces la barra de acero con un imán, siempre en el mis-mo sentido y con el mismo extremo del imán.

Imantación por contacto. Consiste en acercar la barra deacero a un imán de modo que ambos queden en contacto.

Imantación por influencia. Consiste en aproximar la ba-rra de acero a un imán sin que lleguen a tocarse.

Imantación por corriente eléctrica. Consiste en enrollarun hilo de cobre alrededor de la barra de acero procu-rando que las espiras estén bastante próximas, y conectarlos extremos del hilo conductor a los polos de una pila.

• Datos: Q = 150 C; t = 1 min = 60 s

La intensidad de corriente eléctrica es la carga que atra-viesa una sección por unidad de tiempo.

• Decimos que existe un campo de fuerzas en un lugar delespacio si, al colocar en él un cuerpo de prueba, éstequeda sometido a una fuerza.

Para representar esquemáticamente un campo de fuer-zas en una región del espacio se dibujan las líneas decampo. Éstas se trazan de modo que el vector intensidadde campo es tangente a las líneas de campo en cada pun-to y tiene el mismo sentido que éstas. Además, se trazande forma que la densidad de las líneas de campo sea pro-porcional a la intensidad del campo.

IQt

Cs

A= = =15060

2 5,

�F

Armadura Elemento ruptor Varilla elástica

Circuito eléctrico Electroimán Campanilla

8. Campo magnético

Page 105: 9523 gl fis_tx2_cas

110

Sí. Cuando la carga eléctrica se mueve en la direccióndel campo magnético, no actúa ninguna fuerza sobreella.

6. Un campo magnético se describe mediante el vectorcampo magnético o inducción magnética,

�B. La in-

ducción magnética es la fuerza que actúa sobre unacarga de 1 C que se desplaza con una velocidad de 1 m/s perpendicularmente a

�B.

El campo magnético se representa con las líneas de in-ducción magnética, tangentes en cada punto al vectorinducción magnética. La densidad de líneas en unaregión es proporcional al módulo del campo magnéti-co en dicha región.

7. Las líneas de inducción magnética, al igual que las decampo eléctrico, cumplen estas condiciones:

— El vector campo,�B, es tangente a las líneas de in-

ducción en cada punto del espacio y tiene el mis-mo sentido que éstas.

— La densidad de líneas de inducción es una regiónproporcional al módulo de

�B en dicha región.

Las líneas de inducción magnética, a diferencia de lasde campo eléctrico:

— No tienen principio ni fin. Salen del polo nortedel imán y entran por el polo sur, continuando surecorrido por el interior del imán hasta cerrar lalínea.

— Otra diferencia esencial es que las fuerzas magné-ticas no son tangentes a las líneas de inducción,como en el caso del campo eléctrico, sino perpen-diculares en cada punto al campo magnético.

8. Resultados:

Las limaduras se orientan alrededor del hilo forman-do circunferencias centradas en éste, siguiendo las lí-neas de inducción magnética del campo generado porla corriente que circula por el hilo. Las circunferen-cias son más claras cerca del hilo, donde el campomagnético es más intenso.

9. Las líneas de inducción magnética son tangentes alcampo magnético en todo punto, mientras que el cam-po magnético es siempre perpendicular al elementode corriente que lo genera. Por tanto, las líneas de in-ducción magnética de una corriente eléctrica no pue-den ser paralelas a la corriente.

10. Datos: R = 32 cm = 0,32 m; I = 2 A

Calculamos la inducción magnética en el centro de laespira utilizando la expresión hallada en el ejemplo 1(pág. 201) del libro del alumno:

BI

RT m A A

mT= =

⋅ ⋅ ⋅ ⋅

⋅= ⋅

− −−μ π0

7 16

24 10 2

2 0 323 9 10

,,

1. MAGNETISMO

1. Al romper un imán en dos o más trozos obtenemos va-rios imanes porque el magnetismo de los materiales esdebido a la orientación de los dipolos magnéticos de suinterior. El movimiento de los electrones alrededor delnúcleo atómico genera un campo magnético. Aunquerompamos un imán, en cada uno de los nuevos trozossigue habiendo electrones en movimiento que generanel campo magnético correspondiente y con la mismaorientación.

Para determinar con una brújula los polos norte y surde un imán, es necesario tener en cuenta que la punta dela aguja que señala el polo Norte terrestre constituyeel polo norte de la aguja imantada. Por tanto, como lospolos distintos se atraen, la aguja señalará el polo surdel imán.

2. La experiencia que sirvió de base para la teoría electro-magnética fue la de Oersted (1777-1851), quien descu-brió que una corriente eléctrica desviaba la aguja iman-tada de una brújula. Esto significa que la corrienteeléctrica genera un campo magnético.

3. Las propiedades magnéticas de los imanes naturalestienen su origen en el movimiento de los electrones alre-dedor del núcleo del átomo. Como los electronesson cargas eléctricas, podemos entender su movimien-to alrededor del núcleo como una corriente eléctricaque, tal como demostró Oersted, generará un campomagnético. De esta forma, cada átomo es equivalente aun pequeño imán o dipolo magnético. Este hecho escomún a todos los materiales. En los imanes naturaleslos dipolos están orientados en un mismo sentido y su-man sus efectos, mientras que en la mayor parte de losmateriales están orientados al azar y sus efectos se can-celan.

4. El imán atrae a los objetos de hierro, pero no a los decobre o aluminio. El imán no atrae a cualquier tipode metal, sólo atraerá aquellos materiales cuyos dipolosatómicos puedan orientarse según el campo magnéticodel imán.

2. ESTUDIO DEL CAMPO MAGNÉTICO

5. La fuerza magnética que actúa sobre una carga eléctri-ca depende del valor de ésta, de la velocidad con queésta se mueve, del valor del campo magnético y de laorientación de la velocidad respecto al campo mag-nético.

Carcasa externa

BobinasImán

Pabellón

Membrana

Page 106: 9523 gl fis_tx2_cas

111

Aplicando el teorema de Ampère:

16. Un electrón debe entrar en un campo magnético uni-forme moviéndose en una dirección paralela al vectorinducción magnética

�B para que el campo magnético

no ejerza ninguna fuerza sobre él. Como la fuerza deLorentz es proporcional al producto vectorial de la ve-locidad por el campo magnético, si estos vectores sonparalelos, la fuerza es nula.

17. a) Si una carga eléctrica positiva se mueve paralela-mente a un hilo conductor, como el campo mag-nético es perpendicular al hilo (las líneas de in-ducción magnética son circunferencias centradasen el hilo), la velocidad de la carga y el campomagnético serán perpendiculares en todo mo-mento. Por tanto, la fuerza que experimentará lacarga será máxima y orientada en la dirección dela recta que une la carga con el hilo conductor.

b) Según el sentido de la corriente, la fuerza acerca-rá la carga hacia el conductor o la alejará de él. Sila carga se mueve en el mismo sentido que la co-rriente, la fuerza atraerá la carga hacia la corrien-te. En cambio, si la corriente y la velocidad de lacarga tienen sentidos contrarios, la fuerza alejarála carga del conductor.

18. Datos: q = +e = +1,6 ⋅ 10−19 C; mp = 1,67 ⋅ 10−27 kg;

B = 0,2 T; v = 3 ⋅ 107 m/s; α = 90°

a) Calculamos la fuerza mediante la ley de Lorentz:

b) Hallamos el radio de la órbita circular que des-cribe:

19. La principal aplicación del espectrómetro de masasconsiste en identificar los diferentes isótopos de unmismo elemento y determinar su abundancia en la na-turaleza. Al tener todos los isótopos la misma cargapero diferente masa, al penetrar perpendicularmenteen un campo magnético describen órbitas circulares dedistinto radio. Si somos capaces de medir los radiosde las órbitas, podremos determinar la relación masa-carga de los distintos isótopos.

Rm vq B

kg

C= =

⋅ ⋅ ⋅

+ ⋅ ⋅

,

,

1 67 10 3 10

1 6 10 0

27 7

19

m/s

,,,

21 6

Tm=

� � �F q v x B F q v B sen

F C

= =

= + ⋅ ⋅−

( );

,

α

1 6 10 19 33 10 0 2 90

9 6 10

7 1

13

⋅ ⋅ ⋅ ⋅

= ⋅

m s T sen

N

,

,

°

F

○� �⌠

⌡⎮ ⋅ = = =B d a

aC

l μ π μμ

π0 002Ι; Β Ι; Β

Ι

2

11. Datos: a = 30 cm = 0,3 m; B = 4,2 ⋅ 10−6 T

Calculamos la intensidad de corriente eléctrica que ge-nera este campo utilizando la expresión hallada en elejemplo 2 (pág. 201) del libro del alumno para el cam-po generado por una corriente rectilínea indefinida:

12. Teorema de Ampère:

La circulación del campo magnético sobre cualquiercurva cerrada C es igual al producto de la permeabili-dad, μ0, por la intensidad de corriente eléctrica, Ic, queatraviesa la superficie limitada por la curva cerrada C.

La principal utilidad del teorema de Ampère es el cálcu-lo del campo magnético o inducción magnética gene-rado por una corriente en sistemas con una geometríaapropiada, como en el caso del solenoide.

13. Datos: N = 350 espiras; l = 24 cm = 0,24 m; I = 2 A

Para determinar la inducción magnética en el interiordel solenoide utilizamos la expresión deducida en elejemplo 3 (pág. 203) del libro del alumno:

14. Se define como circulación del campo magnético la in-tegral, a lo largo de cierta trayectoria, del producto es-calar del vector inducción magnética,

�B, por el elemen-

to de trayectoria .

15. Las líneas de inducción magnética generadas por unacorriente rectilínea indefinida deben ser, por simetría,circunferencias centradas en el hilo conductor.

Para aplicar el teorema de Ampère escogemos comotrayectoria una circunferencia de radio a centrada enel hilo.

El campo magnético será paralelo al elemento de tra-yectoria a lo largo de toda la trayectoria. Además,como la circunferencia se mantiene a una distanciaconstante del conductor, la intensidad del campo mag-nético será constante a lo largo de toda la trayectoria.Entonces, la circulación del campo magnético será:

○�

○�⌠

⌡⎮

⌡⎮⋅ = =B dl lB d B a

C C

d�l

○� �⌠

⌡⎮ ⋅B d

C

l

d�l

BN

T m Am

A

B

= = ⋅ ⋅ ⋅

= ⋅

− −

μ π07 1

3

4 10350

0 242

3 7 10

,

, TT

○� �⌠

⌡⎮ ⋅ =B d C

C

l μ0 Ι

BIa

a B m T= = =

⋅ ⋅ ⋅

μ

π

π

μ

π

π0

0

6

722 2 0 3 4 2 10

4 10;

, ,Ι

TT m A

I A

⋅ ⋅

=

−1

6 3,

8. Campo magnético

I

Ca

d�l �

B�B

I

+ +

I

�v�F

�B

�B

�F

�v

Page 107: 9523 gl fis_tx2_cas

112

mayor. La diferencia de potencial se va alternandopara que tome el valor adecuado en el momento exac-to en que la partícula pasa de una de a la otra. Cuandoel radio de la trayectoria es igual al radio de las des, lapartícula adquiere su velocidad máxima y sale del ci-clotrón.

24. La base del funcionamiento del sincrotrón es pare-cida a la del ciclotrón. La diferencia esencial está enque en el sincrotrón el radio de la trayectoria semantiene constante. Esto se consigue introducien-do las partículas a gran velocidad y acelerándolasmediante un campo magnético y una frecuencia deoscilaciones variables. De este modo pueden cons-truirse aceleradores en forma anular: sólo es nece-sario generar el campo magnético a lo largo de unanillo.

25. Una corriente eléctrica en un campo magnético uni-forme no experimentará fuerza magnética alguna si esparalela al campo magnético. La fuerza magnética so-bre un elemento de corriente es proporcional al pro-ducto vectorial del elemento de corriente por el cam-po. Si son paralelos, el producto vectorial es nulo y elelemento de corriente no experimenta fuerza magné-tica.

26. Datos: l = 4 m; I = 2,5 A; B = 2 ⋅ 10−2 T; α = 90°

Calculamos la fuerza que experimentará el conductorrectilíneo en presencia del campo magnético:

27. Para determinar si las corrientes eléctricas que circu-lan por dos hilos rectilíneos y paralelos tienen el mis-mo sentido o sentidos contrarios, debemos acercar losdos hilos conductores. Si se atraen, las corrientes ten-drán el mismo sentido; mientras que, si se repelen, lascorrientes tendrán sentidos contrarios.

28. Datos: I1 = 2 A; I2 = 3 A; d = 12 cm = 0,12 m

La fuerza es repulsiva, ya que las dos corrientes tienensentidos contrarios. El módulo de esta fuerza por uni-dad de longitud es:

, repulsiva

3. COMPORTAMIENTO DE LA MATERIA EN CAMPOSMAGNÉTICOS

29. Clasificación de los materiales según sus propiedadesmagnéticas:

— Sustancias paramagnéticas: tienen una permeabi-lidad relativa ligeramente superior a la unidad, μr �1. Entonces, Bint � Bext. Son débilmente atraídaspor un imán.

F Nml

= ⋅ −1 10 5

Fd

T m A A Al

= =⋅ ⋅ ⋅ ⋅ ⋅

− −μ

π

π

π0

7 1

24 10 2 3

2 0 12Ι Ι1 2

, m

F B sen A m T sen= = ⋅ ⋅ ⋅ ⋅

=

−Ι l ,

,

α 2 5 4 2 10 90

0 2

2 °

F NN

20. Para determinar la masa de los diferentes isótopos deun elemento químico que inciden sobre la pantallade un espectrómetro de masas, es necesario medir elradio, R, de la órbita circular que describen a partirde los puntos de la pantalla sobre los que inciden. En-tonces, conociendo el elemento químico de que setrata, relacionamos la masa del isótopo, m, con la car-ga eléctrica del elemento, q, el campo magnético delespectrómetro, B, y la diferencia de potencial aplica-da, ΔV:

21.

22. El selector de velocidades se introduce entre la fuentede iones y el campo magnético. Su función es conse-guir que todos los iones que penetran en el campomagnético lo hagan con la misma velocidad, y determi-nar esta velocidad con exactitud.

El selector de velocidades consiste en una región enla que existen un campo magnético y un campo eléc-trico perpendiculares entre sí y perpendiculares a suvez a la dirección del movimiento de los iones. Unaranura detiene a los iones que son desviados de sutrayectoria, de esta manera sólo atraviesan la ranuralos iones con una velocidad determinada, aquéllospara los que las fuerzas eléctrica y magnética se com-pensan:

23. El ciclotrón es un dispositivo que permite acelerar par-tículas (protones y deutrones) hasta conseguir veloci-dades muy altas. Estas partículas se utilizan en la pro-ducción de materiales radiactivos con aplicacionesmédicas.

El ciclotrón consiste en dos recipientes metálicos semi-circulares o des, D1 y D2, colocados perpendicularmen-te a un campo magnético. En el centro de las des existeuna fuente de iones. Debido a la presencia del campomagnético, los iones se mueven en circunferencias pordentro de las des. En el momento en que el ion pasa deuna de a la otra, se aplica entre ambas una diferenciade potencial adecuada para acelerar la partícula. Ésta,al ganar velocidad, describe órbitas de radio cada vez

� �F F

e v B e E vEB

magn eléctr. .

;

+ =

= =

0

mq

R BV

=2 2

Campo magnético

Pantalla receptora

R

q

+

–+ Cámara de ionizaciónΔV

�B

Page 108: 9523 gl fis_tx2_cas

113

— Cuerpo o desarrollo. Describe mediante una ex-posición lógica y coherente el contenido del es-tudio, organizado en capítulos, apartados y suba-partados. Conviene incluir ejemplos, dibujosfotografías, gráficos... que ilustren y completen laexposición del tema.

— Conclusión. Resume las principales ideas que sehan ido exponiendo e incluye las impresiones per-sonales y los juicios críticos oportunos.

— Bibliografía. Es la relación de los libros que se hanconsultado, ordenados alfabéticamente. Se debenindicar los apellidos y el nombre del autor, el títu-lo del libro, la editorial, y el lugar y la fecha de edi-ción.

FÍSICA Y SOCIEDAD

a) En cualquier lugar sobre la superficie de la Tierra laslíneas de inducción del campo magnético terrestrevan de Sur a Norte geográfico. El polo norte magnéti-co coincide aproximadamente con el polo Sur geográ-fico, mientras que el polo sur magnético se sitúa cercadel polo Norte geográfico.

El eje magnético se mueve actualmente hacia el Oestea razón de un grado de longitud cada 5 años. Se sabe,gracias a los estudios paleomagnéticos, que a lo largode los últimos 2 500 millones de años el campo mag-nético terrestre se ha invertido una vez cada millón deaños.

La intensidad del campo magnético terrestre es muypequeña, cerca de los polos se da el valor máximo, deunos 5 ⋅ 10−5 T.

b) Se define como declinación magnética el ángulo queforman las direcciones Norte-Sur geográfica y Norte-Sur magnética.

c) Respuesta sugerida:

Introducción

Desde hace años los científicos aceptan que el campomagnético terrestre está ligado a los movimientos dela materia en el núcleo de la Tierra. El lento fluir deestos materiales produce corrientes eléctricas, las cua-les inducen el campo magnético. Pero en esta teoríahay muchos problemas por resolver. La forma exactade estos movimientos, la fuente de energía que losmantiene y la forma cómo inducen el campo magné-tico son todavía aspectos desconocidos. Para profundi-zar en este estudio, es necesario conocer las caracterís-ticas del campo magnético terrestre en la superficie ylos datos disponibles sobre la estructura interna delglobo.

Cuerpo o desarrollo

El módulo y la dirección del campo magnético te-rrestre son variables a lo largo de la superficie. El va-lor máximo se da cerca de los polos y es de 0,3 gauss.Su dirección es, en promedio, Norte-Sur, pero pre-senta ligeras variaciones. Estas inhomogeneidadeshacen pensar que el campo está formado por remo-linos irregulares. Por otra parte, desde que se estudiael campo magnético terrestre, su intensidad ha ido

— Sustancias diamagnéticas: su permeabilidad relati-va es ligeramente inferior a la unidad, μr � 1. Portanto, Bint � Bext. Son débilmente repelidas por unimán.

— Sustancias ferromagnéticas: se caracterizan por te-ner una permeabilidad relativa mucho mayor que la unidad, μr >> 1. Por tanto, Bint >> Bext. Además,μr no es constante, sino que depende del campoaplicado y del estado previo de imantación del ma-terial. Estas sustancias son fuertemente atraídaspor los imanes.

30. La diferencia fundamental entre un material paramag-nético y otro ferromagnético es que este último estáformado por pequeñas regiones, llamadas dominiosmagnéticos, en las que todos los dipolos atómicos estánorientados en la misma dirección. Al aplicar al materialun campo magnético externo, varios de sus dominiosse orientan en la dirección del campo, con la totalidadde sus dipolos, por tanto el material ferromagnético seimanta. En cambio, un material paramagnético carecede dominios magnéticos, por lo que al aplicar al mate-rial un campo magnético externo sólo se orienta en ladirección del campo una pequeña fracción de los dipo-los atómicos.

31. Un imán pierde sus propiedades magnéticas si se so-mete a temperaturas muy elevadas, porque los dipo-los que, orientados paralelamente, dotan al imán desus propiedades magnéticas, pierden su orientacióncon la temperatura. La temperatura aumenta laenergía de los átomos, que vibran y pierden su orien-tación. Los dipolos quedan entonces orientados ale-atoriamente y el imán pierde sus propiedades mag-néticas.

32. Respuesta sugerida:

Los materiales paramagnéticos, al igual que los ferro-magnéticos, tienen moléculas con momentos dipola-res permanentes. En concreto, en el caso de los ma-teriales paramagnéticos, estos dipolos no interactúanfuertemente entre sí, de forma que normalmente es-tán orientados al azar. En presencia de un campomagnético externo, estos dipolos pueden orientarseparalelamente al campo y contribuir a incrementarsu intensidad. Sin embargo, si la temperatura no esmuy baja ni el campo muy intenso, el movimiento delos átomos debido a la temperatura tenderá a deso-rientarlos, de forma que sólo una pequeña fracciónde los dipolos contribuirá al campo magnético inte-rior.

En el caso de los materiales diamagnéticos, sus molécu-las no poseen momentos magnéticos permanentes. Enpresencia del campo magnético externo, se induce enlas moléculas un momento dipolar de sentido opuestoal campo que lo genera. De esta forma, el campo en elinterior del material se ve debilitado.

Para la elaboración del informe se recomienda seguiresta estructura:

— Introducción. Plantea el objetivo del trabajo, lajustificación del método seguido y el comentariode las causas que han motivado la selección deltema.

8. Campo magnético

Page 109: 9523 gl fis_tx2_cas

114

Conclusión

En resumen, parece claro que el origen del campomagnético terrestre está relacionado con el movimien-to del material fluido en el núcleo de la Tierra. La for-ma de dichas corrientes puede simularse mediantemodelos experimentales o teóricos, pero quedan mu-chas preguntas por contestar. El estudio del interiordel planeta es difícil y sólo puede obtenerse informa-ción mediante métodos indirectos. Los modelos pue-den ajustar más o menos el campo observado, pero escomplicado saber si reproducen adecuadamente elnúcleo terrestre.

Bibliografía

Carrigan, Ch. R.; Gubbins, D., «Origen del campomagnético terrestre», Investigación y ciencia, abril 1979,pág. 82.

RESOLUCIÓN DE EJERCICIOS Y PROBLEMAS

33. Datos: R = 32 cm = 0,32 m; I = 4 A

Calculamos el campo magnético en el centro del con-ductor en forma de semicircunferencia haciendo usode la expresión hallada en el problema resuelto 1(pág. 276) del libro del alumno:

34. Datos: R = 40 cm = 0,4 m; B = 2 ⋅ 10 −6 T

Calculamos la intensidad de corriente que circula porel conductor en forma de semicircunferencia hacien-do uso de la expresión hallada en el problema resuel-to 1 (pág. 214) del libro del alumno para el campomagnético en el centro de un conductor en forma desemicircunferencia:

35. Datos:

En el punto P, los campos magnéticos creados por lasdos corrientes tienen la misma dirección, perpendicu-lar al plano que contiene los dos hilos, y el mismo sen-tido, ya que son corrientes de sentidos opuestos, comose observa en la figura.

BR

R B m T

T m A= = =

⋅ ⋅ ⋅

⋅ ⋅ ⋅

μ

μ π0

0

6

7

4 4 0 4 2 10

4 10

Ι

4Ι;

,−− =1 2 5, A

BR

T m A Am

T= =⋅ ⋅ ⋅ ⋅

⋅= ⋅

− −−μ π0

7 164 10 4

4 0 323 9 10

Ι

4 ,,

decreciendo y sus remolinos desplazándose hacia elOeste a razón de unos 90 metros por día. Además, losestudios paleomagnéticos indican que el campo haido invirtiendo su sentido aproximadamente cada mi-llón de años.

En cuanto a la estructura del planeta, sabemos queexiste un núcleo interno sólido, de unos 1 220 km deradio, y un núcleo externo fluido (3 485 km de ra-dio). El movimiento de esta parte fluida del interiorde la Tierra puede ser el origen de su campo magné-tico.

El principal problema con el que se encuentran losgeofísicos es la imposibilidad de obtener datos sobre elcampo magnético en el núcleo y sobre sus variaciones.Por ejemplo, se cree que deben existir importantescampos toroidales cuyas líneas son paralelas a las su-perficies esféricas centradas en el núcleo y, por ello, in-detectables.

Por ahora, el modelo más aceptado sobre el origen delcampo magnético es el de la dinamo automantenida.Según este modelo, algún campo magnético inicial (elcampo que llena la galaxia sería suficiente) generó co-rrientes eléctricas en el material en movimiento. Estascorrientes, a su vez, empezaron a inducir un campomagnético. Una vez iniciado el proceso, el mismo cam-po magnético mantiene las corrientes, y viceversa. Pa-rece claro que el movimiento del material está regidopor los efectos conjuntos de la gravedad, la rotación ylos movimientos radiales debidos a las diferencias dedensidad como consecuencia de diferencias de tempe-ratura y composición.

Para estudiar la posible forma de las corrientes, seconstruyen modelos teóricos y experimentales. En es-tos modelos se observa que el movimiento del mate-rial está confinado en un cilindro paralelo al eje derotación. En concreto, el flujo dentro de esta regiónforma largos rodillos paralelos, cuyo diámetro crececon la viscosidad del material. Además, dentro de es-tos rodillos el material se mueve hacia abajo en el he-misferio Norte y hacia arriba en el hemisferio Sur,siendo el movimiento horizontal en el plano ecuato-rial. Los modelos experimentales no pueden repro-ducir los campos magnéticos, pero la teoría indicaque el campo generado sería creciente con el tiem-po. A partir de cierto valor crítico, las fuerzas ejerci-das por el campo magnético sobre el material son ta-les que el flujo sigue un movimiento horizontal únicoa gran escala y genera un campo magnético intenso ytoroidal.

Algunos puntos que los modelos no explican son lasinversiones temporales del sentido del campo y el ori-gen de la energía que mantiene el movimiento delmaterial, contrarrestando los efectos de la viscosidady del mismo campo magnético. En los modelos expe-rimentales la fuente de energía es un gradiente detemperatura entre el centro de la esfera y el exterior.En la Tierra, dicho gradiente de temperatura podríamantenerse gracias a la presencia de elementos ra-diactivos en el núcleo, al calor liberado en el procesode solidificación del núcleo y a la energía gravitatorialiberada en los movimientos radiales del material flui-do.

I1 = 2 A I2 = 3 A

P

d1 = 0,07 m d2 = 0,13 m

d = 0,2 m

�B1

�B2

Page 110: 9523 gl fis_tx2_cas

115

Para calcular la velocidad de la partícula alfa halla-mos primero su masa y su carga:

Entonces, la velocidad de la partícula alfa al pene-trar en el campo magnético será:

b) Determinamos el radio de la circunferencia quedescribe la partícula α a partir de la velocidad y elcampo magnético:

38. Datos: R = 0,9 m; B = 0,4 T

a) Calculamos la masa y la carga del ion 2H+:

Hallamos la velocidad del ion a partir del radio dela órbita circular que describe y del campo magné-tico:

Determinamos la energía cinética del ion:

b) El incremento de energía cinética que ha experi-mentado el ion es igual a su pérdida de energíapotencial eléctrica. Por tanto:

39. Datos: R = 0,6 m; mp = 1,67 ⋅ 10−27 kg;

qp = +e = 1,6 ⋅ 10−19 C; B = 1,5 T

a) La condición para que se produzca resonancia esque la frecuencia del oscilador eléctrico coinci-da con la frecuencia de la partícula aceleradaen el ciclotrón:

feB

mC T

kgp

= =⋅ ⋅

⋅ ⋅=

−21 6 10 1 5

2 1 67 10

19

27π π

, ,

,22 3 107, ⋅ Hz

Ec Ep q V VEcq

J= = = =

−Δ Δ;,

,

4 9 10

1 6 10

13

19

,

C

V VΔ = ⋅3 1 106

Ec m v kg

Ec

= = ⋅ ⋅ ⋅ ⋅

=

−12

12

3 34 10 1 72 10

4

2 27 7, ( , m/s)2

,,9 10 13⋅ − J

Rm vq B

vq B R

mC T m

= = =⋅ ⋅ ⋅

;, , ,

,

1 6 10 0 4 0 9

3 34 10

19

−−

= ⋅

27

71 72 10

kg

v , m/s

q q q e C

m m m mP n

p n p

= + = + = ⋅

= + = = ⋅

−1 1 1 6 10

1 1 2 2 1 67

19,

, ⋅⋅

= ⋅

10

3 34 10

27

27

kg

m kg,

Rm vq B

kg

C= =

⋅ ⋅ ⋅

⋅ ⋅

6 68 10 4 4 10

3 2 10 0

27 5

19

, ,

, ,

m/s

44

0 023 2 3, , cm

T

R m= =

vq Vm

C V

kg

v

= =⋅ ⋅ ⋅ ⋅

=

2 2 3 2 10 2 10

6 68 10

4

19 3

27

Δ ,

,;

,,4 105⋅ m/s

q q q C C

m m mP n

p n

= + = ⋅ ⋅ = ⋅

= + =

− −2 2 2 1 6 10 3 2 10

2 2

19 19, ,

44 4 1 67 10

6 68 10

27

27

m kg

m kg

p = ⋅ ⋅

= ⋅

,

,

Calculamos la inducción magnética creada en el punto Ppor cada uno de los hilos:

La inducción magnética resultante es la suma vectorialde

�B1 y

�B2. Como estos vectores tienen la misma direc-

ción y sentido, el módulo del campo magnético resul-tante será la suma de los módulos de

�B1 y

�B2:

B = B1 + B2 = 5,7 ⋅ 10−6 T + 4,6 ⋅ 10−6 T

B = 1,0 ⋅ 10−5 T

36. Datos:

En el punto P, los campos magnéticos creados por lasdos corrientes tienen la misma dirección, perpendicu-lar al plano que contiene los dos hilos, y sentidosopuestos, ya que las corrientes tienen el mismo senti-do, como se observa en la figura.

Calculamos la inducción magnética creada en el punto Ppor cada uno de los hilos:

La inducción magnética resultante es la suma vectorialde

�B1 y

�B2. Como estos vectores tienen la misma direc-

ción y sentidos opuestos, el módulo del campo magné-tico resultante será la diferencia de los módulos de

�B1

y�B2:

B = B1 − B2 = 5 ⋅ 10−5 T − 1 ⋅ 10−5 T = 4 ⋅ 10−5 T

37. Datos: mp = mn = 1,67 ⋅ 10−27 kg;

qp = +e = +1,6 ⋅ 10−19 C;

qn = 0; ΔV = 2 ⋅ 103 V; B = 0,4 T

a) El incremento de energía cinética de la partículaalfa es igual a su pérdida de energía potencial eléc-trica:

12

22m v q V vq Vm

= ⇒ =ΔΔ

Bd

T m A Am1

0

1

7 1

24 10 5

2 0 025 1= =

⋅ ⋅ ⋅ ⋅

⋅= ⋅

− −μ

π

π

π

Ι1

,00

24 10 1

2 0 02

5

20

2

7 1

− −

= =⋅ ⋅ ⋅ ⋅

T

Bd

T m A Aμ

π

π

π

Ι2

, mmT= ⋅ −1 10 5

Bd

T m A Am1

0

1

7 1

24 10 2

2 0 075 7= =

⋅ ⋅ ⋅ ⋅

⋅=

− −μ

π

π

π

Ι1

,, ⋅⋅

= =⋅ ⋅ ⋅ ⋅

− −

10

24 10 3

2 0 1

6

22

7 1

T

Bd

T m A Aμ

π

π

π0 2Ι

, 334 6 10 6,

mT= ⋅ −

8. Campo magnético

I1 = 5 A

P

I2 = 1 A

d1 = 0,02 m d2 = 0,02 m

d = 0,04 m

�B1

�B2

Page 111: 9523 gl fis_tx2_cas

116

43. Si conocemos la forma de las líneas de inducción mag-nética y la dirección de movimiento de una carga eléc-trica, podemos determinar la dirección de la fuerzaque actuará sobre ella. La ley de Lorentz establece queesta fuerza será proporcional al producto vectorial dela velocidad por el campo magnético. Por tanto, comolas líneas de inducción magnética indican en cadapunto la dirección del campo magnético, sabemosque la fuerza que experimentará la carga será perpen-dicular a su velocidad y a las líneas de inducción mag-nética.

44. Campo magnético creado por un hilo rectilíneo inde-finido por el que circula una corriente de intensidad I

a una distancia a del hilo:

Campo magnético creado por una espira circular deradio R por la que circula una corriente de intensidad

I en su centro:

Campo magnético creado por un solenoide con N es-piras, de longitud l, por el que circula una corriente

de intensidad I en su interior:

45. a) Supongamos un electrón, un protón y un átomode helio que penetran en un campo magnéticouniforme con la misma velocidad y en la misma di-rección, perpendicular al campo.

El radio de la órbita circular que describe cadapartícula es proporcional a su masa e inversamen-te proporcional a su carga. El átomo de helio, alser neutro, no experimenta desviación alguna. Elelectrón y el protón tienen la misma carga, perode signos opuestos. Por esta razón se desvían ensentidos opuestos. Además, el protón tiene unamasa mayor. Por eso, el radio de la órbita del pro-tón es mayor que el de la del electrón.

b) La aceleración centrípeta, gracias a la cual estaspartículas describen una trayectoria circular, es:

La aceleración del átomo de helio es nula, pues sucarga neta es cero. Puesto que las cargas eléctricasdel protón y del electrón son iguales en valor ab-soluto y la masa del electrón es unas dos mil vecesmenor que la del protón, el electrón adquiere unaaceleración que será unas dos mil veces mayor.

El aumento de la energía cinética es proporcionalal valor de la fuerza que actúa sobre cada partícu-la. Por ello, sobre el átomo de helio, por ser su car-

aFm

q v Bmc

c= =

BN

I= μ0 l.

BI

R=

μ0

2.

BIa

π0

2.

b) Cuando los protones adquieren su velocidad máxi-ma salen del ciclotrón, siendo el radio de su órbitaigual al radio de las des del ciclotrón. Por tanto:

40. Datos: R = 0,9 m; f = 1,7 ⋅ 106 Hz

a) Calculamos la masa y la carga del ion 2H+:

Hallamos el campo magnético en el interior del ci-clotrón a partir de la frecuencia de resonancia:

b) Cuando los deuterones adquieren su velocidadmáxima salen del ciclotrón, siendo el radio de suórbita igual al radio de las des del ciclotrón. Portanto:

EJERCICIOS Y PROBLEMAS

41. Las propiedades de un imán se deben a la presencia ensu interior de pequeñas regiones llamadas dominios,en las que todos los dipolos atómicos tienen la mismaorientación. Estos dipolos están orientados paralela-mente, definiendo así un eje magnético en el material.De esta manera los extremos del imán se constituyenen dos polos magnéticos diferentes (norte y sur). Elcampo magnético es aquí más intenso y las propieda-des magnéticas del imán, más acusadas.

42. Una carga eléctrica en reposo y otra carga en movi-miento no producen la misma perturbación en el espa-cio. Una carga en reposo sólo genera a su alrededor uncampo eléctrico. En cambio, un carga en movimientogenera un campo eléctrico y un campo magnético.

ve B R

mC T m

máx = =⋅ ⋅ ⋅

, , ,

,

1 6 10 0 2 0 9

3 34 10

19

227

68 6 10,

kg

vmáx m/s= ⋅

fq B

m

Bm fq

kg

=

= =⋅ ⋅ ⋅ ⋅−, ,

2

2 2 3 34 10 1 7 1027 6

π

π π

,,

Hz

CB T

1 6 100 2

19⋅=

q q q e C

m m m m

p n

p n p

= + = + = ⋅

= + = = ⋅

−1 1 1 6 10

1 1 2 2 1 6

19,

, 77 10

3 34 10

27

27

= ⋅

−,

kg

m kg

ve B R

mC T m

pmáx = =

⋅ ⋅ ⋅

−, , ,

,

1 6 10 1 5 0 6

1 67 10

19

−−

= ⋅

27

78 6 10,

kg

vmáx m/s

S N

�B

Protón Helio

Electrón

�B

�Fp

�Fe

�v

Page 112: 9523 gl fis_tx2_cas

117

a)

b) Hallamos el radio de la órbita circular que des-cribe:

Si se tratara de un protón, la partícula describiríala curva en el otro sentido (sentido antihorario) ycon un radio mayor, ya que R es proporcional a lamasa y ésta es mayor para el protón.

c) Si queremos que el electrón describa un MRU, de-bemos superponer un campo eléctrico perpendicu-lar al campo magnético y a la trayectoria del elec-trón, de forma que la fuerza eléctrica sobre elelectrón sea igual pero de sentido opuesto a la fuer-za que ejerce el campo magnético.

53. Datos: mp = mn = 1,67 ⋅ 10−27 kg;

qp = +e = +1,6 ⋅ 10−19 C;

qn = 0; ΔV = 5 ⋅ 103 V; B = 0,25 T

a) El incremento de energía cinética de la partículaalfa es igual a su pérdida de energía potencialeléctrica:

Para calcular la velocidad de la partícula alfa halla-mos primero su masa y su carga:

q q q C C

m m m

p n

p

= + = ⋅ ⋅ = ⋅

= +

− −2 2 2 1 6 10 3 2 10

2 2

19 19, ,

nn pm kg

m kg

= = ⋅ ⋅

= ⋅

4 4 1 67 10

6 68 10

27

27

,

,

12

22m v q V vq Vm

= ⇒ =ΔΔ

F q v B F qE

q v B qE E vBm e= =

= = = ⋅

;

; ,1 6 106 mm s

N/C

⋅ ⋅ ⋅

= ⋅

− −1 3

3

3 10

4 8 10,

T

E

Rm vq B

kg= =

⋅ ⋅ ⋅

, ,

,

9 1 10 1 6 10

1 6 10

31 6

19

m/s

CC

R m

⋅ ⋅

= ⋅

3 10

3 0 10

3

3,

T

ga nula, no actúa ninguna fuerza y su energía ciné-tica se mantiene constante. El protón y el electrónexperimentan fuerzas iguales, pero en sentidosopuestos. Por tanto, su energía cinética aumentacon la distancia recorrida a razones iguales.

46. El ciclotrón es un dispositivo que permite acelerar partículas hasta adquirir velocidades muy altas. En el momento en que la partícula o ion pasa de una dea la otra, se aplica entre ambas una diferencia de po-tencial adecuada para acelerar la partícula. Ésta, al ganar velocidad, describe órbitas de radio cada vez mayor. La diferencia de potencial se va alternando paraque tome el valor adecuado en el momento exacto en que la partícula pasa de una de a la otra. Cuando elradio de la trayectoria es igual al radio de las des, la par-tícula adquiere su velocidad máxima y sale del ciclo-trón.

47. La fuerza que ejerce un campo magnético sobre unacorriente eléctrica no puede ser nunca paralela a la co-rriente. La fuerza que actúa sobre cada punto del con-ductor es proporcional al producto vectorial del ele-mento de corriente por el campo magnético. Portanto, la fuerza es perpendicular a la corriente en cual-quier punto del conductor.

48. Dos hilos conductores rectos y paralelos se atraen si lascorrientes que circulan por ellos tienen el mismo sen-tido. Los mismos hilos se repelerán si las corrientes quecirculan por ellos tienen sentidos opuestos.

49. Las sustancias ferromagnéticas se caracterizan por te-ner una permeabilidad relativa mucho mayor que launidad, μr >> 1. Por tanto, Bint >> Bext. Además, μr noes constante, sino que depende del campo aplicado ydel estado previo de imantación del material. Estas sus-tancias son fuertemente atraídas por los imanes. Sucomportamiento se debe a que un material ferromag-nético está formado por pequeñas regiones, llamadasdominios magnéticos, en las que todos los dipolos ató-micos están orientados en la misma dirección. En pre-sencia de un campo magnético externo, estos domi-nios se orientan paralelamente a Bext y contribuyen alcampo interno, de forma que Bint >> Bext.

50. Datos: R = 10 cm = 0,1 m; I = 4 A

Calculamos la inducción magnética en el centro de laespira:

51. Datos: a = 28 cm = 0,28 m; B = 3 ⋅ 10−6 T

Calculamos la intensidad de corriente eléctrica que ge-nera este campo:

52. Datos: q = −e = −1,6 ⋅ 10−19 C; me = 9,1 ⋅ 10−31 kg;

B = 3 ⋅ 10−3 T; v = 1,6 ⋅ 106 m/s; a = 90°

BIa

IaB m T

=

= =⋅ ⋅ ⋅

μ

π

π

μ

π

π

0

0

6

2

2 2 0 28 3 10

4 10

,−− −⋅ ⋅

=7 1 4 2,T m A

A

BI

RT m A A

m= =

⋅ ⋅ ⋅ ⋅

⋅= ⋅

− −−μ π0

7 1

24 10 4

2 0 12 5 10

,, 55 T

8. Campo magnético

�F

R

e –

�v

�B

�v

�Fe

�Fm

�E

e

Page 113: 9523 gl fis_tx2_cas

118

56. Datos:

a) En el punto P1 los campos magnéticos creados porlas dos corrientes tienen la misma dirección, per-pendicular al plano que contiene los dos hilos, ysentido contrario, ya que son corrientes con elmismo sentido.

Calculamos la inducción magnética creada en elpunto P1 por cada uno de los hilos:

La inducción magnética resultante es la suma vec-torial de

�B1 y

�B2. Como estos vectores tienen la

misma dirección y sentidos opuestos, el módulodel campo magnético resultante será la diferenciade los módulos de

�B1 y

�B2:

B = B2 − B1 = 2 ⋅ 10−6 T − 2 ⋅ 10−6 T = 0 T

b) Calculamos la inducción magnética creada en elpunto P2 por cada uno de los hilos:

Como los vectores�B1 y

�B2 tienen la misma direc-

ción y sentidos opuestos, el módulo del campomagnético resultante será la diferencia de los mó-dulos de

�B2 y

�B1:

B = B2 − B1 = 2,7 ⋅ 10−6 T − 1,3 ⋅ 10−6 T

B = 1,4 ⋅ 10−6 T

c) Calculamos la inducción magnética creada en elpunto P3 por cada uno de los hilos:

BId

T m A Am

B

10 1

1

7 1

24 10 2

2 0 4= =

⋅ ⋅ ⋅ ⋅

− −μ

π

π

π ,

116

20 2

2

7 1

1 0 10

24 10

= ⋅

= =⋅ ⋅ ⋅

− −

, T

BId

T m Aμ

π

π ⋅⋅

= ⋅ −

42 0 2

4 0 1026

,

,

Am

B T

π

BId

T m A Am

B

10 1

1

7 1

24 10 2

2 0 3= =

⋅ ⋅ ⋅ ⋅

− −μ

π

π

π ,

116

20 2

2

7 1

1 3 10

24 10

= ⋅

= =⋅ ⋅ ⋅

− −

, T

BId

T m Aμ

π

π ⋅⋅

= ⋅ −

42 0 3

2 7 1026

,

,

Am

B T

π

B T

BId

T m A1

6

20 2

2

7 1

2 10

24 10 4

= ⋅

= =⋅ ⋅ ⋅ ⋅

− −μ

π

π

,A

m

B T

2 0 4

2 1026

π ⋅

= ⋅ −

BId

T m A Am1

0

1

7 1

24 10 2

2 0 2= =

⋅ ⋅ ⋅ ⋅

− −μ

π

π

π ,1

Entonces, la velocidad de la partícula alfa al pene-trar en el campo magnético será:

b) Determinamos el radio de la circunferencia quedescribe la partícula alfa a partir de la velocidad yel campo magnético:

54. Datos: l = 3,5 m; I = 4 A; B = 2 ⋅ 10−2 T; α = 90°

Calculamos la fuerza que experimentará el conductorrectilíneo en presencia del campo magnético:

55. Datos: I1 = 1,5 A; I2 = 2,4 A; l = 1 m

a)

b) El campo magnético en el punto D será la sumavectorial del campo creado por cada uno de los doshilos. Hallamos el módulo del campo que generaen D cada uno de los hilos:

Las dos contribuciones son perpendiculares. Portanto, el módulo del campo magnético total será:

La fuerza que se ejercen los hilos es repulsiva, yaque las dos corrientes tienen sentidos opuestos. Elmódulo de esta fuerza por unidad de longitud es:

F I Id

T m A A Al

= =⋅ ⋅ ⋅ ⋅ ⋅− −μ

π

π

π0 1 2

7 1

24 10 1 5 2 4

2, ,

⋅⋅

= ⋅ −

1 41

5 1 10 7

,

,

m

F Nml

B B B T T

B

= + = ⋅ + ⋅

= ⋅

− −12

22 7 2 7 23 10 4 8 10

5 7 10

( ) ( , )

, −−7 T

BIa

T m A Am

B

10

7 1

1

24 10 1 5

2 1= =

⋅ ⋅ ⋅ ⋅

− −μ

π

π

π

,1

== ⋅

= =⋅ ⋅ ⋅ ⋅

− −

3 10

24 10 2 4

7

20

7 1 ,

T

BIa

T m Aμ

π

π2

,

Am

B T

2 1

4 8 1027

π ⋅

= ⋅ −

F I B A m T

F

= = ⋅ ⋅ ⋅ ⋅ °

=

−sen , sen

,

l α 4 3 5 2 10 90

0 28

2

N

Rm vq B

kg= =

⋅ ⋅ ⋅

, ,6 68 10 6 9 10

10

27 5

19

m/s

3,2 ,, ,

C TR m cm

⋅= =

0 250 058 5 8

vq Vm

C V

kg= =

⋅ ⋅ ⋅ ⋅

2 2 3 2 10 5 10

6 68 10

19 3

27

,

,

Δ

vv = ⋅6 9 105, m/s

�F21

�F12

I1 = 1,5 A

a = 1 m

I2 = 2,4 A

BA

D C�B1 �

B2

d a a m= + = =2 2 2 1 41,

�B1

P1 P2 P3

�B2

I1 = 2A I2 = 4A

d = 60 cm = 0,6 m

�B2

�B2

�B1

�B1

Page 114: 9523 gl fis_tx2_cas

119

59. Datos: ΔV = 25 000 V; R = 0,4 m;

q = +e = +1,6 ⋅ 10−19 C; mp = 1,67 ⋅ 10−27 kg

a) El incremento de energía cinética del protón esigual a su pérdida de energía potencial eléctrica:

Entonces, la velocidad del protón al penetrar en elcampo magnético será:

Hallamos la inducción magnética a partir del ra-dio de la órbita circular que describe el protón yde su velocidad:

b) Si la inducción magnética tuviera un valor dobleal anterior, la velocidad del protón no variaría; encambio, el radio de la trayectoria sería ahora:

60. Mediante INTERACTIVE PHYSICS la simulación fun-ciona correctamente con los parámetros indicados:

La definición de campo magnético se puede llevar acabo de forma estándar en el menú de campos defuerza:

Rm vq B

m vq B

R m

R m cm

′′

= = = =

= =

,

,

2 20 4

2

0 2 20

Rm vq B

Bm vq R

kg

=

= =⋅ ⋅ ⋅ ⋅− −, ,1 67 10 2 2 1027 6 1m s

11 6 10 0 4

5 7 10

19

2

, ,

,

⋅ ⋅

= ⋅

C m

B T

vq Vm

C V= =

⋅ ⋅ ⋅ ⋅

2 2 1 6 10 2 5 10

1 67 10

19 4

27

, ,

,

Δ

kkg

v = ⋅1 5 106, m/s

12

22m v q V vq Vm

= ⇒ =ΔΔ

La inducción magnética resultante es la suma vec-torial de

�B1 y

�B2. Como estos vectores tienen la mis-

ma dirección y sentidos opuestos, el módulo delcampo magnético resultante será la diferencia delos módulos de

�B2 y

�B1:

B = B2 − B1 = 4 ⋅ 10−6 T − 1 ⋅ 10−6 T = 3 ⋅ 10−6 T

57. Datos:

a) Dada la geometría del toroide, podemos suponerque, a lo largo de la circunferencia que pasa por elcentro de todas las espiras, el campo magnético essiempre perpendicular al plano de la espira. Portanto, el campo será siempre tangente a dicha cir-cunferencia. Además, su valor será constante sobrela circunferencia por razones de simetría.

Entonces, para aplicar el teorema de Ampère, to-mamos la circunferencia de radio R que pasa porel centro de todas las espiras del toroide.

Por otro lado, la intensidad que atraviesa la super-ficie circular delimitada por la curva C es el pro-ducto del número de espiras del toroide por la in-tensidad que circula por ellas. Por tanto, aplicandoel teorema de Ampère:

b) Para un toroide de 300 espiras, por las que circulauna intensidad de 4 A, a 20 cm del centro:

58. Datos: l = 50 cm = 0,5 m; N = 500; I = 2 A

a) Hallamos el campo magnético en el interior del so-lenoide:

b) Determinamos el campo magnético en el interiorde la barra de hierro dulce situada en el centro delsolenoide, sabiendo que la permeabilidad relativadel hierro dulce es μr = 1 500:

B B T Tr extint , ,= = ⋅ ⋅ =−μ 1 500 2 5 10 3 753

BN

I T m Am

A

B

= = ⋅ ⋅ ⋅ ⋅ ⋅

= ⋅

− −μ π07 14 10

5000 5

2

2 5 1

l ,

, 00 3− T

BNIR

T m A A= =

⋅ ⋅ ⋅ ⋅ ⋅

− −μ

π

π

π0

7 1

24 10 300 4

2 0 2, mm

B T= ⋅ −1 2 10 3,

; ;○� �⌠

⌡⎮ ⋅ = =B d I R B N ICl μ π μ0 02 BB

N IR

C

π0

2

○� �⌠

⌡⎮ ⋅B d

C C C

l

8. Campo magnético

I

I

R

C N espiras

�B

�B

�B

�B

d�l

d�l d

�l

d�l

Círculo 1 Velocidad-X

Círculo 1 Carga

Círculo 1 Masa

10.00 2.00e-004 2.00

vF

Page 115: 9523 gl fis_tx2_cas

120

b) Determinamos la velocidad a partir de la energíacinética, sabiendo que 1 eV = 1,6 ⋅ 10−19 J.

Calculamos la fuerza magnética que actúa sobre elelectrón:

Hallamos el radio de la órbita y la frecuencia an-gular:

Determinamos el período a partir de la frecuenciaangular:

6. Datos: l = 5 m; I1 = 3 A; I2 = 6 A; d = 4 cm = 0,04 m

Como los hilos son mucho más largos que la separa-ción que hay entre ellos, podemos suponer que lafuerza que se ejercen es la misma que si fueran con-ductores rectilíneos indefinidos. Calculamos el módu-lo de esta fuerza por unidad de longitud:

Como la longitud de los conductores es de 5 m, lafuerza que éstos se ejercen mutuamente es

F = 5 m ⋅ 9 ⋅ 10−5 N ⋅ m−1 = 4,5 ⋅ 10−4 N

La fuerza es atractiva, ya que las dos corrientes tienenel mismo sentido.

7. Para construir un imán permanente se puede utilizar elacero. Es un material ferromagnético en el que la pre-sencia de un campo magnético externo induce la orien-tación de los dominios paralelamente al campo magné-tico. Esta ordenación persiste incluso después deeliminar el campo magnético externo. De esta formase obtiene un imán permanente.

Fd

T m A A Al

= =⋅ ⋅ ⋅ ⋅ ⋅

− −μ

π

π

π0 1 2

7 1

24 10 3 6

2 0Ι Ι

,,04

9 10 5

m

F Nml

= ⋅ −

Tf s

s= = =⋅

= ⋅−−1 2 2

8 8 107 1 109 1

10π

ω

π

,,

Rm vq B

kg= =

⋅ ⋅ ⋅

, ,9 1 10 2 9 10

10

31 7

19

m/s

1,6 CC T

R m mm

fq Bm

= ⋅ =

= = =

0 05

3 3 10 3 3

2

3

,

, ,

ω π1,6 ⋅⋅ ⋅

= ⋅

10 0 05

9 1 10

8 8 10

19

31

9 1

,

,

,

C T

kg

� � �

F

F

F

=

= = °

= ⋅ −

q

F q v B

(

sen

,

v x B)

90

1 6 10 199 7 1

13

2 9 10 0 05

2 3 10

, ,

,

C T

N

⋅ ⋅ ⋅ ⋅

= ⋅

m s

F�

E m v vEm

veV

cc= =

=⋅ ⋅ ⋅ ⋅

12

2

2 2 4 10 1 6 10

2

3

;

, , −−

−⋅

= ⋅

19

31

7

9 1 10

2 9 10

,

,

J eV

kg

v m s/

La carga describe trayectorias circulares. El comporta-miento de la carga al modificar los controles se puederesumir de la siguiente manera:

— Cuando aumenta la velocidad inicial, aumenta elradio de la trayectoria (cuando disminuye la carga,el radio aumenta).

— Cuando aumenta la carga, disminuye el radio de latrayectoria (cuando disminuye, éste lo hace tam-bién).

— Cuando aumenta la masa, aumenta el radio de latrayectoria (cuando disminuye, éste lo hace tam-bién).

EVALUACIÓN

1. Es imposible separar los polos de un imán. Al romperun imán en dos o más trozos obtenemos varios imanes,cada uno con sus dos polos. El magnetismo de los ma-teriales es debido a la orientación de los dipolos mag-néticos de su interior. El movimiento de los electronesalrededor del núcleo atómico genera un campo mag-nético. Aunque rompamos un imán, en cada uno delos nuevos trozos sigue habiendo electrones en movi-miento que generan el campo magnético correspon-diente y con la misma orientación.

2. Las líneas de inducción magnética son…

La respuesta correcta es la b.

Las líneas de inducción magnética son siempre cerradas.

3. Datos: a = 10 cm = 0,1 m; I = 4 A

Calculamos la inducción magnética creada por el con-ductor indefinido a una distancia de 10 cm:

4. La fuerza magnética que actúa sobre una carga en mo-vimiento depende del valor de la carga eléctrica, delcampo magnético, de la velocidad de la carga y del án-gulo que forman la velocidad de la carga y el vector in-ducción magnética.

Para que no actúe ninguna fuerza sobre una carga quese mueve por un campo magnético, la velocidad deésta debe ser paralela al campo, ya que la fuerza es pro-porcional al producto vectorial de la velocidad por elcampo.

5. Datos: q = −e = −1,6 ⋅ 10−19 C; m = 9,1 ⋅ 10−31 kg;

B = 0,05 T; Ec = 2,4 ⋅ 103 eV

a)

Ba

T m A Am

B

= =⋅ ⋅ ⋅ ⋅

= ⋅

− −μ

π

π

π0

7 1

24 10 4

2 0 1

8 10

Ι

,−−6 T

�v

�F

�B

Page 116: 9523 gl fis_tx2_cas

121

den su orientación al eliminar el campo externo, deforma que sólo actúa como un imán en presencia de uncampo externo.

Para la construcción de un imán temporal se puede uti-lizar el hierro dulce. También es un material ferromag-nético, pero en este caso los dominios orientados pier-

8. Campo magnético

Page 117: 9523 gl fis_tx2_cas

123

Inducción electromagnética9

• La ley de Ohm se enuncia así:

El cociente entre la diferencia de potencial aplicada alos extremos de un conductor y la intensidad de corrien-te que circula por él es una constante llamada resisten-cia eléctrica del conductor.

— Datos: R = 1,5 kΩ = 1,5 ⋅ 103 Ω; V = 15 V

1. INDUCCIÓN DE LA CORRIENTE ELÉCTRICA

1. La inducción electromagnética consiste en la apari-ción de una corriente eléctrica en un circuito cuandovaría el número de líneas de inducción magnética quelo atraviesan.

2. Para inducir una corriente eléctrica en un circuito esnecesario variar el número de líneas de inducciónmagnética que lo atraviesan. Existen diferentes formasde conseguirlo. Por ejemplo, podemos mover un imánen las proximidades del circuito, o variar la intensidadde corriente en otro circuito próximo a aquél dondedeseamos inducir la corriente.

3. No, no es necesario. Si mantenemos la espira fija, perointroducimos y extraemos sucesivamente un imán enla espira, también se induce una corriente eléctricadurante el movimiento del imán.

4. El físico inglés M. Faraday (1791-1867) observó expe-rimentalmente que la intensidad de la corriente indu-cida aumentaba con la velocidad del movimiento delimán. Faraday interpretó que la corriente se inducecuando varía el número de líneas de inducción mag-nética que atraviesan el circuito. Al aumentar la velo-cidad con que se aproxima el imán al circuito, el nú-mero de líneas de inducción que atraviesan el circuitocrece con mayor rapidez y la intensidad de la corrien-te inducida es mayor.

ΙΩ

= =⋅

= =VR

VA mA

15

1 5 100 01 103,,

�B

�F

I

PREPARACIÓN DE LA UNIDAD

• Dibujamos las líneas de inducción del campo magnéticocreado por una corriente rectilínea e indefinida.

Dibujamos las líneas de inducción del campo magnéticocreado por una corriente eléctrica circular.

• Una bobina o solenoide es un conductor enrollado en es-piral alrededor de un material aislante.

Las líneas de inducción magnética de un solenoide sonidénticas a las de un imán recto.

• Datos: N = 1 000; l = 120 cm = 1,2 m; I = 1,5 A

Calculamos la inducción magnética en el interior del so-lenoide:

BN T m A A

m

B

= =⋅ ⋅ ⋅ ⋅ ⋅

=

− −μ π07 14 10 1 000 1 5

1 2,

l

11 6 10 3, ⋅ − T

+ –

�B

�B

�B

�B

I

9. Inducción electromagnética

Page 118: 9523 gl fis_tx2_cas

124

9. Al cerrar el interruptor, el sentido de la corriente in-ducida en S2 es contrario al que circula en S1.

Al abrir el interruptor, el sentido de la corriente indu-cida en S2 es contrario que en el caso anterior.

10. a) Sí que circula corriente por la espira, pues el nú-mero de líneas de inducción que la atraviesan estáaumentando.

b) No circula corriente por la espira. Aunque la espi-ra se mueve, el número de líneas de inducciónque la atraviesan se mantiene constante.

c) Sí que circula corriente por la espira, pues el nú-mero de líneas de inducción que la atraviesan estádisminuyendo.

11. La fuerza electromotriz (fem) de un generador es eltrabajo que realiza el generador por unidad de carga,o lo que es lo mismo, la energía que proporciona a launidad de carga.

Un campo magnético variable induce una corrienteeléctrica en un circuito por el fenómeno de la induc-ción electromagnética. Tendremos, por consiguiente,en este caso, una fuerza electromotriz denominadafuerza electromotriz inducida, que es la que causa laaparición de esta corriente inducida.

�B

a b c

Iinducida Iinducida

G

I

Iinducida

G

I

Iinducida

+

5. Algunas formas de producir una variación en el campomagnético pueden ser:

— Moviendo un imán en las cercanías de la espira.

— Moviendo la espira en las cercanías de un imán.

— Situando, en las cercanías de la espira, un circuitoeléctrico de intensidad variable.

6. Si la inducción magnética �B forma un ángulo α con el

vector superficie �S de la espira, el flujo magnético vie-

ne dado por la expresión:

que, como vemos, depende directamente de cos α. Te-nemos que, si cos α es máximo, mínimo o nulo, tam-bién lo será φ.

Por lo tanto:

a) Si el vector superficie �S tiene la misma dirección y

el mismo sentido que el campo �B, cos α = 1 y, por

lo tanto, el flujo es máximo.

b) Si el vector superficie �S tiene la misma dirección y

sentido contrario al campo �B, cos α = −1 y, por lo

tanto, el flujo es mínimo.

c) Si el vector superficie �S es perpendicular al campo�

B, cos α = 0 y, por lo tanto, el flujo es nulo.

7. El flujo magnético a través de una superficie cerrada Ses negativo cuando existe una cantidad neta de líneasde inducción que entran en la superficie. De la mismamanera, si hubiera una cantidad neta de líneas de in-ducción que salieran de la superficie, el flujo sería po-sitivo.

8. Datos: N = 120; S = 30 cm2 = 3 ⋅ 10−3 m2; B = 4 ⋅ 10−3 T

a) α = 0°; φ = N B S cos α

φ = 120 ⋅ 4 ⋅ 10−3 T ⋅ 3 ⋅ 10−3 m2 cos 0°

φ = 1,4 ⋅ 10−3 Wb

b) α = 60°; φ = N B S cos α

φ = 120 ⋅ 4 ⋅ 10−3 T ⋅ 3 ⋅ 10−3 m2 cos 60°

φ = 7,2 ⋅ 10−4 Wb

�B

�S

�S

�S

b Flujo mínimoa Flujo máximo

c Flujo nulo

�B

�B

φ α= B S cos

Page 119: 9523 gl fis_tx2_cas

125

cida sobre las cargas del conductor tenga la direccióndel propio conductor. De esta manera, las fuerzas eléc-trica (que se opone a la separación de las cargas) ymagnética que actúan sobre las cargas del conductorquedan compensadas y se obtiene:

Fm = Fe ⇒ e v B = e E

E = v B

ε = E l = v B l

Si el conductor se desplazara en una dirección dife-rente, deberíamos tener en cuenta el ángulo α queformarán los vectores �v y

�B en el cálculo de la fuerza

magnética:Fm = e v B sen α

16. Cuando desplazamos el conductor paralelamente a laslíneas de inducción magnética, no actúa ningunafuerza magnética sobre las cargas del conductor, porser �v y

�B paralelos:

En consecuencia, no se produce una separación entrelas cargas positivas y negativas del conductor, y no exis-te ninguna fem inducida.

17. Datos: l = 25 cm = 0,25 m; v = 6 m ⋅ s−1; α = 90°; B = 0,3 T

a) La fuerza magnética que actúa sobre un electrónde la barra viene dada por la ley de Lorentz:

b) El campo eléctrico en el interior del conductor es:

c) Calculamos la diferencia de potencial o fem indu-cida entre los extremos de la barra:

2. APLICACIONES DE LA INDUCCIÓN ELECTROMAGNÉTICA

18. La diferencia fundamental entre una dinamo y un al-ternador es que la dinamo es un generador que pro-duce corriente eléctrica continua y el alternador es ungenerador que produce corriente alterna.

19. La fem inducida en un alternador se puede expresar:

a) Si duplicamos la velocidad de giro de la bobina, lanueva fem inducida se puede expresar:

Es decir, se duplican la amplitud y la frecuenciaangular de la fem inducida.

ε′ ′ ′

ε′ ε0

= =

=

B S t B S tsen sen

sen

ω ω ω ω2 2

2 22ω t

ε ε= =B S t tsen senω ω ω0

ε = = ⋅ ⋅ ⋅ =−v B T m V, , ,l 6 0 3 0 25 0 451m s

E v Bms

TVm

= = ⋅ =, ,6 0 3 1 8

F e v B C T

Fm = ° = ⋅ ⋅ ⋅ ⋅− −sen , ,90 1 6 10 6 0 319 1m s

mm N= ⋅ −2 9 10 19,

� � �Fm = e x(v B) = 0

12. La intensidad de la corriente inducida viene dada porla expresión:

a) Si doblamos la velocidad de giro de la bobina, la ve-

locidad con la que varía el flujo, , se multiplica

por dos; en consecuencia, la intensidad de la co-rriente inducida aumenta al doble.

b) Si la inducción magnética B se reduce a la mitad, la

velocidad con que varía el flujo, , se divide por

dos; en consecuencia, la intensidad de la corrienteinducida se reduce a la mitad.

c) Si efectuamos los dos cambios anteriores simultá-neamente, la intensidad de la corriente inducidapermanece invariante.

13. Datos: B = 0,4 T; r = 5 cm = 5 ⋅ 10−2 m; R = 15 Ω;Δt = 0,1 s

Inicialmente, el plano de la espira es perpendicular alcampo magnético, por tanto, los vectores

�B y

�S forman

un ángulo α = 0°. En la situación final, la espira hadado un cuarto de vuelta, por tanto,

�B y

�S forman un

ángulo α = 90°.

La variación del flujo viene dada por:

La fem en la espira es:

Calculemos ahora la intensidad de la corriente indu-cida:

14. Datos: N = 200; S = 30 cm2 = 3 ⋅ 10-3 m2; α = 0°;

B = (2t + 0,8)⋅10−3 T

El flujo magnético a través de la bobina varía en eltiempo según la expresión:

La fem inducida en la bobina es:

15. En la experiencia de Henry se utiliza un conductor rec-tilíneo que se mueve perpendicularmente al campomagnético con objeto de que la fuerza magnética ejer-

ε = − = − ⋅ −ddt

1 2 10 3,

φ α

φ

( ) cos cos

( ) ( ,

t N B S N B S

t t

= = °

= +

0

200 2 0 8))

( ) ( , )

⋅ ⋅ ⋅

= + ⋅

− −

10 3 10

12 4 8 10

3 3 2

4

T m

t t Wbφ

ΙΩ

= =⋅

= ⋅ =−

−ε 3,14R

VA mA

1015

2 1 10 2 12

3, ,

ε = − = −− ⋅

= ⋅−

−Δ

Δ

φ

tWb

sV

3 14 100 1

3 14 103

2,,

,

Δ

Δ

φ φ φ α α

φ π

= − = −

= ⋅ ⋅ ⋅ −

0 0

0 4 5 10

B S

T

(cos cos )

, ( 22 2

3

90 0

3 14 10

) (cos cos )

,

m

Wb

° − °

= − ⋅ −Δφ

Δ

Δ

φ

t

Δ

Δ

φ

t

IR t

= −1 Δ

Δ

φ

9. Inducción electromagnética

Page 120: 9523 gl fis_tx2_cas

126

Todos los participantes deben investigar y docu-mentarse sobre el tema con anterioridad.

• Público. Atenderá a las diversas opiniones. Po-drá intervenir al final aportando sus propias opi-niones o preguntando a los participantes algunacuestión.

— Iniciar el coloquio. El moderador presentará a losparticipantes, introducirá el tema y planteará laprimera pregunta a alguno de los participantes.

— Desarrollar y concluir el coloquio. Los distintosparticipantes desarrollarán sus argumentos con-ducidos por el moderador. Cada participantedebe expresar sus opiniones y respetar las de losdemás.

Al final del coloquio, el público podrá exponer susopiniones y efectuar preguntas a los diferentes par-ticipantes. Por último, el moderador puede llevar acabo un breve resumen de las intervenciones.

22. — Mientras el hilo está conectado a la pila, el clavoatrae a los clips cuando se acerca a ellos.

— Si el hilo se desconectara de la pila, el clavo deja-ría de atraer a los clips cuando se acercara a ellos.

a) Sí, el campo magnético generado por el electroi-mán es más intenso que el que crea la bobina porsí sola, ya que el clavo es de hierro dulce y éste seimanta creando, así, su propio campo magnético,que se suma al de la bobina.

b) No, ya que desaparece el campo magnético, ypuesto que el clavo actúa como un imán temporal,desaparecen sus propiedades magnéticas.

23. No, ambos procesos son manifestaciones distintas deun mismo fenómeno físico: la inducción de una femen un circuito debido a la variación del flujo magnéti-co a través de éste.

La diferencia estriba en que en la inducción electro-magnética consideramos el inductor y el inducidocomo sistemas independientes, mientras que en elcaso de la autoinducción estudiamos la creación de uncampo magnético en un circuito debido a la variaciónde la corriente del propio circuito.

24. Un motor es un receptor que transforma algún tipode energía con trabajo mecánico. Llamamos fuerzacontraelectromotriz del motor al trabajo mecánicoque realiza por unidad de carga.

Su unidad es el voltio (V).

25. El coeficiente de autoinducción de una bobina vienedado por la expresión:

Si tenemos una sola espira,

Si tenemos 50 espiras, N L S= =502500

0, μl

N LS

= =1 0, μl

LN

S= μ0

2

l

ε′ =WQ

b) Como la expresión de la frecuencia es:

Es decir, la frecuencia de la corriente inducida tam-bién se duplica.

20. Datos: N = 25; S = 60 cm2 = 6 ⋅ 10−3 m2; f = 50 Hz;

B = 0,4 T; R = 75 Ω

a) La fem inducida en la bobina es la suma de lasfuerzas electromotrices inducidas en cada una delas espiras que la componen:

b) La fem máxima inducida en la bobina es:

c) Calculamos la intensidad máxima:

21. Respuesta sugerida:

Algunas de las ideas que se podrían exponer y desarro-llar en el coloquio son las siguientes:

— La energía eléctrica es la forma de energía másconsumida en la actualidad. Multitud de aparatos,en nuestros hogares y en la industria, funcionancon electricidad.

— El uso de la energía eléctrica está estrechamenterelacionado con la mejora de la calidad de vida deun país.

— La energía eléctrica es muy versátil, pues se puedetransformar en otros tipos de energía (mecánica,térmica, lumínica...) fácilmente y con un alto ren-dimiento.

— La energía eléctrica se puede producir y distribuirde forma económica y eficaz mediante líneas dealta tensión.

— La energía eléctrica no contamina ni produce resi-duos de ninguna clase.

Para la organización del coloquio se recomienda seguirestas pautas:

— Determinar los encargados de las distintas funcio-nes:

• Moderador. Presentará a los participantes e in-troducirá el tema que se va a tratar. Además, con-cederá los turnos de palabra para que el colo-quio se desarrolle de forma ordenada.

• Participantes. Darán sus opiniones sobre el temaelegido y escucharán las de los otros participan-tes. Generalmente, son un máximo de seis perso-nas.

ΙΩ0

0 18 875

0 25= = =ε

RV

A,

,

ε

ε

0

0

= =

= ⋅ ⋅ ⋅ −

N B S N B S f

T m,

ω π2

25 0 4 6 10 3 2 ⋅⋅ ⋅ =2 50 18 8π ,Hz V

ε = =

= ⋅

N B S t N B S f f tsen sen ( )ω ω π π

ε

2 2

25 00 4 6 10 2 50 2 503 2, sen ( )T m Hz Hz t⋅ ⋅ ⋅ ⋅ ⋅ ⋅ ⋅

=

− π π

ε 66 100π πsen ( ) ( )t en unidades SI

f f f= = =ωπ

ω

π2 22; ′

Page 121: 9523 gl fis_tx2_cas

127

N1 = 400; N2 = 50

La tensión de salida disminuye.

La intensidad de salida aumenta.

— Si 50 son las vueltas del circuito primario y 400 lasdel circuito secundario, tendremos:

N1 = 50; N2 = 400

La tensión de salida aumenta.

La intensidad de salida disminuye.

31. Datos: V1 = 3 000 V; I1 = 2 mA = 2 ⋅ 10−3 A;

N1 = 900; N2 = 30

Para resolver este ejercicio utilizaremos la relación detransformación de un transformador:

Calculamos primero la tensión de salida:

La intensidad de salida es:

32. Es debido a las ventajas que ésta presenta en relacióna otras energías:

— Es fácilmente convertible en otras formas de ener-gía.

— Se tiene acceso directo a ella desde nuestros hoga-res.

— Puede ser transportada a largas distancias desdesu lugar de producción.

— No contamina ni produce residuos de ninguna clase.

33. La energía eólica es la energía cinética que procedede la fuerza del viento. Para poder aprovecharla ytransformarla en energía eléctrica se utilizan los aero-generadores, constituidos por unas palas que giran al-rededor de un eje. Éstos transforman la energía ciné-tica del viento en energía cinética de rotación y estánunidos a la parte móvil de un generador (rotor) quetransforma la energía mecánica en eléctrica.

Las principales ventajas de la energía eólica son:

— Es una energía renovable, es decir, de reservas ili-mitadas.

Ι Ι2 11

2

32 1090030

0 06= = ⋅ ⋅ =−NN

A A,

V VNN

V V2 12

1

3 00030900

100= = ⋅ =

VV

NN

2

1

1

2

2

1

= =Ι

Ι

Ι Ι Ι21

21 10 125= =

NN

,

VNN

V V22

11 18= =

Ι Ι Ι21

21 18= =

NN

VNN

V V22

11 10 125= = ,

Es decir, el coeficiente de autoinducción es 2 500 vecesmayor en el caso de una bobina formada por 50 espi-ras que en el de una sola espira.

26. Datos: l = 20 cm = 0,2 m; N = 200;

S = 40 cm2 = 4 ⋅ 10−3 m2; I0 = 4 A; I = 0 A;

Δt = 2 ms = 2 ⋅ 10−3 s

Calculamos primero el coeficiente de autoinducciónde la bobina:

La fem inducida es:

27. Puesto que el coeficiente de inducción mutua se defi-ne como el cociente entre el flujo magnético a travésde un circuito y la intensidad de corriente a través deotro circuito:

En el SI tendrá unidades de:

La unidad del coeficiente de inducción mutua en el SIes el henrio, H. Un henrio es la inductancia mutua en-tre dos circuitos tales que una variación de intensidadde un amperio por segundo en uno de los dos circui-tos induce una fuerza electromotriz de un voltio en elotro circuito.

28. No, puesto que el transformador se basa en el fenóme-no de inducción mutua, y éste sólo se puede producirsi la corriente es variable en el tiempo.

29. Datos: N1 = 100; N2 = 500

La modificación de la tensión es:

La tensión de salida es cinco veces mayor que la ten-sión de entrada, por lo que se trata de un transforma-dor elevador.

Para el caso de la intensidad:

La intensidad de salida es una quinta parte de la inten-sidad de entrada, por lo que, al contrario que en la ten-sión, la intensidad se reduce a la salida.

30. Para resolver este ejercicio utilizaremos la relación detransformación de un transformador:

— Si 400 son las vueltas del circuito primario y 50 lasdel circuito secundario, tendremos:

VV

NN

2

1

1

2

2

1

= =Ι

Ι

Ι Ι Ι21

21 1

15

= =NN

VNN

V V22

11 15= =

WbA

V sA

H=⋅

=

M M12 212

1

= =φ

Ι

ε = − = − ⋅−

⋅=−

−Lt

HA

sV

ΔΙ

Δ10

4

2 102 03

3

( ),

LN

ST m

A mm

L

= = ⋅⋅

⋅ ⋅ ⋅− −μ π0

27

23 24 10

2000 2

4 10l

( ),

== −10 3 H

9. Inducción electromagnética

Page 122: 9523 gl fis_tx2_cas

128

Además, la intensidad también depende de la frecuen-cia:

El campo magnético generado en el interior del con-ductor es variable y tiene la misma frecuencia que lacorriente alterna:

Por tanto, el flujo magnético a través de una seccióndel conductor es:

Según la ley de Faraday, la fem inducida es:

La intensidad de la corriente inducida tendrá una am-plitud proporcional a la fem máxima (ε0 = B0 S ω), esdecir, será proporcional a la frecuencia. Por tanto, lapotencia (P = R I2) y el calor disipados aumentaráncon la frecuencia.

c) Respuesta sugerida:

Entre las varias aplicaciones del fenómeno de la in-ducción electromagnética, además del micrófono y loshornos de inducción que se mencionan en el libro delalumno, pueden proponerse para la redacción del in-forme:

— Los generadores eléctricos (dinamo y alternador).

— El motor eléctrico, que transforma energía eléctri-ca en energía mecánica.

— Dispositivos que utilizan las corrientes de Fou-cault, como frenos de emergencia para camiones.

— El betatrón, que es un dispositivo para acelerarelectrones.

— La bobina o solenoide, un elemento de los circui-tos eléctricos de corriente alterna donde tiene lu-gar el fenómeno de la autoinducción.

— Los transformadores, dispositivos esenciales en eltransporte de la corriente alterna basados en el fe-nómeno de la inducción mutua.

RESOLUCIÓN DE EJERCICIOS Y PROBLEMAS

38. Datos: N = 240; S = 24 cm2 = 2,4 ⋅ 10−3 m2; R = 50 Ω; α0 = 0°; B = 0,5 T; Δt = 4 ms = 4 ⋅ 10−3 s; α = 180°

a) Calculamos en primer lugar la variación del flujomagnético:

La fem inducida es:

εφ

= − = −−

⋅=−

Δ

ΔtWb

sV

0 576

4 101443

,

Δ

Δ

φ φ φ α α

φ

= − = −

= ⋅ ⋅ ⋅

0 0

240 0 5 2 4

N B S

T

(cos cos )

, , 110 180 0

0 576

3 2− ° − °

= −

(cos cos )

,

m

WbΔφ

εφ

ω ω ω= − = =ddt

B S t t0 0sen senε

φ ω= =B S B S tcos0

B B t= 0 cos ω

— No produce residuos tóxicos.

Sus principales inconvenientes son:

— Para poder instalar un parque eólico, se necesitaque la velocidad promedio del viento, en esta zona,sea de 6 m ⋅ s−1.

— Tener una disponibilidad mínima de 2 500 h/año.

3. SÍNTESIS ELECTROMAGNÉTICA

34. La unificación electromagnética de Maxwell consistióen reunir todas las leyes de la electricidad y el magne-tismo en sólo cuatro ecuaciones que relacionan loscampos eléctrico y magnético con sus fuentes: las car-gas eléctricas, las corrientes eléctricas y las variacionesde los propios campos.

35. Una carga eléctrica en movimiento, además de crear uncampo eléctrico, por el hecho de estar en movimientotambién crea un campo magnético. Por lo que las per-turbaciones que produce en el espacio son de tipo elec-tromagnético.

36. Las fuentes o causas del campo eléctrico son las cargaseléctricas, ya estén en movimiento o en reposo, y loscampos magnéticos variables. Por otro lado, el campomagnético puede ser producido por una corrienteeléctrica o por un campo eléctrico variable.

37. El flujo magnético que atraviesa una superficie cerradano puede ser diferente de cero, puesto que tal y comose deduce de la segunda ecuación de Maxwell, el nú-mero de líneas de inducción que entran en la superfi-cie es igual al número de líneas que salen.

El flujo eléctrico a través de una superficie cerrada sólosería cero si en el interior de la superficie no hubieracargas. En caso contrario, éste sería proporcional a lacarga, tal y como se deduce de la primera ecuación deMaxwell.

FÍSICA Y SOCIEDAD

a) Un micrófono es un dispositivo que transforma las vi-braciones sonoras en señales eléctricas que reprodu-cen fielmente el sonido original. A continuación, estacorriente eléctrica es amplificada y transportada hastaun receptor. De esta manera se puede captar un soni-do para amplificarlo o para registrarlo y reproducirloposteriormente.

La ventaja de los micrófonos electromagnéticos frentea los de variación de resistencia está en que los prime-ros no tienen el problema de ruido que presentan losúltimos. Así pueden ser usados en aplicaciones que re-quieran una reproducción fiel del sonido en bandasamplias de frecuencia.

b) Como sabemos, el calor se disipa en el material porefecto Joule; por lo que la potencia disipada depende-rá de la resistencia del material y de la intensidad de lacorriente inducida (P = R I2). A su vez, la resistencia deun material es inversamente proporcional a su conduc-tividad eléctrica, y a su sección, de manera que a me-nor conductividad eléctrica y menor sección, mayorpotencia disipada y, por lo tanto, obtenemos más calor.

Page 123: 9523 gl fis_tx2_cas

129

42. Datos: L = 1,5 ⋅ 10−3 H; I0 = 0 A; I = 10 A;

Δt = 0,5 ms = 5 ⋅ 10−4 s

El signo negativo indica que la fem se opone al au-mento de la intensidad.

43. Datos: r = 10 cm = 0,1 m; S = 10 cm2 = 10−3 m2; μr = 1 500;

Calculamos en primer lugar el coeficiente de autoin-ducción:

A continuación calculamos el número de vueltas quetiene el toroide:

44. Datos: I = 10 sen (100 t) (SI); a = 0,05 m; b = 0,1 m;

d = 0,05 m

Sustituimos la expresión de I en el flujo magnético através de la espira:

Aplicamos la ley de Faraday para hallar la fem indu-cida:

45. Datos: I0 = 1 A; I = 0 A; Δt = 1 ms = 1 ⋅ 10−3 s;

a = 0,05 m; b = 0,1 m; d = 0,05 m

a) Sustituimos los datos del enunciado de la expre-sión del coeficiente de inducción mutua obtenidaen el ejercicio resuelto D.

Mb d a

d

MT A m m

=+

=⋅ ⋅ ⋅ ⋅− −

μ

π

π

π

0

7 1

2

4 10 0 12

0

ln

,ln

,110 05

1 4 10 8

,

,

mm

M H= ⋅ −

επ

= −⋅ ⋅ ⋅ ⋅ ⋅ ⋅ ⋅− −4 10 10 100 100 0 1

2

7 1 cos ( ) ,T A m t mππ

⋅ = − ⋅ ⋅−ln,

,, cos ( )

0 10 05

1 4 10 1005mm

t ( )SI

εφ μ

π= − = −

⋅ +ddt

t b d ad

0 10 100 1002

cos ( )ln

φμ

π=

+0 10 1002

sen ( )ln

t b d ad

LN Sr

Nr L

S

N

r

r

= =

=⋅

μ μ

π

π

μ μ

π

;

,

0

022

2 0 11 1 5 10

1 500 4 10 10

3

7 1 3 2

,m H

T A m m

⋅ ⋅

⋅ ⋅ ⋅ ⋅ ⋅

− − −π== 500 vueltas

Lt

VAs

H= = = ⋅ −εΔΙ Δ/

,,

0 03

201 5 10 3

ΔΙ

ΔtAs

V= =20 0 03; ,ε

ε = − = − ⋅−

⋅= −−

−Lt

HA A

s

ΔΙ

Δ1 5 10

10 0

5 10303

4,( )

VV

b) Hallamos la intensidad de la corriente inducida:

c) La carga total que circula por la bobina es:

Q = I Δt = 2,88 A ⋅ 4 ⋅ 10−3 s = 1,15 ⋅ 10−2 C

39. Datos: N = 240; S = 2,4 ⋅ 10−3 m2; R = 50 Ω; α0 = 0°;

α = 180°; Δt = 4 ms = 4 ⋅ 10−3 s; B = 7 ⋅ 10−5 T

a) Calculamos en primer lugar la variación del flujomagnético:

Δφ = φ − φ0 = N B S (cos α − cos α0)

Δφ = 240 ⋅ 7 ⋅ 10−5 T ⋅ 2,4 ⋅ 10−3 m2 (cos 180° − cos 0°)

Δφ = − 8,06 ⋅ 10−5 Wb

La fem inducida es:

b) Hallamos la intensidad de la corriente inducida:

c) La carga total que circula por la bobina es:

Q = I Δt = 0,40 ⋅ 10−3 A ⋅ 4 ⋅ 10−3 s = 1,6 ⋅ 10−6 C

40. Datos: l = 1 m; R = 20 Ω; v = 1,5 m ⋅ s−1; B = 0,6 T; α = 90°

c) Como �l es perpendicular a

�B (α = 90°):

F = I l B sen α = 0,045 A ⋅ 1 m ⋅ 0,6 T

F = 2,7 ⋅ 10−2 N

d) W = F v t = 2,7 ⋅ 10−2 N ⋅ 1,5 m ⋅ s−1 ⋅ 15 s = 0,61 J

41. Datos: l = 1 m; R = 15 Ω; v = 2 m ⋅ s−1; α1 = 60°; B = 0,5 T

Si la velocidad del alambre �v forma un ángulo α1 = 60°con el campo magnético

�B, el ángulo entre el vector su-

perficie �S (perpendicular al plano determinado por los

conductores y el alambre) y el vector �B serà:

α = 90° − 60° = 30°.

a) ε = B l v cos 30° = 0,5 T ⋅ 1 m ⋅ 2 m ⋅ s−1 ⋅ cos 30°

ε = 0,87 V

b)

c) F = I l B sen 60° = 0,058 A ⋅ 1 m ⋅ 0,5 T ⋅ sen 60°

F = 0,025 N = 2,5 ⋅ 10−2 N

d) W = F v t = 2,5 ⋅ 10−2 N ⋅ 2 m ⋅ s−1 ⋅ 15 s = 0,75 J

ΙΩ

= = =εR

VA

0 8715

0 058,

,

a) l

b)

, , ,ε = = ⋅ ⋅ ⋅ =−B v T m V0 6 1 1 5 0 91m s

ΙΙΩ

= = =εR

VA

0 920

0 045,

,

ΙΩ

= =⋅

= ⋅ =−

−εR

VA mA

2 02 1050

0 40 10 0 402

3,, ,

εφ

ε

= − = −− ⋅

= ⋅

Δ

ΔtWb

s

V

8 06 10

4 10

2 02 10

5

3

2

,

, == 20 2, mV

ΙΩ

= = =εR

VA

14450

2 88,

9. Inducción electromagnética

Page 124: 9523 gl fis_tx2_cas

130

tico creado por la primera bobina hace aumentarel flujo magnético a través de la segunda bobina.

Al desconectar el interruptor se induce una co-rriente eléctrica en la segunda bobina, de sentidoopuesto al caso anterior, pues el flujo magnético através de la segunda bobina disminuye.

Sólo se induce corriente en la segunda bobina sihay variación de la intensidad de corriente de laprimera bobina.

— Movimiento de un circuito eléctrico alrededor deuna bobina

Disponemos de dos bobinas: una primera conecta-da a un generador y una segunda conectada a ungalvanómetro. Ambas bobinas se colocan con susejes paralelos.

Al acercar y alejar la primera bobina, aparece unacorriente eléctrica inducida en la segunda bobina,en un sentido u otro.

Si acercamos la primera bobina a la segunda, el flu-jo magnético a través de esta última aumenta, y seinduce en ella una corriente eléctrica de sentidocontrario a la que circula por la primera bobina.

Si alejamos la primera bobina de la segunda, elflujo magnético a través de esta última disminuye,y se induce en ella una corriente de igual sentidoa la que circula por la primera bobina.

a

bG

G

+ –

+ –

Iinducida

Iinducida

I

I

Iinducida

Iinducida

I

�B a

b

G

G

+ –

+ –

b) En la situación final la intensidad es nula (I = 0),por lo cual el flujo final es cero.

El flujo inicial es:

La fem inducida en la espira es:

EJERCICIOS Y PROBLEMAS

46. — Movimiento de un imán en el interior de un bobina

Si acercamos el imán a la bobina, aparece una corrien-te inducida durante el movimiento del imán, puesel flujo magnético a través de la bobina aumenta.

Si alejamos el imán, se invierte el sentido de la co-rriente inducida, pues el flujo magnético a travésde la bobina disminuye.

Si la bobina y el imán están fijos, no se observa co-rriente inducida.

— Cierre y apertura de un circuito eléctrico

Disponemos de dos bobinas enrolladas alrededorde una misma barra de hierro: la primera forman-do parte de un circuito con generador e interrup-tor y la segunda conectada a un galvanómetro.

Al conectar el interruptor se induce una corrienteeléctrica en la segunda bobina, de sentido contrarioa la corriente de la primera, pues el campo magné-

G

N

Iinducida

b

S

a

G

SN

Iinducida

εφ

= − = −− ⋅

= ⋅−

−−Δ

ΔtWb Wb

sV

0 1 4 10

101 4 10

8

35,

,

φμ

π

φπ

=+

=⋅ ⋅ ⋅ ⋅ ⋅− −

0

0

7 1

2

4 10 1 0 1

ln

,

Ι b d ad

T A m A mm mm

Wb

20 1

0 05

1 4 1008

π

φ

ln,

,

,= ⋅ −

Page 125: 9523 gl fis_tx2_cas

131

47. a) Falso.

La corriente inducida en un circuito se opone a lavariación del flujo magnético que la produce. Si elflujo disminuye, la corriente inducida tiende a au-mentarlo. En cambio, si el flujo aumenta, la co-rriente inducida tiende a disminuirlo.

b) Falso.

Según la ley de Faraday, la fem inducida es directa-mente proporcional a la variación del flujo magné-tico, y no al valor del flujo en sí.

48. El signo negativo nos indica que la fuerza electromo-triz inducida se opone a la variación del flujo magnéti-co (ley de Lenz).

49. Debe moverse perpendicularmente al campo magnéti-co. De esta manera, según observó Henry, aparece unadiferencia de potencial entre los extremos de la barra.

La diferencia de potencial se debe a que la fuerza de Lo-rentz que actúa sobre los electrones del interior del con-ductor arrastra a éstos hasta un extremo de dichoconductor. La acumulación de carga negativa en un ex-tremo y de carga positiva en el extremo opuesto generaun campo eléctrico y la diferencia de potencial corres-pondiente a lo largo del conductor.

50. Tanto el funcionamiento de un generador eléctricocomo el de un motor eléctrico se basan en el fenóme-no de inducción electromagnética. La diferencia estáen que un generador transforma una determinada for-ma de energía en energía eléctrica, y un motor trans-forma energía eléctrica en trabajo mecánico.

51. Para generar una corriente alterna en la espira, hace-mos girar a ésta sobre uno de sus diámetros en el inte-rior del campo magnético, puesto que de esta maneraconseguimos variar el flujo magnético en la espira, ypor el fenómeno de la inducción electromagnética, segenera una corriente alterna en la espira.

52. Datos: 300 rpm

La velocidad angular en el sistema internacional es:

Como la velocidad angular es constante, el ángulo queforman el vector superficie

�S y el campo magnético

�B se

puede escribir como ω t y, por tanto, el flujo magnéticoque atraviesa la espira y la fem inducida en ella son:

Es decir, la fem es periódica y cambia alternativamentede polaridad. La frecuencia de la fuerza coincide conla del movimiento de la espira y viene dada por:

Las leyes en las que nos hemos basado para determinarla frecuencia f son las de la inducción electromagnética:

f Hz= = =ωπ

π

π2102

5

φ ω

εφ

ω

=

= − =

B S t

ddt

t

cos

senε0

300160

21

10rev

s rev sminmin rad rad

⋅ ⋅ =π

π

9. Inducción electromagnética

— Ley de Lenz: El sentido de la corriente inducida estal que se opone a la causa que la produce.

— Ley de Faraday: La fuerza electromotriz inducidaen un circuito es igual a la velocidad con que varíael flujo magnético a través de dicho circuito cam-biada de signo.

53. Al cerrar el circuito, la intensidad de corriente tardaun cierto tiempo en alcanzar su valor estacionario I yel flujo magnético a través de la bobina varía en estetiempo desde cero hasta su valor máximo. En conse-cuencia, se induce una fuerza electromotriz (llamadafuerza contraelectromotriz) que se opone al aumentoinstantáneo de la intensidad en el circuito.

De igual modo, al abrir el circuito, la intensidad tardaun cierto tiempo en anularse. En este caso, la fuerzaelectromotriz inducida se opone a que la intensidadcaiga a cero de forma instantánea.

54. Un transformador es un dispositivo que modifica latensión y la intensidad de corriente alterna.

Su funcionamiento se basa en el fenómeno de induc-ción mutua. Consta de dos bobinas de hilo conductorenrolladas alrededor de un núcleo de hierro dulce yaisladas entre sí. La bobina por la que se hace circularla corriente alterna de entrada recibe el nombre decircuito primario y la otra bobina, por la que circula lacorriente transformada de salida, recibe el nombre decircuito secundario.

La corriente alterna que circula por el circuito pri-mario produce un flujo magnético variable que origi-na una fem inducida alterna en el circuito secunda-rio. La fuerza electromotriz inducida en la bobinasecundaria tiene la misma frecuencia que la corrien-te alterna de entrada. Sin embargo, según las carac-terísticas de las bobinas, la tensión y la intensidadmáximas de la corriente en los dos circuitos puedenser distintas.

55. Datos:

Aplicamos la relación de transformación del transfor-mador para hallar la relación entre las intensidades deentrada y salida:

La intensidad de salida es de 100 veces mayor que lade entrada.

56. La diferencia estriba en la procedencia de la fuerzamotriz que hace girar la turbina. Según la fuente deenergía primaria que se transforma en energía eléctri-ca, tenemos distintos tipos de centrales:

— Centrales hidroeléctricas. Las turbinas son movi-das por el agua que cae por un desnivel. La ener-gía primaria es energía mecánica (energía poten-cial y gravitatoria del agua).

VV

2

1

1

22 1

1100

100= = → =Ι

ΙΙ Ι

V V2 11

100=

εφ

= −ddt

Page 126: 9523 gl fis_tx2_cas

132

b)

El valor positivo obtenido indica que la fem seopone al aumento de flujo.

62. Datos: l = 40 cm = 0,4 m; α = 0°; B = 0,2 T; v = 14 m ⋅ s−1

La diferencia de potencial entre los extremos de la ba-rra es igual a la fem inducida.

ε = B l v = 0,2 T ⋅ 0,4 m ⋅ 14 m ⋅ s−1 = 1,12 V

63. Datos:

ω = 3 000 rpm = 100π rad ⋅ s−1

a) El flujo magnético a través de la bobina es:

La fem inducida es entonces:

b) La fem inducida máxima es la amplitud de la fun-ción ε(t): ε0 = N B S ω

ε0 = 200 ⋅ 0,3 T ⋅ π ⋅ (5 ⋅ 10−2 m)2 ⋅ 100 π rad ⋅ s−1

ε0 = 148,0 V

64. Datos: I0 = 24 A; I = 0 A; ε = 60 V; Δt = 1 ms = 10−3 s

65. Datos: N1 = 2 400; V1 = 220 V; I1 = 4 A; V2 = 10 V

a) Calculamos primero el número de vueltas del cir-cuito secundario:

b) La intensidad de corriente de salida es:

66. Datos:

a) εφ

= − = − + ≤ ≤ddt

t t2 2 0 2;

φ = − ≤ ≤t t t s2 2 0 2;

Ι Ι21

21

2400109

4 88= = ⋅ =NN

A A

NVV

NVV

vueltas22

11

10220

2400 109= = ⋅ =

εε

= − → = − = −⋅−

= ⋅−

Lt

Lt V s

A AΔΙ

Δ

Δ

ΔΙ60 100 24

2 53

, 110 3− H

εφ

ω ω

ε π

= − =

= ⋅ ⋅ ⋅ ⋅ −

ddt

N B S t

T

sen

, (200 0 3 5 10 2 )

sen ( ) , sen

m

t

2 1100

100 148 0

⋅ ⋅ ⋅

⋅ =

−π

π

rad s

(( ) ( )100π t SI

φ ω= N B S tcos

N r cm m B T= = = = ⋅ =−200102

5 5 10 0 32; ; , ;

ε ( )t s V= = − ⋅ −4 4 10 2

εφ

= − = −ddt

t SI10 2 ( )— Centrales térmicas. Las turbinas son movidas por

vapor. El calor necesario para obtener vapor pro-cede de la combustión de materiales fósiles, como carbón, petróleo o gas natural (energía quí-mica).

— Centrales nucleares. Las turbinas son movidas porvapor. El calor necesario para obtener vapor se ob-tiene de la fisión nuclear en un reactor (energíanuclear).

57. Las pérdidas son debidas a las corrientes de Foucaultque aparecen en el núcleo de hierro del transformadorcuando éste es atravesado por un flujo magnético varia-ble. Estas corrientes se manifiestan en el calentamien-to del metal con la consiguiente pérdida de energía y,además, obligan a disipar el calor que generan.

Para reducir estas corrientes se construye el núcleo deltransformador mediante láminas finas de hierro uni-das.

58. Datos: N = 320; r = 4 cm = 4 ⋅ 10−2 m; α = 0°;

B = 0,2 T

φ = N B S cos α = 320 ⋅ 0,2 T ⋅ π ⋅ (4 ⋅ 10−2 m)2

φ = 0,32 Wb

59. Datos: N = 220; S = 30 cm2 = 3 ⋅ 10−3 m2; B = 0,4 T;

α0 = 0°; α = 180°; Δt = 15 ms = 1,5 ⋅ 10−2 s

La variación del flujo es:

Δφ = φ - φ0 = N B S (cos α − cos α0)

Δφ = 220 ⋅ 0,4 T ⋅ 3 ⋅ 10−3 m2 ⋅ (cos 180° − cos 0°)

Δφ = −0,528 Wb

La fem inducida es:

El valor positivo obtenido indica que la fem se opone ala disminución de flujo.

60. Datos: L = 0,4 H; I0 = 2 A; I = 0 A; Δt = 3 ⋅ 10−3 s

Calculamos en primer lugar el flujo magnético a travésde la bobina:

φ = L I0 = 0,4 H ⋅ 2 A = 0,8 Wb

La fem inducida es:

El signo positivo indica que la fem se opone a la dismi-nución del flujo magnético.

61. Datos: l = 5 cm = 5 ⋅ 10−2 m; α = 0°; B = 2t2 (SI); t = 4 s

a) φ = B S cos α = B l 2 cos 0°

φ = 2t2 ⋅ (5 ⋅ 10−2)2 ⋅ cos 0° = 5 ⋅ 10−3 t2 (SI)

εφ

= − = −−

= −−

⋅ −

Δ

Δ

Ι Ι

ΔtL

tH A A( ) , ( )0

3

0 4 0 2

3 10 ssV= 266 7,

εφ

= − = −−

⋅=−

Δ

ΔtWb

sV

( , )

,,

0 528

1 5 1035 22

Page 127: 9523 gl fis_tx2_cas

133

La frecuencia es:

69. Datos: l = 30 cm = 0,3 m; N = 1 000;

S = 60 cm2 = 6 ⋅ 10−3 m2; μr = 1 500

Calculamos el coeficiente de autoinducción de la bo-bina:

Si introdujéramos un núcleo de hierro en su interior,el coeficiente de autoinducción de la bobina se veríamodificado de esta manera:

70. Las celdas de entrada de información pueden ser:

B1: Etiqueta de “f”B2: Etiqueta de “B”B3: Etiqueta de “S”B4: Etiqueta de “N”

C1: valor de “f”C2: valor de “B”C3: valor de “S”C4: valor de “N”

La tabla de la función de la fuerza electromotriz indu-cida en función del tiempo de acuerdo con la fórmu-la: ε(t) = 2π f N B S sin (2π f t) se puede programarcomo:

A20: 0B20: =C$4*C$2*C$3*2*PI()*C$1*SENO(2*PI()*C$1*A20)

A21:=A20+0,001B21: =C$4*C$2*C$3*2*PI()*C$1*SENO(2*PI()*C$1*A21)

A22:=A21+0,001B22: =C$4*C$2*C$3*2*PI()*C$1*SENO(2*PI()*C$1*A22)

... y así hasta llegar a:

A520:=A521+0,001B520:=C$4*C$2*C$3*2*PI()*C$1*SENO(2*PI()*C$1*A520)

El resultado de la programación es:

LN S

L L H

r

r

=

= = ⋅ ⋅ =−

μ μ

μ , ,

02

21500 2 5 10 37 5l

HH

LN

ST m

A mm

L

= = ⋅⋅

=

− −μ π0

27

23 24 10

1 0000 3

6 10l

( ),

22 5 10 2, ⋅ − H

fw

Hz= =⋅

=−

2314 38

250 0

1

π π

,,

rad s

Representamos el flujo magnético y la fem induci-da en función del tiempo.

b) será máximo si:

c) ε = −2t + 2 es una recta comprendida entre 0 y 2,por lo que los dos máximos estarán en t = 0 s y t = 2 s.

El signo de ε sólo nos indica el sentido de la co-rriente.

d) El flujo magnético y la fem inducida no son máxi-mos simultáneamente, puesto que la fuerza elec-tromotriz inducida no se opone al flujo magnéticosino a su variación.

67. Como sabemos, según la experiencia de Henry, si move-mos un barra conductora dentro de un campo magné-tico uniforme, los electrones libres de la barra estaránsometidos a un fuerza que viene dada por la expresión:

y por lo tanto, depende del ángulo queforma el campo con la velocidad lineal que lleva la barra,de modo que: F = q v B sen α. Como la barra describe unmovimiento circular, la velocidad lineal siempre será per-pendicular o normal a la trayectoria y, por lo tanto, alcampo magnético, por lo que el valor de la fuerza será entodo momento F = q v B, su dirección será la direcciónde la barra y su sentido vendrá dado por la regla de lamano derecha, como muestra la figura.

68. Datos: B = 0,6 T; I0 = 2 A; N = 150; r = 3 cm = 3 ⋅ 10−2 m;

R = 40 Ω.

Determinamos la frecuencia angular a partir de la in-tensidad máxima de la corriente inducida.

ΙΙ

Ω

00 0

2 40

150 0

= = → =

=⋅

ε

RN B S

RR

N B SA

,

ωω

ω66 3 10

314 382 2( ),

rad

T m s⋅ ⋅ ⋅=−π

�v�F

� � �F = q (v x B),

ddt

t t sφ

= → − = → =0 2 2 0 1

φ

t(s)t(s)

0

1

23

4

–1

03

842

2

ε(V)φ (Wb) t(s) φ(Wb)8

6

42

0

–2

–2

–4

9. Inducción electromagnética

Page 128: 9523 gl fis_tx2_cas

134

La intensidad a la salida queda aumentada 100 vecesrespecto a la intensidad de entrada.

4. Datos: N = 500; r = 0,005 m; B0 = 0,1 T

a) Si B = 0,2 T en Δt = 0,02 s:

El signo negativo indica que la fem se opone al au-mento del flujo magnético.

b) Si α = 180° en Δt = 0,02 s:

El signo positivo indica que la fem se opone a ladisminución del flujo magnético.

5. Datos: B = 0,4 T; α = 90°; l = 1 m; R = 15 Ω; v = 2 m⋅s−1

a) ε = v B l = 2 m⋅s−1 ⋅ 0,4 T ⋅ 1 m = 0,8 V

b) El sentido de la intensidad puede determinarse apartir de la ley de Lenz. Como el flujo magnéticoa través del circuito aumenta, la corriente induci-da debe crear un campo magnético que contra-rreste el aumento de flujo.

Calculamos la intensidad de la corriente inducida:

c) Calculamos la fuerza magnética sobre la barra:

F B A m T Nm = ° = ⋅ ⋅ =Ι sen , , ,l 90 0 05 1 0 4 0 02

ΙΩ

= = =εR

VA

0 815

0 05,

,

M

N

I�v

�B

εφ φ φ

ε

= − = −−

= −° −

=

Δ

Δ Δ Δt tN B S N B S

t0 0 0180cos

−−° −

= −⋅ ⋅

N S Bt

m

(cos )

( , )

0

2

180 1

500 0 005 0

Δ

επ ,, ( )

,,

1 1 10 02

0 39T

sV

− − −=

εφ φ φ

ε

= − = −−

= −−

= −−

Δ

Δ Δ Δt tN B S N B S

t

N S B B

0 0

0( ))

( , ) ( , , ),

Δt

m T Ts

επ

= −⋅ ⋅ ⋅ −500 0 005 0 2 0 1

0 02

2

== − 0 20, V

El resultado obtenido debe ser el siguiente:

EVALUACIÓN

1. El coeficiente de autoinducción representa la fem au-toinducida en un circuito cuando la intensidad de co-rriente varía un amperio en un segundo.

La unidad del coeficiente de autoinducción en el SI deunidades es el henrio, H:

2. a) Si la corriente en la primera bobina aumenta, se in-crementará también el flujo magnético en la se-gunda bobina, por lo que aparecerá en la segundabobina una corriente inducida de sentido contra-rio al de la primera para contrarrestar este aumen-to del flujo magnético.

b) Si en la primera bobina la corriente disminuye,también disminuirá el flujo magnético que atravie-sa la segunda bobina, por lo que aparecerá una co-rriente inducida en esta segunda bobina del mis-mo sentido que en la primera para contrarrestar ladisminución del flujo magnético.

c) Si la corriente en la primera bobina se mantieneconstante, no habrá variación de flujo en la segun-da bobina y, por la tanto, no aparecerá ninguna co-rriente inducida en la segunda bobina.

3. La tensión a la salida queda disminuida 100 veces res-pecto a su valor a la entrada.

I

I

+–

S

S2

G

R

1 1HV sA

=⋅

t(s)

Page 129: 9523 gl fis_tx2_cas

135

La luz10

PREPARACIÓN DE LA UNIDAD

• Datos: λ = 125 m; c = 3 ⋅ 108 m ⋅ s−1

• a) 6 500 nm = 6 500 ⋅ 10−9 m = 6,5 ⋅ 10−6 m

b) 3,6 ⋅ 10−8 m = 36 ⋅ 10−9 m = 36 nm

c) 320 nm = 320 ⋅ 10−9 m = 3,2 ⋅ 10−7 m

1. NATURALEZA DE LA LUZ

1. Puedes observar las características de la luz explicadaspor cada una de las teorías en la tabla de pie de página.

2. Datos: E = 5,2 ⋅ 10−18 J; h = 6,625 ⋅ 10-−34 J ⋅ s

Determinamos la frecuencia del fotón utilizando la re-lación con la energía a través de la constante de Planck:

3. Las células fotoeléctricas convierten energía luminosa en energía eléctrica. Generalmente consisten en dos

E hf fEh

J

J s= = =

⋅ ⋅=

−;,

,,

5 2 10

6 625 107 85

18

34 ⋅⋅1015 Hz

fc m s

mHz= =

⋅ ⋅= ⋅

λ3 10

1252 4 10

8 16,

10. La luz

semiconductores, en cuya zona de unión existe uncampo eléctrico. Cuando inciden fotones, se generancargas positivas y negativas, que son aceleradas ensentidos opuestos por el campo de la unión. Esta se-paración de cargas crea un potencial eléctrico, deforma que la energía del fotón se convierte en ener-gía eléctrica.

Sus aplicaciones son muchísimas: barreras ópticas dedetección para la protección frente a robos, sistemasde apertura automática, control de alumbrado inte-rior y exterior, máquinas clasificadoras, detección deimpurezas en el proceso de embotellamiento de bebi-das, transmisión de imágenes, cine…

Todas ellas se basan en que la interrupción de la luz que ilumina la célula provoca el cese de la producciónde electricidad o, al contrario, al tener lugar la ilumi-nación de la célula se inicia la producción de energíaeléctrica.

4. a) Verdadero. Las fases de las ecuaciones de cadauno de los campos son iguales en todo momentoy en cada punto del espacio.

b) Falso. Los módulos de los campos eléctrico y mag-

nético verifican la relación con c la veloci-

dad de la onda. Por tanto, no son iguales.

EB

c= ,

Propagaciónrectilínea

Reflexión RefracciónDoble

refracciónDifracción Interferencias Polarización

Efectofotoeléctrico

Propagaciónen el vacío

Teoría corpuscular de Newton

Sí Sí No NoNo

observadaNo

observadasNo

observadaNo

observadoSí

Teoríaondulatoriade Huygens

Sí Sí Sí SíNo

observadaNo

observadasNo

observadaNo

observadoNo

Teoríaondulatoriade Fresnel

Sí Sí Sí Sí Sí Sí SíNo

observadoNo

Teoríaelectromagnéticade Maxwell

Sí Sí Sí Sí Sí Sí SíNo

observadoSí

Naturaleza dual de la luz

Sí Sí Sí Sí Sí Sí Sí Sí Sí

La teoría corpuscular de Einstein fue propuesta para explicar el efecto fotoeléctrico.

Page 130: 9523 gl fis_tx2_cas

136

llegan y otros no, siendo la penumbra una zona par-cialmente iluminada.

9. Respuesta sugerida:

Un experimento posible para verificar la propaga-ción rectilínea de la luz es la cámara oscura. Consis-te en una caja con un pequeño orificio en una caracuya cara opuesta es de material translúcido. Si situa-mos algún objeto, preferentemente luminoso, delan-te del orificio, observaremos su imagen invertida so-bre la cara translúcida. Este fenómeno se explicaatendiendo a la propagación rectilínea de los rayosde luz.

10. Respuesta sugerida:

Identificamos las etapas del método científico en elproceso seguido por Roemer y Fizeau para demos-trar que la velocidad de la luz es finita y medir suvalor:

— Observación. Roemer observó que el intervalode tiempo transcurrido entre dos eclipses conse-cutivos de Io, uno de los satélites de Júpiter, eravariable; se hacía mayor cuando la Tierra se ale-jaba de Júpiter y menor cuando la Tierra se acer-caba a él.

— Formulación de hipótesis. Contra la opinión man-tenida durante siglos, Roemer supone que la velo-cidad de la luz es finita y, por tanto, cuando la dis-tancia entre la Tierra y Júpiter es mayor, tarda mástiempo en llegar hasta nosotros; de ahí la diferen-cia observada entre dos eclipses.

— Experimentación. Para comprobar que la veloci-dad de la luz es finita y calcular su valor de unaforma directa, Fizeau diseña un experimento, elde la rueda dentada y el espejo. En el caso de Ro-emer no podemos hablar propiamente de expe-rimentación, pues no se trata de ensayos contro-lados.

— Organización de los datos experimentales. TantoRoemer como Fizeau repitieron sus observacionespara obtener un número suficiente de datos de losque extraer conclusiones.

— Extracción de conclusiones. Ambos llegaron a laconclusión de que la velocidad de la luz es finita ycalcularon su valor.

— Elaboración de una teoría. Posteriormente, AlbertEinstein (1879-1955) hizo de la conclusión ante-rior la base y el punto de partida de su teoría espe-cial de la relatividad.

11. Datos: c = 3 ⋅ 108 m/s = 3 ⋅ 105 km/s

a) Calculamos los segundos que tiene un año y,con ellos, la distancia recorrida por la luz en unaño:

Δt ad

ah

ds

h= ⋅ ⋅ ⋅ = ⋅1

3651

241

36001

3 15 107, ss

x c tkms

s kmΔ Δ= = ⋅ ⋅ ⋅ = ⋅, ,3 10 3 15 10 9 5 105 7 12

5. Datos: f1 = 3 ⋅ 1010 Hz; f2 = 5 ⋅ 1013 Hz; f3 = 1,07 ⋅ 1015 Hz

6. Datos: λ1 = 760 nm = 7,6 ⋅ 10−7 m; λ2 = 380 nm = 3,8 ⋅ 10−7 m

Determinamos las frecuencias correspondientes:

7. Datos: E = 100 sen (3 ⋅ 1015 t − 1,0 ⋅ 107 x ) (SI)

Si comparamos la expresión del enunciado con laecuación del campo eléctrico de una onda electromag-nética, obtenemos:

E0 = 100 N/C; ω = 3 ⋅ 1015 rad/s; k = 1,0 ⋅ 107 m−1

a) Determinamos la longitud de onda y la frecuenciaa partir del número de ondas y la pulsación:

b) Hallamos la amplitud del campo magnético a par-tir de la amplitud del campo eléctrico:

Entonces, la ecuación del campo magnético será,en unidades del SI:

B = B0 sen(ωt − kx);

B = 3,33 ⋅ 10−7 sen (3 ⋅ 1015 t − 1,0 ⋅ 107x)

8. La sombra es la región no iluminada que aparece de-trás de un cuerpo opaco cuando éste se ilumina con unfoco puntual. Reproduce el contorno del objeto, defi-nido por los rayos luminosos tangentes a éste.

La penumbra es la región parcialmente iluminada queaparece detrás de un objeto cuando éste se iluminacon un foco no puntual. La aparición de la penumbraes debida a las dimensiones finitas del foco, ya que en-tonces algunos rayos de los emitidos en esa dirección

EB

c BEc

N C= = =

⋅ ⋅= ⋅

−−; ,0

01

8 17100

3 103 33 10

m sT

kk m

m= = =⋅

= ⋅

=

−−2 2 2

1 0 106 28 10

2

7 17π

λλ

π π

ω π

;,

,

ff f Hz; ,= =⋅

= ⋅ωπ π2

3 102

4 77 1015

14rad/s

fc

mHz

fc

11

8

714

2

3 10

103 95 10= =

⋅= ⋅

=

−λ,

m/s

7,6

λλ2

8

7143 10

107 9 10=

⋅= ⋅− ,

m/s

3,8 mHz

λ

λ

11

8

10

12

3 10

3 10

10 0 01

= =⋅

= =−

cf Hz

m m, ;

m/s

MMicroondas

cf Hz

λ

λ

22

8

13

2

3 10

5 10

6 10

= =⋅

= ⋅

m/s

−−

= =⋅

6

33

8

15

3 10

1 07 10

;

,

m Infrarrojo

cf

λm/s

HHz

m Ultravioletaλ372 8 10= ⋅ −, ;

Page 131: 9523 gl fis_tx2_cas

137

17. Datos: f = −10 cm = −0,1 m (espejo cóncavo)

a) Si el objeto se sitúa en s1 = −25 cm = −0,25 m, laimagen se situará en:

Calculamos el aumento lateral:

El signo negativo del aumento lateral significa quela imagen aparece invertida. Además, como su va-lor absoluto es menor que la unidad, la imagen esmenor que el objeto. Al ser s2 < 0, la imagen se for-ma por intersección de los rayos reflejados en elespejo y es real.

b) Si el objeto se sitúa en s1 = −10 cm = −0,10 m, laimagen se situará en:

Es decir, no se observa ninguna imagen. En gene-ral, siempre que el objeto se sitúe sobre el foco, nose forma imagen.

c) Si el objeto se sitúa en s1 = −5 cm = −0,05 m, la ima-gen se situará en:

Calculamos el aumento lateral:

Ass

cmcmL = − = −

−=2

1

105

2

1 1 1 11 1

11

0 11

0

2 12

1

2

s s fs

f s

s

m

+ = =−

=

−−

;

, ,005

0 10 10,

m

m cm= =

s

m m

21

10 1

10 10

=

−−

= ∞

, ,

1 1 1 1 1 1 11 12 1 2 1

2s s f s f ss

f s

+ = = − =−

; ;

11

Ass

cmcmL = − = −

−−

= −2

1

16 725

0 67,

,

1 1 1 11 1

11

0 11

0

2 12

1

2

s s fs

f s

s

m

+ = =−

=

−−

;

, ,225

0 167 16 7, ,

m

m cm= − = −

C F

r < 0

f < 0

b) Hallamos los años luz que nos separan de α-Cen-tauri:

2. FENÓMENOS LUMINOSOS

12. a) Verdadero. La velocidad y la longitud de onda de-penden del índice de refracción del medio.

b) Falso. La frecuencia sí es independiente del mediomaterial.

c) Verdadero. El índice de refracción nos proporcio-na la relación entre la velocidad de la luz en el va-cío y la velocidad en el medio.

13. Si un haz de luz láser pasa de un medio a otro de índi-ce de refracción menor, n1 > n2, el ángulo de refrac-ción será mayor que el de incidencia.

Según la ley de Snell, el ángulo de refracción r se rela-

ciona con el de incidencia i por Si

n1 > n2 , el cociente Por tanto, sen r > sen i, lo

que indica que r > i.

14. Datos: n1 = 1,52; n2 = 1; i = 30°

a) Determinamos el ángulo de refracción a partir dela ley de Snell:

b) Calculamos el ángulo límite de la superficie de se-paración entre el vidrio y el aire:

c) Si un rayo incide con un ángulo de 45°, al ser esteángulo mayor que el ángulo límite, se producirá re-flexión total.

15. Datos: f = 1,5 MHz = 1,5 ⋅ 106 Hz; n = 1,2

En el aire, con índice de refracción n = 1, la longitudde onda será:

Entonces, en el medio con n = 1,2, la longitud de onda es:

16. Si queremos ver nuestra imagen ampliada y derecha enun espejo, debemos emplear un espejo cóncavo y si-tuarnos entre el foco y el espejo. Si nos situamos entreel centro de curvatura y el foco, veremos nuestra ima-gen aumentada pero invertida.

nn

mm= = = =

λ

λλ

λ0 0 2001 2

166 7;,

,

λ0

8

6

3 10

1 5 10200= =

⋅=

cf Hz

m,

m/s

sen,

;Lnn

L= = = °2

1

11 52

41 8′

n i n r

rnn

i r

1 2

1

2

1 5

sen sen

sen sen ; sen,

=

= =22 30

10 76

49 28

⋅ °=

= °

sen,

r ′

nn

1

2

1> .

sen sen .rnn

i= 1

2

Δ x kma

kmaño= ⋅ ⋅

⋅=4 085 10

1 1

9 5 104 313

12,

. .

,, ss luz

10. La luz

Page 132: 9523 gl fis_tx2_cas

138

El signo negativo del aumento lateral significa quela imagen aparece invertida. Además, como su valorabsoluto es menor que la unidad, la imagen es me-nor que el objeto. Al ser s2 > 0, la imagen se formapor intersección de los rayos emergentes y es real.

b) s1 = −10 cm

Hallamos la posición de la imagen:

La imagen no se forma (se forma en el infinito),ya que el objeto está situado en el foco de la lente.

c) s1 = −5 cm

Hallamos la posición de la imagen:

Determinamos el aumento lateral:

Como el aumento lateral es mayor que la unidady positivo, la imagen será mayor que el objeto y de-recha. Al ser s2 < 0, la imagen se forma por inter-sección de las prolongaciones de los rayos emer-gentes y es virtual.

21. Datos: f2 = −20 cm (lente divergente); y1 = 2,0 cm;s1 = −30 cm

a) Hallamos la posición de la imagen:

b) Determinamos el aumento lateral:

Calculamos el tamaño de la imagen:

c) Al ser s2 < 0, la imagen se forma por intersecciónde las prolongaciones de los rayos emergentes y esvirtual, como siempre en lentes divergentes.Como el aumento lateral es menor que la unidad

Ayy

y A y cm cmL L= = = ⋅ =2

12 1 0 4 2 0 0 8; , , ,

Ass

AcmcmL L= =

−=2

1

1230

0 4; ,

1 1 1 1 1 1

11 1

2 2 1 2 1 2

2

1 2

f s s s s f

s

s f

= − = +

=+

;

; s

cm cm

cm21

130

120

12=

−+

= −

Ass

AcmcmL L= =

−=2

1

105

2;

1 1 1 1 1 1

11 1

2 2 1 2 1 2

2

1 2

f s s s s f

s

s f

= − = +

=+

;

; s

cm cm

cm21

15

110

10=

−+

= −

1 1 1 1 1 1

11 1

2 2 1 2 1 2

2

1 2

f s s s s f

s

s f

= − = +

=+

;

; s

cm cm

21

110

110

=

−+

= ∞

El signo positivo del aumento lateral significa quela imagen aparece derecha. Además, como su valorabsoluto es mayor que la unidad, la imagen es ma-yor que el objeto. Al ser s2 > 0, la imagen se formapor intersección de las prolongaciones de los rayosreflejados y es virtual.

18. Datos: AL = −2; s2 = −150 cm = −1,5 m (imagen real)

a) Determinamos la posición del objeto a partir de laposición de la imagen y el aumento lateral:

b) Calculamos el radio del espejo a partir de la ecua-ción fundamental de los espejos esféricos:

19. a) La imagen formada por una lente delgada es dere-cha:

— Para una lente convergente si el objeto se sitúaa una distancia de la lente menor que la distan-cia focal.

— Siempre para lentes divergentes.

La imagen formada por una lente delgada esinvertida:

— Para lentes convergentes cuando el objeto sesitúa a una distancia de la lente mayor que ladistancia focal.

b) La imagen formada por una lente delgada es ma-yor que el objeto:

— Para lentes convergentes cuando el objeto se si-túa a una distancia de la lente mayor que la dis-tancia focal pero menor que el doble de ésta.

— Para una lente convergente si el objeto se sitúaa una distancia de la lente menor que la distan-cia focal.

20. Datos: f2 = 10 cm (lente convergente)

a) s1 = −30 cm

Hallamos la posición de la imagen:

Determinamos el aumento lateral:

Ass

AcmcmL L= =

−= −2

1

1530

0 5; ,

1 1 1 1 1 1

11 1

2 2 1 2 1 2

2

1 2

f s s s s f

s

s f

= − = +

=+

;

; s

cm cm

cm21

130

110

15=

−+

=

r

m m

m cm=

−−

= − = −2

11 5

10 75

1 100

, ,

1 1 2 21 12 1

2 1

s s rr

s s

+ = =+

;

Ass

ssA

sm

m

L

L

= −

= − = −−

−= − = −

2

1

12

11 5

20 75 75;

,, cm

Page 133: 9523 gl fis_tx2_cas

139

duzca interferencia es necesario que la luz emitidapor los dos focos sea coherente, es decir, que man-tengan una diferencia de fase constante y que seanmonocromáticas.

28. Los fenómenos de interferencia aparecen cuando sesuperponen las ondas armónicas emitidas por dos fo-cos puntuales, mientras que la difracción se debe alos efectos de un objeto o un orificio en una superfi-cie que se interpone en el camino de las ondas lumi-nosas.

En el caso de que el objeto sea una rendija, el patrónde difracción obtenido se asemeja al patrón de inter-ferencia de dos fuentes puntuales coherentes. Estopuede interpretarse considerando que cada punto dela rendija actúa como una fuente puntual que emiteondas elementales que interfieren entre sí.

29. Datos: λ = 600 nm = 6 ⋅ 10−7 m; L = 1,25 md = 0,090 mm = 9 ⋅ 10−5 m

a) La primera franja oscura corresponde a n = 1. Portanto:

Si suponemos que el ángulo es muy pequeño, po-

demos considerar que

Por tanto, la posición del primer mínimo es:

30. La luz del Sol es dispersada por las moléculas de aire,fenómeno que da origen al color azul del cielo. La luzdispersada está polarizada linealmente. En cambio, laluz solar reflejada por las nubes, al no ser éstas super-ficies planas, no está polarizada. Entonces, si coloca-mos un filtro polarizador en la lente de una cámarafotográfica, la luz del cielo será absorbida en su mayorparte, mientras que la luz reflejada por las nubes, alno ser luz polarizada, no será absorbida y éstas apare-cerán blancas.

31. Datos: n agua = 1,33

Hallamos el ángulo de Brewster, ángulo para el cual laluz reflejada estará totalmente polarizada:

tg i = n; tg i = 1,33; i = 53° 4’

RESOLUCIÓN DE EJERCICIOS Y PROBLEMAS

32. Datos: E(x, t) = 10−3 cos (5 ⋅ 1010 t − 200x) (SI)

Si comparamos la ecuación del enunciado con la delcampo eléctrico de una onda electromagnética expre-sada en función del coseno, E = E0 cos (ωt − kx), ob-tenemos:

E0 = 10−3 N/C; ω = 5 ⋅ 1010 rad/s; k = 200 m−1

y L m m

y m1

3

13

1 25 6 67 10

8 3 10 8

= = ⋅ ⋅

= ⋅ =

sen , ,

,

α

,,3 mm

sen α α= =t gyL

1

sen ,αλ

= = ⋅⋅

⋅= ⋅

−−n

dm

m1

6 10

6 106 67 10

7

53

y positivo, la imagen será menor que el objeto y de-recha.

d)

22. Las aplicaciones de los espejos y de las lentes que se ci-tan en el texto son:

— Espejos: facilitan nuestro cuidado personal o laconducción de vehículos automóviles, se utilizancomo instrumento médico en ortodoncia y comoconstituyente fundamental de los instrumentos óp-ticos (telescopio, cámara fotográfica...).

— Lentes: se emplean en la construcción de instru-mentos ópticos (cámara fotográfica, lupa, prismá-ticos, microscopio, telescopio...) y para la correc-ción de defectos de la visión tales como lamiopía, la hipermetropía, la presbicia o el astig-matismo.

23. La luz azul se desvía más en un prisma óptico que laamarilla, ya que su longitud de onda es menor. Por tan-to, el índice de refracción es mayor para el azul quepara el amarillo. Como la velocidad de propagación enun medio es inversamente proporcional al índice derefracción, en el prisma se propaga a mayor velocidadla luz amarilla que la azul.

24. La formación del arco iris es debida a la dispersiónde la luz por parte de las gotitas de agua en suspen-sión en la atmósfera. Cada una de ellas actúa como unpequeño prisma óptico, desviando la luz del Sol en di-ferentes ángulos según la longitud de onda. De estaforma, la luz blanca del Sol es dispersada y observamossus distintos colores separados. A diferencia del prismaóptico, en las gotas la luz sufre, además de dos refrac-ciones, una reflexión. Por ello la luz que sufre más des-viación es la roja.

25. Los espectros de emisión están formados por la luzemitida por una sustancia química.

En cambio, los espectros de absorción se observan enluz de espectro continuo después de que haya atravesa-do alguna sustancia química donde algunas de sus fre-cuencias son absorbidas. Por ello, los espectros de ab-sorción presentan líneas oscuras sobre el espectrocontinuo de la luz incidente.

26. La condición que tienen que cumplir es que lleguenen fase al punto mencionado. En este caso, la ampli-tud resultante es la suma de las amplitudes de ambasondas.

27. Dos linternas que se mantienen encendidas muy pró-ximas no producen un patrón de interferencia por-que la luz que emiten es incoherente, los trenes deonda que emite cada una de ellas son independientesde los emitidos por la otra linterna. Para que se pro-

F1F2

Oy2

y1

10. La luz

Page 134: 9523 gl fis_tx2_cas

140

c) Como el aumento del sistema es negativo y mayorque la unidad en valor absoluto, la imagen finalserá invertida y mayor que el objeto. Además,como s2N > 0, la imagen se forma por intersecciónde los rayos emergentes y es real.

EJERCICIOS Y PROBLEMAS

36. Los rayos X y las ondas de radio son ondas electro-magnéticas. Las dos consisten en el mismo tipo deonda que la luz y se propagan a la misma velocidaden el vacío, c. Lo que las diferencia es la frecuencia y,por tanto, la longitud de onda y la energía que trans-portan.

37. Leyes de la reflexión:

1. El rayo incidente, la normal a la superficie en elpunto de incidencia y el rayo reflejado están situa-dos en el mismo plano.

2. El ángulo de incidencia y el de reflexión son igua-les.

Leyes de la refracción:

1. El rayo incidente, la normal a la superficie en elpunto de incidencia y el rayo refractado están situa-dos en el mismo plano.

2. La razón entre el seno del ángulo de incidencia y eldel ángulo de refracción es una constante igual a larazón entre las respectivas velocidades de propaga-ción del movimiento ondulatorio.

Principio de Huygens:

Todo punto de un frente de onda se convierte en uncentro puntual productor de ondas elementales se-cundarias, de igual velocidad y frecuencia que la ondainicial, cuya superficie envolvente constituye un nuevofrente de onda.

38. En una refracción se conserva la frecuencia de laonda incidente y el plano que forma ésta con la nor-mal. Al entrar en otro medio, se modifican la direc-ción de propagación, la velocidad y la longitud deonda.

39. El rayo refractado se acerca a la normal cuando pasade un medio a otro con mayor índice de refracción, yaque la velocidad en el nuevo medio es menor. En cam-bio, se aleja de la normal cuando el nuevo medio es deíndice de refracción menor que el primero.

40. El tamaño de la imagen formada por una lente delga-da convergente es igual al tamaño del objeto si éste sesitúa a una distancia de la lente igual al doble de la dis-tancia focal de la lente. Si s1 = −2f2:

sensen

ir

vv

nn

= =1

2

2

1

A A Ass

ss

Acmcm

c

L LM LNM

M

N

N

L

= =

=−

⋅,

2

1

2

1

3015

6 7 mmcm10

1 3,= −

Determinamos la frecuencia y la longitud de onda apartir de la pulsación y del número de ondas:

Hallamos la velocidad de propagación para determinarel índice de refracción del medio:

33. Datos: E0 = 0,5 N/C; λ = 1 m

Calculamos la amplitud del campo magnético a partirde su relación con la amplitud del campo eléctrico:

34.

La imagen final es real, invertida y menor que el objeto.

35. Datos: f2M = 10 cm; f2N = 20 cm; O1O2 = 20 cm; s1M =−15 cm

a) Calculamos dónde se forma la imagen debido a laprimera lente:

Hallamos la posición de la imagen final teniendoen cuenta que la imagen producida por la primeralente es el objeto para la segunda lente:

s1N = s2M −20 cm = 30 cm −20 cm = 10 cm

b) El aumento lateral del sistema será el producto delaumento lateral de cada lente:

1 1 1 1 1 1

11

2 2 1 2 1 2

2

1

f s s s s f

s

s

N N N N N N

N

= − = +

=

;

NN N

N

f

s

cm cm

cm+

=+

=1

11

101

20

6 7

2

2; ,

1 1 1 1 1 1

11

2 2 1 2 1 2

2

1

f s s s s f

s

s

M M M M M M

M

= − = +

=

;

MM M

M

f

s

cm cm

cm+

=

−+

=1

11

151

10

30

2

2;

M

FIM FIN F2M F2N

N

BEc

N CT0

08

90 5

3 101 67 10= =

⋅= ⋅ −, /

,m/s

EB

c=

v f v m Hz= = ⋅ ⋅ = ⋅

=

λ ; , , ,0 0314 7 96 10 2 5 109 8 m/s

nccv

n; ,=⋅

⋅=

3 10

101 2

8

8

m/s

2,5 m/s

ωπ

πωπ π

= = = =⋅ ⋅

= ⋅

=

22

25 10

27 96 10

109

Tf f Hz

k

; ,rad s

22 2 2

2000 03141

π

λλ

π π; ,= = =−k m

m

Page 135: 9523 gl fis_tx2_cas

141

componentes de la luz con el campo eléctricoperpendicular a las cadenas moleculares sontransmitidas, mientras que si el campo es parale-lo a la cadena, genera una corriente eléctrica yes absorbido.

46. El defecto conocido como presbicia o vista cansadaconstituye el alejamiento normal del punto próximocon la edad, hecho que se debe a la disminución de lafacultad de acomodación del ojo causada por la pérdi-da de flexibilidad del cristalino. La consecuencia deello es que no pueden formarse imágenes nítidas delos objetos próximos. Se corrige utilizando lentes con-vergentes adecuadas.

— Un miope usa lentes divergentes porque su pro-blema es que el punto lejano está a una distan-cia finita. Las lentes divergentes desvían los ra-yos procedentes de puntos distantes de maneraque la imagen que forma está más cerca que elobjeto por lo que el ojo miope es capaz de enfo-carla.

— Por el contrario, el ojo hipermétrope y el présbitason incapaces de enfocar objetos próximos. Lalente convergente, al formar la imagen más lejosque la posición del objeto, aleja la imagen y permi-te que el ojo la enfoque.

47. Datos: B0 = 6,5 ⋅ 10−-6 T; c = 3 ⋅ 108 m/s

Determinamos la amplitud del campo eléctrico:

48. Datos: d = 1,5 ⋅ 1011 m; c = 3 ⋅ 108 m/s

El tiempo que tardará la luz del Sol en alcanzar la Tie-rra será:

Δt = 500 s = 8,3 min = 8 min 20 s

49. Datos: d = 2 km; c = 3 ⋅ 108 m/s

El tiempo que tardaría la luz en ir y volver sería:

El experimento fracasó porque este tiempo es inferioral tiempo de reacción del ser humano. Es decir, eltiempo que tarda una persona en tomar conciencia dela imagen de la lámpara, que su cerebro transmita laorden correspondiente al brazo y que éste efectúe elmovimiento de destapar la otra lámpara o cronome-trar es muy superior al tiempo que la luz emplea en re-correr la distancia.

50. Si el rayo reflejado forma un ángulo de 90° con el rayorefractado, aplicando las leyes de la reflexión y de larefracción, tenemos:

Δtdc

ms= =

⋅⋅ −2 2 2000

3 101 33 108

5,m/s

ΔΔ

ts

vdc

m= = =

1 5 10

3 10

11

8

,

m/s

EB

cEB

c E c B= = = = ⋅ ⋅ ⋅ −; ; ,0

00 0

8 63 10 6 5 10m/s TT

B0 1950= N/C

Entonces, el aumento lateral será:

Por tanto, la imagen será real, del mismo tamaño queel objeto e invertida.

41. Sí es posible distinguir por el tacto una lente conver-gente de otra lente divergente. Las lentes convergentesson más gruesas en su parte central que en los extre-mos, mientras que las lentes divergentes tienen los ex-tremos más gruesos que su parte central.

42. El objetivo de una cámara fotográfica no puede serdivergente, ya que la imagen que formaría sería vir-tual. El objetivo debe formar la imagen sobre la pelí-cula, que se encuentra siempre detrás de la lente.Una lente divergente forma la imagen delante deella, en el mismo lado de donde proceden los rayosdel objeto (decimos que la imagen es virtual). Portanto, nunca podría formar la imagen sobre la pelícu-la fotográfica.

43. a) Vemos distintos objetos de diferentes colores por-que cada material es capaz de absorber, reflejar otransmitir distintas longitudes de onda. Cuandoiluminamos un objeto con luz blanca, recibimossólo las frecuencias que ese material es capaz de re-flejar, cuyo efecto en nuestra retina constituye elcolor del objeto.

b) Como el color corresponde a las frecuencias refle-jadas por los objetos, dependerá también de las fre-cuencias con las que los iluminemos. Por ejemplo,un objeto verde, iluminado con luz de otro color,sin la frecuencia verde, parecerá negro, ya que noreflejará luz alguna.

44. El ángulo bajo el que se observan las franjas del patrónde difracción y, por lo tanto, su tamaño sobre la panta-lla aumenta con la longitud de onda. En el microsco-pio, y debido a la abertura del objetivo, también se pro-duce difracción. Por ello se utiliza luz azul en lailuminación del microscopio, para minimizar los efec-tos de la difracción, ya que el azul es la radiación visi-ble de menor longitud de onda.

45. Dos métodos para conseguir luz polarizada linealmen-te son:

— Polarización por reflexión. Cuando la luz incidesobre una superficie pulimentada de vidrio, la luzreflejada está total o parcialmente polarizada. Enconcreto, si la tangente del ángulo de incidenciacoincide con el valor del índice de refracción delmedio, la luz reflejada está totalmente polari-zada.

— Polarización por absorción selectiva. Algunosmateriales formados por láminas que contienenlargas cadenas lineales de moléculas de hidrocar-buros tienen la propiedad de transmitir a su tra-vés la luz sólo en un plano de polarización. Las

Ass

ffL = =

−= −2

1

2

2

22

1

1 1 1 11 1

112

111

2

22 2 1

2

1 2 2 2 2

f s ss

s f f f f

f= − =+

=

−+

= =; 22

10. La luz

Page 136: 9523 gl fis_tx2_cas

142

El campo magnético es perpendicular en todo mo-mento al campo eléctrico y a la dirección de propaga-ción. Por tanto, será paralelo al eje OZ:

Hallamos la pulsación:

Determinamos el número de ondas:

Calculamos la amplitud del campo magnético:

Con esto, las ecuaciones de los campos eléctrico ymagnético serán:

54. Datos: 460 dientes; ω = 20,2 rev ⋅ s−1; d = 7 700 m

Determinamos primero el tiempo que transcurre des-de que la luz atraviesa la rueda hasta que vuelve a al-canzarla, que coincide con el que tarda la rueda enavanzar la mitad del ángulo entre dos dientes conse-cutivos:

Entonces, la velocidad de la luz es:

55. Datos: λ0 = 500 nm = 5 ⋅ 10−7 m; i = 42°; r = 25°; n1 = 1

a) Aplicamos la ley de Snell para determinar el índi-ce de refracción del material:

b) Determinamos la velocidad de la luz en el me-dio a partir de la definición del índice de refrac-ción:

Calculamos la longitud de onda en el medio:

nn

= = =⋅

= ⋅−

−λ

λλ

λ0 07

75 103 16 10; ,

m1,58

m

ncv

vcn

= = =⋅

= ⋅; ,3 10

1 9 108

8m/s1,58

m/s

n i n r nn i

r1 2 21 1 4

sen sen ;sen

sensen

= = =⋅ 22

251 58

°°

=sen

,

cst

dt

m= = =

⋅= ⋅−

Δ

Δ Δ

2 2 7 700

5 38 102 86 105

8

,, m/s

ΔΔ

t

revrevs

= = = ⋅ϕ

ω

π

π

( / ) radrad

,,

2 920

2 20 25 38 10−−5 s

� �

�E = ⋅ −−10 5 10 166 673 10sen ( , ) ( )t x SIj

B == ⋅ ⋅ −−3 33 10 5 10 166 6712 10, sen ( , ) (t x S�k II)

BEc

N o CT0

03 1

81210

3 103 33 10= =

⋅= ⋅

− −−,

m/s

kf

c cs

m= = = =⋅

⋅=

−−2 2 5 10

3 10166 67

10 1

8

π

λ

π ω,

m/s11

ω π π= = ⋅ ⋅ = ⋅ −2 2 7 96 10 5 109 10 1f Hz s,

� � �B k k= = − = =B B t k x B Bz x ysen ( ) ;0 0ω

51. Datos: λ1 = 650 nm = 6,5 ⋅ 10−7 m;λ2 = 480 nm = 4,8 ⋅ 10−7 m

52. Datos: λ1 = 760 nm = 7,6 ⋅ 10−7 m;

λ2 = 380 nm = 3,8 ⋅ 10−7 m

a) Determinamos las frecuencias correspondientes:

b) Si la velocidad del medio es de la velocidad de

la luz en el vacío,

53. Datos: f = 7,96 ⋅ 109 Hz; E0 = 10−3 N/C; dirección depropagación OX en sentido positivo; campo eléctricoparalelo al eje OY.

Como la dirección de propagación es OX, la onda seráperiódica respecto de la variable x. Al ser la propaga-ción en sentido positivo, la variable x irá afectada porun signo negativo. El campo eléctrico será en todo mo-mento paralelo al eje OY. Por tanto:

� � �E = = − = =E E t k x E Ey x zsen ( ) ;j j0 0ω

λ

λ

11 1

8

14

1

34

3 3 10

4 3 95 10

5

= = =⋅ ⋅

⋅ ⋅

=

vf

cf Hz,

,

m/s

770 10 570

34

3 3 10

4

7

22 2

8

⋅ =

= = =⋅ ⋅

− m nm

vf

cf

λm/s

77 9 10

2 85 10 285

14

27

,

,

= ⋅ =−

Hz

m nmλ

v c=34

:

34

fc

HzHz

f

11

8

714

2

3 10

103 95 10= =

⋅= ⋅

=

−λ,

m/s

7,6

cc

HzHz

λ2

8

7143 10

107 9 10=

⋅= ⋅− ,

m/s

3,8

fc

Hz11

8

7143 10

6 5 104 6 10= =

⋅= ⋅−λ ,

,m/s

mVVisible

fc

11

8

713 10

4 8 106 25 10= =

⋅= ⋅−λ ,

,m/s

m44 Hz Visible

i r

r r rr i

n i n

=

= ° − − ° = ° −

⎫⎬⎭

= ° −

=

′′

180 90 9090

1 sen 22 1 2

1

90sen ; sen sen ( )

sen

r n i n i

n i

′ = ° −

== =n i t g inn

n22

121cos ;

i r

r’90°

n1

n2

Page 137: 9523 gl fis_tx2_cas

143

Observamos que la imagen es menor que el objeto ydel mismo signo, por lo tanto, se encuentra derecha.Además, como s2 > 0, la imagen se forma por intersec-ción de las prolongaciones de los rayos reflejados y esvirtual.

60. Datos: espejo cóncavo, r < 0; AL = −3; s2 = −10 cm (ima-gen real)

Determinamos la posición del objeto a partir del au-mento lateral y la posición de la imagen:

Hallamos ahora el radio de curvatura a partir de laecuación fundamental de los espejos esféricos:

61. Datos: f2 = 8 cm (lente convergente)

a) s1 = −32 cm

Hallamos la posición de la imagen:

Determinamos el aumento lateral:

Ass

AcmcmL L= =

−= −2

1

10 732

0 33;,

,

1 1 1 1 1 1

11 1

2 2 1 2 1 2

2

1 2

f s s s s f

s

s f

= − = +

=+

;

; ,s

cm cm

cm21

132

18

10 7=

−+

=

OC

y2

y1 F

s2

s1

1 1 2

21 1

21

101

3 3

2 1

2 1

s s r

r

s s

r

cm

+ =

=+

=

−+

;

, 33

5= − cm

Ass

ssA

scm

LL

= − = − = −−

−= −2

11

21

103

3; ; ,333 cm

O Cy2

y1F

s2

s1

y cm= − ⋅2 33 33, cm

cmcm

−=

101

56. Datos: n1 = 1; n2 = 1,33; s1 = 250 m

La superficie de separación del agua con el aire cons-tituye un dioptrio plano, con n1 = 1 y n2 = 1,33. Así:

El buceador ve el avión a una altura de 332,5 m sobreel agua.

57. Datos: d = 1,5 m; s1 = 0,5 m; n1 = 1,33; n2 = 1

Calculamos la profundidad aparente del pez respecto ala superficie:

Entonces, nosotros lo vemos a una distancia de:

s2 + d = 0,38 m + 1,5 m = 1,88 m

58. Datos: r = −40 cm (espejo cóncavo); s1 = −15 cm

Calculamos la distancia focal del espejo:

Hallamos la posición de nuestra imagen:

59. Datos: y1 = 3 cm; r = 10 cm (espejo convexo);s1 = −10 cm

Calculamos la distancia focal del espejo:

Hallamos la posición de la imagen:

Calculamos el tamaño de la imagen a partir del aumen-to lateral:

Ayy

ss

y yssL = = − = −2

1

2

12 1

2

1

;

1 1 1 1 1 1

11 1

2 1 2 1

2

1

s s f s f s

s

f s

+ = = −

=−

;

; ,s

cm cm

cm21

15

110

3 33=−

=

fr

fcm

cm= = =2

102

5;

OC

y2

y1

F

s2s1

1 1 1 1 1 1

11 1

2 1 2 1

2

1

s s f s f s

s

f s

+ = = −

=−

;

; s

cm cm

cm21

120

115

60=

−−

=

fr

fcm

cm= =−

= −2

402

20;

snn

s s m m22

11 2

11 33

0 5 0 38= = ⋅ =;,

, ,

snn

s s m m22

11 2

1 331

250 332 5= = ⋅ =;,

,

10. La luz

Page 138: 9523 gl fis_tx2_cas

144

Teoría ondulatoria de Huygens: la luz consiste en lapropagación de una perturbación ondulatoria del me-dio. Requiere, por tanto, la presencia de un mediomaterial. Explica fácilmente la propagación rectilínea,la reflexión, la refracción y la doble refracción. Su ma-yor dificultad está en que todavía no se había observa-do la difracción, fenómeno típicamente ondulatorio.

Teoría ondulatoria de Fresnel: considera que la luzconsiste en ondas transversales. Además de todos losanteriores, explica nuevos fenómenos observados, ta-les como la difracción, las interferencias y la polariza-ción.

Teoría electromagnética de Maxwell: la luz no es unaonda mecánica, sino una forma de onda electromag-nética; consiste en la propagación, sin necesidad demedio material, de un campo eléctrico y otro magné-tico perpendiculares entre sí y a la dirección de propa-gación. Explica todos los fenómenos observados hastaentonces.

Naturaleza de la luz según Einstein: la luz está forma-da por fotones, pequeños corpúsculos o cuantos deenergía. Esto explica el efecto fotoeléctrico, cosa queno puede hacer ninguna de las teorías ondulatorias.

Naturaleza dual de la luz: la luz tiene una doble natu-raleza: corpuscular (fotones) y ondulatoria (ondaselectromagnéticas).

2. a) Verdadero. La imagen de una lente divergente essiempre virtual.

b) Falso. Lo que determina si una imagen es real ovirtual es si la imagen se forma por la intersecciónde los rayos luminosos o por la intersección de susprolongaciones. Un valor de s2 < 0 indica en unalente que la imagen es virtual, mientras que en unespejo significa que se trata de una imagen real.

3. La causa de la dispersión de la luz es la dependenciadel índice de refracción con respecto a la longitud deonda. Si un haz de luz blanca es refractado, cada com-ponente con una longitud de onda diferente se refrac-tará con un ángulo diferente, de manera que dará lu-gar a la dispersión.

4. Datos: f = 50 MHz = 5 ⋅ 107 Hz

E0 = 800 N/C

dirección de propagación OX en sentido positivo

dirección de oscilación OY.

a) Hallamos la longitud de onda:

b) Determinamos el período:

c) Calculamos la amplitud del campo magnético:

BEc

N CT0

01

86800

3 102 7 10= =

⋅= ⋅

−−,

m/s

Tf Hz

s= =⋅

= ⋅ −1 1

5 102 107

8

λ = =⋅

⋅=

cf m

m3 10

5 106

8

7

m/s

Por ser s2 > 0, la imagen se forma por intersecciónde los rayos emergentes y es real. Como el aumen-to lateral es menor que la unidad en valor absolu-to y negativo, la imagen será menor que el objeto einvertida.

b) s1 = −6 cm

Hallamos la posición de la imagen:

Determinamos el aumento lateral:

Por ser s2 < 0, la imagen se forma por intersecciónde las prolongaciones de los rayos emergentes y esvirtual. Como el aumento lateral es mayor que launidad en valor absoluto y positivo, la imagen serámayor que el objeto y derecha.

62. Datos: λ = 420 nm = 4,2 ⋅ 10−7 m; y1 = 18 cm = 0,18 m; L = 1 m

a) Calculamos el ángulo bajo el que se observa en lapantalla el primer mínimo:

Determinamos ahora la anchura de la rendija apartir del seno del ángulo de la primera franja os-cura, n = 1:

b) El ángulo que determina la posición del segundomínimo será:

63. Datos: f = 8 cm = 0,08 m

Hallamos el aumento de la lupa cuando el ojo está re-lajado, sin acomodación:

EVALUACIÓN

1. Teoría corpuscular de Newton: la luz está formada pordiminutas partículas que se propagan en línea recta.Explica satisfactoriamente la propagación rectilínea yla reflexión, pero no puede explicar fenómenos típica-mente ondulatorios como la refracción.

Am

fA

mmA A= = =

0 25 0 250 08

3 1,

;,,

,

sen,

,, ;α

λα= = ⋅

⋅= =

−nd

m

m2

4 2 10

2 3 100 365

7

6 221 25° ′

sen ;sen

,,

,

αλ λ

α= = =

⋅ ⋅

=

nd

dn m

d

1 4 2 100 18

2

7

33 10 6⋅ − m

t gyL

t gm

m;

,, ;α α α= = = = °1 0 18

10 18 10 12′

Ass

AcmcmL L= =

−=2

1

246

4;

1 1 1 1 1 1

11 1

2 2 1 2 1 2

2

1 2

f s s s s f

s

s f

= − = +

=+

;

; s

cm cm

cm21

16

18

24=

−−

= −

Page 139: 9523 gl fis_tx2_cas

145

7. Datos: punto remoto = 2,5 m

a) Si la persona no ve claramente los objetos situadosmás allá de 2,5 m, sufre miopía.

b) Debe usar lentes de forma que los objetos situadosen el infinito, s1 = −∞, tengan su imagen respectoa la lente en el punto remoto de la persona, s2 = −2,5 m. Imponemos esta condición para ha-llar la distancia focal de las lentes que debe usaresta persona:

c) La distancia focal es negativa. Se trata, por tanto,de lentes divergentes.

d) Calculamos la potencia:

Pf m

dioptrías= =−

= −1 1

2 50 4

2 ,,

1 1 1

11 1

11

2 51

2 2 1

2

2 1

2

f s s

f

s s

f

m

= −

=−

=

−−

;

, ∞∞

= −2 5, m

5. Datos: n1 = 2; n2 = 1

Calculamos el ángulo límite:

6.

Determinamos el ángulo límite de la superficie del

agua con el aire:

Entonces, el radio del círculo de luz será:

t g LRh

R h t g L m t g m; ( ,= = = ⋅ ° =2 48 45 2 28′)

sen,

;Lnn

L= = = °2

1

11 33

48 45′

Aire, n2 = 1

Agua, n1 = 1,33

L = 48° 45’

R

h = 2 m

sen , ;Lnn

L= = = = °2

1

12

0 5 30

10. La luz

Page 140: 9523 gl fis_tx2_cas

147

Física relativista11

El cuerpo A se moverá hacia arriba con una aceleraciónde 0,07 m/s2.

— La segunda ley de Newton.

• Datos:

Calculamos la cantidad de movimiento:

Calculamos la energía cinética:

1. SISTEMAS DE REFERENCIA

1. a) El autobús.

Sí, la fuerza que los pasajeros ejercen sobre elautobús.

b) Una fuerza ficticia que me impulsa hacia atrás.

No aparece ninguna fuerza de reacción.

2. Un sistema de referencia inercial es aquel que cum-ple la primera ley de Newton o principio de inercia,al contrario de lo que ocurre en los sistemas no iner-ciales.

Para distinguirlos es necesario observar si aparecenfuerzas ficticias, es decir, fuerzas que no tenganreacción. En este caso, estamos ante un sistema noinercial; en el caso contrario, se trata de un sistema iner-cial.

FuerzaO’

Reacción

O

Acción

E m v kg

Ec

c = = ⋅ ⋅ ⋅

=

−12

12

0 025 25000

7 81

2 1 2, ( )

,

m s

225 106⋅ J

� � � �p m p kg= = ⋅ =; ,v k0 025 25000 625m/s

��k

kg ms⋅

m g kg= = =25 0 025 25000, ; � �v k m/s

PREPARACIÓN DE LA UNIDAD

• Sí es posible, porque los estados de movimiento o reposose determinan respecto a un sistema de referencia con-creto. Por tanto, dependiendo del sistema de referencia,un cuerpo puede estar en movimiento o en reposo.

Por ejemplo, una persona sentada en un tren está en re-poso respecto del sistema de referencia del tren, peroestá en movimiento respecto de un sistema de referenciasituado en la estación.

Derivamos respecto al tiempo para hallar la velocidad:

Derivamos de nuevo para obtener la aceleración:

— La tercera ley de Newton o principio de acción y reac-ción.

La fuerza que aparece es la normal, perpendicular a la su-perficie, que impide que el cuerpo se hunda.

— Aplicamos la segunda ley de Newton a los cuerpos Ay B para calcular su aceleración y saber si se move-rán.

T m g m a

m g T m a

T m a m g

T

A A

B B

A A

− =

− =

⎧⎨⎩

= +

sen α

== −

⎧⎨⎩+ = −

m g m a

m a m g m g mB B

A A B B

sen

sen

α

α

sen

( ) ( s

a

m g m g m a m a

m m a mB A A B

A B B

α − = +

+ = een )

sen

sen

α

α

=−

+

=⋅

m g

am m

m mg

akg

A

B A

A B

25 300 1025 10

9 8 0 072 2

° −

+⋅ =, ,

kgkg kg

m

s

m

s

py

px

�N �

T

�T

�p �

p

py

px

� � �a t t( ) = − +50 6j k

� � �v j k( ) ( )t t= − + −50 3 22t

� � � �r i( ) ( )t t t t= − + −15 25 22 3j k

11. Física relativista

Page 141: 9523 gl fis_tx2_cas

148

Cuadro del margen (pág. 279)

La Tierra puede considerarse un sistema inercialen un trayecto por carretera y en el ascenso al Eve-rest. Pero no puede ser considerada como tal en eldescenso y aterrizaje de una nave espacial y en el des-plazamiento de una tormenta por Asia.

Transformaciones inversas (pág. 281)

3. LIMITACIONES DE LA FÍSICA CLÁSICA

9. Los físicos del siglo XIX suponían que el universo esta-ba lleno de éter porque creían que la luz necesitabaun medio material para propagarse. Y, como la luz lle-ga a todo el universo, este medio, el éter, debía dellenarlo todo.

10. Las transformaciones de Galileo no afectan a las leyesde Newton. Pero la física clásica considera que sí afec-tan a las leyes de Maxwel, ya que éstas son válidas úni-camente en un sistema en reposo absoluto o sistemadel éter, el único en que la velocidad de la luz es c.

11. Respuesta sugerida:

El primer esfuerzo por medir la velocidad de la luz sedebe a Galileo. Éste intentó medirla situando a dospersonas, en dos colinas distantes 1 km, cada una conuna linterna. En el momento en que destapaba la lin-terna, el otro debía destapar la suya. El primero debíamedir el tiempo transcurrido entre estas dos acciones.Este método no dio un valor razonable de la velocidadde la luz, dada la gran celeridad de ésta.

En 1676 Ole Römer calculó la velocidad de la luz ob-servando los tiempos de ocultación de los satélites deJúpiter.

Cuando la Tierra se acerca a Júpiter, el eclipse se produ-ce antes de lo esperado y, cuando la Tierra se aleja, seproduce después. Midiendo las diferencias de tiempoentre el primer caso y el segundo se puede determinarla velocidad de la luz.

La primera medida no astronómica de la velocidad dela luz fue realizada por el físico francés A. Fizeau en1849 (método de la rueda dentada). El también físicofrancés J. Foucault mejoró la medida de la velocidadde la luz en 1850 (método del espejo giratorio).

Los dos métodos anteriores se basaron en dividir la luzen destellos, después de rebotar en un espejo lejano, ymedir su desfase con los destellos incidentes.

El método actual se basa en principios similares, aun-que los destellos los proporciona un oscilador eléctri-co (método de la célula de Kerr).

12. Observación. La Tierra se mueve en el universo.

Hipótesis. La Tierra se mueve con una velocidad de-terminada respecto de un sistema en reposo absolutodenominado sistema del éter.

x x ut

y y

z y

t t

x x ut

y y

z z

t

= −

=

=

=

⎪⎪

⎪⎪

= +

=

=

== t ′

2. LA RELATIVIDAD EN LA MECÁNICA CLÁSICA

3. Ven la trayectoria igual que antes, ya que tanto el siste-ma de referencia del tren con MRU como el de la esta-ción son sistemas de referencia inerciales.

4. a) El sistema S corresponde a la persona sentada en elbanco y el S′, al observador situado en el coche.

b)

5. Utilizamos un sistema de referencia fijo en el suelo.

— No, porque hemos cambiado el sistema de referen-cia. En el nuevo sistema, el primer coche está para-do y el segundo circula a 38 km/h. Por tanto, el se-gundo coche circula a mayor velocidad que elprimero.

6. Según las transformaciones de Galileo, la aceleraciónde un cuerpo es la misma en todos los sistemas inercia-les; por tanto, la aceleración en S′ es de 10 m/s2.

7. Datos: v2 = 80 km/h; v1 = 90 km/h

a) Calculamos la velocidad relativa:

b) Si circulan en sentidos contrarios, sus velocidades son:

c) Los dos observadores medirán la misma velocidadpara un móvil en el caso de que la velocidad relati-va entre ellos sea nula.

8. Datos:

a) La velocidad del viento respecto a la Tierra será lavelocidad relativa entre el sistema de referenciaTierra y el sistema de referencia viento.

b) Primero hemos de calcular el tiempo transcurrido,que será el mismo en ambos sistemas. Para ello, su-ponemos x0 = x0′ = 0 y aplicamos la ecuación delMRU en el sistema S:

A continuación, aplicamos la ecuación del MRU,en el sistema S:

x v t x s m

x

′ ′ ′

= = ⋅ =

=

; ,340 57 3 19482

1

m/s

99482 km

x v t

txv

tm

s

=

= = =; ,20000349

57 3m/s

u v v= − = − =( )′ 349 340 9m/s m/s m/s

v = =349 340m/s; v m/s′

v v

u v v1 2

1 2

90 80

90

= = −

= − =

;

( )

km/h km/h

km/h −− − =( 80 170km/h) km/h

u v v= − = − =( )1 2 90 80 10km/h km/h km/h

X XX

S S’

Page 142: 9523 gl fis_tx2_cas

149

que sean las velocidades de la fuente y del obser-vador.

Este postulado nos indica que el valor de la velocidadde la luz, medido desde un sistema inercial, es siempreel mismo.

Estos postulados no son compatibles con la teoría deléter, ya que considera equivalentes todos los sistemasde referencia inerciales y, por tanto, no cree en la exis-tencia de ningún sistema de referencia absoluto.

— El valor de la velocidad de la luz no cambia tantosi nos dirigimos hacia la fuente luminosa como sinos alejamos de ella (segundo postulado). Asípues, no podemos aplicar las transformaciones deGalileo a la velocidad de la luz.

17. Datos: u = 0,6c

Calculamos las constantes

Transformaciones de Galileo:

x′ = x − 0,6c t

y′ = y

z′ = z

t′ = t

Transformaciones de Lorentz:

x′ = 1,25 (x − 0,6c t)

y′ = y

z′ = z

t′ = 1,25 (t − 2 ⋅ 10−9 x)

No, las transformaciones no son equivalentes.

18. No es necesario utilizar las transformaciones de Lo-rentz en la vida cotidiana, ya que las velocidades a quenos movemos son muy pequeñas comparadas con lavelocidad de la luz. Por tanto:

Entonces, las transformaciones de Lorentz se reducena las de Galileo.

19. Datos: x = 100 m; t = 10 s; u = 0,5c

Calculamos la nueva distancia y el tiempo gracias a lastransformaciones de Lorentz:

x′ = 1,155 (x − 0,5c t)

x′ = 1,155 (100 m − 0,5 ⋅ c ⋅ 10 s)

x′ = −1,73 ⋅ 109 m

ββ

γβ

= = = ⋅

=−

=

−uc c

sm

0 5 1 7 10

1

11 155

9

2

, ; ,

,

β γ= ≈ ≈uc

y0 1

ββ

γβ

= = = ⋅

=−

=

−uc c

sm

0 6 2 10

1

11 25

9

2

, ;

,

ββ

γ, :c

y

Experiencia. El experimento de Michelson-Morleyintentó medir la variación de la velocidad de la luz de-bida al movimiento de la Tierra para, de esta manera,hallar la velocidad de la Tierra respecto al sistema deléter.

Resultado. La velocidad de la luz respecto al éter nodepende del movimiento de la Tierra.

Nueva hipótesis, coherente con los resultados experi-mentales. La velocidad de la luz es constante e indepen-diente del movimiento del observador y del movimien-to de la fuente emisora. La comprobación experimentalde las hipótesis permite descartar hipótesis falsas y ela-borar otras nuevas coherentes con los datos experimen-tales.

13. El sistema de referencia S′ corresponde a la Tierra.

El sistema de referencia S′ corresponde al éter.

Por tanto, la velocidad de la luz respecto a S′ se calculade la siguiente manera:

Recorrido P → M1:

Recorrido M1 → P:

14. a) No, no puede explicarse. Si la luz es una onda pa-recida al sonido, necesita un medio material depropagación y su velocidad se suma, o se resta, a lade este medio.

b) No puede existir un sistema del éter, ya que la velo-cidad de la luz no depende del sistema de referen-cia en que se mida. Así, no es necesario un sistemade referencia privilegiado, el sistema del éter, y laluz no nos permite medir velocidades absolutas.

c) Estos resultados permiten concluir que la hipótesisinicial no era correcta. Aunque puede ocurrir quela resolución de los instrumentos de medida sea in-suficiente, que el éter se mueva con la Tierra, o quelos objetos se contraigan en el sentido del movi-miento, hay que estar abierto para aceptar nuevashipótesis que sí estén de acuerdo con los resultadosexperimentales.

4. MECÁNICA RELATIVISTA: RELATIVIDAD ESPECIAL

15. Según las transformaciones de Galileo, si dos sistemas dereferencia S′ y S se mueven con una velocidad relativa v′,cada uno mide una velocidad de la luz diferente: si el pri-mero mide una velocidad c, el segundo mide una veloci-dad c + u. Por tanto, las transformaciones de Galileo nodejan invariante el valor de la velocidad de la luz.

16. Postulado 1. Las leyes de la física son las mismas en to-dos los sistemas de referencia inerciales.

Este postulado señala que todos los sistemas de refe-rencia inerciales son equivalentes en la descripción decualquier fenómeno físico.

Postulado 2. La velocidad de la luz es la misma en to-dos los sistemas de referencia inerciales, cualesquiera

� � � � �v v′ = − − = +( )u c u

� � � � �v v′ = − = −u c u

11. Física relativista

Page 143: 9523 gl fis_tx2_cas

150

Calculamos Δt′ para el viajero:

24. Datos:

a) Calculamos el período visto en la Tierra:

b) El período propio se mide desde un sistema soli-dario con el péndulo; en nuestro caso, el tren. Portanto, el período propio será de 8,4 s.

25. Datos: u = 0,6c; Δx′ = 340 m; Δy′ = 21 m

Calculamos las constantes β y γ:

Calculamos las longitudes que mediría un observadorfijo en la Tierra:

,

26. Datos: u = 0,8c; Δx = 150 m; Δy = 18 m

Calculamos las constantes β y γ:

Calculamos las longitudes que mediría un observadorfijo en la Tierra:

Δy′ = Δy = 18 m de alto

Δx′ = γ Δx = 1,667 ⋅ 150 m = 250 m de largo

27. a) Datos:

Calculamos v′ a partir de la fórmula relativista deadición de velocidades:

Por tanto:

b)

Calculamos v′ a partir de la fórmula de la adiciónrelativista de velocidades:

� �u c c′ = − =( , , ) ; ( , , )0 9 0 0 0 9v

�v ′ = ( , , )0 994 0c

vv u

v u

c

c cc cx

x

x′ =

−=

− −

−−

1

0 9 0 9

10 9 0 9

2

, ( , ), ( , )

ccv cx

2

0 994′ = ,

β = =uc

para las dos naves0 9,

� �u c c= − =( , , ) ; ( , , )0 9 0 0 9 0v

β γβ

= = =−

=0 8

0 81

11 667

2

,, ; ,

cc

Δ Δ

Δ Δ Δ

y y m de alto

x x x

= =

= =

21

1 11 25

3;,γ

440 272 argm m de l o=

β γβ

= = = =−

=uc

cc

0 60 6

1

11 25

2

,, ; ,

Δ Δ Δt t t s s= = ⋅ =γ ; , ,′ 1 67 8 4 14

γβ

=−

=1

11 67

2,

Δt s u cuc

= = = =8 4 0 8 0 8, ; , ; , ;β

Δ Δt t s s

ss

′ = = ⋅ =

⋅ =

γ ,

minm

1 4 300 420

420160

7 iin

La distancia recorrida es el valor absoluto del resulta-do: 1,73 ⋅ 109 m

Calculamos el tiempo:

20. a) Sí, ya que el tiempo es el mismo para todos los sis-temas de referencia.

b) No, ya que el tiempo depende del sistema de refe-rencia.

21. Si la velocidad del globo es cercana a la velocidad de laluz, desde la tierra no verán simultáneamente los re-lámpagos. Primero verán el que ha caído bajo el globoy, después, el que ha caído sobre el globo.

22. Datos:

a) El observador fijo en la Tierra ve los rayos simultá-neamente, pues la velocidad de la luz es constantey ambos rayos recorren la misma distancia.

b) El vagón situado en el sistema de referencia S′ semueve hacia el relámpago anterior y se aleja delposterior. Por ello, verá antes el relámpago ante-rior. Ambos relámpagos no serán simultáneos paraél.

Si el primer relámpago llega al pasajero cuando hapasado un tiempo t′, ha recorrido un espacio: x′ = u t′;por tanto, el relámpago ha recorrido el espacio:d − u t1

Calculamos el tiempo que tarda el relámpago enrecorrer este espacio:

En cambio, el relámpago posterior ha recorrido unespacio:

x + u t2

siendo t2 el tiempo que tarda en llegar el segundorelámpago.

Calculamos t2 a partir de la expresión:

La diferencia de tiempo que percibe O′ es:

t2 − t1 = 5 ⋅ 10−7 s − 5,56 ⋅ 10−8 s

t2 = 4,4 ⋅ 10−7 s

23. Datos:

γβ

=−

=1

11 4

2,

Δt suc

= = = =5 0 7min ; , ;β

x u t c t

tx

c ud

c cm

+ =

=−

=−

=⋅ ⋅, ,

2 2

2 80 830

0 2 3 10 m/ss

t s275 10= ⋅ −

d u t c t d c t u t d c u t

t

− = = + = +; ; ( )1 1 1 1 1

11 8

1

0 830

1 8 3 10

5 56 10

=+

=+

=⋅ ⋅

= ⋅

dc u

dc c

m

t

, ,

,

m/s−−8 s

1 6012

30 0 8′′

= = = =; ; ,m d m u c

t ss

mm s′ = − ⋅ ⋅

⎛⎝⎜

⎞⎠⎟

=−1 155 10 1 7 10 100 11 59, , ,

Page 144: 9523 gl fis_tx2_cas

151

ramos una pelota con velocidad c desde un trenque viaja a la velocidad de la luz c, la pelota saldríadisparada con velocidad 2c.

Mecánica relativista:

— A causa de las transformaciones de Lorentz, la adi-ción de velocidades relativista no es suma vectorialde las velocidades iniciales. No podríamos, en estecaso, tirar una pelota con velocidad c porque nin-gún cuerpo con masa puede alcanzarla.

29. Al acelerar 1 g de oro hasta una velocidad de 0,9c, lamecánica relativista determina que un observador enreposo que vea el oro moviéndose a 0,9c no lo verácon una masa de 1 g, sino con una de 2,3 g. Pero estono significa que aumente su número de átomos, queseguirá siendo siempre el mismo, sino que la masa decada átomo, vista en movimiento a esa velocidad, au-menta en un factor 2,3.

30. a) Datos: me = 9,1 ⋅ 10−31 kg; v = 0,9c

Calculamos β y γ, y la masa vista desde el observa-dor en reposo:

b) Datos: v = 0,99c

Determinamos β y γ para hallar la masa relativista:

c) Datos: v = 250 m/s

Para calcular γ, tomaremos c = 3 ⋅ 108 m⋅s−1:

La velocidad en este caso es mucho más pequeñaque la velocidad de la luz, por lo que los efectos rela-tivistas son despreciables y la masa es prácticamente lamisma que en reposo.

31. Datos: m0 = 270 kg; v = 0,8c

a) Consideramos el Sol en reposo y el meteorito mo-viéndose respecto a él a v = 0,8c. Las constantes β yγ serán:

y la masa del meteorito:

m = γ m0 = 1,667 ⋅ 270 kg = 450 kg

b) La masa propia del meteorito es su masa en re-poso:

m0 = 270 kg

β γβ

= = =−

=vc

0 81

11 667

2, ; ,

β

γβ

= =⋅

⋅ ⋅= ⋅

=−

=

−−v

c250

3 108 33 10

1

1

1

8 17

2

,m s

m s

11 9 1 10 31; ,m m kge≈ = ⋅ −

β γβ

γ

= = =−

=

= = ⋅

vc

m me

0 991

17 089

7 089 9

2, ; ,

, ,, ,1 10 6 45 1031 30⋅ = ⋅− −kg kg

β γβ

γ

= = =−

=

= = ⋅

vc

m me

0 91

12 294

2 294 9

2, ; ,

, ,11 10 2 09 1031 30⋅ = ⋅− −,kg kg

La velocidad de la nave A, vista desde B, será (0,9c,0,392c).

28. Respuesta sugerida:

Aspectos sorprendentes de la mecánica relativista:

— La longitud de los cuerpos depende del sistema dereferencia en que se miden.

— El tiempo también depende del sistema de referen-cia en que lo midamos.

— La suma de velocidades relativista no es una sumade las componentes de las velocidades iniciales.

— La simultaneidad depende del sistema de referen-cia desde el que miro los sucesos.

Simultaneidad

Mecánica clásica:

— La simultaneidad no depende del sistema de refe-rencia desde el que observemos los sucesos, ya queel tiempo es el mismo para todos los observadores.

Mecánica relativista:

— Dos sucesos simultáneos en un sistema de referen-cia no tienen por qué serlo en otro.

Dilatación del tiempo

Mecánica clásica:

— El transcurso del tiempo es independiente del ob-servador que lo mida.

Mecánica relativista:

— La dilatación del tiempo es consecuencia de lastransformaciones de Lorentz.

Contracción de longitudes

Mecánica clásica:

— La longitud de un cuerpo no depende del observa-dor que lo mida.

Mecánica relativista:

— Debido a las transformaciones de Lorentz, un cuer-po situado en un sistema inercial parece contraídosi se observa desde otro sistema de referencia iner-cial en movimiento respecto al primero, aunqueúnicamente en la dirección del desplazamiento re-lativo.

Adición de velocidades

Mecánica clásica:

— La adición de velocidades se efectúa como sumavectorial de las velocidades iniciales. Por tanto, si ti-

vv u

v u

c

cc

c

vv

xx

x

y

y

=−

−=

−⋅

=−

1

0 9 0

10 0 9

1

2 2

,,

β22

2

2

21

0 9 1 0 9

10 0 9

0

( )−

=−( )

−⋅

=

v u

c

c

c

cv

x

y

, ,

,

,′ 3392c

11. Física relativista

Page 145: 9523 gl fis_tx2_cas

152

35. Al proporcionar energía a un cuerpo, éste la utilizapara aumentar su velocidad y su masa. Así, pareceposible aumentar la masa de un cuerpo proporcio-nándole energía. Sin embargo, cada vez necesitará másenergía para aumentar su masa y su velocidad:

m = γ m0

36. m01 = m02 = 0,003 kg

a) Calculamos la masa relativista antes del choque:

m1 = m2 = γ m0 = 1,67 ⋅ 0,003 kg

m1 = m2 = 5 ⋅ 10−3 kg = 5 g

b) Calculamos la energía de las dos partículas antesdel choque:

E1 = Δm1 c2 = 0,002 kg (3 ⋅ 108 m ⋅ s−1)2

E1 = 1,8 ⋅ 1014 J

E2 = Δm2 c2 = 1,8 ⋅ 1014 J

c) La masa final será la suma de las masas relativistas:

FÍSICA Y SOCIEDAD

a) Respuesta sugerida:

El problema principal es que la idea puede ser falsa oparcialmente incorrecta y no ajustarse, por tanto, a larealidad.

b) Respuesta sugerida:

No, las teorías científicas de Newton fueron un granavance para la ciencia y para la humanidad. De hecho,las teorías de Einstein son las únicas válidas para loscuerpos con velocidades próximas a la de la luz perose reducen a las ecuaciones de Newton para velocida-des más pequeñas.

c) Respuesta sugerida:

Es importante destacar que, por mucho que una teo-ría se ajuste a los hechos experimentales, siempre pue-de existir un nuevo experimento o descubrimientoque exija una teoría más completa. También ha de te-nerse en cuenta que es necesario que esta teoría máscompleta incluya la anterior como caso particular.

RESOLUCIÓN DE EJERCICIOS Y PROBLEMAS

37. Datos:

— Aplicamos la ley de adición de velocidades clásica:

v v u c c c

u cx x x= + = + =

= + =

0 7 0 8 1 5

1 5 0

, , ,

( , , )� � �v v

� �u c c= =( , , ) ; ( , , )0 7 0 0 8 0v ′

ME

c

E E

c

m c m c

cM m m g

0 21 2

21

22

2

2

0 1 2 5 5

= =+

=+

= + = + gg g= 10

v v cvc1 2

1 2 2 2

0 8 0 8

1

1

1

1 0 81

= = = =

= =−

=−

=

, ; ,

,

β

γ γβ

,,67

c) Si queremos que la masa en movimiento parezca eldoble de la masa en reposo, debemos imponer m = 2 m0, de donde se deduce que γ = 2. Entonces,obtendremos la velocidad a partir de β:

Despejando β:

Como sabemos que:

v = β c = 0,866c

Es decir, la masa parecerá el doble de la masa en repo-so si se mueve a un 86,6 % de la velocidad de la luz.

32. Datos: m0 = 5 000 kg; u = 0,9c;

Calculamos la masa relativista:

m = γ m0 = 2,29 ⋅ 5 000 kg = 11 450 kg

Calculamos el incremento de masa:

Δm = m − m0 = 11 450 kg − 5 000 kg = 6 450 kg

— Aplicamos la expresión de la energía cinética parahallar la energía suministrada:

Ec = Δm c2 = 6 450 kg (3 ⋅ 108 m ⋅ s−1)2

Ec = 5,8 ⋅ 1020 J

33. Datos: me = 9,1 ⋅ 10−31 kg; u = 0,3c; β = 0,3

Calculamos el incremento de masa:

m = γ m0 = 1,048 ⋅ 9,1 ⋅ 10−31 kg

m = 9,54 ⋅ 10−31 kg

Δm = m − m0 = 4,4 ⋅ 10−32 kg

El incremento de energía será:

E = Δm c2 = 3,96 ⋅ 10−15 J

El incremento de energía cinética clásica será:

Las dos energías son parecidas debido a que la veloci-dad final del electrón es pequeña respecto a la velo-cidad de la luz.

34. La energía de un cuerpo en reposo no es nula desde elpunto de vista relativista, ya que tiene una energía de-bida a su masa en reposo e igual a m0 c2.

ΔΕ

ΔΕ

c m v m c

c kg

c c= =

= ⋅ ⋅ −

12

12

0 3

12

9 1 10

2 2

31

( , )

, ( , )

,

0 3 3 10

3 69 10

8 1 2

15

⋅ ⋅ ⋅

= ⋅

m s

ΔΕc J

γβ

=−

=−

=1

1

1

1 0 31 048

2 2,,

γβ

=−

=−

=1

1

1

1 0 92 29

2 2,,

β = =uc

0 9, ;

β =uc

:

βγ

= − = − =11

11

20 8662 2 ,

γβ

=−

1

12

2

Page 146: 9523 gl fis_tx2_cas

153

b) Calculamos el tiempo visto desde un observadorde la estación, Δt′. Tenemos en cuenta que éste ha-brá observado la contracción relativista del tiemposegún la fórmula relativista:

41. Datos: E = 500 N/C; me = 9,1 ⋅ 10− 31 kg; Q = −1,6 ⋅ 10−19 C

a) Calculamos el valor de la fuerza eléctrica:

b) Aplicamos el teorema del impulso en la direccióndel movimiento:

c) Aplicamos de nuevo el teorema del impulso, te-niendo en cuenta que en la mecánica relativista lamasa varía con la velocidad:

Despejamos β y sustituimos los datos del enuncia-do:

El electrón alcanza el 71 % de la velocidad de laluz.

42. E = 1 000 N/C; t = 1 ms = 10−3 s; mp = 1,67 ⋅ 10−27 kg; e = 1,6 ⋅ 10−19 C

a) Calculamos el valor de la fuerza eléctrica:

F = Q E = 1,6 ⋅ 10−19 C ⋅ 1 000 N/C

F = 1,6 ⋅ 10−16 N

b) Para calcular la velocidad final del protón aplica-mos de nuevo el teorema del impulso, teniendoen cuenta que en la mecánica relativista la masavaría con la velocidad:

β

β

=

+

=

+⋅ ⋅−

1

1

1

19 1 10 3 10

02 2

2 2

31 2 8

m c

F t

kg( , ) ( )

( ) ( , )

,m s⋅

⋅ ⋅

=−

− −

1 2

17 2 6 28 10 3 4 10

0 71

N s

F t m v donde v c y m m

F tm c

;= = =

=

β γ

β0

0

1 −−=

−β

β

β2

2 2

02 2

2

21;

F t

m c

F t p m c m v F t m c

tm c

F

;

,

= = − =

= =⋅

Δ 0 0 0 0

0 9 1 100 3 10

8 103 4 10

31 8 1

17

6

− −

⋅ ⋅ ⋅

= ⋅,

kg

Nt s

m s

F

F

= = − ⋅ ⋅

= ⋅

Q E CNC

N

,1 6 10 500

8 10

19

17

Δ

Δ Δ

t s

t t s s

=

= − = ⋅ − =

6 4

1 6 4 1 0 95 22 2

,

, ,β

Δ Δ

Δ

Δ

x x m

x m de l o

y

= − = ⋅ −

=

,

, arg

1 2 1 0 95

0 6

2 2β

′′ = =Δ y m de ancho2

Por tanto, el electrón saldría despedido a una velo-cidad mayor que la de la luz.

— Aplicamos la adición relativista de velocidades:

Por tanto, la velocidad respecto al laboratorio es:

38. Datos:

Debemos hallar la velocidad v del módulo espacial res-pecto a la Tierra:

39. Datos: Δt′ = 2,6 ⋅ 10−8 s; v = 0,9c; u = v = 0,9 c; β = 0,9

a) Aplicamos la dilatación temporal:

Calculamos la distancia recorrida:

x = v Δt = 0,9c ⋅ 6,0 ⋅ 10−8 s

x = 0,9 ⋅ 3 ⋅108 m ⋅ s−1 ⋅ 6,0 ⋅ 10−8 s = 16,2 m

b) Desde el punto de vista del pion, el laboratorio semueve a 0,9c. Para determinar la distancia recorri-da por el pion en S′ aplicamos la contracción delongitudes:

40. Datos: v = 0,95c; β = 0,95; Δx = 2 m; Δy = 1 m; Δt = 6,4 s

a) Aplicamos la fórmula de contracción de longitu-des:

x x m m′ = ⋅ − = ⋅ − =1 16 2 1 0 9 7 12 2β , , ,

ΔΔ

Δtt

ts

=−

=⋅

−= ⋅

−′

1

2 6 10

1 0 96 0 10

2

8

2β;

,

,, −−8 s

vv u

v u

c

cc

c

c

v

xx

x

y

=+

+=

+

+⋅

=

=

′′

1

0 0 8

10 8 0

0 8

2 2

,,

,

vv

v u

c

vv u

cvy

xy

x;1

11

2

2

2

−= −

⎛⎝⎜

⎞⎠⎟

′ yy

y y

x

v v

v u

c c

c c

c,

, ,

1

1

10 4

10 8 0 8

2

2

2

2

=−

−=

−⋅

β

β′

11 0 80 24

0 8 0 24

2−=

= =

,,

( , ) ( , , , )

c

v v c cx y�v

� �u c cuc

= = =( , , ) ; ( , , ) ;0 8 0 0 0 4v ′ β == 0 8,

�v = ( , , )0 962 0c

vv u

v u

c

vv u

cv u

v

xx

x

xx

x

x

=−

−⎛⎝⎜

⎞⎠⎟

= −

1

1

2

2

vvv u

cv u

v u v vv u

c

v u v

xx

x

x x xx

x x

′′

2

2

1

= −

+ = +

+ = −−⎛⎝⎜

⎞⎠⎟

=+

+=

+

v u

c

vv u

v u

c

c c

x

xx

x

′′

, ,

2

21

0 8 0 7

1 ++⋅

=0 8 0 7

0 962

2

, ,,

c c

c

c

11. Física relativista

Page 147: 9523 gl fis_tx2_cas

154

b) Calculamos la masa relativista del protón a partirde su energía relativista: E = m c2

c) Hallamos la energía cinética de cada protón inci-dente a partir del incremento de masa:

ΔEc = Δm c2 = (3 mp − mp) c2 = 2 mp c2

ΔEc = 3 ⋅ 10−10 J

d) Para hallar la velocidad del protón igualamos lamasa obtenida con la definición de masa relati-vista:

de donde β = 0,94. Por tanto, v = 0,94c.

EJERCICIOS Y PROBLEMAS

45. Decimos que un coche se pone en marcha cuando semueve respecto a un sistema de referencia fijo en elsuelo. Como nosotros somos solidarios con este siste-ma de referencia por estar parados respecto a él, deci-mos que el otro coche es el que se mueve.

No se vulnera el principio de relatividad, puesto quela afirmación que es el otro coche el que se mueve escierta en el sistema de referencia fijo en el suelo.

46. Por el principio de relatividad de Einstein, la veloci-dad de la luz no depende del observador; por tanto,no depende de su dirección ni de la velocidad relativaentre fuente y receptor.

47. Las principales consecuencias de las transformacionesde Lorentz son la dilatación del tiempo, la contrac-ción de longitudes y la relativización del concepto desimultaneidad.

48. Para que un cuerpo alcanzara la velocidad de la luz de-beríamos aplicarle una fuerza infinita, según la mecá-nica relativista.

49. El principio de equivalencia entre la masa y la energíaafirma que la masa y la energía son dos aspectos delmismo fenómeno y que, por tanto, la masa puede pre-sentarse en forma de energía, y viceversa.

50. Datos:

a) Aplicamos las transformaciones de Galileo paracalcular la velocidad de la motocicleta respecto ala autopista:

La velocidad de la motocicleta 10 s después es:�

� � �v

v v

′2

2 2

12

12 100

=

= + = +

k

u k k

m/h

m/h m/h��v2 112= km/h

�v ′2

� � �� � �

v v

v v

v

′1

1 1

1

10 100

= −

= + = +

u

u k km/h m/h

== 110 km/h

Δt s h v k= = ⋅ =−10 2 78 10 123, ; ′2 m/h

� �u = =100 10;km/h km/hv ′1

m mm

pp

= =−

− =31

1132

2

ββ;

mE

c

m c

cJp

= = = ⋅ −2

2

2273

5 01 10,

F t = m v, donde v = c β y m = β m0

Despejamos β y sustituimos los datos del enunciado:

Es decir, el cuerpo alcanza el 30 % de la velocidadde la luz.

43. Datos: me = 9,1 ⋅ 10−31 kg

a) La energía mínima se obtiene cuando todas laspartículas quedan en reposo después del choque:

Einicial = Efinal = 4 me c2

Einicial = 4 ⋅ 9,1 ⋅ 10−31 kg (3 ⋅ 108 m ⋅ s−1)2

Einicial = 3,28 ⋅ 10−13 J

La energía relativista de cada electrón es:

b) Calculamos la masa relativista de un electrón a par-tir de su energía relativista:

Por lo tanto, es el doble de la masa en reposo.

c) Para hallar la velocidad del electrón, igualamos lamasa obtenida con la definición de masa relativista:

de donde β = 0,87. Es decir, la velocidad de cadaelectrón es igual al 87 % de la velocidad de la luz.

44. Datos: mp = 1,67 ⋅ 10−27 kg

a) La energía mínima se obtiene cuando las seis par-tículas quedan en reposo. Aplicando la conserva-ción de la energía:

Einicial = Efinal = 6 mp c2

Einicial = 6 ⋅ 1,67 ⋅ 10−27 kg (3 ⋅ 108 m ⋅ s−1)2

Einicial = 9 ⋅ 10−10 J

La energía mínima ha de ser de 9 ⋅ 10−10 J.

La energía mínima de cada protón ha de ser, portanto:

Em c J

Jp= =

⋅= ⋅

−−

6

29 10

24 5 10

2 1010,

m mm

= =−

− =21

1 0 500

2

2; ,β

β

E m c

mE

c

m c

cm kge

e

=

= = = = ⋅ −,

2

2

2

2302

2 1 82 10

EE J

Jinicial= =⋅

= ⋅−

23 28 10

21 6 10

1313,

,

β

β

=

+

=

+⋅ ⋅−

1

1

1

11 67 10 3 10

02 2

2 2

27 2

m c

F t

kg( , ) ( 88 1 2

16 3 21 6 10 10

0 3)

( , )

,m s⋅

⋅ ⋅

=−

− −N s

F tm c F t

m c;=

−=

−0

2

2 2

02 2

2

21 1

β

β

β

β

Page 148: 9523 gl fis_tx2_cas

155

53. Datos:

a) Calculamos la velocidad del bote visto desde la orilla:

b) Utilizamos las leyes de la cinemática para calcularel tiempo que tarda en atravesar el río:

54. Datos: d = 9 años luz = 9c ⋅ año; v = 0,8c;

a) Calculamos el tiempo que tarda para un observa-dor de la Tierra:

vdt

tdv

c ac

a= = =⋅

=;,

,90 8

11 25

β γβ

= =−

=−

=0 81

1

1

1 0 81 67

2 2, ;

,,

x x v t

tx x

vkm

kh

x

x

= +

=−

= = =

0 0

0

0

24

12

30 minm/h

� �

�v v

v

r= + = −

= + = + −

v

v v

b

x y

( , )

( ) ( ) (

4 3

42 2 2

km/h

33

5

2) km/h

km/h�v =

Y

X

Bote

Corriente del río

1 2 3 4= = − = −; ;km k k� � � �v j vr bm/h i m/h

d u t d u t

d v u t

d d v u t

1 1 2

3 1

3

= =

= −

+ = +

;

( )

( ) ( 22 1

1 2 2 1

1

⎬⎪

⎭⎪

− + = + −

t

v u t u t v u t t

v t

)

( ) ( ) ( )

uu t u t v t v t u t u t

v t v t

t

1 2 2 1 2 1

1 2

2

2

+ = + −

=

= 22 2 15 301 2 2

2

; min ; mint t t

d u t

= ⋅ =

= ;min

min

; ,um

su= =

1 000

3060

1

0 555

1

301

60

2min

min

m/s

ukm

hkmh

= =

t2

d2

d3

d + d3

d1

d

t1Puente

t1

u

y

b) Calculamos la aceleración de la motocicleta respec-to al automóvil:

Utilizamos las transformaciones de Galileo para ha-llar la aceleración respecto a la autopista:

c) Utilizamos las ecuaciones de la cinemática para cal-cular el tiempo que tarda en superar la velocidadlímite de la autopista.

51. Datos: m = 10 kg

Aplicamos las transformaciones de Galileo:

x′ = x − u t = 10 t + t2 − 10 t = t2

y′ = y = 8 t − t2

Por tanto:

Derivamos el vector posición para calcular la velocidad:

Calculamos v′ utilizando las transformaciones de Galileo:

Calculamos la aceleración, que será la misma para losdos sistemas de referencia:

Calculamos la fuerza, que será igual para los dos obser-vadores:

52. Datos: t1 = 15 min = 900 sd = 1 km = 1 000 m

� � ��

F F m/s

F

2′

= = = −

= −

m a kg ( , )

( , )

10 2 2

20 20 NN

��

��

addt

m

s

addt

m

s

= = −

= = −

v

v

( , )

( , )

2 2

2 2

2

2′′

� � ��

v v

v

= − = + − −

= −

u t t

t

( , ) ( , )

( ,

10 2 8 2 10 0

2 8 2tt) m/s

�v( ) ( , )t t t= + −10 2 8 2 m/s

�r( ) ( , )t t t t m= −2 28

��r

m/s

( ) ( , )

( , )

t t t t t

u

= + −

=

10 8

10 0

2 2

� � �v v v0= +

=−

=; ,

t

tv v

aen nuestro caso v0 1200

112 719

1200

kmh

v k a k

tk

= =

=

;m/h m/h

m

2

//h m/hm/h

= =

112719

0 01 40,

kk

t h s

� �a a= = =′ 719 0 062 2

km

h

m

s,

at

k k

h′ =

−=

⋅=−

� �v v2 1

Δ

12 10

1073

m/h m/h

2,78119

7191

3600

1 0001

0

2

2

2

2

km

h

km

h

h

s

mkm

⋅ ⋅ =( )

,006 2

m

s

11. Física relativista

Page 149: 9523 gl fis_tx2_cas

156

Aplicamos la fórmula de adición relativista de veloci-dades:

58. Datos: m = 9m0

La expresión de la masa relativista es:

Por lo tanto, comparando con la expresión anterior:

Esto significa que la velocidad ha de ser:

u = 0,994c

No, porque la masa inicial no se anula. Sólo dependedel factor en que incrementemos la masa.

59. La velocidad de un cuerpo en el sistema S′ es:

A partir de las ecuaciones de transformaciones de Lo-rentz sabemos que:

La velocidad en S es, por lo tanto:

Dividimos el numerador y el denominador por t:

De la misma manera podemos deducir el valor de v′ ya partir de las transformaciones de Lorentz:

y y

t tc

x

vyt

y

tc

x

y

x

′′

=

= −⎛⎝⎜

⎞⎠⎟

= =−

⎛⎝⎜

⎞⎠⎟

=

γβ

γβ

tttt c

xt

vv

cv

v

v u

c

xy

x

y

x

γβ

γβ

β

=−

⎛⎝⎜

⎞⎠⎟

=−

−′

1

1

1

2

22

v

xt

utt

tt c

xt

v u

cv

v uu v

c

xx

x

x′ =−

−=

−=

−β β

1 1 2

vxt

x ut

tc

xx′

′= =

−⎛⎝⎜

⎞⎠⎟

γ

γβ

( )

x x u t

t tc

x

= −

= −⎛⎝⎜

⎞⎠⎟

γ

γβ

( )

vxtx′

′=

m m m= =−

− = − =⎛⎝⎜

⎞⎠⎟

=

91

1

119

119

0 2 0

2 22

β

β β

β

;

1119

0 9942

−⎛⎝⎜

⎞⎠⎟

= ,

m m m= =−

γβ

0 2 01

1

vv u

v u

c

c cc c

c

xx

x′ =

−=

−⋅

=1

0 6 0 4

10 6 0 94

0

2 2

, ,, ,

,,26c

Calculamos el tiempo para un observador de lanave:

Desde el punto de vista del astronauta, la distanciaTierra-Sirio será inferior a la distancia propia del sis-tema. Por ello, la distancia recorrida también serámenor:

55. Datos: Δx′ = 115 cm = 1,15 m; Δy′ = 89 cm = 0,89 m;

v = 0,8c; β = 0,8;

a) Calculamos las dimensiones de la alfombra medi-das por B:

b) Calculamos el tiempo que observa A si para B hatranscurrido 1 min:

Δt = 1 min

Δt′ = γ Δt = 1,67 ⋅ 1 min = 1,67 min

Por tanto, el genio A mide:

56. a)

Calculamos la velocidad de B medida desde A:

b)

c)

El módulo de la velocidad será:

57. Datos: � �v x = =( , , ; ( , , )0 6 0 0 4 0c u c

v v v c cx y′ = ( ′ ′) ( ) , , ,2 2 2 20 8 0 54 0 965+ = + =

vv u

v u

c

c

c

c

c

v

xx

x′

=−

−=

−⋅

= −1

0 0 8

10 0 8

0 8

2

2

2

,

,,

yyy

x

v

v u

c

cc

c

=−

−=

−⋅

= −1

1

0 9 1 0 8

10 0 8

02

2

2

2

β , ,,

,554c

� �v = =( , , ) ; ( , , )0 0 9 0 8 0c u c

vv u

v u

c

c c

c

c

xx

x′ =

−=

− −

−⋅ −1

0 9 0 8

10 8 0 9

2

2

, ,

, ( , )22

0 988= , c

� �u c c= = −( , , ) ; ( , , )0 8 0 0 9 0v

vv u

v u

c

c c

c

c

xx

x′ =

−=

−⋅

=1

0 9 0 8

10 8 0 9

0 3

2

2

2

, ,

, ,, 557c

� �u c c= =( , , ) ; ( , , )0 8 0 0 9 0v

851 67

51, min minpulsaciones pul

=

Δ Δ

ΔΔ

y y m

xx m

m

= =

= = =

0 89

1 151 67

0 69

,

,,

γβ

=−

=−

=1

1

1

1 0 81 67

2 2,,

ΔΔ

xx a

a′ = = =γ

9 11 67

5 4 1. .

,, . .

tt

a a′ = = =γ

11 251 67

6 75,

,,

Page 150: 9523 gl fis_tx2_cas

157

Efectuamos ahora los cálculos desde el punto devista relativista:

Despejamos β:

La energía es debida tanto a la masa en reposocomo a la diferencia de potencial. Por tanto:

c) Calculamos la masa relativista:

Por tanto:

m = 1,783 m0

Calculamos la energía relativista:

62. Datos: p = m v = γ m0v; E = m c2 = γ m0 c2

Elevamos al cuadrado la energía:

E2 = m2 c4 = γ2 m02 c4

Como

Em c

E E m c2 02 4

22 2 2

02 4

1=

−− =;

ββ

γβ

γβ

=−

=−

1

1

1

12

22;

E m c m c m c

E m c

E

= = −

= −

Δ ,

( , )

20

20

2

02

1 783

1 783 1

== =

= ⋅ ⋅ ⋅−

m c m c

E kg

,

, , (

20

2

31

1 783

1 783 9 1 10 3 100

1 46 10

1 46 101

8 1 2

13

13

)

,

,

m s⋅

= ⋅

= ⋅

E J

E JMeVV

JMeV

1 602 100 91213,,

⋅=−

m mmm

mme

= =

=−

=−

=

γ γ

β

;

,,

00

2 2

1

1

1

1 0 8281 783

E m c q V

m c

m c q V

= +

= −+

⎝⎜

⎠⎟

= −

02

02

02

2

1

19

Δ

Δβ

β,, ( )

, ( ) ,

1 10 3 10

9 1 10 3 10 1 6 10

31 8 2

31 8 2

⋅ ⋅ ⋅

⋅ ⋅ ⋅ + ⋅

− −− ⋅

⎝⎜

⎠⎟

=

19

2

400000

0 828β , c

β = −⎛

⎝⎜

⎠⎟1 0

2 2m c

E

m m cm c

= =−

γβ

02 0

2

21

Ec m v

vEc

mJ

kg

=

= =⋅ ⋅

12

2 2 6 41 10

9 1 10

2

14

31

,

,

vv c= ⋅ =3 75 10 1 2508, ,m/s

60. Datos: e = −1,6 ⋅ 10−19 C; me = 9,1 ⋅ 10−31 kg; v = c = 3 ⋅ 108 m/s

a) Consideramos que toda la energía proporcionadase utiliza en aumentar la velocidad:

Despejamos el incremento de potencial:

b) Aplicamos la expresión relativista de la energía:

Despejamos β:

La energía es debida tanto a la masa en reposocomo a la diferencia de potencial aplicada; por tan-to:

c) Calculamos la masa relativista del electrón:

Por lo tanto: m = 1,5 me

61. Datos: ΔV = 400 000 V

a) Calculamos la energía cinética relativista y observa-mos que el resultado coincide con la energía ciné-tica clásica:

b) Calculamos la velocidad que adquiere desde elpunto de vista clásico:

Ec m c q V m c m c

Ec q V

= = + −

= = ⋅ −

Δ Δ

Δ

( )

,

20

20

2

11 6 10 99

14

14

400000

6 41 10

6 41 10

,

,

C V

Ec J

Ec J

= ⋅

= ⋅

,,

1

1 602 100 419

eV

JMeV

⋅=−

mm

mme

e

e

= =−

=−

β ( , ),

1

1

1

1 0 7451 5

2 2

E m c q V

m c

m c q V

= +

= −+

⎝⎜

⎠⎟

= −

02

02

02

2

1

19

Δ

Δβ

β,, ( )

, ( ) ,

1 10 3 10

9 1 10 3 10 1 6 10

31 8 2

31 8 2

⋅ ⋅ ⋅

⋅ ⋅ ⋅ + ⋅

− −− ⋅ ⋅

⎝⎜

⎠⎟

=

19 5

2

2 559 10

0 745

,

, , tan ;β por to ,v c= 0 745

β = −⎛

⎝⎜

⎠⎟1 0

2 2m c

E

E m c m c= =−

γβ

02

2 021

1

ΔVm v

qkg

= =⋅ ⋅ ⋅

− −, ( )

,

2 31 8 1 2

29 1 10 3 10

2 1 6

m s

⋅⋅

= ⋅

−10

2 6 10

19

5,

C

V VΔ

E Ec

q V m v

proporcionada =

=Δ12

2

11. Física relativista

Page 151: 9523 gl fis_tx2_cas

158

Luego la energía relativista de cada electrónserá:

b) Calculamos la masa relativista de un electrón apartir de su energía relativista:

c) Hallamos la energía cinética de cada protón inci-dente a partir de su incremento de masa:

Ec = Δm c2 = (4 me − me) c2 = 3 me c2

Ec = 3 ⋅ 9,1 ⋅ 10−31 kg (3 ⋅ 108 m⋅s−1)2

Ec = 2,5 ⋅ 10−13 J = 1,5 MeV

Para hallar la velocidad del electrón igualamos lamasa obtenida con la definición de masa relati-vista:

de donde β = 0,97. Por tanto, su velocidad será v = 0,97c.

65. Las fórmulas que hay que representar son:

Es delicado fijar el rango de representación para po-der ver las curvas.

Con el rango:

Ec cla v

Ec rel cv c

( )

( )( / )

=

=−

−⎛

⎝⎜⎜

⎠⎟⎟

12

1

11

2

2

2

m mm

ee= =

−− =4

11

142

2;β

β

E m c mE

cm

m c

cm

m

ee= = = =

=

; ;22

2

2

44

33 6 10 30, ⋅ − kg

EE m c

m c

E J

inicial ee= = =

= ⋅ =−

28

24

3 28 10

22

13, 22 05, Mev

Sustituimos el valor de E = γ m0 c2:

Y recordamos la definición del momento, p = γ m0 v:

E2 = p2 c2 + m02 c4

Ésta es la relación que buscábamos.

63. Datos: E = 80 V/m; t = 5 s; Q = 1,6 ⋅ 10−19 C;m0 = 2 ⋅ 10−26 kg

a) Calculamos la fuerza aplicada al ion:

F = q E = 1,6 ⋅ 10−19 C ⋅ 80 V/m = 1,3 ⋅ 10−17 N

b) Aplicamos el teorema del impulso para hallar la ve-locidad, teniendo en cuenta que en mecánica rela-tivista la masa varía con la velocidad:

Despejando β y sustituyendo los datos del enunciado:

c) Calculamos la masa relativista:

Por lo tanto: m = 11,2 m0

m = 11,2 ⋅ 2 ⋅ 10−26 kg = 2,24 ⋅ 10−25 kg

Calculamos la energía cinética relativista:

Ec = Δm c2 = γ m0 c2 − m0 c2 = m0 c2 (γ −1)

Ec = 1,8 ⋅ 10−8 J

64. Datos: me = 9,1 ⋅ 10−31 kg;

Reacción: (e− + e−) → (e− + e−) + 3 (e− + e+)

a) La energía mínima relativista se obtiene cuando laspartículas quedan en reposo después del choque.Aplicamos el principio de conservación de la ener-gía relativista:

Einicial = Efinal = 8 me c2

Einicial = 8 ⋅ 9,1 ⋅ 10−31 kg (3 ⋅ 108 m ⋅ s−1)2

Einicial = 6,55 ⋅ 10−13 J

mm0

2 2

1

1

1

1 0 99611 2= =

−=

−=γ

β ( , ),

β

β

=

+

=

+⋅ ⋅−

1

1

1

12 10 3 10

02 2

2 2

26 2 8

m c

F t

kg( ) ( m ⋅⋅

=

=

s 1 2

17 2 21 3 10 5

0 996)

( , ) ( )

,

;

N s

vc

β ,v c= 0 996

F t m v donde v c m m

F tm c

,= = =

=−

1 0

021

β γ

β

β;

F t

m c

2 2

02 2

2

21=

β

β

E m cv

cm c

E m c v m

2 20

2 42

2 02 4

2 20

2 2 20

2

− =

= +

γ

γ cc4

Page 152: 9523 gl fis_tx2_cas

159

b) Despejamos ahora la velocidad propia del bote, v:

v = 10 km/h − u = 8 km/h

3. El principio de relatividad de Galileo está de acuerdocon la invariabilidad de las leyes de la mecánica clási-ca entre sistemas inerciales; mientras que el de Eins-tein está de acuerdo también con la invariabilidad delas leyes electromagnéticas.

El primero sólo puede aplicarse en sistemas cuya velo-cidad sea muy inferior a la de la luz.

4. La teoría de la relatividad especial nació al estudiar lastransformaciones de las leyes electromagnéticas entresistemas inerciales. En concreto, el experimento deMichelson-Morley hizo necesarias la constancia de lavelocidad de la luz y la no dependencia del sistema dereferencia en que se mida.

5. Datos: m = 2 m0

La fórmula que relaciona la masa en reposo con lamasa relativista es: m = γ m0. Por tanto, γ = 3

Como

entonces:

La velocidad deberá ser de 0,866c.

6. — Veríamos los objetos cotidianos en movimientomás cortos, debido a la contracción de longitudes.

— Los movimientos de los objetos con una cierta ve-locidad se verían como a cámara lenta debido a ladilatación del tiempo.

— El concepto de simultaneidad no sería válido.

7. Datos: m = 7m0

La energía relativista final ha de ser igual a la energíarelativista inicial:

Einicial = Efinal = 7 m0 c2

Por tanto, la energía inicial de cada partícula ha de ser:

Para hallar la velocidad de la partícula, igualamos laenergía obtenida con la definición de energía relativista:

La velocidad de choque necesaria es de 0,958c, y es in-dependiente de las partículas que choquen.

La energía cinética sí cambia porque depende de laspartículas que colisionan:

Ec = Δm c2 = (m − m0) c2 = (γ m0 − m0) c2

Ec = (γ − 1 ) m0 c2

βγ

= − = −⎛⎝⎜

⎞⎠⎟

=11

11

72

0 9582 2 ,

E m c m c= = = =−

γ γ γβ

; ;02

02

2

72

72

1

1

EE

m cfinal= =2

72 0

2

β = − =11

30 9422 ,

;γβ

βγ

=−

= −1

11

12 2

Se obtienen con la curvas:

La gráfica comparativa es:

EVALUACIÓN

1. a) Si el cohete se mueve a velocidad constante respec-to de la Tierra, ambos pueden considerarse siste-mas de referencia inerciales.

b) Las leyes de Newton se cumplen en el sistema S′únicamente si éste es un sistema inercial, es decir,si su velocidad es constante.

2. Datos: d = 6 km; ts = 1 h; tb = 36 min = 0,6 h

a) Aplicamos las leyes de la cinemática al subir:

d = (v − u) ts

donde v es la velocidad del bote, y u es la velocidaddel río.

Aplicamos las leyes de la cinemática al bajar:

d = (v + u) tb

De esta manera, obtenemos un sistema de dos ecua-ciones con dos incógnitas, del que despejamos u:

d v u t

d v u t

km v us

b

= −

= +

⎧⎨⎩

= −( )

( )

( )6 11

6 0 6

6

10

( ) ,

h

km v u h

v k u

v k

= +

⎧⎨⎩

= +

= −

m/h

m/h uu

k u k u u k

k

6 10 2 4

2

;m/h m/h m/h

u m/h

+ = − =

=

11. Física relativista

y x= { }∧0 5 2 0 300000000. * .

y

x

=

− − −

∧ ∧ ∧

( * )*

((( ( /( * )) ) , ) )

9 10 16

1 3 10 8 2 0 5 1 0..300000000{ }

Page 153: 9523 gl fis_tx2_cas

161

Física cuántica12

• Datos: λ = 632 nm = 6,32 ⋅ 10−7 m; c = 3 ⋅ 108 m ⋅ s−1

Calculamos la frecuencia a partir de la relación entre lafrecuencia de una onda y su longitud de onda:

• Tanto el protón, descubierto en 1914, como el neutrón,descubierto en 1932, se encuentran en el núcleo del áto-mo. El protón posee una carga +e = 1,602 ⋅ 10−19 C y elneutrón carece de carga. Sus masas son prácticamenteiguales: mp = 1,673 ⋅ 10−27 kg, mn = 1,675 ⋅ 10−27 kg, y con-centran el 99 % de la masa total del átomo, ya que lamasa de los electrones, descubiertos en 1897 y con carga−e, es unas 2 000 veces menor que la de neutrones y pro-tones: me = 9,110 ⋅ 10−31 kg. El radio nuclear típico se en-cuentra entre 10−14 y 10−15 m, y la distancia entre los electro-nes y el núcleo del átomo es, aproximadamente, del ordende 0,1 nm = 1 Å.

• El descubrimiento del electrón en 1897 condujo al físicoinglés J. J. Thomson a establecer un modelo del átomo.Según este modelo, el átomo estaría formado por un con-junto de electrones incrustados en una masa esférica dedensidad uniforme y carga positiva, de manera que el con-junto sería neutro y estable.

Sin embargo, el descubrimiento del núcleo atómico porel físico británico E. Rutherford llevó a éste a establecerun nuevo modelo atómico. En este modelo, la mayorparte de la masa y toda la carga positiva del átomo seconcentrarían en una minúscula zona central de grandensidad, el núcleo atómico. Por otro lado, el átomo,mucho mayor que el núcleo, incluye la corteza electróni-ca, que es la región donde los electrones describen órbi-tas circulares alrededor del núcleo. Finalmente, el áto-mo es neutro porque el número de electrones es igual alde protones.

1. LIMITACIONES DE LA FÍSICA CLÁSICA

1. Llamamos cuerpos negros a aquellos cuerpos cuya ra-diación térmica depende únicamente de su tempera-tura, y no de su composición.

Son cuerpos que absorben toda la radiación que inci-de sobre ellos y, debido a que la radiación térmica queemiten no es visible a temperaturas ordinarias, se vende color negro. Sin embargo, como la radiación térmi-ca emitida por un cuerpo negro depende de la tempe-ratura, su color no siempre es negro. Así, un bloque

fc m s

mHz= =

⋅ ⋅

⋅= ⋅

−λ

3 10

6 32 104 75 10

8 1

714

,,

PREPARACIÓN DE LA UNIDAD

• Datos: m = 7 kg; v = 10 m⋅s–1

La cantidad de movimiento o momento lineal de uncuerpo es el producto de su masa por su velocidad lineal:

p = m v = 7 kg ⋅ 10 m ⋅ s–1 = 70 kg ⋅ m ⋅ s–1

• Datos: m = 10 g = 0,01 kg; v = 300 m ⋅ s–1

Calculamos la energía cinética:

• Datos: v1 = v; v2 = 2v

Calculamos el cociente entre las energías del cuerpo encada uno de los casos:

Así pues, la energía cinética aumenta cuatro veces al du-plicar el valor de la velocidad.

• Una onda electromagnética es una onda transversal queconsiste en la propagación, sin necesidad de soporte ma-terial alguno, de un campo eléctrico y de un campo mag-nético perpendiculares entre sí y a la dirección de propa-gación.

Como en cualquier onda, existe un transporte neto deenergía sin que haya transporte neto de materia.

Las ondas electromagnéticas son generadas por cargaseléctricas aceleradas. Los vectores de los campos eléctricoy magnético varían sinusoidalmente con el tiempo y laposición, y se encuentran en fase, es decir, ambas alcan-zan el valor máximo y el mínimo al mismo tiempo.

E = E0 sen (ωt − kx); B = B0 sen (ωt − kx)

Además, el cociente de los módulos de los vectores es igual a la velocidad de propagación de la onda, que enel vacío es c = 3 ⋅ 108 m/s, pero que, en general, dependedel medio de propagación.

EB

c=

� �E y B

Ec m v m v

Ec m v m v

1 12 2

2 2 22 2

12

12

12

12

212

= =

= = =( ) mm v m v

EcEc

m v

m v

4 2

212

4

2 2

2

1

2

2

=

= =

Ec m v kg m s J= = ⋅ ⋅ ⋅ =12

12

0 01 300 4502 1 2, ( )

12. Física cuántica

Page 154: 9523 gl fis_tx2_cas

162

b) Calculamos la altura máxima que alcanzará el co-lumpio con la energía de un cuanto:

6. Respuesta sugerida:

El efecto fotoeléctrico se produce al iluminar una su-perficie metálica con radiación electromagnética deuna frecuencia determinada. Con un montaje experi-mental adecuado se puede observar cómo esta radia-ción es capaz de arrancar electrones de la superficiemetálica y generar una corriente eléctrica. Si medimosla intensidad de esta corriente, podemos determinar elnúmero de electrones arrancados, e incluso, su ener-gía de enlace con el metal.

El efecto fotoeléctrico es la base de algunos dispositi-vos de tecnología actual, como la célula fotoeléctricaque controla las puertas de los ascensores o ciertos dis-positivos de seguridad.

7. Existen tres hechos del efecto fotoeléctrico que no te-nían explicación utilizando los argumentos de la físicaclásica. Estos hechos son:

— La emisión de electrones sólo tiene lugar si la fre-cuencia de la radiación incidente supera un deter-minado valor, denominado frecuencia umbral, fu.

Según la teoría clásica, el efecto fotoeléctrico de-bía ocurrir para cualquier frecuencia siempre quela luz fuera lo suficientemente intensa.

— Si la frecuencia de la radiación es superior a estafrecuencia umbral, el número de fotoelectroneses proporcional a la intensidad de la radiación in-cidente. Sin embargo, su energía cinética máximaes independiente de la intensidad de la luz.

— No se observa ningún tiempo de retraso entre lailuminación del metal y la emisión de los fotoelec-trones.

Estos tres aspectos quedaron explicados con la teo-ría cuántica de Einstein para el efecto fotoeléctri-co. La teoría supone que la energía emitida poruna fuente de luz está cuantizada en forma de pa-quetes de energía llamados fotones, en lugar deencontrarse repartida de forma continua a lo lar-go de toda la onda. Los electrones del metal sonarrancados cuando un fotón incide sobre el elec-trón cediéndole toda su energía, y no por la acu-mulación de la energía de la onda sobre una zonadel metal determinada.

— La mínima energía necesaria para arrancar un elec-trón, W0, es igual a la energía de enlace del electrónmás débilmente ligado al metal. Así, el fotón debeaportar una energía mínima de E = h fu = W0. Si la

E m g y yE

m g

yJ

kg

00

342 69 10

20

= =

=⋅ −

;

,

máx máx

máx ⋅⋅ ⋅= ⋅−

9 81 37 102

36

,,

m sm

E n E nEE

nJ

Jcuant

= =

=⋅

= ⋅−

;

,,

00

343598

2 69 103 64 10 oos

de metal de color negro adquiere un color rojizo a me-dida que aumenta su temperatura, y posteriormentepasa a rojo vivo.

2. Respuesta sugerida:

La λmáx es la longitud de onda para la cual se emite unamayor cantidad de energía. Por lo tanto, es la radiacióndominante en la radiación térmica emitida y determi-na el color del que vemos el cuerpo.

Para observar la variación de λmáx con la temperatura,podemos utilizar una barra de hierro y calentarla pro-gresivamente al fuego, retirándola a intervalos para ob-servar el «color» de la radiación que emite. Conformeaumenta la temperatura, la barra se ve de un color rojoopaco, después adquiere un color rojo brillante y, amuy altas temperaturas, toma un intenso color blancoazulado. A medida que aumenta la temperatura, la in-tensidad de la radiación térmica se incrementa y la lon-gitud de onda máxima, λmáx, se hace menor.

3. Las teorías clásicas para explicar la emisión de radia-ción de un cuerpo negro predecían que la energía dela radiación aumentaba indefinidamente al disminuir lalongitud de onda, mientras que experimentalmente sehabía comprobado cómo la energía tendía a cero paralongitudes de onda muy cortas, como las correspon-dientes al ultravioleta.

4. Planck formuló dos hipótesis para explicar la radiaciónde un cuerpo negro:

— Los átomos que emiten la radiación se comportancomo osciladores armónicos.

— Cada oscilador absorbe o emite energía de la radia-ción en una cantidad proporcional a su frecuenciade oscilación: E0 = h f.

a) Esto quiere decir que la energía total emitida o ab-sorbida por los osciladores está cuantizada, es de-cir, sólo puede tener unos valores determinados,múltiplos de E0; E = n E0 = n h f.

b) Así, Planck se limitó a cuantizar la energía de lososciladores armónicos y a suponer que los átomosse comportaban como osciladores, pero no a cuan-tizar propiamente los estados atómicos.

5. Datos: m = 20 kg; L = 1,5 m; ymáx = 50 cm = 0,5 m:h = 6,62 ⋅ 10−34 J ⋅ s

a) Calculamos en primer lugar la frecuencia de osci-lación del columpio, el cual se comporta como unpéndulo, es decir, un oscilador armónico, y así ob-tenemos la energía de un cuanto:

La energía total del columpio es igual a la energía po-tencial en el punto más alto de su trayectoria:

E = m g ymáx = 20 kg ⋅ 9,8 m ⋅ s−2 ⋅ 0,5 m = 98 J

Hallamos el número de cuantos de energía a partirdel cociente entre la energía del columpio y la de uncuanto:

fT

gL

fm s

mHz

E h

= = =⋅

=

=

−1 12

12

9 81 5

0 4071

0

π π;

,,

,

; , ,

,

f E J s Hz

E J0

34

034

6 62 10 0 407

2 69 10

= ⋅ ⋅ ⋅

= ⋅

Page 155: 9523 gl fis_tx2_cas

163

b) Calculamos la energía de los fotones mediante laexpresión de Planck:

E = h f; E = 6,62 ⋅ 10−34 J ⋅ s ⋅ 6 ⋅ 1014 Hz

E = 3,97 ⋅ 10−19 J

13. Bohr formuló un modelo atómico basado en dos pos-tulados fundamentales:

1. El electrón se mueve en órbitas circulares estacio-narias sin absorber ni emitir radiación. Estas órbi-tas sólo pueden tener ciertas energías y radios y secaracterizan por la cuantización del momento an-gular.

2. El electrón sólo puede cambiar de órbita absor-biendo o emitiendo un fotón con una energíaigual a la diferencia energética entre las órbitasentre las que se produce el salto. Ello justifica ladiscretización de los espectros.

a) Un electrón en un estado estacionario no se en-cuentra en reposo, sino que se mueve en una de-terminada órbita alrededor del núcleo del átomo.Lo que ocurre es que las magnitudes físicas que locaracterizan (radio de la órbita, energía, velocidad,momento angular...) no dependen del tiempo.

b) Cuando el número cuántico n aumenta, tambiénlo hace el radio de la órbita del electrón, y dismi-nuye su energía. En el caso límite, el electrón que-daría desligado del átomo (ionización), lo queequivale a decir que su órbita es de radio infinito,y la energía de ligadura con el núcleo se anularía.

14. Datos: m = 1; n = 4; n = 7; RH = 1,097 ⋅ 107 m−1;c = 3 ⋅ 108 m ⋅ s−1; h = 6,62 ⋅ 10−34 J ⋅ s

Las longitudes de onda de la serie de Lyman pueden ob-tenerse a partir de la fórmula de Rydberg para m = 1.

Esta serie corresponde a las transiciones electrónicasen el átomo de hidrógeno desde niveles o estados ex-citados con n > 1 hasta el nivel n = m = 1. La terceralínea de la serie corresponde al salto del nivel con n = 4hasta el m = 1, y la sexta al salto desde el nivel n = 7.Calculamos la longitud de onda de estas dos líneasespectrales mediante la fórmula de Rydberg, y susti-tuimos este valor en la expresión para la energía deun fotón dada por Planck:

E h f hc

E J sm s

= = = ⋅ ⋅ ⋅⋅ ⋅

→−

; ,,λ 4 1

348 1

6 62 103 10

9 72 ⋅⋅

= ⋅

= ⋅

→−

→−

10

2 04 10

1 097 101

1

8

4 118

7 17 1

2

m

E J

m

,

,λ −−⎛⎝⎜

⎞⎠⎟

⎣⎢

⎦⎥ = ⋅

= =

−−

1

79 31 102

18

7

,

;

m

E h f hc

Eλ 11

348 1

8

7 1

6 62 103 10

9 31 10

2

= ⋅ ⋅ ⋅⋅ ⋅

=

−−

,,

J sm s

m

E ,,13 10 18⋅ − J

1 1 1 1 12 2 2 2λ

λ= −⎛⎝⎜

⎞⎠⎟

= −⎛⎝⎜

⎞⎠⎟

⎣⎢

⎦⎥

Rm n

Rm nH H;

11

4 17 1

2 2

1

1 097 101

1

1

49λ →

−−

= ⋅ −⎛⎝⎜

⎞⎠⎟

⎣⎢

⎦⎥ =, ,m 772 10 8⋅ − m

frecuencia del fotón es menor que fu, ningún foto-electrón podrá ser extraído.

— Duplicar la intensidad de la luz equivale a duplicarel número de fotones, y por lo tanto, a duplicar elnúmero de electrones extraído, pero no a variar laenergía cinética de dichos electrones.

— La energía necesaria para arrancar a los electronesse suministra en paquetes (fotones), por lo que loselectrones no necesitan «acumular» energía sufi-ciente para escapar del metal y, por lo tanto, noexiste un tiempo de retraso.

8. Si la frecuencia de la radiación incidente es inferior a lafrecuencia umbral, no se producirá emisión de electro-nes. Por otro lado, si la frecuencia supera el valor umbral,entonces sí que el número de fotoelectrones emitidosserá proporcional a la intensidad de dicha radiación. Elloes debido a que una mayor intensidad en la radiación im-plica una mayor energía por unidad de superficie y uni-dad de tiempo, y por tanto, un mayor número de fotonescapaces de arrancar electrones del metal.

9. Respuesta sugerida:

La confirmación experimental de la existencia de losfotones fue llevada a cabo por el físico norteamericanoArthur H. Compton en 1932, al analizar la colisión en-tre un haz de rayos X y una lámina de grafito. Comp-ton observó que la radiación incidente se dividía tras lacolisión en dos radiaciones de longitudes de onda dife-rentes, una igual a la longitud de onda de la radiaciónincidente y otra de longitud de onda mayor.

Para explicar este hecho, Compton consideró la radia-ción electromagnética como un conjunto de partículasrelativistas de masa en reposo nula, energía E = h f, ycon un momento lineal p = E c−1 = h λ−1. Los fotonesque chocan con un electrón de la lámina de grafito ce-den parte de su energía al electrón en el choque y, portanto, su energía y su frecuencia son menores y su lon-gitud de onda, mayor que antes de la colisión. Sin em-bargo, los fotones que no colisionan con los electronesde la lámina mantienen intactas su energía, frecuenciay longitud de onda.

10. La energía de un fotón es proporcional a su frecuen-cia, según la fórmula de Planck, y puesto que la fre-cuencia de los fotones ultravioleta, del orden de1015 Hz − 1016 Hz, es superior a la de los fotones de luzverde, aproximadamente 1014 Hz, también lo será suenergía.

11. Los fotones de la luz ultravioleta son más energéticosque los de la luz del espectro visible. Así, la energía quesuministran estos últimos no es suficiente para romperlos enlaces moleculares del plástico de las bolsas, perosí lo es la energía proporcionada por los fotonesultravioleta.

12. Datos: λ = 5 ⋅ 10−7 m; c = 3 ⋅ 108 m ⋅ s−1; h = 6,62 ⋅ 10−34 J ⋅ s

a) Calculamos la frecuencia a partir de la relación en-tre la longitud de onda y la frecuencia de unaonda:

fc

fm s

mHz= =

⋅ ⋅

⋅= ⋅

−λ;

3 10

5 106 10

8 1

714

12. Física cuántica

Page 156: 9523 gl fis_tx2_cas

164

Por tanto, a igual energía corresponde la mismafrecuencia.

18. Datos: v = 1 m ⋅ s−1; me = 9,1 ⋅ 10−31 kg; mn = 1,67 ⋅ 10−27 kg; h = 6,62 ⋅ 10−34 J ⋅ s

Calculamos la frecuencia a partir de la relación E = hf y la longitud de onda mediante la relación de DeBroglie:

a) Hallamos la frecuencia y la longitud de onda en elcaso del electrón:

b) Hacemos lo mismo para el neutrón:

19. Datos: Δp = 0

a) Las indeterminaciones en el momento lineal y enla velocidad de una partícula se encuentran rela-cionadas según la expresión:

Por lo tanto, si Δp = 0, la velocidad también se en-cuentra bien definida.

b) El principio de indeterminación de Heisenbergfija el grado de determinación que podemos obte-ner en la medida de la posición y el momento li-neal (o velocidad) de una partícula. Así, un gradode precisión alto en la velocidad implica un errormuy alto en la medida de la posición, y a la inver-sa. En el caso que se nos presenta, la indetermina-ción en el momento lineal es nula, lo que implicauna indeterminación total en la posición. El fijarla velocidad (o el momento lineal) de una partículade forma exacta lleva a desconocer completamentesu posición.

20. Datos: v = 104 m ⋅ s−1; Δv = 0,0005 %; mp = 1,67 ⋅ 10−27 kg; h = 6,62 ⋅ 10−34 J ⋅ s

La imprecisión en la indeterminación de v es del 0,0005por 100, lo que quiere decir que su incertidumbre es:

Δ Δ ΔΔ

p m v vp

m= =;

fm v

h

fkg m s

n=

=⋅ ⋅ ⋅

⋅ ⋅

− −

2

27 1 2

2

1 67 10 1

2 6 62 10

, ( )

, 33461 26 10

1 26

6 62 10

J sHz

f MHz

hm vn

⋅= ⋅

=

= =⋅

,

,

;,

λ λ−−

− −

⋅ ⋅ ⋅

= ⋅ =

34

27 1

7

1 67 10 1

3 96 10 3 9

J s

kg m s

m

,

, ,λ 66 10 4⋅ − mm

fm v

h

fkg m s

e=

=⋅ ⋅ ⋅

⋅ ⋅

− −

2

31 1 2

3

2

9 1 10 1

2 6 62 10

, ( )

, 44

34

31

687

6 62 10

9 1 10

J sHz

hm v

J s

e

⋅=

= =⋅ ⋅

−;,

,λ λ

kkg m s

m mm

⋅ ⋅

= ⋅ =

1

7 27 10 0 727

1

4, ,λ

E h f fEh

m vh

ph h

ph

m v= = = = = =; ;

2

2 λλ

2. MECÁNICA CUÁNTICA

15. a) Utilizamos la fórmula de De Broglie, que relacionala longitud de onda de una partícula con su mo-mento lineal:

Por lo tanto, a medida que aumenta la velocidad, dis-minuye la longitud de onda asociada a la partícula.

b) Calculamos la frecuencia a partir de la relación dePlanck:

Es decir, aumentar la velocidad supone aumentarla frecuencia de la onda asociada al movimientode la partícula.

16. Datos: Ec = 68 eV = 1,09 ⋅ 10−17 J; me = 9,1 ⋅ 10−31 kg;h = 6,62 ⋅ 10−34 J ⋅ s

a) Calculamos la frecuencia de los electrones a partirde la relación de Planck:

b) Para calcular la longitud de onda de los electrones,primero calculamos su momento lineal:

Utilizamos ahora la relación de De Broglie entre elmomento y la longitud de onda:

17. a) Utilizamos la fórmula de De Broglie y la relaciónentre la energía cinética y el momento lineal deuna partícula para determinar la longitud de onda:

De esta expresión se deduce que la partícula conmenor masa será la que tenga asociada una longi-tud de onda mayor.

b) La frecuencia asociada a la partícula es:

E h f fEh

= =;

Ec m vpm

p m Ec

hp

h

m Ec

= = =

= =

12 2

2

2

22

;

λ

λ

λ

=

=⋅ ⋅

⋅ ⋅ ⋅= ⋅

− −

hp

J s

kg m s

6 62 10

4 45 101 49

34

24 1

,

,, 110 1 4910− =m Å,

Ec m vpm

p m Ec

p kg

= = =

= ⋅ ⋅ ⋅ ⋅−

12 2

2

2 9 1 10 1 09

22

31

;

, , 110

4 45 10

17

24 1

− −= ⋅ ⋅ ⋅

J

p kg m s,

fEh

fJ

J ss= =

⋅ ⋅= ⋅

−−;

,

,,

1 09 10

6 62 101 65 10

17

3416 11

161 65 10f Hz= ⋅,

E h f fEh

m v

hm v

h= = = =;

12

2

22

λ = =hp

hm v

Page 157: 9523 gl fis_tx2_cas

165

Así que para los orbitales con l = 1, el módulo de L es

mientras que para los or-

bitales con l = 0, el módulo de L es L = 0.

23. Respuesta sugerida:El experimento de Stern-Gerlach pretendía medir elmomento magnético dipolar de los átomos. Para ellose hacía penetrar un haz de átomos neutros en uncampo magnético no uniforme, con la dirección deleje Z, que variaba en intensidad en esa misma direc-ción. Originalmente el experimento se efectuó conátomos de plata pero es mucho más clarificador en elcaso de átomos de hidrógeno, tal como se repitió pos-teriormente.

Los átomos de hidrógeno poseen un único electrón.Por ello, el momento magnético dipolar del átomo esdebido al giro de este único electrón en su órbita alre-dedor del núcleo (órbita que equivale a una espira decorriente) y, por lo tanto, es proporcional al aumentoangular orbital del electrón. Como los átomos utiliza-dos en el experimento son neutros, la única fuerza netaque actúa sobre ellos es la fuerza Fz proporcional a lacomponente z del momento angular del electrón, queestá caracterizada por el número cuántico ml. Así,mientras que clásicamente cualquier desviación del hazde átomos es posible porque cualquier valor de la com-ponente z del momento angular lo es, cuánticamente elhaz se dividirá en un número de haces divergentesigual al número posible de valores de la componente zdel momento angular, ml, que está cuantizada. El nú-mero ml puede tomar un número impar de valores,ml = −l, − l + 1, ..., 0, l −1, l. Sin embargo, experimental-mente se observó que el haz de átomos se escindía endos componentes: una componente se doblaba hacia elsentido positivo del eje Z y la otra en el sentido opues-to. Es más, los átomos se encontraban en su estado fun-damental, en cuyo caso, l = 0 y ml = 0, y se esperaba queel haz no fuera desviado por el campo magnético.

La explicación estaba en que el electrón posee un mo-mento angular intrínseco llamado espín, de número

cuántico cuya componente z está determinada por

el número cuántico ms y que, análogamente al momento

angular orbital, puede tomar los valores

Este momento angular intrínseco no está relacionadocon el movimiento orbital del electrón y puede pen-sarse que es debido a un movimiento de rotación delelectrón.

ms = −12

12

, .

s =12

,

Resultado real

Resultado clásico

Z

Lh h

= + =1 1 12

22

( ) ,π π

Por lo tanto, la incertidumbre en su momento es:

Δp = m Δv; Δp = 1,67 ⋅ 10−27 kg ⋅ 0,05 m ⋅ s−1

Δp = 8,35 ⋅ 10−29 kg ⋅ m ⋅ s−1

Calculamos la incertidumbre en la posición a partir delprincipio de indeterminación:

21. Respuesta sugerida:

A principios de los años veinte surgieron dos teoríasque pretendían explicar el comportamiento de la ma-teria y la energía de los sistemas microscópicos, incor-porando las nuevas ideas que se habían expuesto hastael momento para resolver los problemas presentadospor la física clásica.

Así, por un lado, encontramos la mecánica cuánticamatricial, desarrollada por W. Heisenberg, M. Born yP. Jordan, que describe las variables físicas asociadas auna partícula (posición, momento lineal…) mediantematrices.

Por otro lado, el físico austríaco, E. Schrödinger, desa-rrolló la denominada mecánica cuántica ondulatoria.Esta teoría describe el comportamiento de la materiamediante funciones de onda. Éstas son soluciones de ladenominada ecuación de Schrödinger, y dependen dela posición y el tiempo.

Poco tiempo más tarde, el inglés Paul A. M. Dirac de-mostró que ambas teorías eran dos representacionesde una misma teoría, la denominada mecánica cuánti-ca o teoría cuántica no relativista.

Según la mecánica cuántica, no tiene sentido decirque los electrones describen órbitas circulares fijas al-rededor del núcleo, sino que existe una serie de zo-nas del espacio donde es más probable hallarlos. Deeste modo se introduce el concepto de orbital, que esuna función de onda correspondiente a ese sistema,y cuyo cuadrado es una medida de la probabilidad deencontrar el electrón en cada punto del espacio.

22. Datos: Z = 1; E0 = −13,606 eV; n = 2

Los orbitales con n = 2 son:

(2, 1, −1), (2, 1, 0), (2, 1, 1), (2, 0, 0)

Calculamos ahora la energía en eV:

El módulo del momento angular es:

Lh

Lh2

2

12

12

= +⎛

⎝⎜⎞

⎠⎟= +l l l l( ) ; ( )

π π

E EZ

nE eV eVn = − = − = −0

2

2 2

2

213 6061

23 40; , ,

Δ Δ ΔΔ

Δ

x ph

xh

p

xJ s

⋅ ≥ ≥

=⋅ ⋅

⋅ ⋅

4 4

6 62 10

4 8 35 1

34

π π

π

;

,

, 006 31 1029 1

7− −

⋅ ⋅= ⋅

kg m sm,

Δv vms

ms

= = ⋅ =0 0005

1000 0005

10010 0 054, ,

,

12. Física cuántica

Page 158: 9523 gl fis_tx2_cas

166

Los transistores bipolares son dispositivos compues-tos por la unión de tres cristales semiconductores do-pados con portadores de carga positivos (tipo P) y ne-gativos (tipo N) en orden PNP o NPN. Cada una delas zonas dispone de un terminal conductor que seconecta al circuito exterior. Éstos reciben los nom-bres de colector, base y emisor. Mediante la aplicaciónde las diferencias de potencial adecuadas entre termi-nales (polarización del transistor) se puede conseguirque la corriente de salida del colector sea directamen-te proporcional a la corriente de entrada en la base yque el transistor actúe como un amplificador de co-rriente.

Los transistores de efecto de campo son dispositivossemiconductores en los que se puede obtener amplifi-cación de corriente y de tensión haciendo variar laconductancia mediante un campo eléctrico exteriortransversal aplicado al material semiconductor.

Las principales aplicaciones de los transistores son:

— Como amplificadores de corriente o tensión, porejemplo, en los amplificadores de los equipos demúsica de alta fidelidad.

— Como interruptores electrónicos en los circuitosintegrados que forman parte de las modernascomputadoras electrónicas.

— Como interruptores de potencia para controlarlas intensidades y potencias en motores y otrosaparatos eléctricos.

b) Respuesta sugerida:

Muchos metales presentan resistividad cero por deba-jo de cierta temperatura crítica Tc, es decir, son super-conductores. Algunos ejemplos son el mercurio (Tc == 4,2 K), el iridio (Tc = 0,1 K), el niobio (Tc = 9,2 K),el aluminio (Tc = 1,2 K) y el plomo (Tc = 7,2 K). Sinembargo, para obtener materiales superconductores aTc mayores, es necesario recurrir a ciertas familias dealeaciones cerámicas.

Las principales aplicaciones de la superconductivi-dad tienen que ver con la utilización de imanes su-perconductores. Son grandes bobinas superconduc-toras que permiten crear campos magnéticos muyintensos sin las pérdidas de energía debidas a la resis-tencia al paso de la corriente de las bobinas conduc-toras normales. Estos electroimanes se emplean en laobtención de imágenes médicas por resonancia mag-nética, en la aceleración y guiado de las partículas enlas grandes instalaciones aceleradoras, en la investi-gación física básica, en el confinamiento magnéticodel plasma, en los reactores experimentales de fusiónnuclear, en trenes que levitan sobre los raíles graciasa los campos magnéticos, en generadores y motoreseléctricos...

Otras aplicaciones futuras de la superconductividad sedarán en el campo de la electrónica, en la fabricaciónde componentes microelectrónicos de computadorasque serían de menor tamaño que los actuales, más ve-loces y disiparían menos energía en forma de calor.

c) Algunas ideas que se podrían expresar y desarrollar enel coloquio son las siguientes:

24. Respuesta sugerida:

El espín es una propiedad intrínseca del electrón y deotras partículas fundamentales y determina cómo lapartícula se alinearía en presencia de un campo mag-nético externo, de forma paralela o antiparalela a di-cho campo. Así pues, es un momento angular intrínse-co que está cuantizado. El espín está caracterizado porel número cuántico de espín, s, propio de cada partícu-la microscópica. En el caso del electrón, el número

cuántico de espín toma el valor de Otras partí-

culas pueden tener distintos valores de s, por ejemplo, elfotón tiene s = 1.

25. — El principio de exclusión de Pauli nos dice que enun átomo multielectrónico nunca podrá existirmás de un electrón en el mismo estado cuántico; esdecir, los electrones en el átomo no pueden teneriguales sus cuatro números cuánticos.

— Clásicamente, es posible distinguir entre dos bolasde billar idénticas. Por ejemplo, podemos fijarnosen ambas bolas en un instante de tiempo determi-nado y marcarlas mentalmente con los números 1y 2. Como ambas siguen trayectorias bien defini-das, observándolas con atención puedo saber entodo momento cuál de ellas es la bola 1 y cuál es labola 2. Sin embargo, cuánticamente, el principiode incertidumbre impide determinar al mismotiempo la posición y la velocidad de las bolas. Así,no es posible determinar sus trayectorias. Por lotanto:

a) Sí puedo identificar (marcar mentalmente) lasdos bolas de billar cuánticas en un momentodeterminado antes del choque, por ejemplo, de-terminando exactamente su posición. Sin em-bargo, para cualquier otro instante de tiempoposterior no puedo distinguir entre ambas,puesto que no es posible conocer sus trayecto-rias.

b) No se puede determinar la trayectoria de cadauna de las bolas.

26. Las partículas con espín semientero se denominan fer-miones, como por ejemplo, el neutrino, el protón, elpositrón, el antiprotón y la partícula Por otro lado,las partículas con espín cero o entero se denominanbosones, como por ejemplo, el fotón, el pión, el gravi-tón y el muón.

FÍSICA Y SOCIEDAD

a) Respuesta sugerida:

Actualmente, mediante diferentes técnicas y procesosde fabricación, es posible obtener una amplia gama detransistores adecuados para las más dispares aplicacio-nes. Así, por ejemplo, se dispone de transistores capa-ces de funcionar a altas frecuencias y otros que sopor-tan corrientes y potencias elevadas.

Desde el punto de vista del principio físico de su fun-cionamiento, podemos distinguir principalmente en-tre muchísimos tipos de transistores, los transistores bi-polares y los de efecto de campo.

−∑ .

s =12

.

Page 159: 9523 gl fis_tx2_cas

167

Sustituimos las dos parejas de valores de λ y VDpara hallar los parámetros a y b:

Del valor del parámetro b obtenemos la funcióntrabajo del potasio:

b) Calculamos la frecuencia umbral mediante la rela-ción W0 = h fu, y con ella obtenemos la longitud deonda:

c) Calculamos ahora la velocidad máxima de los elec-trones:

28. Datos: m = 3; RH = 1,097 ⋅ 107 m−1; h = 6,62 ⋅ 10−34 J ⋅ s;c = 3 ⋅ 108 m ⋅ s−1

a) Escribimos la fórmula de Rydberg para la serie dePaschen (m = 3):

Del estudio de esta expresión se ve que la longitudde onda más corta corresponde al valor más altopara n, es decir, a la transición desde el nivel másalto posible n = ∞. En este caso, la longitud deonda es:

1 19

09

9 9

1 097 107 1

λ

λ λ

∞ ∞ −

= −⎛⎝⎜

⎞⎠⎟

=

= =⋅

RR

R m

HH

H

;,

== ⋅ −8 2 10 7, m

1 1

3

1 19

12 2 2λ

= −⎛⎝⎜

⎞⎠⎟

= −⎛⎝⎜

⎞⎠⎟

Rn

RnH H

Ec e V m v ve Vm

Para v

D eD

emáx máx máx

= = =;

;

12

22

1λ xx

máx

=⋅ ⋅ ⋅

=

2 1 6 10 0 862

9 1 10

5 51

19

31

, ,

,

,

C V

kg

v ⋅⋅ ⋅

=⋅ ⋅ ⋅

10

2 1 6 10 0 469

5 1

2

19

m s

Para vC V

;, ,

λ máx 99 1 10

4 06 10

31

5 1

,

,

= ⋅ ⋅

kg

v m smáx

fWh

fJ

J su u= =

⋅ ⋅

⋅ ⋅=

−0

19

34

2 04 1 6 10

6 62 104 93;

, ,

,, ⋅⋅

= =⋅ ⋅

⋅= ⋅

10

3 10

4 93 106 09 1

14

8 1

14

Hz

cf

m s

Hzλ λ;

,, 00 6097− =m nm

bWe

W b e W eV= − = − =00 0 2 04; ; ,

0 8621

4 34 10

0 4691

5 02 10

7

7

,,

,,

V am

b

V a

=⋅

+

=⋅

− mmb

a m

b V

+

⎬⎪⎪

⎭⎪⎪

= ⋅

= −

−1 26 10

2 04

6,

,

Vhce

We

V a bD D= − = +λ λ

0 1;

— La omnipresencia de la electrónica en nuestras vi-das. Se pueden enumerar casos concretos en casa,en el instituto, en la calle, en los coches, en la me-dicina... de los usos de la electrónica.

— ¿Es posible concebir la vida actual sin la electróni-ca? ¿Qué ventajas nos ha traído?

— La creciente miniaturización de los dispositivoselectrónicos. Ventajas para la vida diaria: comodi-dad, fiabilidad, precio...

— El imparable avance de la electrónica hace que mu-chos aparatos y tecnologías queden obsoletos conrapidez (por ejemplo, ordenadores y algunastecnologías de audio y vídeo que no han tenidoéxito). Este ritmo de aparición de nuevos produc-tos nos obliga a consumir aparatos electrónicos y arenovar los que ya tenemos con más frecuencia delo realmente necesario.

Para la organización del coloquio se recomienda seguirestas pautas:

— Determinar los encargados de las distintas funcio-nes:

• Moderador. Presentará a los participantes e in-troducirá el tema que se va a tratar. Además, con-cederá los turnos de palabra para que el colo-quio se desarrolle de forma ordenada.

• Participantes. Darán sus opiniones sobre el temaelegido y escucharán las de los otros participan-tes. Generalmente, son un máximo de seis perso-nas.

Todos los participantes deben investigar y documentar-se sobre el tema con anterioridad.

• Público. Atenderá a las diversas opiniones. Podráintervenir al final aportando sus propias opinio-nes o preguntando a los participantes algunacuestión.

— Iniciar el coloquio. El moderador presentará a losparticipantes, introducirá el tema y planteará laprimera pregunta a alguno de los participantes.

— Desarrollar y concluir el coloquio. Los distintosparticipantes desarrollarán sus argumentos condu-cidos por el moderador. Cada participante debeexpresar sus opiniones y respetar las de los demás.

Al final del coloquio, el público podrá exponer susopiniones y preguntar a los participantes. Por últi-mo, el moderador puede llevar a cabo un breve re-sumen de las intervenciones.

RESOLUCIÓN DE EJERCICIOS Y PROBLEMAS

27. Datos: λ1 = 434 nm = 4,34 ⋅ 10−7 m; VD1 = 0,862 V;VD2 = 0,469 V; λ2 = 502 nm = 5,02 ⋅ 10−7 m;h = 6,62 ⋅ 10−34 J ⋅ s; c = 3 ⋅ 108 m ⋅ s−1

a) Expresamos la ecuación de Einstein para el efectofotoeléctrico en función de la longitud de onda,

y como además Ecmáx = e VD ,

tenemos:

Echc

Wmáx = −λ 0,

12. Física cuántica

Page 160: 9523 gl fis_tx2_cas

168

como osciladores armónicos y, por otro, supone queestos osciladores únicamente pueden absorber o emi-tir radiación de forma discreta, en cantidades propor-cionales a su frecuencia de oscilación, E = n E0 = n h f.A estos paquetes de energía los denomina cuantos, demodo que la energía de los osciladores se encuentracuantizada.

33. La teoría cuántica de Einstein supone que toda radia-ción se encuentra formada por pequeños paquetes deenergía denominados fotones, cuya energía es pro-porcional a la frecuencia de la radiación. La existenciade los fotones, partículas con masa en reposo nulapero con momento lineal, quedó comprobada experi-mentalmente gracias a las experiencias del físico nor-teamericano A. H. Compton, al estudiar la colisión derayos X sobre una lámina de grafito.

Por lo tanto, la teoría cuántica de Einstein confirma lanaturaleza corpuscular de la luz.

34. La ecuación básica del efecto fotoeléctrico es la expre-sión Ecmáx = h f − W0.

El primer término nos indica la energía cinética máxi-ma que podrá alcanzar un fotoelectrón al ser arranca-do del metal. Esta energía es igual a la diferencia entrela energía del fotón incidente h f y la función trabajodel metal W0, que es el trabajo necesario para extraerel electrón más débilmente enlazado del metal.

35. La longitud de onda de la luz verde es de unos 550 nm,mientras que la de la luz amarilla se encuentra aproxi-madamente en 590 nm. La luz naranja tiene una lon-gitud de onda superior a los 590 nm, lo que quiere de-cir que su frecuencia será menor que la de la luzamarilla y, por lo tanto, no producirá efecto fotoeléc-trico. Sin embargo, la luz azul tiene una longitud deonda menor que la de la luz verde, es decir, una fre-cuencia mayor, y por lo tanto, sí será capaz de produ-cir efecto fotoeléctrico.

36. Si se produce efecto fotoeléctrico al incidir radiaciónde frecuencia f, esto significa que la frecuencia um-bral del metal fu es menor que f. Por lo tanto, si aho-ra incide una radiación de frecuencia 2f, tambiénproducirá efecto fotoeléctrico, pues su frecuenciaserá superior a la umbral.

37. El espectro de emisión lo componen las longitudes deonda de la luz que emite una sustancia química. Es unespectro discreto que presenta una serie de rayas bri-llantes de colores, cada una de distinta longitud deonda y frecuencia, sobre fondo oscuro.

El espectro de absorción lo forman las longitudes deonda de la luz con que se ilumina una sustancia quí-mica que no han sido absorbidas por dicha sustanciaal atravesarla. Es un espectro que presenta una seriede rayas oscuras sobre el espectro continuo de la luzincidente utilizada.

Si colocamos ambos espectros para una misma sustan-cia uno junto al otro, se observa que son complemen-tarios. Las líneas brillantes del espectro de emisión co-rresponden exactamente a las longitudes de onda quefaltan en el de absorción. Ambos espectros son carac-terísticos de cada sustancia química y sirven como mé-todo de identificación de ésta.

Utilizamos la fórmula de Planck para hallar laenergía del fotón:

b) Las tres longitudes de onda mayores de la serie co-rresponden a las transiciones desde los tres nivelesmás cercanos al nivel m = 3, es decir, n = 4, 5 y 6.Aplicamos la fórmula de Rydberg para hallar estaslongitudes de onda:

EJERCICIOS Y PROBLEMAS

29. En el momento de extraer la barra del horno su tem-peratura es muy alta, y, por la ley de desplazamiento deWien, la longitud de onda λmáx para la que se producemayor emisión de energía corresponde al color rojo. Amedida que la barra se enfría, su temperatura descien-de, lo que implica un aumento de λmáx y una disminu-ción de la energía emitida. Esto hace que varíe el colorde la barra y disminuya su brillo, hasta que la tempera-tura es tal que la longitud de onda ya no correspondea radiación del espectro visible.

30. Respuesta sugerida:

Una manera sería establecer una diferencia de poten-cial entre los extremos del metal y conectarlo a un cir-cuito eléctrico, de modo que se generara una corrien-te eléctrica.

Otra forma sería iluminar el metal con una radiaciónde longitud de onda adecuada, de manera que obtuvié-ramos electrones por efecto fotoeléctrico.

Finalmente, una tercera forma sería mediante un pro-ceso de electrólisis, donde el metal que actúa de cáto-do cede electrones a la pila electrolítica.

31. La energía cinética máxima de los fotoelectrones se en-cuentra relacionada con la frecuencia de la radiaciónincidente según la expresión Ecmáx = h f − W0. Por lotanto, triplicar la frecuencia de la radiación no equiva-le a triplicar la energía cinética de los electrones, yaque debemos tener en cuenta la energía que gastamosen desprender los electrones del metal, que no varía enfunción de la frecuencia, sino que es característica delmaterial.

32. La hipótesis de Planck para explicar la radiación delcuerpo negro se basa en dos puntos fundamentales:por un lado, considera que los átomos se comportan

1 1

3

1 19

12 2 2λ

λ= −⎛⎝⎜

⎞⎠⎟

= −⎛⎝⎜

⎞⎠⎟

⎣⎢

⎦⎥

Rn

RnH n H;

11

47 1

2

1

1 097 1019

1

41 88λ = ⋅ −

⎛⎝⎜

⎞⎠⎟

⎣⎢

⎦⎥ = ⋅−

, ,m 110

1 097 1019

1

51

6

57 1

2

1

−−

= ⋅ −⎛⎝⎜

⎞⎠⎟

⎣⎢

⎦⎥ =

m

mλ , ,,

,

28 10

1 097 1019

1

6

6

67 1

2

= ⋅ −⎛⎝⎜

⎞⎠⎟

⎣⎢

⎦⎥

m

mλ−−

−= ⋅1

61 09 10, m

E h f hc

E J sm s

= = = ⋅ ⋅ ⋅⋅ ⋅

⋅−

−; ,,λ

6 62 103 10

8 2 1034

8 1

77

192 42 10 1 51

m

E J eV= ⋅ =−, ,

Page 161: 9523 gl fis_tx2_cas

169

nula, energía E = h f, y momento lineal

Los fotones que chocaban con los electrones de losátomos de grafito cedían parte de su energía y, por lotanto, emergían del material con una longitud deonda mayor.

42. El concepto de órbita del modelo atómico de Bohres heredero del concepto clásico de órbita. Se tratade la trayectoria, en este caso circular, bien definidadel electrón en su movimiento alrededor del núcleo.La novedad es que sólo algunas órbitas son posiblesy que en ellas el electrón se encuentra en un estadoestacionario, es decir, su energía se mantiene cons-tante.

Sin embargo, en el concepto de orbital el electrón yano tiene una localización precisa. Un orbital es unafunción de onda, solución de la ecuación de Schrö-dinger, cuyo cuadrado es una medida de la probabili-dad de hallar el electrón en cada punto del espacio ypara cada instante de tiempo. No podemos hablar dela posición o de la trayectoria del electrón alrededordel núcleo, sino que tan sólo podemos calcular en quézonas del espacio es más probable hallar el electrón.Estas probabilidades pueden representarse mediantesuperficies imaginarias dentro de las cuales la proba-bilidad de encontrar el electrón con una determinadaenergía es elevada.

43. La diferencia fundamental se halla en el número departículas (bosones o fermiones) que puede haber si-multáneamente en un mismo estado cuántico. En elcaso de los fermiones, que tienen espín semientero,este número viene determinado por el principio deexclusión de Pauli, que nos dice que no pueden exis-tir dos fermiones en el mismo estado cuántico. Porotro lado, los bosones, con espín entero, se compor-tan de forma totalmente contraria, y su tendencia es aagruparse en un mismo estado, y así no existe un lími-te para el número de bosones que podemos encontraren el mismo estado cuántico.

Ejemplos de fermiones son el electrón, el protón y el

neutrón, todos ellos con espín Bosones son el

fotón (s = 1), la partícula α (s = 0) y el muón (s = 0).

44. Respuesta sugerida:

El esquema básico de una célula fotoeléctrica es elque se muestra en la figura. La luz incide sobre elcátodo y arranca electrones. El número de electro-nes que llegan al ánodo se mide gracias a la corrien-te que circula por el amperímetro, y modificando elvalor de la polaridad del ánodo podemos variar lacantidad de fotoelectrones emitidos que llegan has-ta él.

Cuando el valor de V es positivo, los electrones sonatraídos por el ánodo, y para valores suficientementegrandes, todos los electrones llegan al ánodo y la co-rriente alcanza un valor máximo.

Interrumpiendo o permitiendo el paso de la luz haciael cátodo hacemos lo propio con la corriente que cir-cula por el circuito, lo que convierte a las células foto-eléctricas en unos útiles sensores.

s =12

.

pEc

h= =

λ.

38. a) En el modelo de Bohr los electrones giran alrede-dor del núcleo atómico describiendo órbitas circu-lares. Estas órbitas sólo pueden tener ciertos valo-res de la energía y ciertos radios, y en ellas loselectrones se encuentran en estados estacionariossin emitir ni absorber energía. Esta absorción oemisión únicamente se produce cuando el elec-trón salta de una órbita a otra, lo que equivale amodificar el nivel energético. Así pues, la órbita enla que se encuentra el electrón nos determina el va-lor de su energía.

b) La cuantización de las energías de las diferentes ór-bitas posibles para el electrón indica que éste nopodrá tener una energía arbitraria, sino que única-mente podrá tener unos valores determinados deenergía, y que sólo podrá cambiar de una órbita aotra emitiendo o absorbiendo un fotón de unaenergía igual a la diferencia de energías entre am-bas. Por ello los espectros atómicos son discretos.

39. La expresión dualidad onda−partícula se utiliza paradescribir el doble comportamiento que presentan lamateria y la radiación. Así, en un experimento de do-ble rendija se puede observar cómo los electrones ad-quieren un comportamiento ondulatorio, mientrasque el efecto Compton nos sirve de prueba del carác-ter corpuscular de la radiación electromagnética.

a) La hipótesis de De Broglie extiende la conocida na-turaleza dual de la radiación al comportamiento dela materia. De Broglie relaciona la energía tantode la materia como de la radiación con la frecuen-cia de la onda asociada a su movimiento según laexpresión: E = h f. Asimismo, relaciona el momento

lineal con la longitud de onda

b) La comprobación experimental de este doble com-portamiento fue realizada por los físicos norteameri-canos Davisson y Germer al observar la difracción delos electrones. El resultado fue una figura de difrac-ción igual a lo que se obtendría al difractar una ondacon una longitud de onda como la predicha por DeBroglie para los electrones del experimento.

40. En el experimento de la doble rendija se observa unafigura de difracción después del impacto de muchoselectrones sobre la pantalla. Pero un único electrón noproduce el patrón de difracción, sino un único impac-to. Por lo tanto, no es posible observar el comporta-miento ondulatorio de la materia a partir del compor-tamiento de un único electrón.

41. El experimento llevado a cabo en 1932 por el físiconorteamericano A. H. Compton fue la confirmacióndefinitiva de la existencia de los fotones, así como unaprueba del comportamiento corpuscular de la radia-ción. El experimento consistía en la observación de lacolisión de un haz de rayos X de longitud de onda l so-bre una lámina de grafito. La radiación dispersada apa-recía dividida en dos haces, uno de longitud de ondamenor a la de la onda incidente, y otro con la mismalongitud de onda.

Para explicar este resultado, Compton consideró la ra-diación electromagnética formada por un conjunto departículas relativistas, los fotones, con masa en reposo

λ =hp

.

12. Física cuántica

Page 162: 9523 gl fis_tx2_cas

170

b) La microscopía electrónica ha permitido poder ob-servar la forma, la función y el comportamiento dediferentes formas de vida o estructuras microscópi-cas que no podían ser estudiadas con el micros-copio óptico o, por lo menos, no con el suficientedetalle. Es el caso, por ejemplo, de los virus, cuyotamaño está comprendido típicamente entre los20 y los 300 nm, o del estudio de la actividad de lascélulas cancerosas.

Estos estudios han redundado en una mejor com-prensión del cuerpo humano, de las enfermedadesque lo atacan y de la forma de enfrentarse a ellas.

46. Respuesta sugerida:

a) Las características fundamentales del láser, y quelo diferencian de las fuentes de radiación conven-cionales, son tres: monocromaticidad, coherenciay direccionalidad. La primera de ellas hace refe-rencia al hecho de que la radiación emergente deun láser posee únicamente una longitud de onda,al contrario de la de una bombilla, que emite luzblanca, formada por una infinidad de longitudesde onda. La coherencia es debida a que todos losátomos del láser radian en fase entre sí, mientrasque en una bombilla cada átomo radia luz inde-pendientemente. Por otro lado, la bombilla emiteen todas las direcciones del espacio, mientras queel haz láser es enfocado en haces muy estrechos enuna sola dirección mediante espejos.

b) Actualmente, se utiliza el láser para leer automáti-camente las etiquetas de los productos en los su-permercados, para operar en los hospitales, en lasimpresoras, para leer discos compactos o para cor-tar piezas en la industria.

47. Datos: λ1 = 4 ⋅10−7 m; λ2 = 7 ⋅ 10−7 m;E = 5,6 eV = 8,96 ⋅ 10−19 J; c = 3 ⋅ 108 m ⋅ s−1

h = 6,62 ⋅ 10−34 J ⋅ s

a) Calculamos el intervalo de frecuencias y utilizamosla fórmula de Planck para hallar el de energías:

fc

fm s

mHz

fc

11

1

8 1

714

2

3 10

4 107 5 10= =

⋅ ⋅

⋅= ⋅

=

−λ

λ

; ,

222

8 1

714

1 1

3 10

7 104 29 10; ,

;

fm s

mHz

E h f

=⋅ ⋅

⋅= ⋅

=

EE J s Hz

E J1

34 14

119

6 62 10 7 5 10

4 96 10

= ⋅ ⋅ ⋅ ⋅

= ⋅ =

, ,

, 33 10

6 62 10 4 28 102 2 234 14

,

; , ,

eV

E h f E J s H= = ⋅ ⋅ ⋅ ⋅− zz

E J eV2192 83 10 1 77= ⋅ =−, ,

Emisión incoherente

Estado metaestable

Emisión coherente (láser)

Haz coherente (láser)

Estado fundamental

Espejo

SemiespejoEnergía de bombeo

a)

b) Las aplicaciones de las células fotoeléctricas sonvariadas: control de la apertura y el cierre depuertas automáticas, en dispositivos de seguri-dad o recuento de unidades en cadenas de mon-taje.

45. Respuesta sugerida:

a) El funcionamiento de un microscopio electrónicose basa en las propiedades ondulatorias de los elec-trones. Estos electrones provienen de un filamentopreviamente calentado (el cañón electrónico) y sonacelerados por una diferencia de potencial muy ele-vada. Se orientan paralelamente formando un úni-co haz mediante unas lentes magnéticas de enfo-que (condensador). Estos electrones incidencontra un blanco (objeto) muy fino y a continua-ción son enfocados por una segunda lente magné-tica que equivale al objetivo de los microscopiosconvencionales. Finalmente, la tercera lente mag-nética hace de ocular y proyecta el haz de electro-nes sobre una pantalla fluorescente de modo que laimagen pueda ser observada.

El microscopio electrónico es muy similar al mi-croscopio compuesto ordinario. En el primero, lafunción de las lentes es efectuada por electroima-nes diseñados específicamente para ello.

Pero la diferencia más importante entre amboses que la resolución del microscopio electrónico esmucho mayor que la del microscopio óptico. Elloes debido a que los microscopios son capaces de re-solver detalles hasta un tamaño comparable a lalongitud de onda de la radiación empleada parailuminar el objeto; más allá de este tamaño la di-fracción nos impide distinguir los detalles. Si acele-ramos los electrones lo suficiente, obtendremosenergías elevadas y longitudes de onda asociadas aéstos muy cortas, del orden de 1 000 veces menoresa las del espectro visible. Por lo tanto, dispondre-mos de una resolución hasta 1 000 veces mayor queutilizando luz visible.

Fuente de luz

Microscopio óptico Microscopio electrónico

Fuente de electrones

CondensadorObjetivo Blanco

Proyector

Luz incidente

Batería

Fotoelectrones− +

Amperímetro

Electrodo colector

Page 163: 9523 gl fis_tx2_cas

171

49. Datos: m = 140 g = 0,14 kg; h = 6,62 ⋅ 10−34 J ⋅ sλ = 1,9 ⋅ 10−24 Å = 1,9 ⋅ 10−34 m

Utilizamos la relación de De Broglie para calcular elmomento lineal de la pelota y así obtener el valor desu velocidad:

Es imposible medir esta longitud de onda, λ = 1,9 ⋅ 10−34

m, ya que es incluso menor que las dimensiones de losradios de los núcleos atómicos, que son del orden de10−15 m.

50. Datos: me = 9,1 ⋅ 10−31 kg; mm = 65 kg; v = 1,5 m ⋅ s−1;Δv = 5 ⋅ 10−3 m ⋅ s−1; h = 6,62 ⋅ 10−34 J ⋅ s

a) Calculamos la energía de ambos debida a su movi-miento, es decir, la energía cinética:

b) Calculamos la indeterminación en el momento li-neal a partir de la indeterminación en la velo-cidad:

c) Utilizamos el principio de indeterminación de

Heisenberg, para hallar la indeter-

minación mínima en la posición de ambos:

ΔΔ

Δ

xh

p

xJ s

kg

ee

e

≥⋅ ⋅

⋅ ⋅

4

6 62 10

4 4 55 10

34

33

π

π

,

, ⋅⋅ ⋅= =

≥⋅

−m sm mm

xh

p

x

mm

m

1 0 0116 11 6

4

6 62 10

, ,

,

ΔΔ

Δ

π−−

−−⋅

⋅ ⋅ ⋅= ⋅

34

134

4 0 3251 62 10

J s

kg m sm

π ,,

Δ Δx ph

⋅ ≥4 π

,

Δ Δ Δ

Δ

p m v p kg m s

pe e e

e

= = ⋅ ⋅ ⋅ ⋅

=

− − −; ,

,

9 1 10 5 10

4 5

31 3 1

55 10

65 5 10

33 1

3

⋅ ⋅ ⋅

= = ⋅ ⋅ ⋅

− −

kg m s

p m v p kg mm m mΔ Δ Δ; ss

p kg m sm

−= ⋅ ⋅

1

10 325Δ ,

Ec m v

Ec kg m s

Ec

e e

e

e

=

= ⋅ ⋅ ⋅− −

12

12

9 1 10 1 5

2

31 1 2, ( , )

== ⋅

=

= ⋅ ⋅

−1 02 10

12

12

65 1 5

30

2

,

( ,

J

Ec m v

Ec kg m s

m m

m−−

− −

=

= = ⋅ ⋅ ⋅

1 2

31 1

73 1

9 1 10 1 5

) ,

; , ,

J

p m v p kg m s

pe e e

ee

m m m

kg m s

p m v p kg m

= ⋅ ⋅ ⋅

= = ⋅ ⋅

− −1 37 10

65 1 5

30 1,

; , ss kg m s− −= ⋅ ⋅1 197 5,

p m vh

vh

m

vJ s

kg

= = =

=⋅ ⋅

⋅ ⋅

;

,

, ,

λ λ

6 62 10

0 14 1 9 10

34

−−−= ⋅

34125

mm s

Por lo tanto, el espectro visible corresponde a las frecuencias comprendidas entre 4,29 ⋅ 1014 Hz y 7,5 ⋅ 1014 Hz, y a las energías fotónicas entre1,77 eV y 3,10 eV.

b) Calculamos la longitud de onda de un fotón de 5,6 eV:

c) El fotón pertenece a la región ultravioleta del es-pectro electromagnético.

48. Datos: λ1 = 300 nm = 3,00 ⋅10−7 m;λ2 = 450 nm = 4,50 ⋅ 10−7 m; h = 6,62 ⋅ 10−34 J ⋅ s;W0 = 3,70 eV = 5,92 ⋅ 10−19 J; c = 3 ⋅ 108 m ⋅ s−1

a) Para que la radiación produzca efecto fotoeléctrico,su energía debe ser superior al valor de la funcióntrabajo del metal, así que calculamos la energía aso-ciada a cada radiación mediante la fórmula dePlanck:

Por lo tanto, sólo producirá efecto fotoeléctrico laradiación de longitud de onda de 300 nm.

b) Calculamos ahora la velocidad máxima de los elec-trones:

Variar la intensidad de la radiación incidente nomodifica la velocidad de los electrones emitidos, yaque ésta sólo depende de la longitud de onda de laradiación.

Ech c

W m vh c

W

vm

h c

máx máx

máx

= − = −

=

;λ λ

λ

02

012

2−−

⎛⎝⎜

⎞⎠⎟

=⋅

⋅⋅ ⋅

W

vkg

J s

0

31

34

2

9 1 10

6 62 10

máx ,

, ⋅⋅ ⋅ ⋅

⋅− ⋅

⎝⎜

⎠⎟

−−3 10

3 00 105 92 10

8 1

719m s

mJ

v

,,

máxx = ⋅ ⋅ −3 9 105 1, m s

E hc

E J sm s

m

11

134

8 1

76 62 103 10

3 00 10

=

= ⋅ ⋅ ⋅⋅ ⋅

⋅−

λ

,,

EE J W

E hc

E J s

119

0

22

234

6 62 10

6 62 103

= ⋅ >

=

= ⋅ ⋅ ⋅

,

,

λ

⋅⋅ ⋅

= ⋅ <

10

4 50 10

4 41 10

8 1

7

219

0

m s

m

E J W

,

,

E h f hc

hcE

J sm s

= =

=

= ⋅ ⋅ ⋅⋅ ⋅−

,,

λ

λ

λ 6 62 103 10

8 934

8 1

66 10

2 22 10

19

7

= ⋅

J

mλ ,

12. Física cuántica

Page 164: 9523 gl fis_tx2_cas

172

c) Calculamos ahora la velocidad máxima de los elec-trones:

53. Datos: me = 9,1 ⋅ 10−31 kg; λ = 1 Å = 10−10 m;h = 6,62 ⋅ 10−34 J ⋅ s

Utilizamos la relación de De Broglie para calcular elmomento del electrón y así obtener su energía ciné-tica:

Su energía proviene del potencial que lo ha acelera-do, E = e V. Por lo tanto, lo aceleró una diferencia depotencial de 150 V.

54. La longitud de onda de De Broglie para una partículade masa relativista m viene dada por:

Por otro lado, la longitud de onda Compton es

Por lo tanto:

55. La fórmula (de Bohr) que hay que utilizar es:

donde E0 = 13,6057 eV

E EZ

nn = − 0

2

2

λ

λB

C

hv

cm v

hm c

cv

cv

cv

=

=−

=⎛⎝⎜

⎞⎠⎟

11

1

2

2

0

0

2

2 2

λCh

m c=

0

.

λBhp

hm v

hm

v

c

v

hv

cm v

= = =

=−

02

2

2

2

0

1

1

ph

pJ s

mkg m s= =

⋅ ⋅= ⋅ ⋅ ⋅

−−

λ;

,,

6 62 10

106 62 10

34

1024 −−

− −

= = =⋅ ⋅ ⋅

1

22 24 1 21

2 26 62 10

2Ec m v

pm

Eckg m s

;( , )

⋅⋅ ⋅

= ⋅ =

9 1 10

2 41 10 150

31

17

,

,

kg

Ec J eV

Ec e V m v ve Vm

Para v

D eD

emáx máx máx= = =;

;

12

22

1λ mmáx

máx

=⋅ ⋅ ⋅

=

2 1 6 10 4 732

9 1 10

1

19

31

, ,

,

,

C V

kg

v 229 10

2 1 6 10 2 919

6 1

2

19

⋅ ⋅

=⋅ ⋅ ⋅

m s

Para vC

;, ,

λ máx ,

,

V

kg

v m s

9 1 10

1 01 10

31

6 1

= ⋅ ⋅

−máx

51. a) Orbital 3f. Su número cuántico principal esn = 3, lo que quiere decir que l puede tomar losvalores l = 0, 1, 2, mientras que la letra f corres-ponde a l = 3. Por lo tanto, el orbital 3f no exis-te.

b) Orbital 5p. En este caso, n = 5, y l puede tomar losvalores l = 0, 1, 2, 3, 4. La letra p corresponde a l= 1, así que sí es posible este orbital.

c) Orbital 2s(−1). Los números cuánticos de este orbi-tal son n = 2, l = 0 y ml = −1. Sin embargo, para l = 0,sólo es posible el valor ml = 0, ya que para un l fijo,los valores de ml oscilan entre −l y +l. Este orbital noes posible.

d) Orbital 4f(+3). Este orbital tiene n = 4, l = 3 y ml = 3.Los tres valores son posibles, ya que para n = 4, l po-drá tomar los valores l = 0, 1, 2, 3. Además, paral = 3, ml podría tomar los valores ml = −3, −2, −1, 0,1, 2, 3. Por lo tanto, este orbital sí es posible.

52. Datos: λ1 = 1 850 nm = 1,85 ⋅ 10−6 m; c = 3 ⋅ 108 m ⋅ s−1;λ2 = 2 536 nm = 2,536 ⋅ 10−6 m; VD1 = 4,732 V;VD2 = 2,919 V; h = 6,62 ⋅ 10−34 J ⋅ s

a) Expresamos la ecuación de Einstein para el efectofotoeléctrico en función de la longitud de onda,

y como además Ecmáx = e VD,

tenemos:

Sustituimos las dos parejas de valores de λ y VDpara hallar los parámetros a y b:

Del valor del parámetro b obtenemos la función traba-

jo del mercurio:

W0 = −b e = 1,97 eV

b) Calculamos la frecuencia umbral mediante la rela-ción W0 = h fu, y con ella obtenemos la longitud deonda umbral:

fWh

fJ

J s

u

u

=

=⋅ ⋅

⋅ ⋅= ⋅

0

19

34

1 97 1 6 10

6 62 104 76

, ,

,, 110

3 10

4 76 106 30

14

8 1

14

Hz

cf

m s

Hz

uu

u

λ

λ

=

=⋅ ⋅

⋅= ⋅

,, 110 6307 m nm=

bWe

= − 0

4 7321

1 85 10

2 9191

2 536 10

6

6

,,

,,

V am

b

V am

=⋅

+

=⋅

−++

⎬⎪⎪

⎭⎪⎪

= ⋅

= −

b

a m

b V

1 24 10

1 97

5,

,

Vhce

We

V a bD D= − = +λ λ

0 1;

Ech c

Wmáx = − ,λ 0

Page 165: 9523 gl fis_tx2_cas

173

2. Si la radiación incidente es capaz de extraer electronesdel metal, esto significa que su frecuencia f es superiora la frecuencia umbral fu. La energía cinética de loselectrones arrancados se relaciona con esta frecuencia

mediante la expresión

Por lo tanto, a medida que aumenta la longitud deonda de la radiación incidente, disminuye la ener-gía cinética de los electrones emitidos. Sin embar-go, si la longitud de onda sigue aumentando, llega-rá a ser mayor que la correspondiente a lafrecuencia umbral, momento en el que cesará laemisión de fotoelectrones, pues la energía de la ra-diación será inferior a la necesaria para arrancarlosdel metal.

3. Datos: λ = 560 nm = 5,6 ⋅ 10−7 m; Im = 10−10 W ⋅ m−2;Øpup = 8 mm = 8 ⋅ 10−3 m; h = 6,62 ⋅ 10−34 m;c = 3 ⋅ 108 m ⋅ s−1

a) Utilizamos la fórmula de Planck expresada en fun-ción de la longitud de onda para hallar la energíadel fotón de λ = 5,6 ⋅ 10−7 m:

E h f hc

E J sm s

= = = ⋅ ⋅ ⋅⋅ ⋅

⋅−

−; ,,λ

6 62 103 10

5 6 1034

8 1

77

193 55 10

m

E J= ⋅ −,

Ec h f Wh c

Wmáx = − = − .0 0λ

EVALUACIÓN

1. Datos: λ1 = 75 pm = 7,5 ⋅ 10−11 m; λ2 = 750 nm = 7,5 ⋅ 10−7 m; λ3 = 7,5 mm = 7,5 ⋅ 10−3 m; σ = 5,67 ⋅ 10−8 W ⋅ m−2 ⋅ K−4

a) Utilizamos la ley de desplazamiento de Wien parahallar la temperatura de la cavidad:

Sustituyendo en esta expresión los valores de λ1, λ2

y λ3 obtenemos:

b) Para calcular la potencia emitida por unidad deárea utilizaremos la ley de Stefan-Boltzmann:

Sustituyendo en esta expresión los valores de T1, T2

y T3 obtenemos:

PS

W mPS

W m

P

1 23 2 2 7 21 26 10 1 264 10= ⋅ ⋅ = ⋅ ⋅− −, ; ,

33 9 21 26 10S

W m= ⋅ ⋅− −,

P T SPS

T= =σ σ4 4;

T K T K T K17

2 33 86 10 3 864 0 386= ⋅ = =, ; ; ,

λ

λ

máx

T m K

Tm K

= ⋅ ⋅

=⋅ ⋅

2 897755 10

2 897755 10

3

3

,

,

xx

12. Física cuántica

La programación de las celdas visibles (entrada de datos, devolución de resultados) es:

Para Z = 1 y n = 2 obtenemos la serie de Balmer:

Page 166: 9523 gl fis_tx2_cas

174

7. En 1927, los físicos norteamericanos C. Davisson y L.A. Germer comprobaron experimentalmente la hipó-tesis de De Broglie sobre la dualidad onda-partículade la materia al observar de forma casual la difracciónde un haz de electrones al incidir sobre una placa don-de se habían practicado dos pequeñas incisiones. Elpatrón de difracción coincidía con el que se obtendríaal difractar fotones de la misma longitud de onda quela predicha por De Broglie para los electrones.

8. Datos: me = 9,1 ⋅ 10−31 kg; mb = 30 g = 0,03 kg;Δv = 10−3 m ⋅ s−1; h = 6,62 ⋅ 10−34 J ⋅ s

a) Calculamos la indeterminación en el momento li-neal a partir de la indeterminación en la veloci-dad:

Δpe = me Δv; Δpe = 9,1 ⋅ 10−31 kg ⋅ 10−3 m ⋅ s−1

Δpe = 9,1 ⋅ 10−34 kg ⋅ m ⋅ s−1

Δpb = mb Δv; Δpb = 0,03 kg ⋅ 10−3 m ⋅ s−1

Δpb = 3 ⋅ 10−5 kg ⋅ m ⋅ s−1

Calculamos la indeterminación en la posición a par-tir del principio de incertidumbre de Heisemberg

9. n = 1. El único orbital posible es el (1, 0, 0). Así quehay 1 = 12 orbitales.

n = 2. Los orbitales posibles son (2, 1, −1), (2, 1, 0),(2, 1, 1), (2, 0, 0), es decir, 4 = 22 orbitales.

n = 3. Le corresponden los siguientes orbitales:

(3, 2, −2), (3, 2, −1), (3, 2, 0), (3, 2, 1), (3, 2, 2), (3, 1,−1), (3, 1, 0), (3, 1, 1), (3, 0, 0). En total son 9 = 32 or-bitales.

n = 4. En este caso tenemos los orbitales: (4, 3, −3), (4,3, −2), (4, 3, −1), (4, 3, 0), (4, 3, 1), (4, 3, 2), (4, 3, 3),(4, 2, −2), (4, 2, −1), (4, 2, 0), (4, 2, 1), (4, 2, 2), (4, 1,−1), (4, 1, 0), (4, 1, 1), (4, 0, 0). En total 16 = 42 orbi-tales.

n = 5. En este caso tenemos los siguientes orbitales:(5, 4, −4), (5, 4, −3), (5, 4, −2), (5, 4, −1), (5, 4, 0),(5, 4, 1), (5, 4, 2), (5, 4, 3), (5, 4, 4), (5, 3, −3), (5,3, −2), (5, 3, −1), (5, 3, 0), (5, 3, 1), (5, 3, 2), (5, 3,3), (5, 2, −2), (5, 2, −1), (5, 2, 0), (5, 2, 1), (5, 2, 2),(5, 1, −1), (5, 1, 0), (5, 1, 1), (5, 0, 0). En total 25 = 52

orbitales.

xh

px

Jb

bb≥ ≥

⋅ ⋅−34

46 62 10

ΔΔ

Δ;,

π

ss

kg m s

x mb

4 3 10

1 76 10

5 1

30

π ⋅ ⋅ ⋅ ⋅

≥ ⋅

− −

−Δ ,

ΔΔ

Δxh

px

J s

kge

ee≥ ≥

⋅ ⋅

⋅ ⋅ ⋅

−46 62 10

4 9 1 10

34

34π π;

,

, mm s

x me

−1

0 06Δ ,

Δ Δx ph

⋅ ≥4 π

:

= =⋅ ⋅ −8 13 10

4 86λ;

,f

cf

m s

⋅⋅= ⋅−10

6 17 10714

mHz,

b) Calculamos el número de fotones por segundo ne-cesarios para que el ojo detecte la radiación. Paraello comparamos la energía de un fotón con la queatraviesa la superficie de la pupila en un segundocorrespondiente a una radiación de intensidad(energía por unidad de superficie y tiempo) iguala la mínima que el ojo es capaz de detectar:

4. Datos: E = 1020 eV =16 J; h = 6,62 ⋅ 10−34 J ⋅ s; c = 3 ⋅ 108 m ⋅ s−1

Calculamos la longitud de onda del fotón gamma apartir de la fórmula de Planck expresada en función dela longitud de onda:

5. Datos: W0 = 1 eV = 1,6 ⋅ 10−19 J; h = 6,62 ⋅ 10−34 J ⋅ s;Ecmáx = 1,5 eV = 2,4 ⋅ 10−19 J

Podemos calcular la frecuencia de los fotones incidentesa partir de la expresión de Einstein para el efecto fotoeléc-trico:

6. Datos: ni = 4; nf = 2; RH = 1,096776 ⋅ 107 m−1; c = 3 ⋅ 108 m ⋅ s−1

a) Utilizamos la expresión de Bohr para calcular laenergía estacionaria de un electrón del átomo dehidrógeno en un nivel n:

b) Calculamos la frecuencia del fotón emitido me-diante la fórmula de Rydberg:

1 1 1 1 12 2 2 2λ

λ= −⎛

⎝⎜

⎠⎟ = −

⎝⎜

⎠⎟

⎣⎢R

n nR

n nHf i

Hf i

;⎢⎢

⎦⎥⎥

= ⋅ −⎛⎝⎜

⎞⎠⎟

→−

1

4 27 1

2 21 096776 101

2

1

4λ , m⎢⎢

⎦⎥

= ⋅

→−

1

4 274 86 10λ , m

EeV

n

EeV eV

n = −

= − = − = −

13 6

13 6

4

13 616

0 85

2

4 2

,

, ,,

, ,,

eV

EeV eV

eV2 2

13 6

2

13 64

3 40= − = − = −

Ec h f W fEc W

h

fJ

máxmáx= − =

+

=⋅ + ⋅−

;

, ,

00

192 4 10 1 6 10−−

−⋅ ⋅= ⋅

19

3414

6 62 106 10

J

J sHz

,

E h f hc

hcE

J sm s

= = =

= ⋅ ⋅ ⋅⋅ ⋅−

;

,

λλ

λ 6 62 103 10

1634

8 1

JJm= ⋅ −1 24 10 26,

E I S t I r t

E W m m sm m= =

= ⋅ ⋅ ⋅ ⋅ ⋅− − −( )

π

π

2

10 2 3 210 4 10 1

EE J

E n E nE

E

n

fotónfotón

= ⋅

= =

=⋅

−5 03 10

5 03 10

15,

;

, −−

−⋅=

15

193 55 1014 169

J

J

fotoness,

Page 167: 9523 gl fis_tx2_cas

17512. Física cuántica

Page 168: 9523 gl fis_tx2_cas

175

Física nuclear13

3. Datos: N = 7/8 N0; t = 1,54 días

a) Primero pasamos el tiempo de días a segundos:

Ahora sustituimos los datos en la ley de emisión ra-diactiva:

Y tomando logaritmos neperianos, resulta:

b) El período de semidesintegración T se relacionacon la constante radiactiva λ según:

Su valor en días es:

4. Para fuentes externas al organismo, la más peligrosaes la radiación γ, le sigue la radiación β, y la menos pe-ligrosa de las tres es la radiación α.

Para fuentes internas al organismo, la más peligrosa esla radiación α, le sigue la radiación β, y la menos peli-grosa de las tres es la radiación γ.

Para prevenir los peligros de la radiación hay que mini-mizar la exposición del organismo a la radiación. Estose consigue aumentando la distancia de separaciónentre la fuente radiactiva y el organismo, reduciendo eltiempo de exposición a la radiación y utilizando panta-llas o escudos protectores que eviten que la radiaciónpenetre en el organismo.

5. Respuesta sugerida:

Los principales usos de las radiaciones ionizantes son:

— En el campo de la medicina, se utilizan en el trata-miento y la diagnosis del cáncer, el examen de ór-ganos y la esterilización de material médico.

— En el campo de la industria, se emplean en radio-grafías para detectar fracturas y defectos en plan-chas de acero, en soldaduras y en materiales deconstrucción.

T sh

sdh

d= ⋅ ⋅ =6931471

36001

240 8,

T Ts

s= ⇒ =⋅

=− −

ln ln

,

2 2

1 0 106931476 1λ

ln ln

,

78

1330561

13305687

1 0 10 6

= − ⋅ ⇒ =

= ⋅ −

λ λ

λ

ss

ss−1

N N e N N et s= ⋅ ⇒ = ⋅− − ⋅0 0 0

13305678

λ λ

t dh

ds

hs= ⋅ ⋅ =1 54

241

36001

133056,

PREPARACIÓN DE LA UNIDAD

• Los elementos químicos correspondientes a los distintosnúmeros atómicos son: Z = 2: helio (He); Z = 13: alumi-nio (Al); Z = 26: hierro (Fe); Z = 48: cadmio (Cd); Z = 62:samario (Sm); Z = 84: polonio (Po), y Z = 92: uranio (U).

• Un megaelectronvoltio (MeV) es una unidad de ener-gía igual a 106 electronvoltios (eV). A su vez, un elec-tronvoltio es el valor absoluto de la energía que ad-quiere un electrón cuando es acelerado a lo largo deuna diferencia de potencial eléctrico de un voltio. Esdecir:

Esto es, un electronvoltio equivale a 1,6 ⋅ 10–19 julios (J).

Por tanto, un megaelectronvoltio equivale a:

106 eV ⋅ (1,6 ⋅ 10−19 J/1 eV) = 1,6 ⋅ 10−13 J.

1. RADIACTIVIDAD

1. Las sustancias radiactivas pueden emitir radiaciones ca-paces de penetrar en cuerpos opacos, ionizar el aire ha-ciéndolo conductor, impresionar placas fotográficas yexcitar la fluorescencia de ciertas sustancias. Las sus-tancias radiactivas, además, pueden producir cambiosquímicos en la materia orgánica.

2. Tanto la radiación α como la β están asociadas a par-tículas materiales, mientras que la radiación γ es unaforma de radiación electromagnética.

— La radiación α está formada por núcleos de helio 4(es decir, conjuntos de dos neutrones y dos protonesque reciben el nombre de partículas alfa). Por tanto,cada partícula α posee la carga eléctrica correspon-diente a dos protones, Q = +2e = +3,2 ⋅ 10−19 C, y lamasa de un núcleo de helio 4, m = 6,7 ⋅ 10−27 kg.

— La radiación β está formada por electrones rápidosprocedentes de la desintegración de neutrones delnúcleo. Así pues, cada partícula β posee la carga eléc-trica y la masa de un electrón (Q = −e = −1,6 ⋅ 10−19 C;m = 9,1 ⋅ 10−31 kg).

— La radiación γ es una radiación electromagnéticade mayor frecuencia (y menor longitud de onda)que los rayos X. Las partículas asociadas a la radia-ción γ son fotones con la correspondiente frecuen-cia. Por tanto, el valor de su masa y su carga eléctri-ca es nulo (Q = 0; m = 0).

1 1 6 10 1 1 6 1019 19, ,eV C V J= − ⋅ ⋅ = ⋅− −� �

13. Física nuclear

Page 169: 9523 gl fis_tx2_cas

176

3. REACCIONES NUCLEARES

10. La reacción nuclear asociada a la emisión de partícu-las α es:

Esta reacción indica que cuando un núcleo padre (desímbolo N) con número atómico Z y número másico A

emite una partícula se transforma en un nú-

cleo hijo (de símbolo Y). El número atómico del núcleohijo es dos unidades inferior al del núcleo padre; y elnúmero másico del núcleo hijo es cuatro unidades in-ferior al del núcleo padre.

La reacción nuclear asociada a la emisión de partícu-las β es:

Esta reacción indica que cuando un núcleo padre (desímbolo N) con número atómico Z y número másico A

emite una partícula se transforma en un núcleo

hijo (de símbolo Y). El número atómico del nú-cleo hijo es una unidad superior al del núcleo padre;y el número másico del núcleo hijo es igual al del núcleopadre.

En ambos tipos de reacciones nucleares, la suma delos números atómicos y la suma de los números mási-cos son iguales en uno y otro miembro de la reacción.

11. Datos: Z = 92

Las reacciones sucesivas que tienen lugar son:

12. La fisión nuclear consiste en la división de un núcleode gran masa en otros dos núcleos más ligeros cuandoel núcleo pesado es bombardeado con neutrones. Eneste proceso se libera gran cantidad de energía y másneutrones. Los neutrones liberados pueden fisionarotros núcleos pesados dando lugar a una reacción encadena. En las centrales nucleares se produce fisiónnuclear en cadena controlada. La explosión de lasbombas atómicas de fisión es un ejemplo de fisión nu-clear en cadena fuera de control.

La fusión nuclear consiste en la unión de dos nú-cleos ligeros para formar otro más pesado. En esteproceso se libera gran cantidad de energía (superiora la reacción de fisión). Las reacciones de fusión encadena se producen en las estrellas gracias a las altastemperaturas y presiones de su interior. El ser huma-no aún no ha conseguido producir de forma renta-ble la fusión nuclear en cadena controlada. La ex-plosión de las bombas atómicas de hidrógeno es unejemplo de fusión nuclear en cadena fuera de con-trol.

13. Datos: P = 1 200 MW; t = 1 año = 3,1536 ⋅ 107 s;Ar(U) = 235,0439 u; NA = 6,022 ⋅ 1023

92238

01

92239

93239

10

94239

10U n U Np e Pu e+ → → + → +− −

β −( )10e ,

ZA

ZAN Y e→ ++ −1 1

0

α 24 He( ),

ZA

ZAN Y He→ +−

−24

24

ΔEA

MeVMeV= =

1692 18226

7 5,

,— En el campo de la química, se utilizan para fabricar

productos químicos y para estudiar los mecanismosde reacción.

— En otros campos, se utilizan para esterilizar espe-cies nocivas (agricultura), para datar muestras or-gánicas (paleontología) y para la fabricación derelojes atómicos de precisión y generadores auxi-liares para satélites artificiales (ingeniería).

6. Respuesta sugerida:

Los radicales libres son moléculas químicas neutrascon un electrón desapareado que no forma parte deun enlace químico. Por esta razón son moléculas muyactivas, ya que intentan aparear su electrón libre. Alatraer electrones de otras moléculas, provocan la oxi-dación de estas últimas.

Las partículas α provocan la formación de radicales li-bres procedentes de moléculas de agua del organismo;y estos radicales libres reaccionan con moléculas com-plejas de tejidos.

2. EL NÚCLEO ATÓMICO

7. El núcleo atómico está formado por protones y neutro-nes. Ambos tipos de partículas reciben el nombre denucleones.

8. La energía de enlace por nucleón es el cociente en-tre la energía de enlace del núcleo y el número totalde nucleones que forman el núcleo (número má-sico).

El orden de magnitud de la energía de enlace por nu-cleón es de varios MeV. Su valor medio es, aproximada-mente, de 8,3 MeV.

9. Datos: A(Ra) = 226; Ar(Ra) = 226,0254 u; Z(Ra) = 88;mp = 1,0073 u; mn = 1,0087 u

a) El defecto de masa Δm vale:

Sustituimos los valores para el radio 226, tomandocomo masa nuclear MN, la masa atómica Ar:

b) Calculemos primero la energía de enlace ΔE, te-niendo en cuenta que la energía asociada a unamasa de 1 u es de 931 MeV:

El valor de la energía de enlace por nucleón es, portanto:

ΔE uMeVu

MeV= ⋅ =1 8176931

11 692 18, ,

Δm u u= ⋅ + − ⋅ −[ , ( ) , ] ,88 1 0073 226 88 1 0087 226 0254

,

u

m uΔ = 1 8176

Δm Z m A Z m Mp n N= + − −[ ( ) ]

Protones Neutrones

Masa 1,673 · 10–27 kg mn = 1,675 · 10−27 kg

Carga +e = +1,602 ⋅ 10–19 C 0 C

Page 170: 9523 gl fis_tx2_cas

177

Emín = 2 mp c2 = 2 ⋅ 1,67 ⋅ 10−27 kg (3 ⋅ 108 m ⋅ s-1)2

Emín = 3,01 ⋅ 10−10 J

Así pues, la frecuencia mínima del fotón es:

FÍSICA Y SOCIEDAD

a) El período de semidesintegración del carbono 14 esde 5 730 años. Así, en un período de tiempo muchomayor, por ejemplo, de treinta mil años, transcurren:30 000/5 730 = 5,24 períodos de semidesintegración.

Es decir, treinta mil años equivale a más de cinco vecesel período de semidesintegración. Al cabo de cinco pe-ríodos de semidesintegración, la cantidad de carbono 14del resto orgánico se ha reducido a (1/2)5 = 1/32 par-tes de su valor inicial, ya de por sí pequeño. En estascondiciones, la técnica del carbono 14 es poco reco-mendable por la poca cantidad de carbono 14 presenteen la muestra y porque el error en la medida es com-parable al valor medido.

b) El cobalto 60 es una fuente de rayos γ. Dado que éstaes el tipo de radiación más penetrante, se usa comofuente externa para penetrar en el organismo y llegarhasta los tejidos y órganos afectados de cáncer.

El yodo 131 emite rayos β y γ. En este caso debe usar-se como fuente interna ya que los rayos β no tienentanto poder de penetración como los rayos γ. Por tan-to, la fuente de radiación β debe situarse cerca delórgano o tejido que se va a tratar. Usando el yodo 131como fuente interna, gran parte de la radiación γemitida atraviesa el cuerpo humano sin interactuar,pero, en cambio, casi toda la radiación β puede ac-tuar en las células cancerígenas.

c) Respuesta sugerida:

Para organizar el debate, puede procederse de la si-guiente manera:

— Determinar los encargados de las diferentes fun-ciones:

• Ponentes. Defenderán una determinada postu-ra exponiéndola y argumentándola.

• Oponentes. Rebatirán las opiniones de los po-nentes con sus propios argumentos y opiniones.

Tanto los ponentes como los oponentes deben in-vestigar y documentarse sobre el tema con ante-rioridad al debate.

• Moderador. Presentará el tema y las opiniones deambos grupos. Además, concederá los turnosde palabra para que el debate se desarrolle deforma ordenada. Guiará el debate para que nose produzcan desórdenes y todos tengan las mis-mas oportunidades de participar.

• Secretario. Tomará nota de las diversas opinio-nes expuestas y resumirá los argumentos pre-sentados y las conclusiones al final del debate.

fE

hJ

J smínmín= =

⋅ ⋅= ⋅

3 01 10

6 62 104 5 1

10

34

,

,, 0023 Hz

Calculemos primero la energía E liberada en un año(suponiendo que el reactor funciona sin interrupcio-nes):

El valor de esta energía en megaelectronvoltios es:

Calculamos la masa de uranio 235 que se precisa paraobtener esta energía, teniendo en cuenta que en la fi-sión de un núcleo de uranio 235 se liberan 200 MeV:

14. La principal dificultad técnica que presenta la fusiónnuclear controlada es la del confinamiento del mate-rial que se debe fusionar. Este material ha de llevarse amuy altas temperaturas para conseguir la energía deactivación necesaria para la fusión. El problema es quea estas temperaturas los reactivos se encuentran en es-tado de plasma y es difícil su confinamiento en un re-cipiente o espacio como el reactor de fusión.

15. Respuesta sugerida:

La fusión nuclear controlada, cuando se logre explo-tar de forma rentable, presentará frente a la fisión lasventajas de que sus productos de reacción no son con-taminantes y, además, se dispone de enormes reservasde combustible para la fusión controlada (el hidróge-no del agua de los océanos). Además, en la fusión seobtiene (a partir de la misma cantidad de masa dereactivos) una energía más de tres veces superior queen la fisión.

4. PARTÍCULAS SUBATÓMICAS Y FUERZASFUNDAMENTALES

16. Respuesta sugerida:

Las partículas subatómicas más conocidas son el elec-trón (cuya antipartícula es el positrón), el protón (cuyaantipartícula es el antiprotón), el neutrón (cuya anti-partícula es el antineutrón) y el fotón (cuya antipartí-cula es el mismo fotón).

17. Los seis tipos de leptones que existen son: el electrón,el muón, el tauón, el neutrino del electrón, el neutrinodel muón y el neutrino del tauón.

Los seis tipos de quarks que existen son: up, down, stran-ge, charmed, bottom y top.

18. Datos: mp = 1,67 ⋅ 10−27 kg; h = 6,62 ⋅ 10−34 J ⋅ s;c = 3 ⋅ 108 m ⋅ s−1

La energía mínima que debe tener un fotón para gene-rar un par protón-antiprotón es la energía asociada a lamasa de las dos partículas. Teniendo en cuenta que lasdos tienen la misma masa:

m MeVnucl

MeVg

= ⋅ ⋅ ⋅2 6352 101

200235 043929,

. ,

66 022 10

4 616 10 461 6

23

5

, .

, ,

= ⋅ =

nucl

m g kg

E JeV

J

E

= ⋅⋅

= ⋅

−3 7843 101

1 6 10

2 3652 10

1619

35

,,

, ,eV MeV= ⋅2 3652 1029

E P t W s

E J

= = ⋅ ⋅ ⋅

= ⋅

, ,

,

1 2 10 3 1536 10

3 7843 10

9 7

16

13. Física nuclear

Page 171: 9523 gl fis_tx2_cas

178

Teniendo en cuenta que las masas m y m0 se rela-cionan con el número de núcleos en el instantet (N) y en el instante inicial (N0) según:

donde M es la masa

molar y NA la constante de Avogadro, la ley deemisión radiactiva se puede escribir como:

Sustituyendo valores y tomando logaritmos nepe-rianos hallamos el valor de t:

Es decir, t = 990 210 s = 11,46 días.

En este caso concreto, se habría podido obtener elresultado de una manera más directa a partir delvalor del cociente entre m0 y m:

Esto significa que la masa se ha reducido a la mi-tad tres veces sucesivas. Por tanto, ha transcurridoun tiempo t que es tres veces el período de semi-desintegración:

b) Calculamos el número de núcleos iniciales, N0, yfinales, N.

Calculamos ahora los valores de la actividad ini-cial, A0, y final A.

A0 = λ N0 = 2,1 ⋅ 10−6 s−1 ⋅ 5,42 ⋅ 1018

A0 = 1,14 ⋅ 1013 Bq

A = λ N = 2,1 ⋅ 10−6 s−1 ⋅ 6,78 ⋅ 1017 = 1,42 ⋅ 1012 Bq

20. Datos: M = 209,9829 u; m0 = 2 ⋅ 10−3 g; m = 5 ⋅ 10−4 g

a) Sustituimos los datos del enunciado en la ley de emisión radiactiva para hallar la constante radiac-tiva λ:

m m e

g g e

t

s

=

⋅ = ⋅ ⋅ ⇒

− − − ⋅ ⋅

0

4 3 2 38 105 10 2 107,

λ

λ

lln ,

,

5 10

2 102 38 10

5 82 10

4

37⋅

⋅= − ⋅ ⋅

= ⋅

g

gsλ

λ 88 1s−

t dhoras

d horas

= ⋅ ⋅ ⋅ =27624

1601

601

minmin

22 38 107, ⋅ s

Nm N

Mg

gA

00

3 232 10 6 022 10222 0175

5 4= =⋅ ⋅ ⋅

=− ,

,, 22 10

2 5 10 6 022 10222 017

18

4 23

= =⋅ ⋅ ⋅−

Nm N

MgA , ,, 55

6 78 1017,g

= ⋅

t T d d= = ⋅ =3 3 3 82 11 46, ,

mm

g

g0

4

3

32 5 10

2 100 125

12

=⋅

⋅= =

⎛⎝⎜

⎞⎠⎟

,,

0 25 10 2 10

0

3 3 2 1 10 6 1

,

ln,

,⋅ = ⋅ ⋅ ⇒

− − − ⋅ ⋅− −

g g e s t

2252

2 1 10

20 25

2 1 10

6 1

6 1

= − ⋅ ⋅ ⇒

⇒ =⋅

− −

− −

,

ln,

,

s t

ts

== 990 210 s

m m e t= −0

λ

mN MN

mN MNA A

= =; ;00

• Público. Atenderá a las diversas opiniones. Podráintervenir al final del debate exponiendo suspropios argumentos y valorando qué grupo hasido más convincente y cuál ha defendido mejorsu postura.

Los ponentes defenderán la postura de que la ra-diactividad presenta más beneficios que peligros.Pueden apoyarse en las múltiples aplicaciones de laenergía nuclear, principalmente como fuente deenergía alternativa a los combustibles fósiles. Gra-cias a su desarrollo, los países que no cuentan conreservas de carbón y petróleo no han tenido quedepender de las demandas arbitrarias de los paísesproductores de éstos. Además, con pocas excepcio-nes, las centrales nucleares garantizan la seguridadde la población por la gran cantidad de medidas deseguridad y control que poseen.

Los oponentes rebatirán las opiniones anterioresafirmando que la radiactividad presenta más peli-gros que beneficios. Se pueden apoyar en los efec-tos de las explosiones de bombas atómicas y de losaccidentes nucleares. También se opondrán a lascentrales nucleares por los desechos radiactivosque producen, que conllevan un importante pro-blema de almacenamiento. Además, la rápidaproliferación de centrales de fisión a partir de ladécada de los setenta ha perjudicado el desarrolloy la investigación de otras fuentes de energíaalternativas.

El moderador del debate puede intentar un acer-camiento de ponentes y oponentes introduciendola discusión sobre la fusión nuclear. Se puede con-siderar tanto la fusión natural (es decir, la que tie-ne lugar en el Sol y permite la vida en la Tierra)como la fusión artificial controlada (aún no conse-guida).

— Iniciar el debate. El moderador presentará el temay dará la palabra en primer lugar a alguno de losponentes.

— Desarrollar y concluir el debate. Los distintos po-nentes y oponentes desarrollarán sus argumentos,conducidos por el moderador.

Al final del debate, el público podrá expresar susopiniones. Por último, el secretario resumirá lasconclusiones.

RESOLUCIÓN DE EJERCICIOS Y PROBLEMAS

19. Datos: M = 222,0175; m0 = 2 ⋅ 10−3 g; m = 2,5 ⋅ 10−4 g

a) La ley de emisión radiactiva es:

Hallamos primero la constante radiactiva λ a partirdel período de semidesintegración:

λ = = = ⋅ − −ln ln,

2 2330048

2 10 10 6 1

T ss

N N e t= −0

λ

T dhoras

d horas

= ⋅ ⋅ ⋅3 8224

1601

601

,min

min== 330048 s

Page 172: 9523 gl fis_tx2_cas

179

Se trata de una reacción de emisión de partículasα inducida por el bombardeo de núcleos de nitró-geno con protio (hidrógeno 1).

22.

a) La reacción nuclear dada: esuna reacción de fusión nuclear.

b) Hallamos el defecto de masa Δm asociado a lareacción nuclear anterior:

Es decir, al producirse la reacción tiene lugar unapérdida de masa de 0,0212 u por cada átomo de

reaccionante; por tanto, se libera una ener-

gía E de valor:

23. Datos:

Isótopo inicial: 23592U

Isótopos finales: 13250Sn, 101

42Mo

Escribimos la reacción nuclear que tiene lugar y de-jamos indeterminado el número de neutrones libera-dos.

23592U + 1

0n → 13250Sn + 101

42Mo + x 10n

Efectuamos el balance de los números atómicos y losnúmeros de masa.

La reacción de fisión es:

23592U + 1

0n → 13250Sn + 101

42Mo + 3 10n

Se liberan tres neutrones por núcleo de uranio.

24. Datos: E = 210 MeV/núcleo

P = 6 700 kW = 6,7 ⋅ 106 W

r = 58 % = 0,58

t = 1 día = 86 400 s

Calculamos las energías producida y consumida por elreactor en un día.

Aplicamos los factores de conversión necesarios parahallar la masa de uranio consumida.

E P tproducida = ⋅ = ⋅ ⋅ ⋅ =

= ⋅

6 7 10 8 64 10

5 79 1

6 4, ,

,

W s

00

1 10058

5 79 10

11 J

Er

Econsumida producida= ⋅ = ⋅ ⋅, 111

119 98 10

J

J

=

= ⋅,

235 1 132 101 1

92 0 50 42 03

+ = + + ⋅

+ = + + ⋅

⎫⎬⎭

=x

xx

E uMeVu

MeV= ⋅ =0 0212931

119 7, ,

11H( )

Δ

Δ

m M H M H M He

m u

= + −

= +

( ( ) ( )) ( )

( , ,11

13

24

1 0078 3 01660 4 0026 0 0212) , ,u u u− =

11

13

24H H He+ → ,

Datos M H u M H u: , ; ,11

131 0078 3 0160( ) = ( ) = ;;

,M H u24 4 0026( ) =

El período de semidesintegración vale:

El resultado que obtenemos es muy similar al valorreal del período de semidesintegración del polo-nio 210, que se puede consultar en las tablas y es de139 días.

b) Calculamos el número de núcleos iniciales, N0, y fi-nales, N.

Calculamos ahora los valores de la actividad inicial,A0, y final A.

A0 = λ N0 = 5,82 ⋅ 10−8 s-1 ⋅ 5,73 ⋅ 1018 = 3,3 ⋅ 1011 Bq

A = λ N = 5,82 ⋅ 10−8 s−1 ⋅ 1,43 ⋅ 1018 = 8,3 ⋅ 1010 Bq

21. a)

Toda reacción nuclear debe cumplir que la sumade los números atómicos y la suma de los númerosmásicos en ambos miembros de la reacción seaniguales. Esto es:

Suma de números atómicos:

13 + 2 = Z + 0 ⇒ Z = 15

Suma de números másicos:

27 + 4 = A + 1 ⇒ A = 30

El elemento de número atómico 15 es el fósforo.Además, como su número másico es 30, se trata delfósforo 30. La reacción es, pues:

Se trata de una reacción de emisión de neutronesinducida por el bombardeo de núcleos de alumi-nio con partículas α.

b)

Análogamente al caso anterior, tenemos:

Suma de números atómicos:

7 + 1 = 2 + Z ⇒ Z = 6

Suma de números másicos:

14 + 1 = 4 + A ⇒ A = 11

El elemento de número atómico 6 es el carbono.Además, como su número másico es 11, se trata delcarbono 11. La reacción es, pues:

714

11

24

611N H He C+ → +

714

11

24N H He XZ

A+ → +

1327

24

1530

01Al He P n+ → +

1327

24

01Al He X nZ

A+ → +

Nm N

Mg

gA

00

3 232 10 6 022 10209 9829

5 7= =⋅ ⋅ ⋅

=− ,

,, 33 10

5 10 6 022 10209 9829

18

4 23

= =⋅ ⋅ ⋅−

Nm N

MgA ,, gg

= ⋅1 43 1018,

Ts

s

T

= =⋅

= ⋅

=

− −

ln ,

,,

,

2 0 693

5 82 101 19 10

1 19

8 17

λ

⋅⋅ ⋅ ⋅ ⋅ =10160

160

124

1387 minmin

ss

h dhoras

d

13. Física nuclear

Page 173: 9523 gl fis_tx2_cas

180

EJERCICIOS Y PROBLEMAS

27. La radiactividad consiste en la emisión de radiacionesionizantes procedentes de los núcleos atómicos inesta-bles de ciertas sustancias denominadas radiactivas.

Las radiaciones ionizantes pueden ser de tres tipos:radiación α (son núcleos de helio), radiación β (sonelectrones de gran energía cinética, del orden delMeV, procedentes de la desintegración de neutronesdel núcleo) y radiación γ (es una radiación elec-tromagnética de frecuencia superior a la de los ra-yos X).

28. El individuo B corre mayor peligro que el individuo Aporque, dentro del organismo, una fuente radiactivaemisora de radiación α es más peligrosa que una fuen-te radiactiva emisora de radiación γ.

29.

30. Existen dos tipos de fuerzas nucleares:

— La fuerza nuclear fuerte es una fuerza de atracciónentre cualquier tipo de nucleones (ya sean proto-nes o neutrones). Es la responsable de la cohesióndel núcleo.

— La fuerza nuclear débil es una fuerza que actúa entodo tipo de partículas, aunque sus efectos sonmás apreciables en las partículas no sometidas a lainteracción nuclear fuerte. Es la responsable de laemisión β.

Estas fuerzas son de muy corto alcance: la fuerzanuclear fuerte es nula para distancias superiores a10−15 m, y la fuerza nuclear débil es nula para distan-cias superiores a 10−17 m.

A las distancias donde las fuerzas nucleares no se anulan,la fuerza nuclear fuerte es superior en intensidad a lafuerza electromagnética, mientras que la fuerza nucleardébil es inferior en intensidad a la fuerza electromagné-tica.

31. La masa de un núcleo es siempre inferior a la sumade las masas que tienen los protones y neutrones aisladosque se unen para formar el núcleo. La diferencia demasas recibe el nombre de defecto de masa (Δm).

La energía liberada al formarse un núcleo se deno-mina energía de enlace y proviene del defecto demasa; es decir, de la masa que pierden sus nucleonesal enlazarse para formar el núcleo. La energía de en-

Átomo Núcleo

Masa

MátomoLa masa atómica esdel orden de launidad de masaatómica, u.1 u = 1,661 · 10–27 kg

Mnúcleo ≥ 0,99 Mátomo

Volumen

VátomoEl átomo tiene unradio aproximado de10−10 m.

Vnúcleo ∼∼10−5 Vátomo

El núcleo tiene unradio aproximado de10–15 m.

Nivelesenergéticos

Son del ordendel eV.

Son del ordendel MeV.

25. Datos:

Isótopos iniciales: 21H, 2

1H

Isótopos finales: AZX,

11H

E � 4,03 MeV/núcleo

a) Escribimos la reacción nuclear y efectuamos el ba-lance de los números atómicos y los números demasa.

El número atómico Z � 1 corresponde al hidró-geno. El isótopo que falta es, pues, 3

1H. La ecua-ción corresponde a una reacción nuclear de fu-sión.

b) Aplicamos los factores de conversión necesariospara hallar la energía liberada por kg de 3

1H.

26. Datos:

Isótopos iniciales: 21H, A

ZX

Isótopo final: 42He

E � 17,59 MeV/núcleo

a) Escribimos la reacción nuclear y efectuamos el ba-lance de los números atómicos y los números demasa.

El isótopo que falta es el 31H. La reacción nuclear esuna fusión.

b) Aplicamos los factores de conversión necesariospara hallar la energía liberada en la formación de3 kg de helio.

EMeV

núcleonúcleos

mol= ⋅

⋅⋅17 59

6 022 101

123

,, mmol

g

g

kgkg MeV

4

1 000

13 7 94 1027,

⋅ ⋅ = ⋅

12

24

01

2 4 1 4 1 2 3

1

H X He nZA

;

+ → +

+ = + = + − =

+ =

A A

Z 22 0 2 0 1 1+ = + − =; Z

EMeV

núcleonúcleos

molm

= ⋅⋅

⋅4 036 022 10

1123

,, ool

g

g

kgkg MeV

3

1 000

11 5 1 21 1027, ,

⋅ ⋅ = ⋅

12

12

11 2 2 1 3

1 1 1 1

H H X HZA ;

;

+ → + + = + =

+ = + =

A A

Z Z

9 98 101

1 6 10

1

10

1

1119 6,

,⋅ ⋅

⋅⋅ ⋅

−JeV

J

MeV

eV

núccleo UMeV

mol U

núcl,

235 235

232101

6 022 10⋅

⋅ eeos U

g U

mol Ug,

235

235

235235235

111 6

⋅ = UU

Page 174: 9523 gl fis_tx2_cas

181

culas elementales pueden unirse para formar partícu-las más complejas.

Las antipartículas se definen en relación con las partí-culas, de forma que la antipartícula de una partículadada es otra partícula de masa y espín iguales que laprimera pero con carga eléctrica y momento angularde signo contrario a los de la partícula.

Las partículas elementales se clasifican en leptones(no están sometidas a la interacción nuclear fuerte) yquarks (sí están sometidas a la fuerza nuclear fuerte).Los quarks no existen aislados, sino que se combinanpara formar otras partículas, los hadrones.

Las antipartículas se clasifican en leptones (no someti-das a la fuerza nuclear fuerte) y en mesones (partícu-las sí sometidas a la fuerza nuclear fuerte).

36. Las fuerzas fundamentales de la naturaleza son: lafuerza gravitatoria, la fuerza electromagnética, la fuer-za nuclear fuerte y la fuerza nuclear débil.

Tanto la fuerza gravitatoria como la fuerza electro-magnética son de largo alcance (sus efectos se perci-ben a escalas astronómicas). En cambio, las fuerzasnucleares son de corto alcance (sus efectos se perci-ben sólo a escalas nucleares: a una distancia menor de10−15 m para la nuclear fuerte y a una distancia menorde 10−17 m para la débil).

A distancias menores de 10−17 m donde la fuerza nu-clear débil no es nula, la intensidad de los distintos tiposde fuerza decrece en este orden: fuerza nuclear fuerte,fuerza electromagnética, fuerza nuclear débil y fuerzagravitatoria.

37. Datos: en unidades SI

Comparando con la ley de emisión radiactiva:

se deduce el valor de la constante radiac-tiva λ de la muestra: λ = 2,1 ⋅ 10−6 s−1

Por tanto, el período de semidesintegración T vale:

38. Datos: m0 = 3 ⋅ 10−3 g;

a) Hallamos la constante radiactiva del yodo 131, λ, yconocida ésta, aplicamos la ley de emisión radiac-tiva para hallar el tiempo t que tarda la muestra enreducirse a 0,5 g:

λλ

= =⋅

= ⋅

=

− −ln ,

,,

2 0 693

6 91 101 00 105

6 1

0

ss

m m e−−

− − −

⋅ = ⋅ ⋅ ⇒

⇒⋅

− −

λt

s tg g e5 10 3 10

5 10

4 3 10 6 1

ln44

36 1 6

6

3 1010 1 79 10

1 79 10

⋅= − ⇒ = ⋅

= ⋅

−− − ,

,

s t t s

tmin

min,s

sh d

hd⋅ ⋅ ⋅ =

160

160

124

20 7

T dh

d hs

= ⋅ ⋅ ⋅ = ⋅8241

601

601

6 91 105minmin

, ss

Ts

s= =⋅

= ⋅− −

ln ,

,,

2 0 693

2 1 103 3 106 1

5

λ

N N e t= −0 ,λ

N N e t= − ⋅ −

02 1 10 6,

lace ΔE se relaciona con el defecto de masa Δm me-diante la fórmula de Einstein:

ΔE = Δm c2

32. Una reacción nuclear es un proceso mediante el cualnúcleos atómicos se transforman en otros distintos.

Las diferentes emisiones radiactivas que dan lugar al fe-nómeno de la radiactividad se deben a reacciones nu-cleares en núcleos atómicos que son inestables:

— En el caso de la emisión α, un núcleo atómico ines-

table se transforma en otro más estable

y emite una partícula α

— En el caso de la emisión β, un núcleo atómico ines-

table se transforma en otro más estable

y emite una partícula

— En el caso de la emisión γ, un núcleo que se hallaen un nivel energético excitado pasa a otro nivelmenos energético y emite la diferencia de energíaen forma de radiación electromagnética.

33. La fisión nuclear es una reacción nuclear en la que unnúcleo pesado se divide en dos núcleos más ligeros alser bombardeado con neutrones. En el proceso se libe-ran más neutrones y gran cantidad de energía.

El ejemplo típico de reacción de fisión es la fisión deluranio 235:

En esta reacción se liberan 200 MeV por átomo de ura-nio 235.

En las centrales nucleares, la energía se obtiene me-diante fisión nuclear en cadena controlada de uranio235 o plutonio 239. En la fisión en cadena dentro delreactor de la central, los neutrones liberados en la fi-sión de un átomo pueden fisionar a más átomos. Todoel proceso se controla mediante un controlador quesirve para capturar el exceso de neutrones en caso deque la velocidad de reacción sea muy rápida y haya ries-go de explosión nuclear.

34. La fusión nuclear es una reacción nuclear en la quedos núcleos ligeros se unen para formar otro más pesa-do. En este proceso se libera gran cantidad de energía.

Un ejemplo de reacción de fusión es la fusión del pro-tio y el tritio:

En esta reacción se liberan 19,8 MeV de energía porátomo de helio 4.

La fusión nuclear en cadena controlada aún no se uti-liza como fuente de energía porque no se ha consegui-do llevar a cabo de forma rentable, debido a la dificul-tad técnica que supone confinar los reactivos, que, atemperaturas tan elevadas, están en estado de plasma.

35. Las partículas elementales son aquellas que no puedenser descompuestas en otras más simples. Algunas partí-

11

13

24H H He+ →

92235

01

56141

3692

013U n Ba Kr n+ → + +

β −( )10e .

ZA Y+( )1Z

A N( )

24 He( ).Z

A Y−−( )2

4

ZA N( )

13. Física nuclear

Page 175: 9523 gl fis_tx2_cas

182

b) Calculamos la vida media τ del radioisótopo a partirde su relación con el período de semidesintegra-ción T:

41. Datos: Ar(16O) = 15,9949 u; A = 16; Z = 8;mp = 1,0073 u; mn = 1,0087 u

a) Tomamos como valor de la masa nuclear del oxí-geno 16 el valor de su masa atómica por el peque-ño valor de la masa de los electrones:MN(16O) ≈ Ar(16O), y sustituimos los datos delenunciado en la expresión para el defecto demasa Δm:

b) La energía de enlace ΔE puede calcularse a partirde la relación ΔE = Δm c2, o bien de forma más di-recta a partir del equivalente energético de la uni-dad de masa atómica (1 u = 931 MeV):

c) La energía de enlace por nucleón es:

42. Datos: A = 234; Z = 90

a) Al emitirse la primera partícula β, el torio 234 setransforma en protactinio 234, ya que su númeroatómico Z se incrementa en una unidad y su nú-mero másico A no varía:

Análogamente, al emitirse la segunda partícula β,el protactinio 234 pasa a ser uranio 234:

Al emitirse la primera partícula α, el uranio 234 seconvierte en torio 230, ya que su número atómico Z = 92 se reduce en dos unidades y su número má-sico A = 234 se reduce en cuatro unidades:

Análogamente, al emitirse la segunda partícula α,el torio 230 se transforma en radio 226:

La reacción global es:

b) El isótopo resultante es el radio 226.

43. Datos C H H C E MeV

A Cr

: 612

12

11

613

12

2 71+ → + =; , ;

( )) ; ( ) , ; ( ) ,= = =12 1 0078 2 01411 2u A H u A H ur r

90234

88226

10

242 2Th Ra e He→ + +−

90230

88226

24Th Ra He→ +

92234

90230

24U Th He→ +

91234

92234

10Pa U e→ + −

90234

91234

10Th Pa e→ + −

ΔEA

MeVMeV= =

12316

7 74,

ΔE uMeVu

MeV= ⋅ =0 1331931

1123 9, ,

Δ

Δ

m Zm A Z m M

m up n N= + − −

= ⋅ + − ⋅

( ( ) )

( , ( ) ,8 1 0073 16 8 1 00087 15 9949

0 1331

) ,

,

u u

m u

τ = = =T

ln, min,

, min2

2 50 693

3 6

b) Para que la actividad A se reduzca a la cuarta partede su valor inicial, el número de núcleos debe re-ducirse en la misma proporción:

puesto que la actividad y el número de núcleos serelacionan según: A = λ N.

El tiempo t necesario para que el número de núcle-os se reduzca a la cuarta parte del valor inicial es,pues, de dos períodos de semidesintegración T, yaque el número de núcleos ha de reducirse a la mi-tad dos veces sucesivas:

t = 2 T = 2 ⋅ 8 d = 16 d

39. Datos: m = (9/10) m0; t1 = 20 s

a) Sustituimos los datos del enunciado en la ley deemisión radiactiva escrita en términos de las masasm y m0, para hallar la constante radiactiva λ de lamuestra:

Y ahora calculamos el período de semidesintegra-ción T de la muestra radiactiva:

b) A partir de la ley de emisión radiactiva:y teniendo en cuenta la relación en-

tre actividad y número de núcleos en los distintosinstantes de tiempo: A0 = λ N0; A = λ N, la ley deemisión radiactiva se puede escribir en términos delas actividades:

Y sustituyendo los datos del enunciado en la ecua-ción anterior, hallamos el tiempo t2 necesario paraque la actividad se reduzca a una tercera parte desu valor inicial:

40. Datos: A(t) = A0/8; t = 7,5 min

a) En nuestro caso, el cociente de actividades es unmúltiplo entero de (1/2). Por tanto, el tiempotranscurrido es un múltiplo entero del período desemidesintegración T del radioisótopo:

Se obtiene el mismo resultado si se sustituyen losdatos del enunciado en la ley de emisión radiactivaescrita en términos de las actividades, aunque esteúltimo procedimiento es más largo.

AA

t T

Tt0

312

3

37 5

32 5

=⎛⎝⎜

⎞⎠⎟

⇒ =

= = =, min

, min

AA e

s

s t00

5 27 10

33

3 5 27 10

3 12= ⋅ ⇒

⇒ = ⋅

− ⋅

− −,

ln , −− ⋅ ⇒ =12 2 208t t s

A A e t= −0

λ

N N e t= −0 ,λ

Ts

s= =⋅

=−

ln ,

,

2 0 693

5 27 101323λ

m m e m m e

s

t s= ⇒ = ⇒

⇒ = − ⋅

− − ⋅0 0 0

2019

109

1020ln

λ λ

λ ⇒⇒ = ⋅ −λ 5 27 10 3, s

AA

NN

= ⇒ =0 0

4 4

Page 176: 9523 gl fis_tx2_cas

183

c) Hallamos la masa m de carbono 14 al cabo de1010 s haciendo uso de la ley de emisión radiactivaen términos de las masas:

46. Datos: NA0 = NB0; t = 1 350 s; NA = 2 NB; TA = 150 s

Escribimos las leyes de emisión radiactiva de los dosradioisótopos A y B:

Ahora dividimos las dos ecuaciones:

Y sustituimos los datos del enunciado en esta últimaexpresión:

Tomando logaritmos neperianos, resulta:

ln 2 = (λB − λA) ⋅ 1 350 s

Teniendo en cuenta la relación entre la constanteradiactiva λ y el período T para cada radioisótopo,

la expresión anterior puede escribirse en

términos de los períodos:

47. Datos:

En esta reacción nuclear el valor del número atómicopasa de Z0 = 92 a Z = 82; mientras que el valor del nú-mero másico cambia de A0 = 238 a A = 206.

Sabemos que cada partícula α emitida reduce en 2 uni-dades el valor de Z y reduce en 4 unidades el valor de A.A su vez, cada partícula β emitida aumenta el núme-ro atómico Z en una unidad y deja invariante el nú-mero másico A.

Así pues, el cambio global en el número másico es de-bido solamente a las partículas α emitidas. Sea x el nú-mero de partículas α emitidas, se cumple:

En nuestro caso es: A0 − A = (238 − 206) = 32

Por tanto, se tiene: 32 = 4x ⇒ x = 8; es decir, se emiten8 partículas α.

El cambio global en el número atómico se debe tantoa las partículas α emitidas como a las partículas β emi-

A A x A A x− = − − =0 04 4;

92238

82206U Pb→

ln ln2 21 1

1350

11 1

150

= −⎛

⎝⎜⎞

⎠⎟⋅

= −⎛

T Ts

T s

B A

B⎜⎜

⎠⎟⋅ ⇒

⇒ + = ⇒ =

1350

11350

1150

1135

s

s s TT s

BB

λ =⎛⎝⎜

⎞⎠⎟

ln,

2T

2 0

0

1350NN

NN

eB

B

B

B

sB A= ⋅ −( )λ λ

NN

NN

eA

B

A

B

tA B= ⋅ − −0

0

( )λ λ

N N e

N N eA A

t

B Bt

A

B

=

=

0

0

λ

λ

m m e

m g e

t

s s

=

= ⋅ ⋅ =

− − ⋅ ⋅− −

0

4 3 829 10 105 10 412 1 10

λ

, ,88 10 4⋅ − g

Hallamos primero el defecto de masa Δm asociado a lareacción a partir de la energía liberada:

Calculamos ahora la masa atómica del carbono 13 apartir de la expresión del defecto de masa:

Δm = (M(12C) + M(2H)) − (M(1H) + M(13C));

0,0029 u = (12 u + 2,0141 u) − 1,0078 u − M(13C) ⇒

⇒ M(13C) = 13,0034 u

El valor obtenido coincide con el valor real de la masaatómica del carbono 13, que podemos consultar en lastablas.

44. Datos: me = 9,1 ⋅ 10−31 kg; h = 6,62 ⋅ 10−34 J ⋅ s

a) La masa total del par electrón-positrón vale: m = 2me= 2 ⋅ 9,1 ⋅ 10−31 kg = 18,2 ⋅ 10−31 kg

La energía E asociada a esta masa total es la ener-gía total de los dos fotones:

Los dos fotones tienen la misma energía Ef de va-lor la mitad de la energía total E:

Por tanto, la frecuencia f de cada fotón es:

45. Datos: T = 5 730 años = 1,807 ⋅ 1011 s;

Ar(14C) = 14,0032 u; t = 1010 s;

a) Calculamos primero la constante radiactiva λ delcarbono 14:

Calculamos ahora la masa inicial de la muestra:

b) Hallamos la actividad A al cabo de 1010 s haciendouso de la ley de emisión radiactiva en términos delas actividades:

A A e

A Bq e

t

s

=

= ⋅ ⋅

− ⋅ ⋅− −

0

7 3 829 10 108 217 1012 1 10

λ

, , ss

A Bq= ⋅7 9 107,

mN MN

NA

mM AN

gA

A

00

00

00 14 0032 8 217 10

= =

= =⋅ ⋅

;

, ,

λ

λ

77 1

23 12 1

04

6 022 10 3 829 10

5 0 10

s

s

m g

− −

⋅ ⋅ ⋅

= ⋅

, ,

,

λ = =⋅

= ⋅ − −ln ,

,,

2 0 693

1 807 103 829 10

1112 1

T ss

As0

94 93 10160

8= ⋅ ⋅ =,min

min,

desintegraciones2217 107⋅ Bq

fEh

J

J sHzf= =

⋅ ⋅ ⋅= ⋅

8 2 10

6 62 101 24 10

14

3420,

,,

EJ

Jf =⋅

= ⋅1 64 10

28 2 10

1314,

,

E m c kg m s

E

= = ⋅ ⋅ ⋅ ⋅

= ⋅

− −

, ( )

,

2 31 8 1 218 2 10 3 10

1 64 10 113 J

Δm MeVuMeV

u= ⋅ =2 711

9310 0029, ,

13. Física nuclear

Page 177: 9523 gl fis_tx2_cas

184

Calculamos la masa de uranio necesaria para pro-porcionar esta energía, sabiendo que 100 g de ura-nio dan una energía de 5,12 ⋅ 1031 eV y que unelectronvoltio equivale a 1,6 ⋅ 10−19 J:

50. El cuadro final debería tener un aspecto similar al si-guiente:

51. Para el carbono 14 (de gran interés en arqueología) lagráfica de desintegración se obtiene trazando la fun-ción:

Siendo

Se obtiene:

EVALUACIÓN

1. La ley de emisión radiactiva establece que, en unamuestra radiactiva, el número de emisiones radiactivasque se producen por unidad de tiempo es proporcionalal número de núcleos N sin desintegrar presentes en lamuestra.

λ = = −lnaños

25 700

1

N t N e t( ) = −0

λ

6 048 101

1 6 10

100

5 12 1013

19 31,, ,

⋅ ⋅⋅

⋅⋅−J

eV

J

g

eV== 738 g

8 64 10 7 10 6 048 104 8 13, ,⋅ ⋅ ⋅ = ⋅s W Jtidas. Llamando y al número de partículas β emitidas,se cumple:

Z − Z0 = −2x + 1y

Z0 − Z = 2x − y

En nuestro caso es:

Z0 − Z = (92 − 82) = 2 ⋅ 8 − y

Por tanto, se tiene: 10 = 16 − y ⇒ y = 6; es decir, se emi-ten 6 partículas β.

48. Datos: Ec(α) = 9,5 MeV; Ar(4He) = 4,0026 u;Ar(1H) = 1,0078 u

a) La reacción producida al bombardearse el litio 7con un protón y emitirse una partícula α es:

b) De la reacción anterior, observamos que en reali-dad el litio 7 emite dos partículas α, cada una deellas con una energía cinética de 9,5 MeV. Por tan-to, la energía total E liberada en la reacción vale:E = 2 ⋅ 9,5 MeV = 19 MeV

Hallamos ahora el defecto de masa Δm asociado aesta reacción:

Calculamos finalmente la masa atómica del litio 7a partir de la expresión del defecto de masa de lareacción:

Δm = (M(7Li) + M(1H)) − (M(4He) + M(4He));

0,0204 = M(7Li) + 1,0078 u − (2 ⋅ 4,0026 u) ⇒

⇒ M(7Li) = 7,0178 u

El valor obtenido es muy similar al valor real de lamasa atómica del litio 7, que se puede consultar enlas tablas y es de 7,0160.

49. Datos: E = 200 MeV; Ar(235U) = 235,0439 u;

m = 100 g; Potencia = 700 MW = 7 ⋅ 108 W

a) Calculamos el número de núcleos de uranio 235 fi-sionables, N, de una muestra de 100 g de uranio235:

Cada núcleo fisionable proporciona una energíade 200 MeV, por tanto, la energía total, E, que sepuede obtener con 100 g de uranio 235 es:

b) Calculamos la energía consumida en un día poruna central nuclear de 700 MW de potencia, supo-niendo que funciona ininterrumpidamente:

E nuclMeV

nuclMeV= ⋅ = ⋅2 56 10

2001

5 12 1023 25, ..

,

N gnuclg

= ⋅⋅

= ⋅1006 022 10

235 04392 56 10

2323, .

,,

t dh

d hs

s= ⋅ ⋅ ⋅ = ⋅1241

601

601

8 64 104minmin

,

NNA = ⋅6 022 1023,

Δm MeVuMeV

u= ⋅ =191

9310 0204,

37

11

24

24Li H He He+ → +

Page 178: 9523 gl fis_tx2_cas

185

El número atómico, Z, es el número de protonesen un núcleo y es característico de cada elementoquímico. El número másico, A, es el número denucleones (protones y neutrones) de un núcleo, yvaría para cada isótopo distinto de un mismo ele-mento.

Al emitir una partícula α, un núcleo atómico se trans-forma en otro distinto de número atómico dos unida-des inferior y número másico cuatro unidades infe-rior. Esto es debido a que el núcleo padre emite unnúcleo de helio 4 (pierde dos protones y dos neutro-nes).

Al emitir una partícula β, un núcleo atómico se trans-forma en otro distinto de número atómico una uni-dad superior y del mismo número másico. Esto es de-bido a que un neutrón del núcleo padre se desintegradando lugar a un protón y un electrón.

Al emitir radiación γ, un núcleo atómico pierde ener-gía y pasa de un estado excitado a otro menos energé-tico. Sin embargo, sus números atómico y másico nose alteran.

5. Datos: Ar(138Ba) = 137,9050 u; A = 138; Z = 56;

mp = 1,0073 u; mn = 1,0087 u

a) Tomamos como valor de la masa nuclear del bario138 el valor de su masa atómica por el pequeño valorde la masa de los electrones: MN (138Ba) ≈ Ar(138Ba),y sustituimos los datos del enunciado en la expresiónpara el defecto de masa Δm:

b) La energía de enlace ΔE puede calcularse a partirde la relación: ΔE = Δm c2, o bien de forma más di-recta a partir del equivalente energético de unaunidad de masa atómica(1 u = 931 MeV):

c) La energía de enlace por nucleón es:

6.

a) Toda reacción nuclear debe cumplir que la sumade los números atómicos y la suma de los númerosmásicos en ambos miembros de la reacción seaniguales. Esto es:

Suma de números atómicos:

Z + 1 = 3 ⋅ 2 = 6 ⇒ Z = 5

Suma de números másicos:

A + 1 = 3 ⋅ 4 = 12 ⇒ A = 11

Datos X X He E MeV

A HZA

r

: ; , ;

( ) ,

+ → =

=

11

24

1

3 11 47

1 00078 4 00264; ( ) ,u A He ur =

ΔEA

MeVMeV= =

1133 2138

8 21,

,

ΔE uMeVu

MeV= ⋅ =1 2172931

11133 2, ,

Δ

Δ

m Zm A Z m M

m up n N= + − −

= ⋅ + −

( ( ) )

( , ( )56 1 0073 138 56 ⋅⋅ −

=

1 0087 137 9050

1 2172

, ) ,

,

u u

m uΔ

λ = constante radiactiva, característicade cada isótopo radiactivo.

Como consecuencia, el número de núcleos N disminu-ye de forma exponencial con el tiempo.

N0 = número de núcleos sin desin-tegrar en el instante inicial.

El período de semidesintegración, T, de una sustanciaradiactiva es el tiempo que debe transcurrir para que lamasa de cualquier muestra de dicha sustancia se reduz-ca a la mitad.

La constante radiactiva, λ, es una característica de cadaisótopo radiactivo igual a la constante de proporcionali-dad entre el número de emisiones por unidad de tiem-po y el número de núcleos presentes en la muestra.

La vida media, τ, de un radioisótopo es el tiempo me-dio que tarda en desintegrarse un núcleo de dicha sus-tancia tomado al azar.

2. La exposición a altas dosis de radiación en los seres vi-vos provoca un aumento de la tasa de cáncer y otrostrastornos de tipo genético que se manifiestan en lasgeneraciones posteriores.

— Algunos de los radioisótopos más peligrosos para elser humano son el estroncio 90, el potasio 40 y el car-bono 14.

— La radiactividad no siempre es perjudicial. Utiliza-da en las dosis y forma correctas presenta muchosbeneficios, como la detección y el tratamiento delcáncer y diversas aplicaciones tecnológicas e indus-triales.

3. Datos: λ = 1,7 ⋅ 105 s−1

a) La vida media τ del radioisótopo es:

τ = 1/λ = 1/(1,7 ⋅ 105 s−1) = 5,9 ⋅ 10−6 s

b) Si una muestra del radioisótopo se reduce a unacuarta parte de su masa inicial, se tiene:

Para que esto ocurra debe transcurrir un tiempo tigual a dos veces el período de semidesintegraciónT. Por tanto:

4. Los núcleos atómicos están formados por protones yneutrones. Los protones y neutrones tienen una masasimilar, que es aproximadamente 104 veces la del elec-trón. Los protones tienen carga eléctrica positiva deigual valor absoluto que la del electrón; mientras quelos neutrones no poseen carga eléctrica. Los protonesy neutrones se mantienen unidos gracias a la fuerzanuclear fuerte.

El núcleo posee más del 99% de la masa del átomo, sinembargo, su volumen es de sólo 10−5 veces el volumenatómico.

t Ts

s= = =⋅

⋅= ⋅−

−2 22 2 0 693

1 7 108 105 1

6ln ,

mm

m= = ⋅⎛⎝⎜

⎞⎠⎟

00

2

412

N N e t= −0

λ

dNdt

N= −λ

13. Física nuclear

Page 179: 9523 gl fis_tx2_cas

186

núcleo pesado le proporciona la energía de activaciónnecesaria.

Las reacciones de fusión consisten en la unión de dosnúcleos ligeros para formar otro núcleo más pesado.La fusión se produce de forma natural en las estrellas.Las altas temperaturas y presiones de su interior pro-porcionan la energía de activación necesaria.

Como fuentes de energía a gran escala, la fisión pre-senta la ventaja de que su uso es económicamente ren-table y la técnica necesaria se conoce desde hace másde 50 años. En cambio, presenta el grave riesgo de unaccidente nuclear. Otro inconveniente de la fisión esel problema del almacenamiento y eliminación de losresiduos contaminantes que produce.

La fusión, en cambio, presenta el inconveniente deque su uso hoy en día no es aún económicamente ren-table. Se requiere todavía una gran inversión para suestudio y desarrollo. No obstante, cuando se logre lafusión en cadena controlada y económicamente renta-ble, constituirá una fuente de energía limpia (no pro-duce residuos contaminantes) y sin problemas de es-casez de combustible.

8. Las cuatro fuerzas fundamentales son: la fuerza gravi-tatoria, la fuerza electromagnética, la fuerza nuclearfuerte y la fuerza nuclear débil.

La unificación de las fuerzas fundamentales consisteen suponer que los cuatro tipos de fuerzas fundamen-tales son distintas manifestaciones de un único campode fuerzas. Hasta la fecha se ha conseguido la unifica-ción de la fuerza electromagnética y la fuerza nucleardébil (campo electrodébil), y las predicciones han sidocomprobadas.

El elemento de número atómico 5 es el boro. Ade-más, como su número másico es 11, se trata delboro 11. La reacción es, pues:

b) Hallamos el defecto de masa Δm asociado a la reac-ción a partir de la energía liberada:

Calculamos ahora la masa atómica del boro 11 apartir de la expresión del defecto de masa:

Δm = (M(11B) + M(1H)) − (3 M(4He));

0,0123 u = M(11B) + 1,0078 u − 3 ⋅ 4,0026 u ⇒

⇒ M(11B) = 11,0123 u

El valor obtenido es muy similar al valor real de lamasa atómica de boro 11, que se puede consultaren las tablas y es de 11,0093.

7. Respuesta sugerida:

Las reacciones de fisión y fusión son reacciones nuclea-res en las que se libera gran cantidad de energía (delorden de 10 o 102 MeV por núcleo reaccionante). Estaenergía proviene del defecto de masa asociado a lareacción. Ambas reacciones necesitan de una ciertaenergía de activación para producirse.

Las reacciones de fisión consisten en la rotura de unnúcleo pesado para formar otros dos núcleos más lige-ros de masa similar. La fisión se induce de forma artifi-cial cuando se bombardean núcleos pesados con neu-trones lentos. La captura del neutrón por parte del

Δm MeVuMeV

u= ⋅ =11 471

9310 0123, ,

511

11

243B H He+ →

Page 180: 9523 gl fis_tx2_cas

187

queña sea la relación entre la masa de la polea y la delos cuerpos, mejor será la aproximación que se obtie-ne en caso de la polea de masa negligible.

d) La máquina de Atwood podría utilizarse para determi-nar la aceleración de la gravedad mediante el procedi-miento seguido en esta experiencia.

A partir de los valores experimentales obtenidos se re-presenta gráficamente s en función de t2 y se calcula lapendiente de la recta. Según la ecuación del MRUA,s = 1/2 a t2, y la expresión teórica de la aceleración encaso de la polea inercial, la pendiente viene dada por:

De la pendiente se obtiene el valor experimental de laaceleración de la gravedad.

CONSERVACIÓN DE LA CANTIDAD DE MOVIMIENTO EN UN SISTEMA DE DOS MASAS

a) Las ecuaciones del movimiento de una bola respectodel sistema de referencia indicado en el libro delalumno son:

En primer lugar, encontramos el tiempo de movimien-to imponiendo la condición y = 0.

El alcance se obtiene sustituyendo en la ecuación de xel tiempo de movimiento.

b) Sí. Se cumple el principio de conservación de la canti-dad de movimiento porque mientras las bolas estánencima de la plataforma, no actúa ninguna fuerzaneta externa al sistema (el peso se compensa con lanormal).

Las posibles desviaciones de este principio tienen lu-gar cuando se aplica involuntariamente una fuerza ex-terna al sistema. Por ello, cortamos el hilo con unas ti-jeras sin tocar las bolas ni el muelle.

x v t vh

g= =

2

012

22= − =h g t th

g;

x v t y h g t= = −;12

2

12 2

( )( )

am m

m m mgA B

A B

=−

+ +

POLEAS DE MASA NEGLIGIBLE Y POLEAS INERCIALES

a) Polea de masa negligible

Partimos del sistema de ecuaciones que se deduce en eltexto.

En este caso TA = TB. Sumamos las dos ecuaciones delsistema para obtener:

Polea inercial

Partimos del sistema de ecuaciones que se deduce en eltexto.

En este caso La suma de ambas ecua-

ciones del sistema nos da:

b) Para valores de t elevados, los datos experimentales sepueden desviar apreciablemente de la recta ajustada.Esto es debido principalmente a dos razones:

— El error experimental cometido en la medida detiempo con el cronómetro crece para valores detiempo elevados.

— La fuerza de frotamiento entre la polea y la cuerdano es menospreciable.

c) El valor experimental que se obtiene para la acelera-ción se acerca más al valor teórico deducido en caso dela polea inercial. Esto es así porque la polea real utili-zada tiene cierta masa m que no es menospreciableante las masas de los cuerpos mA y mB. Cuanto más pe-

m g m g T T m m a

m g m g m

A B B A A B

A B A

( )

(

− + − = +

− = + mm a m a

m m g m m m a

a

B

A B A B

)

( ) [ ( ) ]

+

+ = + +

=

12

2 2

222

4( )

( )( )

m mm m m

gA B

A B

+ +

T T m aA B− =12

.

m g T m a

T m g m aA A A

B B B

− =

− =

⎫⎬⎭

m g m g m m a

am mm m

g

A B A B

A B

A B

( )

( )

− = +

=−

+3

m g T m a

T m g m aA A A

B B B

− =

− =

⎫⎬⎭

Prácticas

Prácticas

Page 181: 9523 gl fis_tx2_cas

188

CONSERVACIÓN DE LA ENERGÍA MECÁNICA EN EL MOVIMIENTO ARMÓNICO SIMPLE

a) Sí, la masa m cumple el principio de conservación dela energía mecánica. Esto puede comprobarse verifi-cando experimentalmente que se cumple la ecuación(1) del texto:

El efecto del frotamiento con el aire puede hacer quela amplitud de oscilación vaya disminuyendo hastaque el sistema se pare, razón por la que el principio deconservación de la energía mecánica no se cumpleexactamente. Para reducir el error experimental me-diremos las posiciones extremas x1 y x2 dentro de laprimera oscilación del sistema.

b) Los valores de K obtenidos mediante las determinacio-nes estática y dinámica pueden no coincidir exacta-mente debido al error experimental cometido. Enprincipio, se tiene que suponer que el error experi-mental que se comete al medir el período con el cro-nómetro será más grande que el que se comete al me-dir la posición final con la regla, por lo que el valorobtenido mediante la determinación estática será másfiable.

c) Para calcular la velocidad en un punto cualquiera x si-tuado entre x1 y x2 podemos aplicar el principio deconservación de la energía mecánica entre los puntosx1 y x.

De esta ecuación podemos deducir la velocidad v, yaque todos los otros datos son conocidos.

FENÓMENOS DE INTERFERENCIA EN UNA CUBETADE ONDAS

a) El modelo teórico supone que las dos ondas que inter-fieren son armónicas y coherentes, que tienen la mis-ma frecuencia, amplitud, longitud de onda y veloci-dad, y que la vibración de ambas se produce en lamisma dirección. Todas estas condiciones se cumplenen la experiencia porque ambas ondas proceden deuna misma fuente y se obtienen al interponer un lis-tón con dos orificios iguales.

b) Respuesta libre.

c) La velocidad de propagación de las ondas v se relacio-na con la longitud de onda λ y la frecuencia de oscila-ción f mediante la ecuación:

v = λ f

d) La imagen obtenida en la pantalla es una reproduc-ción de la que se observa en la cubeta de ondas con uncierto aumento o factor de escala. Por lo tanto, pode-mos medir distancias sobre la pantalla y después divi-dirlas por el factor de escala para obtener el valor real.En caso de la longitud de onda se tendría que medirsobre la pantalla la distancia entre dos máximos de laamplitud de la onda.

m g x x K x x m v( ) ( )− = − +12

12 21

212

m g x x K x x( ) ( ) ( )2 1 22

121

21− = −

CONSERVACIÓN DE LA ENERGÍA MECÁNICA EN UN PLANO INCLINADO

a) Las ecuaciones del movimiento parabólico permitenescribir para el punto de impacto (xC, yC):

Donde vBx y vBy son las componentes de la velocidad enel punto B (vBx = vB cos α; vBy = vB sen α)

De la primera ecuación aislamos el tiempo para susti-tuirlo en la segunda:

b) Si suponemos que las fuerzas de rozamiento son des-preciables, se cumple el principio de conservación dela energía mecánica. Sin embargo, normalmente lasfuerzas de rozamiento son apreciables, tanto en el tra-yecto A-B como en el trayecto B-C. En consecuencia, laenergía mecánica en el punto final suele ser algo me-nor que en el punto inicial, EA � EB � EC.

c) Experimentalmente puede comprobarse que al aumentarla inclinación del plano inclinado la variación de la ener-gía mecánica entre los puntos A y B se aproxima más al va-lor teórico cero. Esto pasa porque al aumentar el ánguloα la fuerza de rozamiento (Fr = μ m g cos α) disminuye.

d) Podemos aplicar de nuevo el principio de conservaciónde la energía mecánica, ahora entre los puntos B y C,para determinar la velocidad de la bola en el punto C.

Si escogemos el origen de energía potencial en B, laenergía potencial en C es negativa e igual a −m g yC.

12

012

12

12

2 2

2 2 ;

m v m v m g y

v v g y v

B C C

B C C

+ = −

= − BB C C

C B C

C B C

v g y

v v g y

v v g y

2 2

2 2

2

2

2

2

= −

= +

= +

E E

E E E EB C

C P C PB B C C

=

+ = +

txv

y vxv

gxv

y xv

C

Bx

C ByC

Bx

C

Bx

C CB

=

= +⎛

⎝⎜⎞

⎠⎟

=

12

2

ssencos cos

α

α α

α

vg x

v

y x tg

BC

B

C C

+

= = +

12

122 2

112

1

2

22 2

2

2

cos

co

g xv

y x tgg x

v

CB

C CC

B

α

α− =ss

cos

cos

2

2 22

22

2

2

α

αα

vg x

y x tg

vx

BC

C C

BC

=−

=( )

cos ( )

α α

α α

gy x tg

vx g

y x tg

C C

BC

C C

2

2

=−

x v t y v t g tC Bx C By= = +;12

2

Page 182: 9523 gl fis_tx2_cas

189

— Para concentrar los fotones del pulso en un únicosentido se dirigen mediante un espejo y un se-miespejo paralelos. El haz láser se transmite unavez alcanzada la amplificación suficiente.

El láser emite radiación visible. Actualmente, elláser se aplica en campos tan dispares como las te-lecomunicaciones (fibras ópticas), los lectores dediscos compactos, la cirugía (bisturís), la industria(soldadores y cortadores de precisión)…

b) Si en la trayectoria del láser se interpone un obs-táculo longitudinal, por ejemplo una rendija ver-tical, se produce difracción en los bordes vertica-les del obstáculo, ya que la separación entre éstoses del orden de la longitud de onda mientras quela separación entre los bordes horizontales es mu-cho más grande. Por lo tanto, la posición de losmínimos de difracción dependerá de la anchurade la rendija (en dirección horizontal) y la figurade difracción estará constituida por franjas hori-zontales brillantes y oscuras alternadas.

c) El fenómeno de la difracción puede interpretarsefísicamente a partir del principio de Huygens.Cada punto de la rendija o del contorno del obs-táculo se convierte en emisor de ondas elementa-les que interfieren entre sí. Por ello, se observacualitativamente la misma figura de difracción encaso de un cabello que de una rendija, ya que am-bos obstáculos tienen el mismo contorno.

d) Sí. Experimentalmente se obtiene que la posición

del primer mínimo viene dada para

e) Se observa que en caso de la rendija más estrechala franja luminosa central es más ancha. Asimis-mo, el resto de franjas están más separadas y sonmenos luminosas al repartirse la luz entre una su-perficie más grande.

DESCARGA DE UN CONDENSADOR

a) De todos los condensadores, el condensador electrolíti-co es el que ofrece más capacidad con menos volumen.

Se fabrica a partir de óxidos de metales dieléctricos,como el aluminio. En este condensador, el dieléctricoes la finísima capa de óxido de aluminio adherida almetal. Debido al escaso espesor de esta capa, la capa-cidad de un condensador electrolítico es muy superiora la de cualquier otro condensador con el mismo ta-maño.

Es importante recordar que los condensadores electro-líticos tienen polaridad y que la capacidad y la tensión

yLa

= λ .

Espejo Semiespejo

Energía de bombeo

Hazcoherente

(láser)

Para determinar el factor de escala medimos sobre lapantalla una distancia conocida, por ejemplo la distan-cia entre los dos orificios que actúan como foco.

FENÓMENOS DE DIFRACCIÓN CON EL LÁSER

a) Respuesta sugerida:

El láser (amplificación de luz por emisión estimuladade radiación) es un dispositivo óptico que permite cre-ar luz monocromática, es decir, de una frecuencia de-terminada, coherente, muy intensa y concentrada.

Generación del pulso láser

— Se hace incidir una radiación electromagnéticamuy intensa sobre un gas (láser de helio-neón) oun sólido (láser de rubí o de cromo) ionizados. Deeste modo se consigue excitar los átomos ioniza-dos hasta diferentes estados de energía de vidacorta.

— Los iones pierden energía y pasan a un estadometaestable, en el que pueden permanecer mástiempo.

— En la situación de inversión de población (núme-ro más elevado de iones en el estado excitado queen el estado fundamental), la emisión de un fotónpor un ion provoca una reacción en cadena, lacual genera el pulso láser.

Emisión coherente (láser)

Emisión incoherente

Estado metaestable

Estadofundamental

Prácticas

Page 183: 9523 gl fis_tx2_cas

190

distancia. Conectamos un amperímetro en seriecon la bobina para poder medir la intensidad decorriente que circula. Conectamos la fuente de co-rriente y fijamos un valor de la intensidad I.

— Sustituimos los valores de x e I en la expresiónmatemática de la inducción magnética de la bobi-na para obtener el valor de Bb.

— Observamos la desviación de la aguja de la brúju-la y anotamos el ángulo α. Calculamos el valor dela inducción magnética terrestre Bt a partir de Bby α.

INDUCCIÓN ELECTROMAGNÉTICA.PRIMERA EXPERIENCIA DE FARADAY

a) Respuesta sugerida:

Sabido que la intensidad de la corriente inducida de-pende de la velocidad del imán, tendríamos que man-tener una velocidad fija al estudiar la dependenciacon el número de espiras de la bobina y con la inten-sidad del campo magnético del imán. Sin embargo,como el movimiento del imán se lleva a cabo manual-mente, es difícil repetir la misma velocidad en todoslos casos, sobre todo en velocidades altas.

b) La intensidad de la corriente inducida es proporcio-nal a la fem inducida y ésta se define como la veloci-dad con la que varía el flujo magnético a través del cir-cuito, cambiada de signo. Así:

— Como el flujo magnético es proporcional al nú-mero de espiras, la intensidad de la corriente in-ducida aumenta al aumentar el número de es-piras.

— La velocidad con la que varía el flujo magnéticodepende directamente de la velocidad del imán.Por lo tanto, la intensidad de la corriente induci-da aumenta con la velocidad del imán.

— Finalmente, el flujo magnético es proporcional ala inducción magnética. Por esta razón, la intensi-dad de la corriente inducida aumenta con la in-tensidad del campo magnético.

c) Sí. El sentido de la corriente inducida depende delsentido del movimiento del imán. Mientras introduz-camos el imán en la bobina, el flujo magnético au-menta y la fem inducida tiende a compensar este au-mento originando una corriente inducida. Por contra,al extraer el imán de la bobina, el flujo magnético dis-minuye y la fem inducida compensa esta disminuciónoriginando una corriente eléctrica en sentido opuestoal anterior.

d) El flujo magnético es proporcional al radio de la bobi-na. Por esta razón, la intensidad de la corriente indu-cida aumenta al aumentar el radio de la bobina.

BB

tgtb=α

BN I R

x Rb =+( ) /

μ02

2 2 3 22

se especifican en el cuerpo. Si aumentemos la tensión detrabajo más de la cuenta o no respetemos la polaridad,el dieléctrico se perfora y se destruye el condensador.

b) La ecuación obtenida por V(t) es una exponencial ne-gativa y, por lo tanto, la ecuación de ln V(t) será unarecta de pendiente negativa. Esto indica que la tensiónse reduce desde el valor inicial V0 hasta llegar a ser ceroen un tiempo infinito. Experimentalmente puede com-probarse como en los primeros instantes de tiempo Vdecae mucho rápidamente y ln V se ajusta mejor alcomportamiento lineal previsto.

c) La energía eléctrica almacenada en un condensador

cargado es En el proceso de descarga esta

energía se transforma, en primer lugar, en energía dela corriente eléctrica E = V I t, y después en energía di-sipada en la resistencia R por efecto Joule, E = R I2 t.

d) Necesitamos la resistencia para descargar a través deésta el condensador. A través de la resistencia, circulauna corriente porque existe una diferencia de poten-cial entre sus extremos.

Se utiliza una resistencia óhmica (que cumple la ley deOhm) porque, según que se comprueba al aplicar laley de Ohm, la carga y el potencial eléctricos en el con-densador disminuyen exponencialmente durante ladescarga.

— La velocidad de descarga es inversamente propor-cional al producto RC, valor que recibe el nombrede constante de tiempo. La constante de tiempo esel tiempo que tarda la diferencia de potencial a re-ducirse un factor e (es decir, un 35,8 %).

CAMPO MAGNÉTICO DE UNA BOBINA

a) El campo magnético en el interior de la bobina es másintenso que fuera de ésta, y se reduce al aumentar ladistancia al centro de la bobina.

b) Cuanto más cerca nos situemos del centro de la bobi-na (x → 0), los valores de la inducción magnética ob-tenidos experimentalmente se acercarán más a la rec-ta ajustada. La inducción magnética de la bobina(Bb) es más intensa en el centro de ésta, donde valeN μο I/2 y va disminuyendo a medida que nos alejamosdel centro hasta hacerse inapreciable a grandes distan-cias x.

c) Para valores negativos de x se obtienen los mismos re-sultados que para valores positivos de x, ya que el pro-blema es simétrico y la expresión de Bb no depende delsigno de x. La única diferencia es el sentido de las líne-as de inducción; si para valores positivos de x salen dela bobina, para valores negativos de x entrarán.

— Si se invierte el sentido de la intensidad de corrien-te, también se invertirá el sentido de la inducciónmagnética de la bobina Bb.

d) Un procedimiento para medir la inducción magnéticaterrestre a partir del montaje experimental de que dis-ponemos podría ser el siguiente:

— Situamos la brújula en un punto del eje de la bobi-na a una distancia x de su centro y medimos esta

E C V= .12

2

Page 184: 9523 gl fis_tx2_cas

191

Tomando g = 10, las ecuaciones del movimientoy de la velocidad son:

El tiempo de vuelo de la pelota es:

Tomando t = 1 s la distancia (d) a la canasta será:

b) El tiempo hasta alcanzar el máximo de la trayec-toria se calcula:

La altura máxima:

c) En el momento de entrar en la canasta:

d) Y el ángulo de con la horizontal:

3. a) Teniendo en cuenta que:

Aplicamos la ecuación de la velocidad:

b) En 20 minutos dará:

c) La velocidad del surco a 14,5 cm del centro degiro será:

�v( , ) , , ,14 5 0 145 3 46 0 50= ⋅ = m/s

2033

660min ⋅ =rev

revmin

3,463,46

rad/s2= + = =0 55

0 69α α; ,

wrev

srad

rev= ⋅ ⋅ =33

160

21min

min3,46 rad/s

π

tag ;β β= = =48

12

0arc tg 26,6

�v( )1

� � � �v i j v( ) ; ( ) ,1 8 4 1 8 4 8 942 2= − = + = m/s

h 0,6 0,6máx = + ⋅ − ⋅ =2 6 5 3 82 , m

0 6 106

100 6= − = =t; t ,

d m= ⋅ =8 1 8

3 2 6 51

0 22= + − =

⎧⎨⎩

t t ts

s;

,

� � �

� � �r t t i t t j

v t i t j

( ) ( )

( ) ( )

= + + −

= + −

8 2 6 5

8 6 10

2

UNIDAD 1

DINÁMICA DE TRASLACIÓN Y DE ROTACIÓN

1. a) Representamos entre t = 0 h y t = 5 h:

También se puede representar la gráfica a partir dela ecuación de la trayectoria entre los valores x = 1y x = 6:

b) A partir de la gráfica es evidente que las coordena-das de las islas deben ser:

A(1,2), B(3,11) y C(5,7)

c)

d)

2. a) Teniendo en cuenta que:

los valores iniciales son:� �

� � � �r j

v i j i

( ) ;

( )

0 2

0 10 10 8 6

=

= + = +cos senα α��j

sen α = − =1 0 8 0 62, ,

� � �

� � � �v t i t j

v i j v

( ) ( ) ;

( ) ; ( )

= + −

= − =

6 2

4 2 4 11� � ++ =2 52 mill/h

�� � � � � �

�v

r r i j i jA B,

( ) ( ) ( )=

−=

+ − +

−=

3 03 0

4 11 23 0

ii j

vr r i j

B C

+

=−

−=

+ −

3

5 35 3

6 7

�� � � �( ) ( ) (

,

mill/h

44 115 3

2

� �� �i ji j

+

−= −

)mill/h

y x x= − + −2 8 5

t

r

0 1 2 3 4 5

1 2 2 7 3 10 4 11 5 10 6 7�

( , ) ( , ) ( , ) ( , ) ( , ) ( , )

Propuestas de evaluación

Propuestas de evaluación

t = 0 A(1, 2)

t = 3

B(4, 11)

t = 5 C(6, 7)

Page 185: 9523 gl fis_tx2_cas

b)

c) El campo total se obtiene:

d)

e) Sumando los módulos:

Esta suma no coincide con el módulo de campoporque el campo se suma vectorialmente (no esigual a la suma de módulos).

f) El potencial se obtiene sumando los potencialesparciales, dado que es escalar:

2. a) Consideremos un punto interior de la esfera situa-do a una distancia r del centro. Según el teoremade Gauss:

Sustituyendo:

siendo ρ la densidad. Por tanto:

De donde:

Finalmente:

�g G

M

Rr= 3

� �g r G

M

Rr g2

33= ;

− = −�g r G

M

Rr4 4

43

43

2

3

3π ππ

π

− = −�g r G r4 4

43

2 3π π ρ π

⌠⌡⎮ = = −s

� �gdS GMsΦ 4 π

V a V a V a V a( ) ( ) ( ) ( )

,

= + +

= − ⋅⋅−

1 2 3

1111

6 67 102 10

3�ii j

i j

+−

− ⋅⋅

− +−

− ⋅

3

6 67 102 10

3 3

6 67 10

1111

1

� �,

, 11112 10

3 314 15

−= −� �

i jV,

� � �g a g a g a1 2 3

0 74 1 11 1 48 3 34

( ) ( ) ( )

, , , ,

+ + =

= + + = N//kg

�g a( ) , , ,= + =0 79 0 26 0 832 2 N/kg

� � �

� �

g a i j

i j

( ) , ,

, ,

,

= − − +

+ − −

0 52 0 52

0 79 0 79

1 05 ii j

i j

+ =

= − −

1 05

0 79 0 26

,

, ,

� �N/kg

g a

g a

12 2

22

0 52 0 52 0 74

0 79 0

( ) , , ,

( ) , ,

= + =

= +

N/kg

779 1 11

1 05 1 05 1 48

2

32 2

=

= + =

,

( ) , , ,

N/kg

N/kg�g a

Y a 6,5 cm:

d) Como en 20 min da 660 vueltas, podemos conside-rar 660 circunferencias.

Las longitudes de la primera y la última serán:

Si la disminución del radio es lineal y tenemos encuenta que en una progresión aritmética de 660términos hay 659 saltos, la longitud de la circunfe-rencia n será:

Por tanto, la longitud total del surco rozado por laaguja es:

UNIDAD 2

CAMPO GRAVITATORIO

1. Situamos los puntos de acuerdo al siguiente esquema:

Usando la fórmula:

a) �� �

� ��g a

i j

i j

i111

11

26 67 102 10

3 3

3 3

3( ) ,= − ⋅

+

+

+−

33

0 52 0 521

2

� � �

j

g a i j

g a

⎝⎜⎜

⎠⎟⎟

= − −( ) , ,

(

N/kg

)) ,= − ⋅⋅

− +

− +

− +−6 67 10

3 10

3 3

3 3

3 311

11

2� �

� ��

i j

i j

i��

� � �

j

g a i j

g a

⎝⎜⎜

⎠⎟⎟

= −

=

2

3

0 79 0 79( ) , ,

( )

N/kg

−− ⋅⋅

⎝⎜−6 67 10

4 10

3 3

3 3

3 311

11

2, � �

� �� �

i j

i j

i j⎜⎜

⎠⎟⎟

= − +� � �g a i j3 1 05 1 05( ) , , N/kg

� �g a Gmd

uii

i ai( )

,

=

m

a

3

m21m

o

6

6

3

3

j i

L L L L

L

= + + ⋅ ⋅ ⋅ +

= ⋅ ⋅ −−

1 2 660

2 0 145 6600 145 0 065

6π ,

, ,559

0 1 659

2 0 145 6600 145

+ + ⋅ ⋅ ⋅ +( )⎡

⎣⎢⎤

⎦⎥=

= ⋅ ⋅ −−

π ,, 00 065

6596592

660

2 6600 145 0 065

2

,

, ,

⋅⎡

⎣⎢⎤

⎦⎥=

= ⋅+

π⎡⎡

⎣⎢⎤

⎦⎥= 435 42, m

L nn = ⋅ −⋅ − ⋅⎛

⎝⎜⎞⎠⎟

−22 2

6591π

π π(0,145

0,145 0,065))

L m

L m1

660

2 0 145

2 0 065

= ⋅ ⋅

= ⋅ ⋅

π

π

,

,

�v( , ) , , ,6 5 0 065 3 46 0 22= ⋅ = m/s

192

Page 186: 9523 gl fis_tx2_cas

193

La aceleración en B es nula, ya que la intensidaddel campo gravitatorio creado en B por las dosmasas de 10 kg se anula, por ser la resultante dedos vectores de la misma intensidad y direcciónpero de sentido contrario.

b) Aplicando el teorema de la conservación de laenergía mecánica:

y haciendo las sustituciones correspondientes:

Teniendo en cuenta que la masa es de 100 g = 0,1 kg,se obtiene finalmente:

UNIDAD 3

GRAVITACIÓN EN EL UNIVERSO

1. a)

b) Como:

En la Tierra salta con una v0:

12

10 1 6 2 101 6 3202

0m v m v� �= ⋅ = ⋅ =, ; ,

12 0

2m v mg h�

=

g GM

RG

M

RG

M

R

GM

pp

p

T

T

T

T

T

= = = =

=

( ) ( ) ( )

(

2 2 2

5

5

5

25

525 RR

gT

T)2

525

525

10 2= = = N/kg

v m s= ⋅ ⋅− −1 5 10 4 1,

− ⋅ ⋅ ⋅ ⋅ ⋅ + =

= − ⋅ ⋅

− −2 6 67 10100 3

0

2 6 67

11 2 2,,

,

N m kgkgm

11010

0 2412

11 2 2 2− −⋅ ⋅ ⋅ + ⋅N m kgkg

mm v

,

E E E EpA cA pB cB+ = +

a N m kgkg

A = ⋅ ⋅ ⋅ ⋅+

− −2 6 67 1010

0 18 0 24

11 2 2

2 2,

( , ) ( , )⎡⎡⎣⎢

⎤⎦⎥

= ⋅ ⋅ ⋅ ⋅− −

22

11 2 22 6 67 1010

m

a N m kgkg

A

cos

,

θ

(( , ) ( , )

,,

,

0 18 0 24

0 180 30

8 89 1

2 22

2+⎡⎣⎢

⎤⎦⎥

= − ⋅

m

a A 00 9 2− −⋅�j m s

A

B

m = 10 kg m = 10 kg48 cm

m’ = 100 g

18 cm

θ

b) Dado que la esfera se comporta como si toda lamasa estuviera en el centro:

c) El potencial gravitatorio en el interior de la esfe-ra se comporta como un potencial elástico dadoque el campo es proporcional a la distancia. Noobstante, hay que definir un origen de poten-ciales compatible con el potencial en la super-ficie:

La constante A debe satisfacer:

De donde:

Por tanto:

O sea:

d) Aplicando las fórmulas a 2 000 km de profundi-dad:

e) El potencial en la superficie:

f) Con r = 4,37 ⋅ 106 m, el potencial a 2 000 km de pro-fundidad será:

3. a) La componente X de la intensidad del campo gra-vitatorio creado en A por las dos masas de 10 kgse anula; por tanto, la aceleración de m′ en A se-ría la resultante de las dos componentes Y (soniguales, por ello, sería el doble del valor de una deellas):

V( , )

,,

( ,

4 37 10

6 67 105 98 10

2 6 37 10

6

1124

⋅ =

= − ⋅⋅

⋅ ⋅−

66 36 2 6 3

6

4 37 10 3 6 37 10

4 37 10

)( , ) ( , )

( ,

⋅ − ⋅ ⋅⎡⎣

⎤⎦

⋅V )) ,= − ⋅7 92 107 J/kg

V S( ) ,,

,,= − ⋅

⋅= − ⋅−6 67 10

5 98 10

6 37 106 26 1011

24

67 JJ

r m

g

= ⋅ − ⋅ = ⋅

= ⋅ −

6 37 10 2 10 4 37 10

6 67 105 98

6 6 6

11

, ,

,,� ⋅⋅

⋅⋅ =

10

6 37 104 37 10 6 74

24

6 36

( , ), , N/kg

V r GM

Rr R( ) ( )= −

12

332 2

V r GM

Rr G

MR

( ) = −12

323

2

A GMR

GMR

GMR

= − − = −12

32

12 3

2GM

RR A V R G

MR

+ = = −( )

V r GM

Rr A( ) = +

12 3

2

V S GMR

( ) = −

Propuestas de evaluación

Page 187: 9523 gl fis_tx2_cas

194

Luego:

Por tanto:

La segunda ley de Newton se formula:

Despejando R0

Sustituyendo:

De donde:

b) El módulo de la velocidad del satélite:

c) La energía mecánica por kg vale:

Efectuando operaciones:

Operando y sustituyendo:

d) Igualando con la energía en órbita:

Despejando:

12

12

2 1

2

2

v GMR

GMR

v GMR R

LT

T

T

o

L TT o

= −

= −⎛

⎝⎜⎞

⎠⎟=

= GGM

R

RR

RR

g RRR

T

T

T

T

T

oT

T

o2

2 2 222−

⎝⎜

⎠⎟ = −

⎝⎜

⎠⎟

�vv g RRRL T

T

o

= −⎛

⎝⎜

⎠⎟2

2

12

12

2m v G

m MR

Gm M

Rs Ls T

T

s T

o

�− = −

Em

GMR

GM

R

RR

gRR

Em

s

T

o

T

T

T

o

T

o

s

= − = − = −

= −

12

12

12

1

2

2 2

229 8

6 378 10

1 45 102 76 10

6 2

77,

( , )

,,

⋅= − ⋅ J

Em

v GMR

GMR

GMR

GMRs

sT

o

T

o

T

o

T

o

= − = − = −12

12

12

2�

Em

m v Gm M

Rms

s ss T

o

s

=

−12

2�

�v R ws o s= = ⋅ ⋅ ⋅ = ⋅−1 45 10 3 64 10 5 25 107 4 3, , , m/s

h R R mo T= − = ⋅ − ⋅ = ⋅1 45 10 6 378 10 8 07 107 6 6, , ,

Ro =⋅

⋅= ⋅−9 8 1 45 10

23 7,

),

(6,378 10

(3,64 10 )

6

4 2

RGM GM

R

Rg

Ro

T

s

T

T

T

s

T

s

= = = =ω ω ω23 2

2

23

2

23

Gm M

Rm

v

RG

M

R

RR

s T

os

s

o

T

o

o s

o2

2

2

2 2

= =�

; ;ω

ωs = − ⋅ = ⋅− −214 400

7 3 10 3 64 105 4π, , rad/s

ω ω ω ωT s s t⋅ + ⋅ = = −14 400 14 400 22

14 400π

π;

La altura en el planeta será:

c) Dado que:

En este caso:

d) El dato del enunciado es:

De donde:

Por tanto, la relación será:

Deberá aumentar su altura un 41% del radio del pla-neta.

Este valor no depende de ninguno de los datos queidentifican al planeta; por tanto, es válido paracualquier planeta esférico.

2. a) La función angular la rotación terrestre con es:

donde:

La función angular de rotación del satélite congiro en sentido contrario:

Si el satélite debe sobrevolar un punto fijo delecuador cada 4 h (14 400 s) se debe cumplir:

ϕ ϕ πT st t( ) ( )− = 2

ϕs st t( ) = −ω

ωTrevd

radrev

ds

= ⋅ ⋅⋅

= ⋅ −111

124 3 600

7 3 10 5π, rad/s

ϕT Tt t( ) = ω

ϕ0 0=

hR

R

Rp

p

p

=−

= − =( )

( ) ,2 1

2 1 0 41

hR R R

R R R

p p p

p p p

=− ± +

=

=− ±

=−

⋅ ⋅ ⋅

⎧⎨⎪

2 4 4

2

2 2 2

2

2 1

2 2

( )

⎩⎩⎪

g h g

GM

R hG

M

R

R h R

p

p

p

p

p p

( ) ( )

( ) ( )

( )

=

+=

+ =

12

0

2

2

2 2

2 2

RR R h h R

h R h R

p p p

p p

2 2 2

2 2

2 2

2 0

+ + =

+ − =

0 3212

2 0 32

0 32 32 5 66

2= − = −

= − = =

t t t t

t t s

; ( );

; ,

y t y v t ato( ) = + +021

2

12

32 214

32 82 2

m m h h mP p= = =;

Page 188: 9523 gl fis_tx2_cas

195

Con los datos de problema:

De donde:

y:

Por tanto:

b) Sustituyendo en la ecuación de la velocidad ten-dremos:

c) A 10 cm del punto de equilibrio:

De donde:

De las múltiples soluciones tomamos por simplici-dad la positiva más pequeña (el resultado seráidéntico con cualquier solución del arcsen).

Sustituyendo:

De donde:

d) Con cálculos energéticos:

La energía cinética es inmediata:

2. a) De acuerdo con los datos del enunciado:

ω π ππ

= = =2 28

16N rad/s

Ec E Ep J( , ) ( , ) , , ,− = − − = − =0 1 0 1 0 448 0 112 0 336

E K A

E J

Ep

=

= ⋅ =

− =

12

12

22 4 0 20 0 448

0 112

22

2

2, , ,

( , ) ,44 0 10 0 1122⋅ =, , J

Ec J( , ) , ( , ) ,0 06512

0 35 1 39 0 3362= − =

� � �v i i( , ) , cos ( , ) ,0 065 8 0 2 8 0 065 1 39= ⋅ ⋅ + = −π m/ss

t s= 0 065,

0 10 2

80 10 2

8

8

,,

( );,,

;= + = +

+ =

sen arcsen

ar

t t

t

π π

π ccsen arcsen0 10 2

18

0 10 2

,,

;,,

t = −⎛⎝⎜

⎞⎠⎟

π

0 1 0 2 8, , ( )� �i t i= +sen π

� �

�v t t i

v

( ) , cos ( )

( , ) , (

= ⋅ +

= ⋅ ⋅

8 0 2 8

4 5 8 0 2 8

π

cos 44 5 0 20, ) ,+ =π� �i i m/s

� �

�r t t i

r

( ) , ( )

( , ) , ( ,

= +

=

0 2 8

1 5 0 2 81 5

sen

sen

π

++ =π) ,� �i i m0 11

− = − = − = =1 6 8 1 6 81 68

0 2, cos ; , ;,

,A A A mπ

0 0 20 0= ⇒ =sen( ) , , , ...ϕ ϕ π π

0

1 6 80

0

=

− =

⎧⎨⎪

⎩⎪

A sen i

i A i

( )

, ( )

ϕ

ϕ

� �cos

Sustituyendo:

3. a) De acuerdo con la definición de g:

Por tanto:

b) De acuerdo con la definición de :

Sustituyendo:

c) La segunda Ley de Newton y

De donde se obtiene la tercera ley de Kepler:

Por tanto:

Sustituyendo:

UNIDAD 4

MOVIMIENTOS VIBRATORIOS

1. a)

De donde:� �

� �r(t) A t i

v(t) A t i

= +

= +

⎧⎨⎪

sen

cos

( )

( )

8

8 80

0

ϕ

ϕ⎪⎪

ω = = =Km

22 40 35

8,

,rad/s

T ap = ⋅⋅

⎝⎜

⎠⎟ =1

7 7833 10

1 4960 1011 87

11

11

3,

,,

TG M

RG M

RR

RT

R

Rps

ops

OTop

OTT

op

OT

= = =4 42

32

33

3

3

3

π π==

TGM

Ro2

234

Gm M

Rm

v

RG

M

R

R

R TT

o

m

o o

o

o2

2

2

2 2

2

4= =

�; ;

π

ω = 2πT :

�ve = ⋅ ⋅⋅

⋅=2 318 9 8

6 378 10

7 1492 1059 552

6 2

7,( , )

,m//s

12

0

2 2318

2

2

2

m v GmM

R

v GM

RG

M

R

RR

ep

p

ep

p

T

T

T

p

− =

= = = 22 3182

⋅ gRRT

T

p

�ve

gp = ⋅⋅

⎝⎜

⎠⎟ =318 9 8

6 378 10

7 1492 1024 80

6

7

2

,,

,, NN/kg

g GM

RG

M

R

R

Rg

R

Rpp

p

T

T

T

pT

T

p

= = =2 2

2

2

2

2

318318

�vL = ⋅ ⋅ −⋅⎛

⎝⎜

⎠9 8 2 6 378 10

6 378 10

1 45106

6 2

7, ,( , )

, ⎟⎟ =

= ⋅9 87 103, m/s

Propuestas de evaluación

Page 189: 9523 gl fis_tx2_cas

196

Sustituyendo:

Si situamos la Ep = 0 en el punto más bajo de la os-cilación, la energía potencial en el punto más altode la oscilación será:

Como en el punto más alto no hay velocidad:

b) La constante de elasticidad asociada a la oscila-ción de un péndulo simple cuando se hace laaproximación en el cálculo de sen α ≈ α vale deacuerdo con la teoría:

Si tomamos α en radianes:

la amplitud de oscilación o elongación máximavaldrá:

Por tanto, la energía mecánica del sistema valdrá:

El error relativo efectuando los cálculos con todoslos decimales valdrá:

Es decir, del 0,57%.

4. a) En la posición inicial, antes de liberar el muelle,la energía potencial (Ep) es máxima mientrasque la energía cinética (Ec) es nula. Una vez li-berado el muelle, la energía potencial tenderáa 0 mientras que la energía cinética se hará má-xima.

b) Suponiendo que no hay pérdidas por rozamiento,el cuerpo tiene velocidad máxima (vmáx) cuandola energía cinética es máxima. Calculamos la ener-gía total del sistema:

Dado que Rcmáx = E = 1,25 J y que Rcmáx =

= 1/2 m (vmáx)2 se obtiene que:

vEcm

msmáx

máx= ± =⋅

=2 2 1 25

0 52 24

,,

,

E K A N m m J= = ⋅ ⋅ =−12

12

250 0 102 1 252 1 2, ,

ΔEE

=−

=0 1234 0 1227

0 12270 0057

, ,,

,

E J= ⋅ =12

9 6 0 16 0 12342, , ,

A L= ⋅ = ⋅ =α 0 6125 0 26 0 16, , ,

απ

= =15180

0 2600 ,

radrad

KmgL

= =⋅

=0 6 9 80 6125

9 6, ,,

, N/m

E Ep J= + =0 0 1227,

Ep J= ⋅ ⋅ =0 6 9 8 0 02 0 1227, , , ,

h m= − =0 6125 1 15 0 020, ( cos ) ,

De donde:

Lo que equivale a:

Eliminando A y despejando ϕ0:

Despejando A:

La ecuación del movimiento será:

b) Aplicando la fórmula de ω:

c) Aplicando la fórmula de la velocidad:

d) Con la fórmula de la energía mecánica:

3. a) La altura de la masa en el punto más elevado de laoscilación vale de acuerdo con la figura:

h L L L= − = −cos ( cos )15 1 150 0

L cos15o

15o

h

LL

E =12

K A

E =12

2,4,8 0,40 = 16,384 J

2

2⋅

� � �v i i

Ec

( ) , cos ,

(

2 16 0 4 16 26

2 86= ⋅ ⋅ +⎛⎝⎜

⎞⎠⎟

m/s

2212

0 8 2 86 3 232) , , ,= ⋅ = J

ω = = =

= ⋅ =

Km

K K

K

;,

;,

;

, ,

160 8

160 8

16 0 8 204 8

2

2 N/m

� �r t sen t i SI( ) , ( ) ( )= +0 4 16

0 26

0 26

0 20 5

0 4, ;,

( / ),,

,= = = =A sen Asen

π

0 2

3 2 3 16

0 2

3 2 3 160

0

,

, cos;

,

,

=

=

⎧⎨⎪

⎩⎪=

A sen

A

AA

ϕ

ϕttg

tg tg

arctg

ϕ

ϕ ϕ

ϕ

0

0 0

0

16 0 2

3 2 3

1

3

1

3

;

,

,;

⋅= =

=⎛⎝⎝⎜

⎞⎠⎟

=⎛

⎝⎜

⎠⎟ =arctg rad

33 6

π

0 2 2

3 2 3 16 20

0

, ( )

, ( )

= +

= +

⎧⎨⎪

⎩⎪

A

i A

sen

cos

π ϕ

π ϕ�

0 168

3 2 3 16 168

0,

, cos

2 i A i

i A

� �

= +⎛⎝⎜

⎞⎠⎟

= ⋅

senπ

ϕ

π++

⎛⎝⎜

⎞⎠⎟

⎨⎪⎪

⎩⎪⎪

ϕ0

�i

Page 190: 9523 gl fis_tx2_cas

197

c) A partir de la función de onda:

El máximo de la aceleración es:

d) Teniendo en cuenta:

y sustituyendo:

Que equivale a:

(Dado que: )

3. a) Con las fórmulas básicas calculamos las constantescaracterísticas de la onda:

Despejando β:

Por tanto, la función de onda en mb:

El valor que se pide:

b) Aplicando la fórmula que refleja la absorción:

Despejando x y sustituyendo β:

0 01 0 01

0 01 0 01

2 36

, ; ln ,

ln , ln ,

,

= − =

=−

=−

−e x

x

xβ β

β 1101952− = m

A Ae x1100

= −β

y

e

( , ; )

( ,,

6 0003 15

30 3 142 6 002 36 10 152

=

= ⋅− ⋅ ⋅−

sen 003 9 24 15

11 41

− ⋅ =

=

, )

, mb

y t x e t xx( , ) ( , ),= −− ⋅ −

30 3 142 9 242 36 10 2

sen

β =−

= ⋅ − −ln ,,

0 351

2 36 10 2 1m

ω = = =

= =

= =

2 2 500 3 142

510 15

340

π π

λ

f

v

vf

,

rad/s

m/s

3340500

0 68

3070

10030 30 0 351 51

=

− = =− −

,

; ,

m

e eβ β ;;

225 2 35/ , ...π =

225 225 2 35 5 02rad rad≡ − ⋅ =π ,

ϕ ϕ( , ; , ) ( , )

( , ) ( ,

t x t x

t x

+ + − =

= + + +

0 45 4 5

300 0 45 20 4 55 300 20

300 0 45 20 4 5 225

)

, ,

− − =

= ⋅ + ⋅ =

t x

rad

ϕ( , )t x t x= +300 20

a t xmáx2m/s( , ) ,= ⋅ =1 2 300 10 8002

y t x t x

v t x

( , ) , ( )

( , ) , cos(

= +

= ⋅

1 2 300 20

1 2 300 30

sen

00 20

1 2 300 300 202

t x

a t x t x

+

= − ⋅ +

)

( , ) , ( )sen

UNIDAD 5

MOVIMIENTO ONDULATORIO

1. a) Calculamos en primer lugar las constantes caracte-rísticas de la onda:

b) Sustituyendo en la fórmula de la velocidad:

c) La fase de un punto en un instante es:

La diferencia de fase en un instante de dos puntosseparados 68,45 m será:

Que equivale a:

(Dado que ).

d) Sustituyendo en la función de la aceleración:

2. a) Aplicando las fórmulas básicas:

b) Teniendo en cuenta que la frecuencia indica las os-cilaciones por s:

n f t n osc= ⋅ = ⋅ =; ( min) , , .1 47 75 60 2 864 79

λπ π

π π

= = =

= = = =

=

2 220

0 314

12

3002

47 75

km

fT

Hz

v

,

λλf = ⋅ =0 314 47 75 15, , m/s

a(t,x) 4,09 10 sen(1 508t 134 x)

a(6,89;89,41

5= − ⋅ −

99)

sen

=

= − ⋅ ⋅ − ⋅ =

=

4 09 10 1 508 6 89 134 89 419

13

5, ( , , )

7714 m/s2

9 175 2 1 460/ ,...π =

9 175 9 175 2 1 460 1 68( ) ,rad rad≡ − ⋅ =π

Δϕ = − − − +⎡⎣ ⎤⎦ =

= ⋅

1 508 134 1 508 134 68 45

134 6

t x t x( , )

88 45 9 175, = rad

ϕ( , )t x t x= −1 508 134

v(t,x) 271 cos (1 508t 134 x)

v(8,56;35,5)

271 c

= −

=

= oos (1 508 8,56 134 35,5) m/s⋅ − ⋅ = 110

ω = = ⋅ =

= =

= =

−2 2 240 1 508

36032

11 25

1πν π

λ

s

vms

vf

, m/s

111 25240

0 047

2 20 047

134 1

,,

,

( , )

=

= = =

=

m

k m

y t x

π

λπ

00 18 1 508 134, ( ) ( )sen t x SI−

Propuestas de evaluación

Page 191: 9523 gl fis_tx2_cas

198

2. a) De acuerdo con las condiciones de una cuerda in-movilizada por sus dos extremos:

De donde:

Por tanto:

b) Las frecuencias de los armónicos responden a laecuación:

Por tanto:

c) De acuerdo con la ecuación

La longitud de onda del 5º armónico:

Como los nodos se producen en la cuerda cada, sus coordenadas serán:

d) Dado que:

Tendremos:

En consecuencia:

f = 810 Hz

f = 1 620 Hz

f = 2 430 Hz

1

2

3

f n nn′ = =810 1 2 3; , , , ...

fv v

L

v

L

vL

vL

f

11

1

2 2 2 2 3

32 2

32

32

5

′′ ′ ′= = = = =

= = =

λ ( )/

440 810=

x m

x m

x m

x m

x m

x

1

2

3

4

5

0

0 16

0 36

0 48

0 64

=

=

=

=

=

,

,

,

,

66 0 8= , m

λ5 /2

λ525

2 0 85

0 32= =⋅

=L ,

,

λ = =2

1 2 3L

nn; , , , ...

f Hz

f Hz

f

3

7

20

3 540 1 620

7 540 3 780

20 540

= ⋅ =

= ⋅ =

= ⋅ = 110 800 Hz

f nf nn = =1 1 2 3; , , , ...

v f= ⋅ = ⋅⋅

=1 1 5402 0 8

1864λ

,m/s

λn2 Ln

n= =; , , , ...1 2 3

L n nn= =λ

21 2 3; , , , ...

4. Sabemos que la velocidad es de donde deduci-

mos que por lo tanto, se tiene:

Para el hierro (Fe):

Para el aire:

Concluimos de este modo que la razón de los tiem-

pos será:

UNIDAD 6

FENÓMENOS ONDULATORIOS

1. a) Aplicando las fórmulas básicas:

Las ecuaciones que se piden son:

b) La onda resultante de la interferencia oscila esta-cionariamente a 800 Hz; por tanto:

c) De acuerdo con las fórmulas deducidas en la teoría:

Por tanto:

d) De acuerdo con las fórmulas deducidas en la teo-ría:

Por tanto:

x n n nn = + = + =( ),

( ) , ; , , , , ...2 10 44

2 1 0 1 0 1 2 3

x n nn = + =( ) ; , , , , ...2 14

0 1 2 3λ

x n n nv = = =20 44

0 2 0 1 2 3,

, ; , , , , ...

x n nv = =24

0 1 2 3λ

; , , , , ...

t osc Tf

s

N

( .)

,

(

100 100 1001

1001

8000 125

2

= ⋅ = =

= =

mmin) .= ⋅ = ⋅ =f osc120 800 120 96 000

y t x t x SI

y t x

1

2

0 6 5 027 15 7

0

( , ) , ( , ) ( )

( , ) ,

= −

=

sen

66 5 027 15 7

1 2 15

sen

cos

( , ) ( )

( , ) , (

t x SI

y t xi

+

= ,, ) s n ( ) ( )7 5 027x e t SI⋅

ω = = ⋅ =

= = =

2 2 800 5 927

2 20 4

15 7

1

1

π π

π

λ

π

f s

k m,

,

TT

ss

Fe

aire

= =0 00580 0882

0 0657,,

,

Tmms

saire = =30

3400 0882,

Tm

ms

sFe = =30

5 1300 0058,

tev

= ;

vet

=

Page 192: 9523 gl fis_tx2_cas

199

b)

UNIDAD 7

CAMPO ELÉCTRICO

1. a) Es evidente que si llamamos d a la distancia entrelas cargas y q al valor absoluto de las cargas:

De donde:

b) Si tenemos en cuenta, además, que:

Despejando y sustituyendo:

c) Evaluamos la energía potencial eléctrica y la gra-vitatoria.

Dado que

Sumando ambas energías:

2. a) De acuerdo con la definición de densidad de car-ga superficial:

b) Del teorema de Gauss aplicado al campo eléctricose deduce fácilmente:

Sustituyendo:

E = ⋅ ⋅ ⋅ =−2 5 10 9 10 282 69 9π , N/C

�E

K

K= = =σε

σ

π

π σ2 2

14

20

σ = =⋅⋅

= ⋅−

−QS

2 1 100 6 0 7

5 109

9,, ,

C/m2

Ep J= + =0 18 0 042 0 22, , ,

h m= − =0 8 0 8 30 0 69, , º ,cos

Ep e Kqd

J

Ep g mgh

( ),

,

( )

-

= = ⋅⋅

=

= =

29

6

9 104 10

0 80 18

9,, , , ,8 0 04 0 69 0 042⋅ ⋅ = J

qTd sen

KC= =

⋅ ⋅

⋅= ⋅ −

2 3

9630 0 45 0 8

9 104 10

º , , 1/2

sen/

sen302

0 82 0 8 30 0 8= = ⋅ ⋅ =

dd

,; , º ,

Tmg

N= =⋅

=cos º

, ,,

300 04 9 8

3 20 45

/

T Kq

dT mg

sen 30

30

2

cos º

=

=

⎨⎪

⎩⎪

f fv vv v

Hzm s m s

m sRR

F

=−

+=

⋅ − ⋅

⋅ +

− −

−450340 24

340

1 1

1 330

450316

370450 0 85

1

1

1

m s

Hzm s

m sHz

⋅=

=⋅

⋅= ⋅ =

− , 3382 5, Hz

3. Los módulos de las velocidades del primer coche y delcamión valen en el SI:

a) La fórmula que relaciona las frecuencias y las velo-cidades en el efecto Doppler es:

De acuerdo con los sentidos de las velocidades enel caso del coche 1 y el camión se plantea:

Sustituyendo:

En el caso del receptor en reposo:

b) Si planteamos la ecuación para los dos coches:

Sustituyendo:

Despejando;

4. Dado que tanto la fuente del sonido (el claxon del ca-mión) como el receptor (el motorista) están en movi-miento uno respecto del otro, la frecuencia vendrádada por la expresión:

donde en el numerador el signo será + si el receptor seaproxima y − si de aleja, mientras que en el denomina-dor será − si la fuente se aproxima y + si se aleja. Así,obtenemos:

a) f fv vv v

Hzm s m s

m sRR

F

=+

−=

⋅ + ⋅

⋅ −

− −

−450340 24

340

1 1

1 330

450364

310450 1 17

1

1

1

m s

Hzm s

m sHz

⋅=

=⋅

⋅= ⋅ =

−, 5527 87, Hz

f fv vv vR

R

F

±,

340 2 750340 30

800

2 340 750340 30

80

− =−

= −−

v

v

F

F

( )

( )00

49 38 177= =, ...m km/h

800 750340 30

340 2=

−− v F( )

f fv vv vR F

R

F

( ) ( )( )( )

1 212

±

f r HzR( ) =−

=750340

340 30823

f c HzR( ) =+−

=750340 20340 30

871

f c fv v cv vR F

R

F

( ) ( )( )( )

±1

1

f fv vv vR F

R

F

±

vkmh

mkm

hs

v ckmh

( )

( )

1 1081 000

3 60030

72

= =

=

m/s

11 0003 600

20m

kmh

s= m/s

Propuestas de evaluación

Page 193: 9523 gl fis_tx2_cas

200

UNIDAD 8

CAMPO MAGNÉTICO

1. a) De acuerdo con la teoría, el campo magnético crea-do en el centro de la circunferencia por un frag-mento F (asociado a α) en el plano x-y vale:

Por tanto:

Si el ángulo se expresa en grados:

b) Para un fragmento de circunferencia correspon-diente a un ángulo de 70º:

c) Si queremos que el módulo del campo magnéticovalga 5 ⋅10−5 T, la intensidad se calcula:

De donde:

2. a) Teniendo en cuenta que la energía suministradapor el potencial se convierte en energía ciné-tica:

De donde:

b) De acuerdo con las fórmulas de la fuerza magnéti-ca y la aceleración centrípeta:

De donde:

c) El potencial de aceleración se relaciona con elradio de la circunferencia mediante la expre-sión:

Rm v

q v B

m v

q B=

⋅ ⋅=

⋅=

⋅ ⋅ ⋅ ⋅

−�

� ���

2 27 57 1 67 10 5 10

3 1

,

,, ,,

6 10 0 80 01519⋅ ⋅

=− m

q v B F mv

Rm⋅ ⋅ = =� � � � 2

Vmq

v= =⋅ ⋅

⋅ ⋅ ⋅⋅ =

−27 1 67 10

2 3 1 6 105 10

227

195 2� ,

,( ) 33 344 V

12

2m v q V�

=

I A=⋅ ⋅ ⋅

⋅ ⋅ ⋅=

720 0 2 5 10

4 10 7081 85

5

7

,,

π

��

BIR

IR B

o

o

= ′ =′

μα

μ α720

720;

�B T=

⋅ ⋅ ⋅⋅

= ⋅−

−2 10 5720 0 20

70 3 05 107

6π,

,

�B

IR

en gradoso= ′ ′μ

α α720

;

�B

IR

en radianeso=μ

πα α

4;

� � �B

I

Rdl k

I

RR k

IRF

o o o= = =⌠⌡⎮

( )α

μ

π

μ

πα

μ

πα

4 4 42 2

��k

c) Sabiendo que el trabajo que realizará la fuerzaeléctrica se invierte en energía cinética:

Despejando y sustituyendo:

3. a) De acuerdo con las fórmulas del condensador pla-no con aire como dieléctrico:

Despejando y sustituyendo:

b) La relación entre carga y diferencia de potencialentre las placas es:

Despejando y sustituyendo:

c) La capacidad de un sistema de condensadores co-nectados en paralelo se obtiene:

Y en serie:

4. a) Teniendo en cuenta que los condensadores se co-nectan en paralelo uniendo entre sí las placas posi-tivas, la capacidad final del sistema será:

b) La energía potencial eléctrica acumulada en un

condensador es expresión que vale 0

al inicio del proceso de carga ya que V = 0, mien-tras que al final de dicho proceso su valor es:

E C V C V F Vfinal = + = +

+

−12

12

12

5 10 100

12

1

1 12

2 22 4 2 2⋅

⋅⋅10 200 2 5 20 22 53 2 2− = + =F V J J J, ,

E CVP =12

2,

C C C 0,5 mF 1 mF 1,5 mF1 2= + = + =

C pF=+ + +

= = =1

120

120

120

120

14 20

204

5/

C pF= + + + = ⋅ =20 20 20 20 4 20 80

Q CV C= = ⋅ ⋅ = ⋅− −20 10 220 4 4 1012 9,

CQV

=

dS

Cm= =

⋅=−

ε

π0

9 12

1

4 9 10

0 1 0 2

20 100 0088

, ,,

CS

d=

ε0

��

vq E d

m= =

=⋅ ⋅ ⋅ ⋅

=−

2

2 6 10 282 7 0 20 004

4 124 , ,,

, m/s

12

2m v q E d� �

=

Page 194: 9523 gl fis_tx2_cas

201

UNIDAD 9

INDUCCIÓN ELECTROMAGNÉTICA

1. a) Calculamos previamente la superficie de cada es-pira:

El flujo valdrá:

De acuerdo con la teoría, la fuerza electromotrizse calcula:

En t = 5 s:

b) En t = 7 s la fem inducida valdrá:

Teniendo en cuenta la ley de Ohm:

c) Es evidente que:

De donde:

2. a) Calculamos previamente S y ω:

Dado que el ángulo formado por las espiras con elcampo magnético varía según la expresión α = ωt,el flujo será:

La fem generada será:

b) De acuerdo con la expresión de ε(t), la fem máxi-ma es 22,7 V. Por tanto:

IR

A00 22 7

800 28= = =

ε ,,

e tddt

t

t t

( ) ,

( ) , (

= − = ⋅

=

Φ0 06 377 377

22 7 377

sen

senε SSI)

Φ

Φ

= = ⋅ ⋅ ⋅

=

−n B S t t�

cos , , cos

,

ω 50 0 8 1 5 10 377

0 06

3

cos 377 t

S m= ⋅ = ⋅

= ⋅ ⋅ =

−0 03 0 05 1 5 10

2 60 377

3 2, , ,

ω π rad/s

0 1140

0 850 1 400 85

4 72, , ;,,

,= =⋅

=t t s

I tt

Rt( )

( ),= =

ε 140

0 85

IR

A( )( ) ,

,77 5 94

400 15= = =

ε

ε( ) , ,7 0 85 7 5 94= ⋅ = V

ε( ) , ,5 0 85 5 4 24= ⋅ = V

ε(t)ddt

t t= − = ⋅ =Φ

7 07 0 12 0 85, , ,

Φ

Φ

= = ⋅ − ⋅ ⋅

= =

−n B S t

n B S

�900 6 0 06 7 85 10

7 07

2 3( , ) ,

, (66 0 06 2− , )t

S r m= = ⋅ = ⋅ −π π2 2 30 05 7 85 10, ,

Sustituyendo:

3. a) De acuerdo con los datos del problema las distan-cias del punto b a los conductores son:

De acuerdo con la teoría, los campos magnéticoscreados por cada corriente en el punto b tienen lamisma dirección y sentido; por lo tanto, el módulodel campo resultante será:

Sustituyendo:

b) Teniendo en cuenta que las distancias entre el pun-to a y los conductores son:

El campo en el punto a se calcula:

De donde se deduce que:

Despejando:

c) Las distancias del punto c a los conductores son:

Teniendo en cuenta que los campos tiene direccio-nes iguales y sentidos contrarios:

�B T=

⋅ ⋅−

⋅ ⋅= ⋅

− −−4 10 6

2 0 54 10 2

2 1 32 09 10

7 76π

π

π

π, ,,

d = 0,8 + 0, 5 = 1,3 m; d = 0, 5 m1,c 2,b

2 0 4 6 0 8 6 2 4 0 8

0 84

0 2

( , ) ; , ; ,

,,

d d d d d

d m

+ = = − =

= =

4 10 22

4 10 62 0 4

7 7⋅ ⋅=

⋅ ⋅

+

− −π

π

π

πd d( , )

04 10 2

24 10 62 0 4

7 7

= =⋅ ⋅

−⋅ ⋅

+

− −�B a

d d( )

( , )π

π

π

π

d d d da a1 2 0 4, ,; ,= = +

�B T=

⋅ ⋅+

⋅ ⋅= ⋅

− −−4 10 2

2 0 44 10 6

2 0 44 10

7 76π

π

π

π, ,

�B

Id

Id

o

b

o

b

= +μ

π

μ

π1

1

2

22 2, ,

d d mb b1 2 0 8 2 0 4, , , ,= = =/

V =⋅ ⋅ ⋅ ⋅

⋅ ⋅ ⋅=

3 1 6 10 0 8 0 04

2 7 1 67 1021 02

19 2 2

27

, , ,

,33 V

12 2

2

2

2

m v q V

q v B mv

R

Vm v

q

v

� � �

=

=

⎨⎪⎪

⎩⎪⎪

=;

==

⎪⎪

⎪⎪

= =

q B R

m

Vmq B R

qm

q B R

m

� �2 2 2

2

2 2

2 2

Propuestas de evaluación

Page 195: 9523 gl fis_tx2_cas

202

c) Según la teoría:

Sustituyendo:

El momento de cortar la alimentación se aproxi-ma por t = 0 despreciando el tiempo que tarda enactivarse la fuerza electromotriz inducida.

d) En t = 10 ms = 0,010 s

4. a) Sabemos que en los transformadores se cumple lallamada relación de transformación:

de donde deducimos que

Despejando se obtiene finalmente que:

b) A partir de la misma relación obtenemos que:

De aquí se puede deducir que la intensidad de sa-lida del transformador I2 es:

UNIDAD 10

LA LUZ

1. a) La velocidad de la luz en el vacío vale: c = 3 ⋅ 108

m/s, por tanto:

Por tanto:

b) Los parámetros de la onda son:

k m

w f

= =⋅

=

= = ⋅ = ⋅

−2 21 11

2 98

2 2 9 5 10 5 97

1

7

π

λπ

π π

,,

, , 1108 rad/s

v fvf

m= = =⋅

⋅=λ λ;

,,

2 10

9 5 102 11

8

7

ncv

vcn

= = =⋅

= ⋅;,

3 101 5

2 108

8 m/s

I INN

Aespiras

espiras2 11

2

2 518 000

82548 7= = =, , 88 A

II

NN

2

1

2

1

= .

NV N

VV espiras

Vespiras2

2 1

1

10 1 800220

82= = =

VV

NN

2

1

2

1

= .VV

II

NN

2

1

2

1

2

1

= =

ε( , ) ,,0 01 136 0 18662 0 01= =− ⋅e V

ε( )0 136 136662 0= =− ⋅e V

ε( ) ,( , )

td e

dte

tt= − =

−−0 27

0 75136

662662

ε( )( )

t LdI t

dt= −

c) El flujo en función de ω y t vale:

La fem en función de ω y t vale:

La fem máxima en función de ω:

Por tanto:

Despejando ω y sustituyendo:

De donde:

3. a) Primero calculamos la S de cada espira:

De acuerdo con la teoría, el coeficiente de autoin-ducción L vale:

b) La intensidad en régimen continuo coincide con laintensidad máxima de la fase de apagado:

La función intensidad en la fase de apagado vale:

Si en 10 ms cae a 1 mA:

De donde:

Por tanto:

I t e SIt( ) , ( )= −0 75 662

− =

=−

=

k

k

0 010 0010 75

0 75 0 0010 01

662

, ln,,

ln , ln ,,

0 001 0 01 0 75 0 01, ( , ) , ,= = −I e k

I t e kt( ) ,= −0 75

IVR0

3040

0 75= = = ,

LNl

S H= = =−μ π0

27

2

4 102 500

0 90 031 0 274

,, ,

LL N

Nm

rL

( )( )

,

( ) ,

117512 500

0 628

12

0 6282

= = =

= =π π

==

= = =

0 1

0 1 0 0312 2 2

,

, ,

m

S r mπ π

f Hz= =2 0002

318π

ω = =⋅

=800 06

80 1 50 06

2 0000I,

,,

IR0 = =ε0 0 06

80, ω

ε0 0 06( ) ,ω ω=

ε( , ) ,ω ωt w t= 0 06 sen

Φ( , ) , cosw t t= 0 06 ω

Page 196: 9523 gl fis_tx2_cas

203

b) De acuerdo con los cálculos elementales de losanillos de difracción, la desviación α de los rayosque producen la primera franja oscura se obtiene:

De donde:

Sustituyendo:

c) Si combinamos las fórmulas utilizadas en los apar-tados anteriores, obtendremos:

Es decir:

Sustituyendo:

d) Despejamos λ en la fórmula anterior:

Sustituimos α por el valor obtenido a 2 000 au-mentos:

La frecuencia correspondiente a esta longitud deonda:

Se trata de luz ultravioleta.

4. a) Deberemos colocar una lente convergente cuyafocal calculamos con la expresión:

Despejando y sustituyendo:

Las dioptrías se calculan:

Pf

dioptrías= = =1 1

0 0499320

,

f

s s

=−

=−

=1

1 11

140

10 05

0 04993

2 1 ,

,

1 1 1

2 1f s s= −

f Hz=⋅

⋅= ⋅−

3 10

4 87 106 16 10

8

815

,,

λπ

=⋅

= ⋅−

−2 4 89 1030 000

0 00074 87 10

28, ,

,sen

m

λα

π=

2 sena

S

απ

=⋅ ⋅⎛

⎝⎜

⎠⎟ =arcsen ra

7 3 10 30 0002 0 0007

0 827,

,, dd

αλ π

=⎛

⎝⎜

⎠⎟arcsen

aS2

αλ λ λ

π=

′= =arcsen arcsen

d/aarcsen

S/da

2

α =⋅

⋅= ⋅

−−arcsen rad

7 3 10

1 5 104 89 10

7

52,

,,

αλ

=′

arcsend

sen αλ

=′d

dda

m′ = = = ⋅ −0 032 000

1 5 10 5,,

Suponemos que se propaga en la dirección del ejeOZ en sentido positivo; por tanto:

c) La relación entre los módulos máximos de los cam-pos eléctrico y magnético es:

Por lo tanto:

De donde:

2. a) Situando el origen de coordenadas en la franjacentral, la coordenada de la primera franja brillan-te es:

Despejando en esta expresión la distancia entre lasrejillas (d) para n = 1:

b) Si aplicamos la fórmula anterior a las cinco prime-ras franjas luminosas a partir del valor de y1, obte-nemos:

c) Debemos despejar L en la fórmula y sustituir:

La distancia que separa las franjas corresponde ay1; por tanto, sustituiremos en la ecuación los valo-res con n = 1:

3. a) El diámetro de la muestra valdrá:

Si tenemos en cuenta que el microscopio aumenta2 000 veces el tamaño de los objetos, el tamaño dela parte de la muestra cuya imagen llega al objetivoserá la fracción a = 2 000 del diámetro de la mues-tra. Si llamamos d’ al diámetro del círculo cuya luzllega al objetivo:

d rS

= = = =2 2 20 0007

0 03π π

,,

Ly d

m=⋅

=⋅ ⋅

⋅=

−1

5

7

0 05 4 86 10

6 104 05

λ, ,

,

Ly dn

n=λ

y m

y y m

y y m

y

1

2 1

3 1

4

0 021

2 0 042

3 0 063

=

= ⋅ =

= ⋅ =

=

,

,

,

44 0 084

5 0 105

1

5 1

⋅ =

= ⋅ =

y m

y y m

,

,

dL

ym= =

⋅ ⋅= ⋅

−−λ

1

751

6 10 1 70 021

1 4 86 10,

,,

yL

dnn =

λ

� �B z t t z j SI( , ) ( , , ) ( )= ⋅ −−10 5 97 10 2 988 8sen

BEvo

0 882

2 1010= =

⋅= −

EB

vo

0

=

� �E z t t z i SI( , ) ( , , ) ( )= ⋅ −2 5 97 10 2 988sen

Propuestas de evaluación

Page 197: 9523 gl fis_tx2_cas

204

c) Si en la Tierra se contabilizan 7 años, el tiempocontabilizado en la nave se obtiene con la expre-sión:

Sustituyendo:

Lo que equivale a 4 meses y 6 días.

2. a) De acuerdo con la mecánica clásica, si E es el mó-dulo del campo eléctrico, mo la masa en reposo delprotón, q su carga, t el tiempo de aceleración y v elmódulo de la velocidad al final de la aceleración:

Despejando:

Sustituyendo:

O bien: 0,64 c

b) Si operamos con las ecuaciones relativistas:

Despejando:

Sustituyendo:

v =⋅

+⋅ ⋅ ⋅

⋅ ⋅ ⋅

3 10

11 67 10 8 10

400 1 7 10 0 00

8

27 8

19

,

, , 55

1 61 10 0 52

2

8

⎝⎜

⎠⎟

=

= ⋅ =, ,m/s c

1

1

2

2 2

−⎛⎝⎜

⎞⎠⎟

=

−⎛⎝⎜

⎞⎠⎟

=⎛

⎝⎜⎞

⎠⎟

vc

m vEqt

vc

mEqt

v

o

o 22

2 22

2

2 22

1

c vm cEqt

v

c vm cEqt

o

o

− =⎛

⎝⎜⎞

⎠⎟

= +⎛

⎝⎜⎞

⎠⎟

⎡⎡

⎢⎢

⎥⎥

=

+⎛

⎝⎜⎞

⎠⎟

vc

m cEqt

o12

Eqt mvm v

vc

o= =

−⎛⎝⎜

⎞⎠⎟

12

v =⋅ ⋅ ⋅

⋅= ⋅

400 1 6 10 0 005

1 67 101 92 10

19

278, ,

,, m/s

vEqtmo

=

Eqt m vo=

Δt años′ = −⋅

⎝⎜

⎠⎟ =7 1

2 9962 10

3 100 35

8

8

2,

,

Δ Δt tvc

′ = −⎛⎝⎜

⎞⎠⎟

12

b) El tamaño de la imagen se calcula con la fórmula:

Despejando y2 y sustituyendo:

c) Se trata de volver a calcular s2 para el valor s1 = −0,05 − 0,0001 = −0,0501. Utilizamos de nuevo:

Despejando y sustituyendo:

El aumento será:

Es decir, la imagen será 307,5 veces mayor e invertida.

UNIDAD 11

FÍSICA RELATIVISTA

1. a) De acuerdo con la teoría de la relatividad, los inter-valos de tiempo en la nave (Δt’) se relacionan conlos intervalos de tiempo en la Tierra (Δt)

Sustituyendo:

b) Si el tiempo de travesía medido en la Tierra es de10 años (120 meses), se cumplirá la expresión

Despejando:

O también: 0,99875 c

13 10

6120

16

120 3 10

8

2

2

8

−⋅

⎛⎝⎜

⎞⎠⎟

=

−⎛⎝⎜

⎞⎠⎟

=⋅

⎛⎝⎜

v

v ⎞⎞⎠⎟

= ⋅ −⎛⎝⎜

⎞⎠⎟

= ⋅

2

82

83 10 16

1202 9962 10v , m/s

1206

13 108

2=

−⋅

⎛⎝⎜

⎞⎠⎟

v

Δt meses=− ( )

=6

1 0 810

2,

ΔΔ

tt

vc

=′

−⎛⎝⎜

⎞⎠⎟

12

AssL = =

−= −2

1

15 400 0501

307 5,

,,

s

f s

m2

1

11 1

11

0 04991

0 0501

15 40=+

=+

=

, ,

,

1 1 1

2 1f s s= −

y yss

m2 12

1

0 035400 05

28= = − −−

=,,

yy

AssL

2

1

2

1

= =

Page 198: 9523 gl fis_tx2_cas

205

Despejando:

b) Calculamos para cada una E, E(0) y Ec.

c) La energía cinética clásica vale:

La relativista con cuatro decimales:

La similitud está justificada porque la velocidad esel 1% de c.

4. En primer lugar, calculamos la constante:

Aplicando la fórmula relativista de la adición de velo-cidades, obtenemos:

La velocidad de las partículas será:�

v c c

v c

′ = − −

= − + − =

( , , , )

( , ) ( , )

0 45 0 55

0 45 0 552 2 00 71, c

vv u

v u

c

cc

c

c

v

xx

x

y

=−

−=

−= −

=

1

0 0 45

10 45

0 45

2 2

,,

,

vv

v u

c

c

c

cy

x

1

1

0 75 1 0 45

10 45

0 552

2

2

2

−=

−= −

β , ,,

,

β = =0 45

0 45,

,c

c

Ec relativista J1 79 0865( ) ,=

Ec clásica c J111 21

21 76 10 0 01 79 0806( ) , ( , ) ,= ⋅ ⋅ =−

E J

E

1

11 8 2

2

6

1

1 76 10 3 10

1 0 11 5816 10=

⋅ ⋅ ⋅

−= ⋅

−, ( )

,,

(00 1 76 10 3 10 1 5816 1011 8 2 6

1 1

) , ( ) ,= ⋅ ⋅ ⋅ = ⋅

= −

− J

Ec E E11

2

11 8 2

2

0 79

1 76 10 3 10

1 0 21 581

( )

, ( )

,,

=

=⋅ ⋅ ⋅

−=

J

E 99 10

0 1 76 10 3 10 1 5816 10

6

211 8 2 6

= ⋅ ⋅ ⋅ = ⋅−

J

E ( ) , ( ) , JJ

Ec E E J

E

2 2 2

3

11 8 2

0 316

1 76 10 3 10

1

= − =

=⋅ ⋅ ⋅

( )

, ( )

00 31 5823 10

0 1 76 10 3 10

2

6

211 8 2

,,

( ) , ( )

= ⋅

= ⋅ ⋅ ⋅ =−

J

E 11 5816 10

0 712

6

3 3 3

,

( )

= − =

J

Ec E E J

mo =

−+ ⋅

−+

−+

−−3 10

1 0 42 10

1

1 0 01

1

1 0 02

1

11

2

11

2 2

,

, , 11 0 03

1 76 10

2

11

= ⋅ −

,

,m kgo

c) La energía relativista se calcula con la fórmula:

Sustituyendo:

d) Con la velocidad calculada en el apartado b), secalcula:

Sustituyendo:

Con la aproximación de la física clásica:

3. a) De acuerdo con el principio de conservación de laenergía:

Simplificando:

3 10

1 0 42 10

1 0 01 1 0 02

11

2

11

02

02

−+ ⋅ =

=−

+−

+

−−

,

, ,

m m mm021 0 03− ,

E E

c

cc

inicial final=

−⎛⎝⎜

⎞⎠⎟

+⋅−3 10

10 4

2 1011 2

2,

−−

−=

=

−⎛⎝⎜

⎞⎠⎟

+

11 2

2

02

20

2

1 0

10 01

10 0

c

m c

cc

m c

( )

, , 221

0 0320

2

2cc

m c

cc

⎛⎝⎜

⎞⎠⎟

+

−⎛⎝⎜

⎞⎠⎟

,

Ec clásica m vo( )

, ( , )

= =

= ⋅ ⋅ ⋅−

12

12

1 67 10 1 6 10

2

27 8 2 == ⋅ −2 18 10 11, J

E = ⋅ ⋅ ⋅ ⋅

⋅− ⋅ − ⋅

−1 67 10 3 10

1 3 10 1 61 10

27 8 2

8 2 8

, ( )

( ) ( , ))

( ) ( , ),

2

8 2 8 2

10

3 10 1 61 103 29 10

⋅ − ⋅= ⋅ − J

Ec relativista mc m c

m c

vc

m

o

o

( ) = − =

=

−⎛⎝⎜

⎞⎠⎟

2 2

2

2

1

oo

o

o

c

m c

m c

2

22

2

2

1 1

1

1

=

=− −

⎝⎜⎜

⎠⎟⎟

=−

( )

( )

v/c

v/c

cc v

c v

2 2

2 2

⎝⎜⎜

⎠⎟⎟

E =⋅ ⋅

−⋅

⎝⎜

⎠⎟

=−1 67 10 3 10

11 61 10

3 10

27 8 2

8

8

2

, ( )

,11 78 10 10, ⋅ − J

E mcm c

vc

o= =

−⎛⎝⎜

⎞⎠⎟

22

2

1

Propuestas de evaluación

Page 199: 9523 gl fis_tx2_cas

206

b) La velocidad de los protones (sin aplicar la teoríade la relatividad):

Sustituyendo:

Por otro lado:

Ambos valores no deben coincidir porque, aun-que parezca contradictorio, la teoría cuántica noidentifica la velocidad de la partícula con la veloci-dad de propagación de la onda asociada (v = f ⋅ λ).

c) Despejando V en la relación obtenida anterior-mente:

Tendremos:

Sustituyendo:

3. a) De acuerdo con la fórmula de física cuántica:

donde Eo = 13,606 eV

Tendremos:

b) Las frecuencias correspondientes a las transi-ciones entre estos niveles se calculan con la fór-mula:

fE

h

f

m nm n

,,

,, ,

,

=

=⋅ − ⋅− −

Δ2

1 2

17 181 96 10 4 90 10

6 625 ⋅⋅= ⋅−10

2 22 103416, Hz

E eV J

E

1

2

217

2

13 6063

1122 5 1 96 10

13

= − = − = − ⋅

= −

−, , ,

,, , ,

,

6063

230 6 4 90 10

13 6063

2

217

3

2

= − = − ⋅

= −

−eV J

E33

13 6 2 18 10217= − = − ⋅ −, ,eV J

E EZ

nn o= −2

2

V =⋅

⋅ ⋅ ⋅ ⋅

− − −

( , )

, , (

6 625 10

2 1 67 10 1 6 10 10

34 2

27 19 122 2 821)

= V

Vh

mq=

2

22 λ

λ = =h

mqV2

f λ = ⋅ ⋅ ⋅ = ⋅−9 7 10 1 4 10 1 38 1016 12 5, , , m/s

v =⋅ ⋅ ⋅

⋅= ⋅

2 1 6 10 400

1 67 102 77 10

19

275,

,, m/s

vEm

qVm

= =2 2

=⋅

⋅ ⋅ ⋅ ⋅ ⋅= ⋅

− −

6 625 10

2 1 67 10 1 6 10 4001 4

34

27 19

,

, ,, 110 12− m

UNIDAD 12

FÍSICA CUÁNTICA

1. a) La relación entre el potencial de detención (VD),la longitud de onda de la radiación (λ) y la funciónde trabajo (Wo) está dada por la expresión:

De donde:

Sustituyendo:

b) La longitud de onda umbral es aquella para la cuala los electrones no les queda Ec; por tanto:

De donde:

Sustituyendo:

c) El módulo de la velocidad máxima (vmáx) de loselectrones emitidos se relaciona con la longitud deonda de la radiación mediante:

Despejando:

Sustituyendo:

2. a) Utilizando las fórmulas que relacionan las magni-tudes ondulatorias con las corpusculares:

fEh

qVh

H= = =⋅ ⋅

⋅= ⋅

1 6 10 400

6 625 109 7 10

19

3416,

,, zz

hp

h

mE

h

mqVλ = = = =

2 2

vmáx =⋅

⋅⋅

⋅−−

−−

2

9 1 106 625 10

3 10

1 27 10331

348

7,,

,,445 10

1 64 10

19

6

⋅⎛

⎝⎜

⎠⎟

= ⋅

vmáx m/s,

vm

hc

Wmáxe

o= −⎛⎝⎜

⎞⎠⎟

12

m v Ec hc

We m x2

m x oá á= = −Δλ

λu m= ⋅⋅

= ⋅− −6 625 103 103 45

5 77 10348

7,,

,

λuo

hc

W=

0 = −hc

Wu

W

W

o

o

= ⋅⋅

⋅− ⋅ ⋅−

−−6 625 10

3 10

3 4 101 6 10 1 534

8

719,

,, ,

== ⋅ =−3 47 10 2 1519, ,J eV

W hc

qVo D= −λ

qV Ec hc

WD máx o= = −Δλ

Page 200: 9523 gl fis_tx2_cas

207

UNIDAD 13

FÍSICA NUCLEAR

1. a) Expresamos en primer lugar la actividad radiacti-va en Bq

La constante λ se calcula a partir del período desemidesintegración:

La ecuación que relaciona la actividad radiactiva(A) con el número de átomos radiactivos (N):

Por tanto:

Sustituyendo:

Cuya masa vale:

b) De modo similar:

c) La antigüedad se calcula fácilmente con la ley de-sintegración:

Despejando el tiempo:

Sustituyendo:

Que equivale a 1295 años.

d) Dentro de 1 000 años el tiempo transcurrido des-de el inicio de la desintegración será:

1295 + 1000 = 2295 a

t = −⋅

⋅= ⋅−

1

3 836 10

8 25 10

9 64 104 0812

19

19,ln

,

,, 11010 s

tN tNo

= −1λ

ln( )

N t N eot( ) = −λ

0 013 7 10

13 710

3 7 10

3 836

108

8

,,

,

,

,

CuBq

CuBq

No

⋅=

=⋅

⋅⋅= ⋅

−109 65 10

9 65 101

6 023

1219

19

,

,,

átomos

atmol

⋅⋅= ⋅ −

10

14 00321

2 24 10233

at

gmol

g,

,

8 247 101

6 023 10

14 00321

11923,

,

,,⋅

⋅=at

mol

at

gmol

992 10 3⋅ − g

N átomos=⋅

⋅= ⋅−

3 164 10

3 836 108 247 10

8

1219,

,,

N tA t

( )( )

A t N t( ) ( )= λ

λ = =⋅

= ⋅ − −ln ln

,,

2 25 730

1

1 807 103 836 1011

12 1

T aa

ss

8 55 103 7 10

13 164 103

108,

,,⋅

⋅= ⋅Cu

BqCu

Bq

c) Las correspondientes longitudes de onda son:

d) De acuerdo con las fórmulas cuánticas:

En los orbitales p el número cuántico l vale 1; portanto:

4. a) Por la ley de Stefan-Boltzmann sabemos que lapotencia total emitida viene dada por la expresión:P = σ T4 S, donde sustituyendo los valores cono-cidos se obtiene que:

b) La masa del Sol disminuye de forma constante acausa de la radiación que emite.

Para calcular dicha disminución tomamos la ecua-ción E = m ⋅ c2 y despejamos la masa, es decir:m = E / c2.

Sustituyendo los valores se obtiene una pérdida deenergía del Sol por segundo de:

4,289 ⋅ 1012 g/s = 4 289 000 t/s.

Para calcular la pérdida anual hay que multiplicarpor los segundos que equivalen a un año, es decir:

4 289 000 31 536 000

135 257 904 000 000

ts

saño

tañ

=

=oo

taño

=

= ⋅135 1012

5 6703 10 5 800

4 6 96 10

8 2 4 4, ( )

( ,

⋅ ⋅ ⋅ ⋅ ⋅

⋅ ⋅ ⋅

− − −W m K

π 88 2

23 26 13 904 10 3 9 10

m

W W s

)

,

=

= ⋅ = ⋅ ⋅ −

Ll = +⋅

=

=⋅

=

1 1 16 625 10

2

26 625 10

21 49

34

34

( ),

,,

π

π⋅⋅ ⋅−10 34 J s

L l lh

l = +( )12π

λ

λ

1 21 2

8

168

1 3

3 10

2 22 101 35 10,

,

,

,,= =

⋅= ⋅

=

−cf

m

cf11 3

8

168

2 32 3

3 10

2 63 101 14 10

3

,

,,

,,=

⋅= ⋅

= =⋅

− m

cf

λ110

4 11 107 30 10

8

158

,,

⋅= ⋅ − m

=⋅ − ⋅− −1 96 10 2 18 10

1 3

17, ,,f

118

3416

2 3

18

6 625 102 63 10

4 90 10 2

,,

,,

⋅= ⋅

=⋅ −

Hz

f,,

,,

18 10

6 625 104 11 10

18

3415⋅

⋅= ⋅

− Hz

Propuestas de evaluación

Page 201: 9523 gl fis_tx2_cas

208

El nuevo núcleo que aparece es el isótopo radiac-tivo del carbono de A = 14.

b) La fórmula que da el defecto de masa es:

Sustituyendo:

c) El intercambio de energía vale:

El signo negativo indica que se libera energía enel proceso; por tanto, se trata de una reacción exo-térmica.

d) El valor de energía liberada calculado correspon-de a un átomo; el correspondiente a 500 g será:

Lo que equivale a 26 TJ.

4. Parte de la energía del fotón se transforma en masa yel resto en energía cinética.

Conocemos la energía de un fotón:

Energía que se transforma en masa (electrón-posi-trón) es: Ee-p = (me + mp) ⋅ c2

Es decir:

Dado que conocemos la equivalencia entre julios yMeV: 1 MeV = 1,6 ⋅ 10-13 J

⇒ = ,E MeVc 6 87

( ,

( , )

6 62 10 3 10

1 6 10

34 8 1

13

⋅ ⋅ ⋅ ⋅ ⋅

− −

)J s m s

m==

= ⋅ ⋅ ⋅ ⋅ ⋅ + ⇒

⇒ =

− −2 9 1 10 3 10

1

31 8 1 2, ( )kg m s E

E

c

c ,,1 10 12⋅ − J

E h= ⋅ c/λ

5001

14 01

6 023 10

114

14

14

23 14

g Nmol N

g N

átomos N

,

, ⋅

mmol N

MeV

átomo N

JMeV

14

14

137 6342

1

1 602 101

⋅⋅ −, ,

== ⋅2 629 1013, J

ΔE uMeVu

MeV= ⋅ = −0 0082931

17 6342, ,

Δm

u

= + − +[ ] =

= −

1 0073 14 0031 1 0087 14 01

0 0082

, , , ,

,

Δm M(p ) M(C ) M(n ) M(N )11

614

01

714= + − +⎡

⎣⎤⎦

La ley aplicada a la actividad es:

Sustituyendo t = 2 295 a = 7,24 ⋅ 1010 s

2. a) Teniendo en cuenta que el valor absoluto de la car-ga del electrón vale:

Obtenemos:

Por tanto:

b) Utilizando la equivalencia aproximada estándar:931 MeV equivalen a 1 u.

Por tanto:

c) La relación entre el defecto de masa y la masa ató-mica es:

Despejando Z:

Sustituyendo:

3. a) La reacción es fácil de escribir teniendo en cuentala conservación de la carga y de la masa:

n N p C01

714

11

614+ → +

Z =− ⋅ +

−=

0 88 84 1 0087 83 81 0073 1 0087

36 001, , ,

( , , ), 88 36≈

Δ

Δ

Δ

m Zm Am Zm M

m m M m m

Zm m M

p n

n p n

n

= + − −

− + = −

=− +

A Z

A

( )

(mm mp n− )

Δm Zm A Z m Mp n= + − −( )

Δm u= 0 88,

819 651

9310 88, ,MeV

uMeV

u=

819 651 602 10

11 31 10

1310,

,,MeV

JMeV

J⋅

= ⋅−

1 10 101 602 10

11 602 106 6

1913MeV eV eV

CeV

= =⋅

= ⋅−

−,, JJ

q Ce = ⋅ −1 602 10 19,

A e( , ) , ,, ,7 24 10 0 01 010 3 836 10 7 24 1012 10

⋅ = =− ⋅ ⋅ ⋅−

00076 Cu

A t A eot( ) = −λ